Academia.eduAcademia.edu
SOLUTION MANUAL FOR TABLE OF CONTENTS 1. Introduction. Engineering and Mechanics. Learning Mechanics. Fundamental Concepts. Units. Newtonian Gravitation. 2. Vectors. Vector Operations and Definitions. Scalars and Vectors. Rules for Manipulating Vectors. Cartesian Components. Components in Two Dimensions. Components in Three Dimensions. Products of Vectors. Dot Products. Cross Products. Mixed Triple Products. 3. Forces. Types of Forces. Equilibrium and Free-Body Diagrams. Two-Dimensional Force Systems. Three-Dimensional Force Systems. 4. Systems of Forces and Moments. Two-Dimensional Description of the Moment. The Moment Vector. Moment of a Force About a Line. Couples. Equivalent Systems. Representing Systems by Equivalent Systems. 5. Objects in Equilibrium. The Equilibrium Equations. Two-Dimensional Applications. Statically Indeterminate Objects. Three-Dimensional Applications. Two-Force and Three-Force. 6. Structures in Equilibrium. Trusses. The Method of Joints. The Method of Sections. Space Trusses. Frames and Machines. 7. Centroids and Centers of Mass 316. Centroids. Centroids of Areas. Centroids of Composite Areas. Distributed Loads. Centroids of Volumes and Lines. The Pappus-Guldinus Theorems. Centers of Mass. Definition of the Center of Mass. Centers of Mass of Objects. Centers of Mass of Composite Objects. 8. Moments of Inertia. Areas. Definitions. Parallel-Axis Theorems. Rotated and Principal Axes. Masses. Simple Objects. Parallel-Axis Theorem. 9. Friction. Theory of Dry Friction. Applications. 10. Internal Forces and Moments. Beams. Axial Force, Shear Force, and Bending Moment. Shear Force and Bending Moment Diagrams. Relations Between Distributed Load, Shear Force, and Bending Moment. Cables. Loads Distributed Uniformly Along Straight Lines. Loads Distributed Uniformly Along Cables. Discrete Loads. Liquids and Gasses. Pressure and the Center of Pressure. Pressure in a Stationary Liquid. 11. Virtual Work and Potential Energy. Virtual Work. Potential Energy. Problem 1.1 The value of  is 3.14159265. . . If C is the circumference of a circle and r is its radius, determine the value of r/C to four significant digits. Problem 1.2 The base of natural logarithms is e D 2.718281828 . . . (a) (b) (c) Express e to five significant digits. Determine the value of e2 to five significant digits. Use the value of e you obtained in part (a) to determine the value of e2 to five significant digits. Solution: C D 2r ) r 1 D D 0.159154943. C 2 To four significant digits we have r D 0.1592 C Solution: The value of e is: e D 2.718281828 (a) To five significant figures e D 2.7183 (b) e2 to five significant figures is e2 D 7.3891 (c) Using the value from part (a) we find e2 D 7.3892 which is not correct in the fifth digit. [Part (c) demonstrates the hazard of using rounded-off values in calculations.] Problem 1.3 A machinist drills a circular hole in a panel with a nominal radius r D 5 mm. The actual radius of the hole is in the range r D 5 š 0.01 mm. (a) To what number of significant digits can you express the radius? (b) To what number of significant digits can you express the area of the hole? Solution: a) The radius is in the range r1 D 4.99 mm to r2 D 5.01 mm. These numbers are not equal at the level of three significant digits, but they are equal if they are rounded off to two significant digits. Two: r D 5.0 mm b) The area of the hole is in the range from A1 D r1 2 D 78.226 m2 to A2 D r2 2 D 78.854 m2 . These numbers are equal only if rounded to one significant digit: One: A D 80 mm2 Problem 1.4 The opening in the soccer goal is 24 ft wide and 8 ft high, so its area is 24 ft ð 8 ft D 192 ft2 . What is its area in m2 to three significant digits? Solution:  A D 192 ft2 1m 3.281 ft 2 D 17.8 m2 A D 17.8 m2 Problem 1.5 The Burj Dubai, scheduled for completion in 2008, will be the world’s tallest building with a height of 705 m. The area of its ground footprint will be 8000 m2 . Convert its height and footprint area to U.S. customary units to three significant digits. Solution:  3.281 ft D 2.31 ð 103 ft 1m   3.218 ft 2 A D 8000 m2 D 8.61 ð 104 ft2 1m  h D 705 m h D 2.31 ð 103 ft, A D 8.61 ð 104 ft2 c 2008 Pearson Education, Inc., Upper Saddle River, NJ. All rights reserved. This material is protected under all copyright laws as they  currently exist. No portion of this material may be reproduced, in any form or by any means, without permission in writing from the publisher. 1 Problem 1.6 Suppose that you have just purchased a Ferrari F355 coupe and you want to know whether you can use your set of SAE (U.S. Customary Units) wrenches to work on it. You have wrenches with widths w D 1/4 in, 1/2 in, 3/4 in, and 1 in, and the car has nuts with dimensions n D 5 mm, 10 mm, 15 mm, 20 mm, and 25 mm. Defining a wrench to fit if w is no more than 2% larger than n, which of your wrenches can you use? Solution: Convert the metric size n to inches, and compute the percentage difference between the metric sized nut and the SAE wrench. The results are:  5 mm 1 inch 25.4 mm   D 0.19685.. in, 0.19685  0.25 0.19685  100 D 27.0%  10 mm  15 mm n  20 mm  25 mm 1 inch 25.4 mm 1 inch 25.4 mm 1 inch 25.4 mm 1 inch 25.4 mm   D 0.3937.. in,   D 0.5905.. in,   D 0.7874.. in,   D 0.9843.. in, 0.3937  0.5 0.3937 0.5905  0.5 0.5905  100 D 27.0%  100 D C15.3% 0.7874  0.75 0.7874 0.9843  1.0 0.9843  100 D C4.7%  100 D 1.6% A negative percentage implies that the metric nut is smaller than the SAE wrench; a positive percentage means that the nut is larger then the wrench. Thus within the definition of the 2% fit, the 1 in wrench will fit the 25 mm nut. The other wrenches cannot be used. Problem 1.7 Suppose that the height of Mt. Everest is known to be between 29,032 ft and 29,034 ft. Based on this information, to how many significant digits can you express the height (a) in feet? (b) in meters?. Solution: a) h1 D 29032 ft h2 D 29034 ft The two heights are equal if rounded off to four significant digits. The fifth digit is not meaningful. Four: h D 29,030 ft b) In meters we have   1m D 8848.52 m h1 D 29032 ft 3.281 ft   1m D 8849.13 m h2 D 29034 ft 3.281 ft These two heights are equal if rounded off to three significant digits. The fourth digit is not meaningful. Three: h D 8850 m Problem 1.8 The maglev (magnetic levitation) train from Shanghai to the airport at Pudong reaches a speed of 430 km/h. Determine its speed (a) in mi/h; (b) ft/s. Problem 1.9 In the 2006 Winter Olympics, the men’s 15-km cross-country skiing race was won by Andrus Veerpalu of Estonia in a time of 38 minutes, 1.3 seconds. Determine his average speed (the distance traveled divided by the time required) to three significant digits (a) in km/h; (b) in mi/h. 2 Solution: a) v D 430 b) v D 430 km h  0.6214 mi 1 km  D 267 mi/h v D 267 mi/h     1 ft 1h km 1000 m D 392 ft/s h 1 km 0.3048 m 3600 s v D 392 ft/s Solution: 15 km  1.3 38 C min 60 a) vD  b) v D 23.7 km/h   60 min 1h 1 mi 1.609 km  D 23.7 km/h v D 23.7 km/h  D 14.7 mi/h v D 14.7 mi/h c 2008 Pearson Education, Inc., Upper Saddle River, NJ. All rights reserved. This material is protected under all copyright laws as they  currently exist. No portion of this material may be reproduced, in any form or by any means, without permission in writing from the publisher. Problem 1.10 The Porsche’s engine exerts 229 ft-lb (foot-pounds) of torque at 4600 rpm. Determine the value of the torque in N-m (Newton-meters). Solution: Problem 1.11 The kinetic energy of the man in Active Example 1.1 is defined by 12 mv2 , where m is his mass and v is his velocity. The man’s mass is 68 kg and he is moving at 6 m/s, so his kinetic energy is 1 2 2 2 2 (68 kg)(6 m/s) D 1224 kg-m /s . What is his kinetic energy in U.S. Customary units? Solution:  T D 229 ft-lb 1N 0.2248 lb  T D 1224 kg-m2 /s2  1m 3.281 ft 1 slug 14.59 kg   D 310 N-m 1 ft 0.3048 m Solution: Use Table 1.2. The result is: Problem 1.13 A furlong per fortnight is a facetious unit of velocity, perhaps made up by a student as a satirical comment on the bewildering variety of units engineers must deal with. A furlong is 660 ft (1/8 mile). A fortnight is 2 weeks (14 nights). If you walk to class at 2 m/s, what is your speed in furlongs per fortnight to three significant digits? Solution: Solution: A D 200 mm2  280 mm120 mm D 20800 mm2  2 1m a) A D 20800 mm2 D 0.0208 m2 A D 0.0208 m2 1000 mm  b) A D 20800 mm2 1 in 25.4 mm g D 9.81 m  v D 12,000 D 903 slug-ft2 /s 1 ft 0.3048 m 1 ft 0.3048 m    D 32.185 . . . 1 furlong 660 ft  ft s2  3600 s hr  D 32.2  24 hr 1 day ft s2   14 day 1 fortnight  furlongs fortnight y 40 mm x 120 mm 40 mm 40 mm 200 mm 2 A D 32.2 in Problem 1.15 The cross-sectional area of the C12ð30 American Standard Channel steel beam is A D 8.81 in2 . What is its cross-sectional area in mm2 ? s2 v D 2 m/s 2 D 32.2 in2 2 T D 903 slug-ft2 /s Problem 1.12 The acceleration due to gravity at sea level in SI units is g D 9.81 m/s2 . By converting units, use this value to determine the acceleration due to gravity at sea level in U.S. Customary units. Problem 1.14 Determine the cross-sectional area of the beam (a) in m2 ; (b) in in2 . T D 310 N-m y A Solution:  A D 8.81 in2 25.4 mm 1 in 2 D 5680 mm2 x c 2008 Pearson Education, Inc., Upper Saddle River, NJ. All rights reserved. This material is protected under all copyright laws as they  currently exist. No portion of this material may be reproduced, in any form or by any means, without permission in writing from the publisher. 3 Problem 1.16 A pressure transducer measures a value of 300 lb/in2 . Determine the value of the pressure in pascals. A pascal (Pa) is one newton per meter squared. Solution: Convert the units using Table 1.2 and the definition of the Pascal unit. The result:  300 lb in2  4.448 N 1 lb   D 2.0683 . . . 106  12 in 1 ft N m2 2  1 ft 0.3048 m 2  D 2.07106  Pa Problem 1.17 A horsepower is 550 ft-lb/s. A watt is 1 N-m/s. Determine how many watts are generated by the engines of the passenger jet if they are producing 7000 horsepower. Solution:  P D 7000 hp  550 ft-lb/s 1 hp 1m 3.281 ft  1N 0.2248 lb  D 5.22 ð 106 W P D 5.22 ð 106 W Problem 1.18 Chapter 7 discusses distributed loads that are expressed in units of force per unit length. If the value of a distributed load is 400 N/m, what is its value in lb/ft?. Problem 1.19 The moment of inertia of the rectangular area about the x axis is given by the equation I D 13 bh3 . The dimensions of the area are b D 200 mm and h D 100 mm. Determine the value of I to four significant digits in terms of (a) mm4 ; (b) m4 ; (c) in4 . Solution:  w D 400 N/m 0.2248 lb 1N  1m 3.281 ft  D 27.4 lb/ft w D 27.4 lb/ft Solution: 1 200 mm100 mm3 D 66.7 ð 106 mm4 3 (a) ID (b) I D 66.7 ð 106 mm4 (c) I D 66.7 ð 106 mm4   y 1m 1000 mm 1 in 25.4 mm 4 D 66.7 ð 106 m4 4 D 160 in4 h x b Problem 1.20 In Example 1.3, instead of Einstein’s equation consider the equation L D mc, where the mass m is in kilograms and the velocity of light c is in meters per second. (a) What are the SI units of L? (b) If the value of L in SI units is 12, what is its value in U.S. Customary base units? 4 Solution: a) L D mc ) b) L D 12 kg-m/s Units L D kg-m/s  0.0685 slug 1 kg  3.281 ft 1m  D 2.70 slug-ft/s L D 2.70 slug-ft/s c 2008 Pearson Education, Inc., Upper Saddle River, NJ. All rights reserved. This material is protected under all copyright laws as they  currently exist. No portion of this material may be reproduced, in any form or by any means, without permission in writing from the publisher. Problem 1.21 The equation D Solution: My I (a) is used in the mechanics of materials to determine normal stresses in beams. (a) (b) D (N-m)m My N D D 2 I m4 m D 2000 N-m0.1 m My D I 7 ð 105 m4 (b) When this equation is expressed in terms of SI base units, M is in newton-meters (N-m), y is in meters (m), and I is in meters to the fourth power (m4 ). What are the SI units of ? If M D 2000 N-m, y D 0.1 m, and I D 7 ð 105 m4 , what is the value of  in U.S. Customary base units? D 59,700  1 lb 4.448 N  0.3048 m ft 2 lb ft2 Problem 1.22 The acceleration due to gravity on the Solution: a) The mass does surface of the moon is 1.62 m/s2 . (a) What would the  on location. The mass in kg is  not depend 14.59 kg mass of the C-clamp in Active Example 1.4 be on the surface mass D 0.397 kg D 0.397 kg 0.0272 slug 1 slug of the moon? (b) What would the weight of the C-clamp in newtons be on the surface of the moon? b) The weight on the surface of the moon is W D mg D 0.397 kg1.62 m/s2  D 0.643 N Problem 1.23 The 1 ft ð 1 ft ð 1 ft cube of iron weighs 490 lb at sea level. Determine the weight in newtons of a 1 m ð 1 m ð 1 m cube of the same material at sea level. Solution: The weight density is  D The weight of the 1 m3 cube is:  W D V D 490 lb 1 ft3   1 m3 W D 0.643N 1 ft 490 lb 1 ft3 1 ft 0.3048 m 3  1N 0.2248 lb  1 ft 1 ft D 77.0 kN Problem 1.24 The area of the Pacific Ocean is 64,186,000 square miles and its average depth is 12,925 ft. Assume that the weight per unit volume of ocean water is 64 lb/ft3 . Determine the mass of the Pacific Ocean (a) in slugs; (b) in kilograms Solution: The volume of the ocean is  V D 64,186,000 mi2 12,925 ft  64 lb/ft3 32.2 ft/s2 2 D 2.312 ð 1019 ft3  (a) m D V D (b) m D 4.60 ð 1019 slugs 2.312 ð 1019 ft3  D 4.60 ð 1019 slugs  Problem 1.25 The acceleration due to gravity at sea level is g D 9.81 m/s2 . The radius of the earth is 6370 km. The universal gravitational constant is G D 6.67 ð 1011 N-m2 /kg2 . Use this information to determine the mass of the earth. 5,280 ft 1 mi Solution: Use Eq (1.3) a D 14.59 kg 1 slug  D 6.71 ð 1020 kg GmE . Solve for the mass, R2  m 2 9.81 m/s2 6370 km2 103 gR2 mE D D  km  G N-m2 11 6.6710  kg2 D 5.9679 . . . 1024  kg D 5.971024  kg c 2008 Pearson Education, Inc., Upper Saddle River, NJ. All rights reserved. This material is protected under all copyright laws as they  currently exist. No portion of this material may be reproduced, in any form or by any means, without permission in writing from the publisher. 5 Problem 1.26 A person weighs 180 lb at sea level. The radius of the earth is 3960 mi. What force is exerted on the person by the gravitational attraction of the earth if he is in a space station in orbit 200 mi above the surface of the earth? Solution: Use Eq (1.5).  W D mg RE r 2  D WE g   g RE RE C H 2  D WE 3960 3960 C 200 2 D 1800.90616 D 163 lb Problem 1.27 The acceleration due to gravity on the surface of the moon is 1.62 m/s2 . The moon’s radius is RM D 1738 km. (a) What is the weight in newtons on the surface of the moon of an object that has a mass of 10 kg? (b) Using the approach described in Example 1.5, determine the force exerted on the object by the gravity of the moon if the object is located 1738 km above the moon’s surface. Solution: Problem 1.28 If an object is near the surface of the earth, the variation of its weight with distance from the center of the earth can often be neglected. The acceleration due to gravity at sea level is g D 9.81 m/s2 . The radius of the earth is 6370 km. The weight of an object at sea level is mg, where m is its mass. At what height above the earth does the weight of the object decrease to 0.99 mg? Solution: Use a variation of Eq (1.5). W D mgM D 10 kg1.26 m/s2  D 12.6 N a)  b) Adapting equation 1.4 we have aM D gM then  F D maM D 10 kg1.62 m/s2  W D 12.6 N RM r 2 . The force is 1738 km 1738 km C 1738 km 2 D 4.05 N F D 4.05 N  W D mg RE RE C h 2 D 0.99 mg Solve for the radial height,  h D RE p 1 0.99   1 D 63701.0050378  1.0 D 32.09 . . . km D 32,100 m D 32.1 km Problem 1.29 The planet Neptune has an equatorial diameter of 49,532 km and its mass is 1.0247 ð 1026 kg. If the planet is modeled as a homogeneous sphere, what is the acceleration due to gravity at its surface? (The universal gravitational constant is G D 6.67 ð 1011 N-m2 /kg2 .) Solution: Problem 1.30 At a point between the earth and the moon, the magnitude of the force exerted on an object by the earth’s gravity equals the magnitude of the force exerted on the object by the moon’s gravity. What is the distance from the center of the earth to that point to three significant digits? The distance from the center of the earth to the center of the moon is 383,000 km, and the radius of the earth is 6370 km. The radius of the moon is 1738 km, and the acceleration due to gravity at its surface is 1.62 m/s2 . Solution: Let rEp be the distance from the Earth to the point where the gravitational accelerations are the same and let rMp be the distance from the Moon to that point. Then, rEp C rMp D rEM D 383,000 km. The fact that the gravitational attractions by the Earth and the Moon at this point are equal leads to the equation mN m  mN  mN D G 2 m ) gN D G 2 rN 2 r rN Note that the radius of Neptune is rN D 12 49,532 km D 24,766 km     N-m2 1.0247 ð 1026 kg 1 km 2 Thus gN D 6.67 ð 1011 24766 km2 1000 m kg2 We have: W D G D 11.1 m/s2  gE RE rEp gN D 11.1 m/s2 2  D gM RM rMp 2 , where rEM D 383,000 km. Substituting the correct numerical values leads to the equation 9.81  m   1738 km 2  m   6370 km 2 D 1.62 , s2 rEp s2 rEM  rEp where rEp is the only unknown. Solving, we get rEp D 344,770 km D 345,000 km. 6 c 2008 Pearson Education, Inc., Upper Saddle River, NJ. All rights reserved. This material is protected under all copyright laws as they  currently exist. No portion of this material may be reproduced, in any form or by any means, without permission in writing from the publisher. Problem 2.1 In Active Example 2.1, suppose that the vectors U and V are reoriented as shown. The vector V is vertical. The magnitudes are jUj D 8 and jVj D 3. Graphically determine the magnitude of the vector U C 2V. 45⬚ U V Solution: Draw the vectors accurately and measure the resultant. R D jU C 2Vj D 5.7 R D 5.7 Problem 2.2 Suppose that the pylon in Example 2.2 is moved closer to the stadium so that the angle between the forces FAB and FAC is 50° . Draw a sketch of the new situation. The magnitudes of the forces are jFAB j D 100 kN and jFAC j D 60 kN. Graphically determine the magnitude and direction of the sum of the forces exerted on the pylon by the cables. Solution: Accurately draw the vectors and measure the magnitude and direction of the resultant jFAB C FAC j D 146 kN ˛ D 32° c 2008 Pearson Education, Inc., Upper Saddle River, NJ. All rights reserved. This material is protected under all copyright laws as they  currently exist. No portion of this material may be reproduced, in any form or by any means, without permission in writing from the publisher. 7 Problem 2.3 The magnitude jFA j D 80 lb and the angle ˛ D 65° . The magnitude jFA C FB j D 120 lb. Graphically determine the magnitude of FB . FB FC ␤ a FA Solution: Accurately draw the vectors and measure the magnitude of FB . jFB j D 62 lb Problem 2.4 The magnitudes jFA j D 40 N, jFB j D 50 N, and jFC j D 40 N. The angle ˛ D 50° and ˇ D 80° . Graphically determine the magnitude of FA C FB C FC . FB FC ␤ a FA Solution: Accurately draw the vectors and measure the magnitude of FA C FB C FC . R D jFA C FB C FC j D 83 N 8 c 2008 Pearson Education, Inc., Upper Saddle River, NJ. All rights reserved. This material is protected under all copyright laws as they  currently exist. No portion of this material may be reproduced, in any form or by any means, without permission in writing from the publisher. Problem 2.5 The magnitudes jFA j D jFB j D jFC j D 100 lb, and the angles ˛ D 30° . Graphically determine the value of the angle ˇ for which the magnitude jFA C FB C FC j is a minimum and the minimum value of jFA C FB C FC j. FB FC ␤ a FA Solution: For a minimum, the vector FC must point back to the origin. R D jFA C FB C FC j D 93.2 lb ˇ D 165° Problem 2.6 The angle  D 50° . Graphically determine the magnitude of the vector rAC . 150 mm 60 mm B rAB A ␪ rBC rAC C Solution: Draw the vectors accurately and then measure jrAC j. jrAC j D 181 mm c 2008 Pearson Education, Inc., Upper Saddle River, NJ. All rights reserved. This material is protected under all copyright laws as they  currently exist. No portion of this material may be reproduced, in any form or by any means, without permission in writing from the publisher. 9 Problem 2.7 The vectors FA and FB represent the forces exerted on the pulley by the belt. Their magnitudes are jFA j D 80 N and jFB j D 60 N. Graphically determine the magnitude of the total force the belt exerts on the pulley. Solution: Draw the vectors accurately and then measure jFA C FB j. jFA C FB j D 134 N FB 45⬚ FA 10⬚ Problem 2.8 The sum of the forces FA C FB C FC D 0. The magnitude jFA j D 100 N and the angle ˛ D 60° . Graphically determine the magnitudes jFB j and jFC j. Solution: Draw the vectors so that they add to zero. jFB j D 86.6 N, jFC j D 50.0 N FB 30 a FA FC Problem 2.9 The sum of the forces FA C FB C FC D 0. The magnitudes jFA j D 100 N and jFB j D 80 N. Graphically determine the magnitude jFC j and the angle ˛. FB 30 a FA FC Solution: Draw the vectors so that they add to zero. jFC j D 50.4 N, ˛ D 52.5° 10 c 2008 Pearson Education, Inc., Upper Saddle River, NJ. All rights reserved. This material is protected under all copyright laws as they  currently exist. No portion of this material may be reproduced, in any form or by any means, without permission in writing from the publisher. Problem 2.10 The forces acting on the sailplane are represented by three vectors. The lift L and drag D are perpendicular. The magnitude of the weight W is 500 lb. The sum of the forces W C L C D D 0. Graphically determine the magnitudes of the lift and drag. L 25⬚ D W Solution: Draw the vectors so that they add to zero. Then measure the unknown magnitudes. jLj D 453 lb jDj D 211 lb Problem 2.11 A spherical storage tank is suspended from cables. The tank is subjected to three forces, the forces FA and FB exerted by the cables and its weight W. The weight of the tank is jWj D 600 lb. The vector sum of the forces acting on the tank equals zero. Graphically determine the magnitudes of FA and FB . FA 40˚ Solution: Draw the vectors so that they add to zero. Then measure the unknown magnitudes. jFA j D jFB j D 319 lb FB 20˚ 20˚ W c 2008 Pearson Education, Inc., Upper Saddle River, NJ. All rights reserved. This material is protected under all copyright laws as they  currently exist. No portion of this material may be reproduced, in any form or by any means, without permission in writing from the publisher. 11 Problem 2.12 The rope ABC exerts forces FBA and FBC of equal magnitude on the block at B. The magnitude of the total force exerted on the block by the two forces is 200 lb. Graphically determine jFBA j. Solution: Draw the vectors accurately and then measure the unknown magnitudes. jFBA j D 174 lb FBC C 20⬚ B B FBA A Problem 2.13 Two snowcats tow an emergency shelter to a new location near McMurdo Station, Antarctica. (The top view is shown. The cables are horizontal.) The total force FA C FB exerted on the shelter is in the direction parallel to the line L and its magnitude is 400 lb. Graphically determine the magnitudes of FA and FB . Solution: Draw the vectors accurately and then measure the unknown magnitudes. jFA j D 203 lb jFB j D 311 lb L FA 30⬚ 50⬚ FB Top View Problem 2.14 A surveyor determines that the horizontal distance from A to B is 400 m and the horizontal distance from A to C is 600 m. Graphically determine the magnitude of the vector rBC and the angle ˛. Solution: Draw the vectors accurately and then measure the unknown magnitude and angle. jrBC j D 390 m ˛ D 21.2° North B a rBC C 60⬚ 20⬚ A 12 East c 2008 Pearson Education, Inc., Upper Saddle River, NJ. All rights reserved. This material is protected under all copyright laws as they  currently exist. No portion of this material may be reproduced, in any form or by any means, without permission in writing from the publisher. Problem 2.15 The vector r extends from point A to the midpoint between points B and C. Prove that C r D 12 rAB C rAC . rAC r rAB B A Solution: The proof is straightforward: C r D rAB C rBM , and r D rAC C rCM . rAC r Add the two equations and note that rBM C rCM D 0, since the two vectors are equal and opposite in direction. Thus 2r D rAC C rAB , or r D 1 2 rAC C rAB  A M B rAB Problem 2.16 By drawing sketches of the vectors, explain why U C V C W D U C V C W. Solution: Additive associativity for vectors is usually given as an axiom in the theory of vector algebra, and of course axioms are not subject to proof. However we can by sketches show that associativity for vector addition is intuitively reasonable: Given the three vectors to be added, (a) shows the addition first of V C W, and then the addition of U. The result is the vector U C V C W. V U V+W (a) W U+[V+W] V (b) shows the addition of U C V, and then the addition of W, leading to the result U C V C W. U W The final vector in the two sketches is the same vector, illustrating that associativity of vector addition is intuitively reasonable. Problem 2.17 A force F D 40 i  20 j N. What is its magnitude jFj? U+V (b) [U+V]+W p Solution: jFj D 402 C 202 D 44.7 N Strategy: The magnitude of a vector in terms of its components is given by Eq. (2.8). Problem 2.18 An engineer estimating the components of a force F D Fx i C Fy j acting on a bridge abutment has determined that Fx D 130 MN, jFj D 165 MN, and Fy is negative. What is Fy ? Solution:  jFj D jFy j D jFx j2 C jFy j2  jFj2  jFx j2 D  165 MN2  130 MN2 D 101.6 MN Fy D 102 MN c 2008 Pearson Education, Inc., Upper Saddle River, NJ. All rights reserved. This material is protected under all copyright laws as they  currently exist. No portion of this material may be reproduced, in any form or by any means, without permission in writing from the publisher. 13 Problem 2.19 A support is subjected to a force F D Fx i C 80j (N). If the support will safely support a force of 100 N, what is the allowable range of values of the component Fx ? Solution: Use the definition of magnitude in Eq. (2.8) and reduce algebraically. 100 ½  Fx 2 C 802 , from which 1002  802 ½ Fx 2 . Thus jFx j  Problem 2.20 If FA D 600i  800j (kip) and FB D 200i  200j (kip), what is the magnitude of the force F D FA  2FB ? Solution: Take the scalar multiple of FB , add the components of the two forces as in Eq. (2.9), and use the definition of the magnitude. F D 600  2200i C 800  2200j D 200i  400j jFj D Problem 2.21 The forces acting on the sailplane are its weight W D 500jlb, the drag D D 200i C 100j(lb) and the lift L. The sum of the forces W C L C D D 0. Determine the components and the magnitude of L. p 3600, or 60  Fx   C60 (N)  2002 C 4002 D 447.2 kip y L Solution: D L D W  D D 500j  200i C 100j D 200i C 400jlb jLj D  200 lb2 C 400 lb2 D 447 lb W L D 200i C 400jlb, jLj D 447 lb x 14 c 2008 Pearson Education, Inc., Upper Saddle River, NJ. All rights reserved. This material is protected under all copyright laws as they  currently exist. No portion of this material may be reproduced, in any form or by any means, without permission in writing from the publisher. Problem 2.22 Two perpendicular vectors U and V lie in the x-y plane. The vector U D 6i  8j and jVj D 20. What are the components of V? (Notice that this problem has two answers.) Solution: The two possible values of V are shown in the sketch. y The strategy is to (a) determine the unit vector associated with U, (b) express this vector in terms of an angle, (c) add š90° to this angle, (d) determine the two unit vectors perpendicular to U, and (e) calculate the components of the two possible values of V. The unit vector parallel to U is eU D  6i 62 C 82 8j  D 0.6i  0.8j 2 6 C 82 V2 6 V1 U x 8 Expressed in terms of an angle, eU D i cos ˛  j sin ˛ D i cos53.1°   j sin53.1°  Add š90° to find the two unit vectors that are perpendicular to this unit vector: ep1 D i cos143.1°   j sin143.1°  D 0.8i  0.6j ep2 D i cos36.9°   j sin36.9°  D 0.8i C 0.6j Take the scalar multiple of these unit vectors to find the two vectors perpendicular to U. V1 D jVj0.8i  0.6j D 16i  12j. The components are Vx D 16, Vy D 12 V2 D jVj0.8i C 0.6j D 16i C 12j. The components are Vx D 16, Vy D 12 Problem 2.23 A fish exerts a 10-lb force on the line that is represented by the vector F. Express F in terms of components using the coordinate system shown. Solution: We can use similar triangles to determine the components of F.  F D 10 lb y  11 7 p i p j D 5.37i  8.44j lb 72 C 112 72 C 112 F D 5.37i  8.44j lb 7 11 F x c 2008 Pearson Education, Inc., Upper Saddle River, NJ. All rights reserved. This material is protected under all copyright laws as they  currently exist. No portion of this material may be reproduced, in any form or by any means, without permission in writing from the publisher. 15 Problem 2.24 A man exerts a 60-lb force F to push a crate onto a truck. (a) Express F in terms of components using the coordinate system shown. (b) The weight of the crate is 100 lb. Determine the magnitude of the sum of the forces exerted by the man and the crate’s weight. Solution: (a) F D 60 lbcos 20° i C sin 20° j D 56.4i C 20.5j lb F D 56.4i C 20.5jlb (b) W D 100 lbj y F C W D 56.4i C [20.5  100]j lb D 56.4i  79.5j lb F 20⬚ jF C Wj D  56.4 lb2 C 79.5 lb2 D 97.4 lb jF C Wj D 97.4 lb x Problem 2.25 The missile’s engine exerts a 260-kN force F. (a) Express F in terms of components using the coordinate system shown. (b) The mass of the missile is 8800 kg. Determine the magnitude of the sum of the forces exerted by the engine and the missile’s weight. y F 3 4 Solution: (a) We can use similar triangles to determine the components of F.  F D 260 kN p  3 iC p j D 208i C 156j kN 42 C 32 42 C 32 4 x F D 208i C 156j kN (b) The missile’s weight W can be expressed in component and then added to the force F. W D 8800 kg9.81 m/s2 j D 86.3 kNj F C W D 208i C [156  86.3]j kN D 208i  69.7j kN jF C Wj D  208 kN2 C 69.7 kN2 D 219 kN jF C Wj D 219 kN Problem 2.26 For the truss shown, express the position vector rAD from point A to point D in terms of components. Use your result to determine the distance from point A to point D. y rAD D 0  1.8 mi C 0.4 m  0.7 mj D 1.8i  0.3j m  rAD D 1.8 m2 C 0.3 m2 D 1.825 m B A 0.6 m D 0.7 m 0.4 m C 0.6 m 16 Solution: Coordinates A(1.8, 0.7) m, D(0, 0.4) m x 1.2 m c 2008 Pearson Education, Inc., Upper Saddle River, NJ. All rights reserved. This material is protected under all copyright laws as they  currently exist. No portion of this material may be reproduced, in any form or by any means, without permission in writing from the publisher. Problem 2.27 The points A, B, . . . are the joints of the hexagonal structural element. Let rAB be the position vector from joint A to joint B, rAC the position vector from joint A to joint C, and so forth. Determine the components of the vectors rAC and rAF . Solution: Use the xy coordinate system shown and find the locations of C and F in those coordinates. The coordinates of the points in this system are the scalar components of the vectors rAC and rAF . For rAC , we have rAC D rAB C rBC D xB  xA i C yB  yA j y C xC  xB i C yC  yB j E D 2m rAC D 2m  0i C 0  0j C 2m cos 60°  0i or C 2m cos 60°  0j, F giving C rAC D 2m C 2m cos 60° i C 2m sin 60° j. For rAF , we have A B x rAF D xF  xA i C yF  yA j D 2m cos 60° xF  0i C 2m sin 60°  0j. Problem 2.28 For the hexagonal structural element in Problem 2.27, determine the components of the vector rAB  rBC . Solution: rAB  rBC . The angle between BC and the x-axis is 60° . rBC D 2 cos60° i C 2sin60° j m rBC D 1i C 1.73j m rAB  rBC D 2i  1i  1.73j m rAB  rBC D 1i  1.73j m Problem 2.29 The coordinates of point A are (1.8, 3.0) ft. The y coordinate of point B is 0.6 ft. The vector rAB has the same direction as the unit vector eAB D 0.616i  0.788j. What are the components of rAB ? y Solution: The vector rAB can be written two ways. A rAB rAB D jrAB j0.616i  0.788j D Bx  Ax i C By  Ay j Comparing the two expressions we have By  Ay  D 0.6  3.0ft D 0.788jrAB j jrAB j D B x 2.4 ft D 3.05 ft 0.788 Thus rAB D jrAB j0.616i  0.788j D 3.05 ft0.616i  0.788j D 1.88i  2.40j ft rAB D 1.88i  2.40j ft c 2008 Pearson Education, Inc., Upper Saddle River, NJ. All rights reserved. This material is protected under all copyright laws as they  currently exist. No portion of this material may be reproduced, in any form or by any means, without permission in writing from the publisher. 17 y Problem 2.30 (a) Express the position vector from point A of the front-end loader to point B in terms of components. 98 in 45 in C (b) Express the position vector from point B to point C in terms of components. A 55 in (c) Use the results of (a) and (b) to determine the distance from point A to point C. B 50 in 35 in x 50 in Solution: The coordinates are A(50, 35); B(98, 50); C(45, 55). (a) The vector from point A to B: rAB D 98  50i C 50  35j D 48i C 15j (in) (b) The vector from point B to C is rBC D 45  98i C 55  50j D 53i C 5j (in). (c) The distance from A to C is the magnitude of the sum of the vectors, rAC D rAB C rBC D 48  53i C 15 C 5j D 5i C 20j. The distance from A to C is jrAC j D 18  52 C 202 D 20.62 in c 2008 Pearson Education, Inc., Upper Saddle River, NJ. All rights reserved. This material is protected under all copyright laws as they  currently exist. No portion of this material may be reproduced, in any form or by any means, without permission in writing from the publisher. Problem 2.31 In Active Example 2.3, the cable AB exerts a 900-N force on the top of the tower. Suppose that the attachment point B is moved in the horizontal direction farther from the tower, and assume that the magnitude of the force F the cable exerts on the top of the tower is proportional to the length of the cable. (a) What is the distance from the tower to point B if the magnitude of the force is 1000 N? (b) Express the 1000-N force F in terms of components using the coordinate system shown. A 80 m 40 m y Solution: In the new problem assume that point B is located a distance d away from the base. The lengths in the original problem and in the new problem are given by  Loriginal D Lnew D (a)  40 m2 C 80 m2 D  A Force exerted on the tower by cable AB 8000 m2 d2 C 80 m2 80 m F The force is proportional to the length. Therefore  d2 C 80 m2 1000 N D 900 N  8000 m2  dD B  8000 m2  1000 N 900 N B x 40 m 2  80 m2 D 59.0 m d D 59.0 m (b) The force F is then  F D 1000 N  d d2 C 80 m2 i  80 m d2 C 80 m2 j D 593i  805j N F D 593i  805j N c 2008 Pearson Education, Inc., Upper Saddle River, NJ. All rights reserved. This material is protected under all copyright laws as they  currently exist. No portion of this material may be reproduced, in any form or by any means, without permission in writing from the publisher. 19 Problem 2.32 Determine the position vector rAB in terms of its components if (a)  D 30° , (b)  D 225° . y 150 mm 60 mm B rAB rBC C θ x A Solution: (a) y rAB D 60 cos30° i C 60 sin30° j, or 150 mm 60 mm rAB D 51.96i C 30j mm. And B (b) FAB rAB D 60 cos225° i C 60 sin225° j or A rAB D 42.4i  42.4j mm. θ FBC C F x Problem 2.33 In Example 2.4, the coordinates of the fixed point A are (17, 1) ft. The driver lowers the bed of the truck into a new position in which the coordinates of point B are (9, 3) ft. The magnitude of the force F exerted on the bed by the hydraulic cylinder when the bed is in the new position is 4800 lb. Draw a sketch of the new situation. Express F in terms of components. y B B 30⬚ F A 30⬚ A x Solution:   D tan1 2 ft 8 ft  D 14.04° F D 4800 lb cos i C sin j. F D 4660i C 1160j lb 20 c 2008 Pearson Education, Inc., Upper Saddle River, NJ. All rights reserved. This material is protected under all copyright laws as they  currently exist. No portion of this material may be reproduced, in any form or by any means, without permission in writing from the publisher. y Problem 2.34 A surveyor measures the location of point A and determines that rOA D 400i C 800j (m). He wants to determine the location of a point B so that jrAB j D 400 m and jrOA C rAB j D 1200 m. What are the cartesian coordinates of point B? B A N rAB rOA Proposed roadway x O Solution: Two possibilities are: The point B lies west of point A, or point B lies east of point A, as shown. The strategy is to determine the unknown angles ˛, ˇ, and . The magnitude of OA is jrOA j D  B 4002 C 8002 D 894.4. α The angle ˇ is determined by tan ˇ D A B y α θ β 800 D 2, ˇ D 63.4° . 400 0 x The angle ˛ is determined from the cosine law: cos ˛ D 894.42 C 12002  4002 D 0.9689. 2894.41200 ˛ D 14.3° . The angle  is  D ˇ š ˛ D 49.12° , 77.74° . The two possible sets of coordinates of point B are rOB D 1200i cos 77.7 C j sin 77.7 D 254.67i C 1172.66j (m) rOB D 1200i cos 49.1 C j sin 49.1 D 785.33i C 907.34j (m) The two possibilities lead to B(254.7 m, 1172.7 m) or B(785.3 m, 907.3 m) Problem 2.35 The magnitude of the position vector rBA from point B to point A is 6 m and the magnitude of the position vector rCA from point C to point A is 4 m. What are the components of rBA ? y Thus rBA D xA  0i C yA  0j ) 6 m2 D xA 2 C yA 2 rCA D xA  3 mi C yA  0j ) 4 m2 D xA  3 m2 C yA 2 3m B Solution: The coordinates are: AxA , yA , B0, 0, C3 m, 0 x C Solving these two equations, we find xA D 4.833 m, yA D š3.555 m. We choose the “-” sign and find rBA D 4.83i  3.56j m A c 2008 Pearson Education, Inc., Upper Saddle River, NJ. All rights reserved. This material is protected under all copyright laws as they  currently exist. No portion of this material may be reproduced, in any form or by any means, without permission in writing from the publisher. 21 Problem 2.36 In Problem 2.35, determine the components of a unit vector eCA that points from point C toward point A. Strategy: Determine the components of rCA and then divide the vector rCA by its magnitude. Solution: From the previous problem we have rCA D 1.83i  3.56j m, rCA D  1.832 C 3.562 m D 3.56 m Thus eCA D rCA D 0.458i  0.889j rCA Problem 2.37 The x and y coordinates of points A, B, and C of the sailboat are shown. (a) Determine the components of a unit vector that is parallel to the forestay AB and points from A toward B. (b) Determine the components of a unit vector that is parallel to the backstay BC and points from C toward B. Solution: rAB D xB  xA i C yB  yA j rCB D xB  xC i C yC  yB j Points are: A (0, 1.2), B (4, 13) and C (9, 1) Substituting, we get rAB D 4i C 11.8j m, jrAB j D 12.46 m y B (4, 13) m rCB D 5i C 12j m, jrCB j D 13 m The unit vectors are given by eAB D rAB rCB and eCB D jrAB j jrCB j Substituting, we get eAB D 0.321i C 0.947j eCB D 0.385i C 0.923j A (0, 1.2) m 22 C (9, 1) m x c 2008 Pearson Education, Inc., Upper Saddle River, NJ. All rights reserved. This material is protected under all copyright laws as they  currently exist. No portion of this material may be reproduced, in any form or by any means, without permission in writing from the publisher. Problem 2.38 The length of the bar AB is 0.6 m. Determine the components of a unit vector eAB that points from point A toward point B. y B 0.4 m A 0.3 m x Solution: We need to find the coordinates of point Bx, y B We have the two equations m y m 0.6 0.4 0.3 m C x2 C y 2 D 0.6 m2 x 2 C y 2 D 0.4 m2 Solving we find x D 0.183 m, y D 0.356 m A 0.3 m O x Thus eAB D rAB 0.183 m  [0.3 m]i C 0.356 mj D  rAB 0.183 m C 0.3 m2 C 0.356 m2 D 0.806i C 0.593j y Problem 2.39 Determine the components of a unit vector that is parallel to the hydraulic actuator BC and points from B toward C. 1m D C Solution: Point B is at (0.75, 0) and point C is at (0, 0.6). The vector 1m rBC D xC  xB i C yC  yB j 0.6 m B A x rBC D 0  0.75i C 0.6  0j m rBC D 0.75i C 0.6j m jrBC j D eBC D  0.15 m 0.6 m Scoop 0.752 C 0.62 D 0.960 m 0.75 0.6 rBC D iC j jrBC j 0.96 0.96 eBC D 0.781i C 0.625j c 2008 Pearson Education, Inc., Upper Saddle River, NJ. All rights reserved. This material is protected under all copyright laws as they  currently exist. No portion of this material may be reproduced, in any form or by any means, without permission in writing from the publisher. 23 Problem 2.40 The hydraulic actuator BC in Problem 2.39 exerts a 1.2-kN force F on the joint at C that is parallel to the actuator and points from B toward C. Determine the components of F. Solution: From the solution to Problem 2.39, eBC D 0.781i C 0.625j The vector F is given by F D jFjeBC F D 1.20.781i C 0.625j k Ð N F D 937i C 750j N Problem 2.41 A surveyor finds that the length of the line OA is 1500 m and the length of line OB is 2000 m. y N (a) Determine the components of the position vector from point A to point B. (b) Determine the components of a unit vector that points from point A toward point B. A Proposed bridge B Solution: We need to find the coordinates of points A and B rOA D 1500 cos 60° i C 1500 sin 60° j 60⬚ 30⬚ rOA D 750i C 1299j m Point A is at (750, 1299) (m) River x O rOB D 2000 cos 30° i C 2000 sin 30° j m rOB D 1732i C 1000j m Point B is at (1732, 1000) (m) (a) The vector from A to B is rAB D xB  xA i C yB  yA j rAB D 982i  299j m (b) The unit vector eAB is eAB D 982i  299j rAB D jrAB j 1026.6 eAB D 0.957i  0.291j 24 c 2008 Pearson Education, Inc., Upper Saddle River, NJ. All rights reserved. This material is protected under all copyright laws as they  currently exist. No portion of this material may be reproduced, in any form or by any means, without permission in writing from the publisher. Problem 2.42 The magnitudes of the forces exerted by the cables are jT1 j D 2800 lb, jT2 D 3200 lb, jT3 j D 4000 lb, and jT4 j D 5000 lb. What is the magnitude of the total force exerted by the four cables? y T4 51⬚ T3 40⬚ T2 29⬚ T1 9⬚ x Solution: The x-component of the total force is Tx D jT1 j cos 9° C jT2 j cos 29° jT3 j cos 40° C jT4 j cos 51° Tx D 2800 lb cos 9° C 3200 lb cos 29° C 4000 lb cos 40° C 5000 lb cos 51° Tx D 11,800 lb The y-component of the total force is Ty D jT1 j sin 9° C jT2 j sin 29° C jT3 j sin 40° C jT4 j sin 51° Ty D 2800 lb sin 9° C 3200 lb sin 29° C 4000 lb sin 40° C 5000 lb sin 51° Ty D 8450 lb The magnitude of the total force is  jTj D Tx 2 C Ty 2 D  11,800 lb2 C 8450 lb2 D 14,500 lb jTj D 14,500 lb c 2008 Pearson Education, Inc., Upper Saddle River, NJ. All rights reserved. This material is protected under all copyright laws as they  currently exist. No portion of this material may be reproduced, in any form or by any means, without permission in writing from the publisher. 25 Problem 2.43 The tensions in the four cables are equal: jT1 j D jT2 j D jT3 j D jT4 j D T. Determine the value of T so that the four cables exert a total force of 12,500-lb magnitude on the support. y T4 51⬚ T3 40⬚ T2 29⬚ T1 9⬚ x Solution: The x-component of the total force is Tx D T cos 9° C T cos 29° C T cos 40° C T cos 51° Tx D 3.26T The y-component of the total force is Ty D T sin 9° C T sin 29° C T sin 40° C T sin 51° Ty D 2.06T The magnitude of the total force is  jTj D Tx 2 C Ty 2 D Solving for T we find 26  3.26T2 C 2.06T2 D 3.86T D 12,500 lb T D 3240 lb c 2008 Pearson Education, Inc., Upper Saddle River, NJ. All rights reserved. This material is protected under all copyright laws as they  currently exist. No portion of this material may be reproduced, in any form or by any means, without permission in writing from the publisher. Problem 2.44 The rope ABC exerts forces FBA and FBC on the block at B. Their magnitudes are equal: jFBA j D jFBC j. The magnitude of the total force exerted on the block at B by the rope is jFBA C FBC j D 920 N. Determine jFBA j by expressing the forces FBA and FBC in terms of components. FBC C 20° B B FBA A Solution: FBC FBC D Fcos 20° i C sin 20° j 20° FBA D Fj FBC C FBA D Fcos 20° i C [sin 20°  1]j Therefore 920 N2 D F2 cos2 20° C [sin 20°  1]2  ) F D 802 N FBA Problem 2.45 The magnitude of the horizontal force F1 is 5 kN and F1 C F2 C F3 D 0. What are the magnitudes of F2 and F3 ? y F3 30˚ F1 Solution: Using components we have Fx : 5 kN C F2 cos 45°  F3 cos 30° D 0 45˚ Fy : F2 sin 45° C F3 sin 30° D0 F2 Solving simultaneously yields: x ) F2 D 9.66 kN, F3 D 13.66 kN c 2008 Pearson Education, Inc., Upper Saddle River, NJ. All rights reserved. This material is protected under all copyright laws as they  currently exist. No portion of this material may be reproduced, in any form or by any means, without permission in writing from the publisher. 27 Problem 2.46 Four groups engage in a tug-of-war. The magnitudes of the forces exerted by groups B, C, and D are jFB j D 800 lb, jFC j D 1000 lb, jFD j D 900 lb. If the vector sum of the four forces equals zero, what are the magnitude of FA and the angle ˛? y FB FC 70° 30° Solution: The strategy is to use the angles and magnitudes to determine the force vector components, to solve for the unknown force FA and then take its magnitude. The force vectors are FB D 800i cos 110° C j sin 110°  D 273.6i C 751.75j FC D 1000i cos 30° C j sin 30°  D 866i C 500j α 20° FD FA x FD D 900i cos20°  C j sin20°  D 845.72i  307.8j FA D jFA ji cos180 C ˛ C j sin180 C ˛ D jFA ji cos ˛  j sin ˛ The sum vanishes: FA C FB C FC C FD D i1438.1  jFA j cos ˛ C j944  jFA j sin ˛ D 0 From which FA D 1438.1i C 944j. The magnitude is jFA j D  14382 C 9442 D 1720 lb The angle is: tan ˛ D 28 944 D 0.6565, or ˛ D 33.3° 1438 c 2008 Pearson Education, Inc., Upper Saddle River, NJ. All rights reserved. This material is protected under all copyright laws as they  currently exist. No portion of this material may be reproduced, in any form or by any means, without permission in writing from the publisher. Problem 2.47 In Example 2.5, suppose that the attachment point of cable A is moved so that the angle between the cable and the wall increases from 40° to 55° . Draw a sketch showing the forces exerted on the hook by the two cables. If you want the total force FA C FB to have a magnitude of 200 lb and be in the direction perpendicular to the wall, what are the necessary magnitudes of FA and FB ? Solution: Let FA and FB be the magnitudes of FA and FB . The component of the total force parallel to the wall must be zero. And the sum of the components perpendicular to the wall must be 200 lb. A 40⬚ 20⬚ B FA cos 55°  FB cos 20° D 0 FA sin 55° C FB sin 20° D 200 lb Solving we find FA 40⬚ FA D 195 lb FB D 119 lb 20⬚ FB c 2008 Pearson Education, Inc., Upper Saddle River, NJ. All rights reserved. This material is protected under all copyright laws as they  currently exist. No portion of this material may be reproduced, in any form or by any means, without permission in writing from the publisher. 29 Problem 2.48 The bracket must support the two forces shown, where jF1 j D jF2 j D 2 kN. An engineer determines that the bracket will safely support a total force of magnitude 3.5 kN in any direction. Assume that 0  ˛  90° . What is the safe range of the angle ˛? F2 α F1 F2 Solution: Fx : 2 kN C 2 kN cos ˛ D 2 kN1 C cos ˛ F1 β α α F1 + F2 Fy : 2 kN sin ˛ Thus the total force has a magnitude given by F D 2 kN  p 1 C cos ˛2 C sin ˛2 D 2 kN 2 C 2 cos ˛ D 3.5 kN Thus when we are at the limits we have  2 C 2 cos ˛ D 3.5 kN 2 kN 2 D 49 17 ) cos ˛ D ) ˛ D 57.9° 16 32 In order to be safe we must have 57.9°  ˛  90° 30 c 2008 Pearson Education, Inc., Upper Saddle River, NJ. All rights reserved. This material is protected under all copyright laws as they  currently exist. No portion of this material may be reproduced, in any form or by any means, without permission in writing from the publisher. Problem 2.49 The figure shows three forces acting on a joint of a structure. The magnitude of Fc is 60 kN, and FA C FB C FC D 0. What are the magnitudes of FA and FB ? y FC FB 15° x 40° FA Solution: We need to write each force in terms of its components. FA 195° FA D jFA j cos 40i C jFA j sin 40j kN 40° x FB D jFB j cos 195° i C jFB j sin 195j kN FC D jFC j cos 270° i C jFC j sin 270° j kN FB Thus FC D 60j kN 270° Since FA C FB C FC D 0, their components in each direction must also sum to zero. FC FAx C FBx C FCx D 0 FAy C FBy C FCy D 0 Thus, jFA j cos 40° C jFB j cos 195° C 0 D 0 jFA j sin 40° C jFB j sin 195°  60 kN D 0 Solving for jFA j and jFB j, we get jFA j D 137 kN, jFB j D 109 kN Problem 2.50 Four forces act on a beam. The vector sum of the forces is zero. The magnitudes jFB j D 10 kN and jFC j D 5 kN. Determine the magnitudes of FA and FD . FD 30° FA FB FC Solution: Use the angles and magnitudes to determine the vectors, and then solve for the unknowns. The vectors are: FA D jFA ji cos 30° C j sin 30°  D 0.866jFA ji C 0.5jFA jj FB D 0i  10j, FC D 0i C 5j, FD D jFD ji C 0j. Take the sum of each component in the x- and y-directions: Fx D 0.866jFA j  jFD ji D 0 and Fy D 0.5jFA j  10  5j D 0. From the second equation we get jFA j D 10 kN . Using this value in the first equation, we get jFD j D 8.7 kN c 2008 Pearson Education, Inc., Upper Saddle River, NJ. All rights reserved. This material is protected under all copyright laws as they  currently exist. No portion of this material may be reproduced, in any form or by any means, without permission in writing from the publisher. 31 Problem 2.51 Six forces act on a beam that forms part of a building’s frame. The vector sum of the forces is zero. The magnitudes jFB j D jFE j D 20 kN, jFC j D 16 kN, and jFD j D 9 kN. Determine the magnitudes of FA and FG . FA 70⬚ FC FD 40⬚ 50⬚ 40⬚ FB Solution: Write each force in terms of its magnitude and direction FG FE y as F D jFj cos i C jFj sin j where  is measured counterclockwise from the Cx-axis. Thus, (all forces in kN) θ FA D jFA j cos 110° i C jFA j sin 110° j kN FB D 20 cos 270° i C 20 sin 270° j kN x FC D 16 cos 140° i C 16 sin 140° j kN FD D 9 cos 40° i C 9 sin 40° j kN FE D 20 cos 270° i C 20 sin 270° j kN FG D jFG j cos 50° i C jFG j sin 50° j kN We know that the x components and y components of the forces must add separately to zero. Thus FAx C FBx C FCx C FDx C FEx C FGx D 0 FAy C FBy C FCy C FDy C FEy C FGy D 0 jFA j cos 110° C 0  12.26 C 6.89 C 0 C jFG j cos 50° D 0 jFA j sin 110°  20 C 10.28 C 5.79  20 C jFG j sin 50° D 0 Solving, we get jFA j D 13.0 kN 32 jFG j D 15.3 kN c 2008 Pearson Education, Inc., Upper Saddle River, NJ. All rights reserved. This material is protected under all copyright laws as they  currently exist. No portion of this material may be reproduced, in any form or by any means, without permission in writing from the publisher. Problem 2.52 The total weight of the man and parasail is jWj D 230 lb. The drag force D is perpendicular to the lift force L. If the vector sum of the three forces is zero, what are the magnitudes of L and D? y Solution: Let L and D be the magnitudes of the lift and drag forces. We can use similar triangles to express the vectors L and D in terms of components. Then the sum of the forces is zero. Breaking into components we have p L 5 p 2 2 22 C 52 5 22 C 52 L p L p 5 22 C 52 2 22 C 52 DD0 D  230 lb D 0 Solving we find D jDj D 85.4 lb, jLj D 214 lb x W Problem 2.53 The three forces acting on the car are shown. The force T is parallel to the x axis and the magnitude of the force W is 14 kN. If T C W C N D 0, what are the magnitudes of the forces T and N? Solution: Fx : T  N sin 20° D 0 Fy : N cos 20°  14 kN D 0 Solving we find N D 14.90 N, T D 5.10 N 20⬚ y T W x 20⬚ N c 2008 Pearson Education, Inc., Upper Saddle River, NJ. All rights reserved. This material is protected under all copyright laws as they  currently exist. No portion of this material may be reproduced, in any form or by any means, without permission in writing from the publisher. 33 Problem 2.54 The cables A, B, and C help support a pillar that forms part of the supports of a structure. The magnitudes of the forces exerted by the cables are equal: jFA j D jFB j D jFC j. The magnitude of the vector sum of the three forces is 200 kN. What is jFA j? Solution: Use the angles and magnitudes to determine the vector components, take the sum, and solve for the unknown. The angles between each cable and the pillar are:  A D tan1 B D tan1 FC FA FB  D 33.7° ,   8 D 53.1° 6  C D tan1 4m 6m 12 6  D 63.4° . 6m A B Measure the angles counterclockwise form the x-axis. The force vectors acting along the cables are: C FA D jFA ji cos 303.7° C j sin 303.7°  D 0.5548jFA ji  0.8319jFA jj 4m 4m 4m FB D jFB ji cos 323.1° C j sin 323.1°  D 0.7997jFB ji  0.6004jFB jj FC D jFC ji cos 333.4° C j sin 333.4°  D 0.8944jFC ji0.4472jFC jj The sum of the forces are, noting that each is equal in magnitude, is F D 2.2489jFA ji  1.8795jFA jj. The magnitude of the sum is given by the problem:  200 D jFA j 2.24892 C 1.87952 D 2.931jFA j, from which jFA j D 68.24 kN Problem 2.55 The total force exerted on the top of the mast B by the sailboat’s forestay AB and backstay BC is 180i  820j (N). What are the magnitudes of the forces exerted at B by the cables AB and BC ? Solution: We first identify the forces: FAB D TAB  4.0 mi  11.8 mj 4.0 m2 C 11.8 m2 FBC D TBC  y B (4, 13) m 5.0 mi  12.0 mj 5.0 m2 C 12.0 m2 Then if we add the force we find 4 5 TAB C p TBC D 180 N Fx :  p 155.24 169 11.8 12 TAB  p TBC D 820 N Fy :  p 155.24 169 Solving simultaneously yields: ) TAB D 226 N, A (0, 1.2) m 34 C (9, 1) m TAC D 657 N x c 2008 Pearson Education, Inc., Upper Saddle River, NJ. All rights reserved. This material is protected under all copyright laws as they  currently exist. No portion of this material may be reproduced, in any form or by any means, without permission in writing from the publisher. Problem 2.56 The structure shown forms part of a truss designed by an architectural engineer to support the roof of an orchestra shell. The members AB, AC, and AD exert forces FAB , FAC , and FAD on the joint A. The magnitude jFAB j D 4 kN. If the vector sum of the three forces equals zero, what are the magnitudes of FAC and FAD ? y B (– 4, 1) m FAB FAC (4, 2) m C x A FAD D (–2, – 3) m Solution: Determine the unit vectors parallel to each force: B 2 3 eAD D p iC p j D 0.5547i  0.8320j 22 C 32 22 C 32 C A 4 1 iC p j D 0.9701i C 0.2425j eAC D p 42 C 12 42 C 12 D 4 2 iC p j D 0.89443i C 0.4472j eAB D p 42 C 22 42 C 22 The forces are FAD D jFAD jeAD , FAC D jFAC jeAC , FAB D jFAB jeAB D 3.578i C 1.789j. Since the vector sum of the forces vanishes, the x- and y-components vanish separately: Fx D 0.5547jFAD j  0.9701jFAC j C 3.578i D 0, and Fy D 0.8320jFAD j C 0.2425jFAC j C 1.789j D 0 These simultaneous equations in two unknowns can be solved by any standard procedure. An HP-28S hand held calculator was used here: The results: jFAC j D 2.108 kN , jFAD j D 2.764 kN Problem 2.57 The distance s D 45 in. Solution: (a) (a) (b) Determine the unit vector eBA that points from B toward A. Use the unit vector you obtained in (a) to determine the coordinates of the collar C. The unit vector is the position vector from B to A divided by its magnitude rBA D [14  75]i C [45  12]jin D 61i C 33j in jrBA j D y eBA D A (14, 45) in  61 in2 C 33 in2 D 69.35 in 1 61i C 33j in D 0.880i C 0.476j 69.35 in eBA D 0.880i C 0.476j C s (b) B (75, 12) in x To find the coordinates of point C we will write a vector from the origin to point C. rC D rA C rAC D rA C seBA D 75i C 12j in C 45 in0.880i C 0.476j rC D 35.4i C 33.4j in Thus the coordinates of C are C 35.4, 33.4 in c 2008 Pearson Education, Inc., Upper Saddle River, NJ. All rights reserved. This material is protected under all copyright laws as they  currently exist. No portion of this material may be reproduced, in any form or by any means, without permission in writing from the publisher. 35 Problem 2.58 In Problem 2.57, determine the x and y coordinates of the collar C as functions of the distance s. Solution: The coordinates of the point C are given by xC D xB C s0.880 and yC D yB C s0.476. Thus, the coordinates of point C are xC D 75  0.880s in and yC D 12 C 0.476s in. Note from the solution of Problem 2.57 above, 0  s  69.4 in. Problem 2.59 The position vector r goes from point A to a point on the straight line between B and C. Its magnitude is jrj D 6 ft. Express r in terms of scalar components. y B (7, 9) ft r A (3, 5) ft C (12, 3) ft x Solution: Determine the perpendicular vector to the line BC from y point A, and then use this perpendicular to determine the angular orientation of the vector r. The vectors are B[7,9] P rAB D 7  3i C 9  5j D 4i C 4j, jrAB j D 5.6568 r rAC D 12  3i C 3  5j D 9i  2j, jrAC j D 9.2195 rBC D 12  7i C 3  9j D 5i  6j, jrBC j D 7.8102 A[3,5] C[12,3] x The unit vector parallel to BC is eBC D rBC D 0.6402i  0.7682j D i cos 50.19°  j sin 50.19° . jrBC j Add š90° to the angle to find the two possible perpendicular vectors: eAP1 D i cos 140.19°  j sin 140.19° , or eAP2 D i cos 39.8° C j sin 39.8° . Choose the latter, since it points from A to the line. Given the triangle defined by vertices A, B, C, then the magnitude of the perpendicular corresponds to the altitude when the base is the line 2area . From geometry, the area of BC. The altitude is given by h D base a triangle with known sides is given by area D p ss  jrBC js  jrAC js  jrAB j, where s is the semiperimeter, s D 12 jrAC j C jrAB j C jrBC j. Substituting values, s D 11.343, and area D 22.0 and the magnitude of the 222 D 5.6333. The angle between the perpendicular is jrAP j D 7.8102 5.6333 vector r and the perpendicular rAP is ˇ D cos1 D 20.1° . Thus 6 the angle between the vector r and the x-axis is ˛ D 39.8 š 20.1 D 59.1° or 19.7° . The first angle is ruled out because it causes the vector r to lie above the vector rAB , which is at a 45° angle relative to the x-axis. Thus: r D 6i cos 19.7° C j sin 19.7°  D 5.65i C 2.02j 36 c 2008 Pearson Education, Inc., Upper Saddle River, NJ. All rights reserved. This material is protected under all copyright laws as they  currently exist. No portion of this material may be reproduced, in any form or by any means, without permission in writing from the publisher. Problem 2.60 Let r be the position vector from point C to the point that is a distance s meters along the straight line between A and B. Express r in terms of components. (Your answer will be in terms of s). y B (10, 9) m s r Solution: First define the unit vector that points from A to B. A (3, 4) m rB/A D [10  3]i C [9  4]j m D 7i C 5j m jrB/A j D  C (9, 3) m p 7 m2 C 5 m2 D 74 m x 1 eB/A D p 7i C 5j 74 Let P be the point that is a distance s along the line from A to B. The coordinates of point P are  xp D 3 m C s  yp D 4 m C s 7 p 74 5 p 74  D 3 C 0.814s m  D 4 C 0.581s m. The vector r that points from C to P is then r D [3 C 0.814s  9]i C [4 C 0.581s  3]j m r D [0.814s  6]i C [0.581s C 1]j m Problem 2.61 A vector U D 3i  4j  12k. What is its magnitude? Strategy: The magnitude of a vector is given in terms of its components by Eq. (2.14). Problem 2.62 The vector e D 13 i C 23 j C ez k is a unit vector. Determine the component ez . (Notice that there are two answers.) Solution: Use definition given in Eq. (14). The vector magnitude is jUj D  32 C 42 C 122 D 13 Solution: eD 2 1 i C j C ez k ) 3 3  2  2 1 2 4 C C ez 2 D 1 ) e2 D 3 3 9 Thus ez D Problem 2.63 An engineer determines that an attachment point will be subjected to a force F D 20i C Fy j  45k kN. If the attachment point will safely support a force of 80-kN magnitude in any direction, what is the acceptable range of values for Fy ? y 2 3 or ez D  2 3 Solution: 802 ½ Fx2 C Fy2 C F2z 802 ½ 202 C Fy2 C 452 To find limits, use equality. Fy2LIMIT D 802  202  452 F Fy2LIMIT D 3975 Fy LIMIT D C63.0, 63.0 kN jFy LIMIT j  63.0 kN  63.0 kN  Fy  63.0 kN z x c 2008 Pearson Education, Inc., Upper Saddle River, NJ. All rights reserved. This material is protected under all copyright laws as they  currently exist. No portion of this material may be reproduced, in any form or by any means, without permission in writing from the publisher. 37 Problem 2.64 A vector U D Ux i C Uy j C Uz k. Its magnitude is jUj D 30. Its components are related by the equations Uy D 2Ux and Uz D 4Uy . Determine the components. (Notice that there are two answers.) Solution: Substitute the relations between the components, deter- U D C3.61i C 23.61j C 423.61k mine the magnitude, and solve for the unknowns. Thus D 3.61i  7.22j  28.9k U D Ux i C 2Ux j C 42Ux k D Ux 1i  2j  8k where Ux can be factored out since it is a scalar. Take the magnitude, noting that the absolute value of jUx j must be taken: U D 3.61i C 23.61j C 423.61k D 3.61i C 7.22j C 28.9k p 30 D jUx j 12 C 22 C 82 D jUx j8.31. Solving, we get jUx j D 3.612, or Ux D š3.61. The two possible vectors are Problem 2.65 An object is acted upon by two forces F1 D 20i C 30j  24k (kN) and F2 D 60i C 20j C 40k (kN). What is the magnitude of the total force acting on the object? Solution: F1 D 20i C 30j  24k kN F2 D 60i C 20j C 40k kN F D F1 C F2 D 40i C 50j C 16k kN Thus FD Problem 2.66 Two vectors U D 3i  2j C 6k and V D 4i C 12j  3k. (a) Determine the magnitudes of U and V. (b) Determine the magnitude of the vector 3U C 2V.  40 kN2 C 50 kN2 C 16 kN2 D 66 kN Solution: The magnitudes: (a) jUj D p p 32 C 22 C 62 D 7 and jVj D 42 C 122 C 32 D 13 The resultant vector 3U C 2V D 9 C 8i C 6 C 24j C 18  6k D 17i C 18j C 12k (b) 38 The magnitude j3U C 2Vj D p 172 C 182 C 122 D 27.51 c 2008 Pearson Education, Inc., Upper Saddle River, NJ. All rights reserved. This material is protected under all copyright laws as they  currently exist. No portion of this material may be reproduced, in any form or by any means, without permission in writing from the publisher. y Problem 2.67 In Active Example 2.6, suppose that you want to redesign the truss, changing the position of point D so that the magnitude of the vector rCD from point C to point D is 3 m. To accomplish this, let the coordinates of point D be 2, yD , 1 m, and determine the value of yD so that jrCD j D 3 m. Draw a sketch of the truss with point D in its new position. What are the new directions cosines of rCD ? D (2, 3, 1) m rCD (4, 0, 0) m Solution: The vector rCD and the magnitude jrCD j are C rCD D [2 m  4 m]i C [yD  0]j C [1 m  0]k D 2 mi C yD j z C 1 mk (a)  jrCD j D x 2 m2 C yCD 2 C 1 m2 D 3 m Solving we yCD D 2 m find yCD D  3 m2  2 m2  1 m2 D 2 m The new direction cosines of rCD . cos x D 2/3 D 0.667 cos y D 2/3 D 0.667 cos z D 1/3 D 0.333 Problem 2.68 A force vector is given in terms of its components by F D 10i  20j  20k (N). Solution: F D 10i  20j  20k N (a) (b) What are the direction cosines of F? Determine the components of a unit vector e that has the same direction as F. FD  10 N2 C 20 N2 C 20 N2 D 30 N cos x D (a) 10 N D 0.333, 30 N cos z D (b) cos y D 20 N D 0.667, 30 N 20 N D 0.667 30 N e D 0.333i  0.667j  0.667k c 2008 Pearson Education, Inc., Upper Saddle River, NJ. All rights reserved. This material is protected under all copyright laws as they  currently exist. No portion of this material may be reproduced, in any form or by any means, without permission in writing from the publisher. 39 Problem 2.69 The cable exerts a force F on the hook at O whose magnitude is 200 N. The angle between the vector F and the x axis is 40° , and the angle between the vector F and the y axis is 70° . y 70° (a) What is the angle between the vector F and the z axis? (b) Express F in terms of components. Strategy: (a) Because you know the angles between the vector F and the x and y axes, you can use Eq. (2.16) to determine the angle between F and the z axis. (Observe from the figure that the angle between F and the z axis is clearly within the range 0 < z < 180° .) (b) The components of F can be obtained with Eqs. (2.15). F 40° x O z Solution: (a) cos 40° 2 C cos 70° 2 C cos z 2 D 1 ) z D 57.0° F D 200 Ncos 40° i C cos 70° j C cos 57.0° k (b) F D 153.2i C 68.4j C 108.8k N Problem 2.70 A unit vector has direction cosines cos x D 0.5 and cos y D 0.2. Its z component is positive. Express it in terms of components. Solution: Use Eq. (2.15) and (2.16). The third direction cosine is  cos z D š 1  0.52  0.22 D C0.8426. The unit vector is u D 0.5i C 0.2j C 0.8426k Problem 2.71 The airplane’s engines exert a total thrust force T of 200-kN magnitude. The angle between T and the x axis is 120° , and the angle between T and the y axis is 130° . The z component of T is positive. (a) What is the angle between T and the z axis? (b) Express T in terms of components. l D cos 120° D 0.5, m D cos 130° D 0.6428 from which the z-direction cosine is n D cosz D š  1  0.52  0.64282 D C0.5804. Thus the angle between T and the z-axis is y y Solution: The x- and y-direction cosines are (a) z D cos1 0.5804 D 54.5° , and the thrust is T D 2000.5i  0.6428j C 0.5804k, or: 130⬚ x x (b) T D 100i  128.6j C 116.1k (kN) 120⬚ T z 40 z c 2008 Pearson Education, Inc., Upper Saddle River, NJ. All rights reserved. This material is protected under all copyright laws as they  currently exist. No portion of this material may be reproduced, in any form or by any means, without permission in writing from the publisher. Problem 2.72 Determine the components of the position vector rBD from point B to point D. Use your result to determine the distance from B to D. Solution: We have the following B5, 0, 3 m, C6, 0, 0 m, D4, 3, 1 m coordinates: A0, 0, 0, rBD D 4 m  5 mi C 3 m  0j C 1 m  3 mk y D (4, 3, 1) m rBD D i C 3j  2k m  D 1 m2 C 3 m2 C 2 m2 D 3.74 m A C (6, 0, 0) m x z B (5, 0, 3) m Problem 2.73 What are the direction cosines of the position vector rBD from point B to point D? Solution: cos x D 1 m D 0.267, 3.74 m cos z D Problem 2.74 Determine the components of the unit vector eCD that points from point C toward point D. cos y D 3m D 0.802, 3.74 m 2 m D 0.535 3.74 m Solution: We have the following B5, 0, 3 m, C6, 0, 0 m, D4, 3, 1 m coordinates: A0, 0, 0, rCD D 4 m  6 mi C 3 m  0j C 1 m  0k D 2i C 3j C 1k rCD D  2 m2 C 3 m2 C 1 m2 D 3.74 m Thus eCD D 1 2i C 3j C k m D 0.535i C 0.802j C 0.267k 3.74 m Problem 2.75 What are the direction cosines of the unit vector eCD that points from point C toward point D? Solution: Using Problem 2.74 cos x D 0.535, cos y D 0.802, cos z D 0.267 c 2008 Pearson Education, Inc., Upper Saddle River, NJ. All rights reserved. This material is protected under all copyright laws as they  currently exist. No portion of this material may be reproduced, in any form or by any means, without permission in writing from the publisher. 41 Problem 2.76 In Example 2.7, suppose that caisson shifts on the ground to a new position. magnitude of the force F remains 600 lb. In the position, the angle between the force F and the x is 60° and the angle between F and the z axis is Express F in terms of components. the The new axis 70° . y 40⬚ F x 54⬚ Solution: We need to find the angle y between the force F and the y axis. We know that z cos2 x C cos2 y C cos2 z D 1  cos y D š 1  cos2 x  cos2 z D š  1  cos2 60°  cos2 70° D š0.7956 y D š cos1 0.7956 D 37.3° or 142.7° We will choose y D 37.3° because the picture shows the force pointing up. Now Fx D 600 lb cos 60° D 300 lb Fy D 600 lb cos 37.3° D 477 lb Fz D 600 lb cos 70° D 205 lb Thus F D 300i C 477j C 205k lb Problem 2.77 Astronauts on the space shuttle use radar to determine the magnitudes and direction cosines of the position vectors of two satellites A and B. The vector rA from the shuttle to satellite A has magnitude 2 km, and direction cosines cos x D 0.768, cos y D 0.384, cos z D 0.512. The vector rB from the shuttle to satellite B has magnitude 4 km and direction cosines cos x D 0.743, cos y D 0.557, cos z D 0.371. What is the distance between the satellites? B rB x Solution: The two position vectors are: A y rA rA D 20.768iC 0.384jC 0.512k D 1.536i C 0.768j C 1.024k (km) rB D 40.743iC 0.557j 0.371k D 2.972i C 2.228j  1.484k (km) z The distance is the magnitude of the difference: jrA  rB j D  1.5362.9272 C 0.7682.2282 C 1.0241.4842 D 3.24 (km) 42 c 2008 Pearson Education, Inc., Upper Saddle River, NJ. All rights reserved. This material is protected under all copyright laws as they  currently exist. No portion of this material may be reproduced, in any form or by any means, without permission in writing from the publisher. Problem 2.78 Archaeologists measure a pre-Columbian ceremonial structure and obtain the dimensions shown. Determine (a) the magnitude and (b) the direction cosines of the position vector from point A to point B. y 4m 10 m 4m 10 m A 8m Solution: (a) B The coordinates are A (0, 16, 14) m and B (10, 8, 4) m. b rAB D [10  0]i C [8  16]j C [4  14]k m D 10i  8j  10k m jrAB j D  p 102 C 82 C 102 m D 264 m D 16.2 m 8m z C x jrAB j D 16.2 m (b) 10 D 0.615 cos x D p 264 8 D 0.492 cos y D p 264 10 cos z D p D 0.615 264 Problem 2.79 Consider the structure described in Problem 2.78. After returning to the United States, an archaeologist discovers that a graduate student has erased the only data file containing the dimension b. But from recorded GPS data he is able to calculate that the distance from point B to point C is 16.61 m. y 10 m 4m 4m 10 m A 8m B (a) (b) What is the distance b? Determine the direction cosines of the position vector from B to C. b 8m z C x Solution: We have the coordinates B (10 m, 8 m, 4 m), C (10 m C b, 0 18 m). rBC D 10 m C b  10 mi C 0  8 mj C 18 m  4 mk rBC D bi C 8 mj C 14 mk (a) (b) We have 16.61 m2 D b2 C 8 m2 C 14 m2 ) b D 3.99 m The direction cosines of rBC are 3.99 m D 0.240 16.61 m 8 m cos y D D 0.482 16.61 m 14 m cos z D D 0.843 16.61 m cos x D c 2008 Pearson Education, Inc., Upper Saddle River, NJ. All rights reserved. This material is protected under all copyright laws as they  currently exist. No portion of this material may be reproduced, in any form or by any means, without permission in writing from the publisher. 43 Problem 2.80 Observers at A and B use theodolites to measure the direction from their positions to a rocket in flight. If the coordinates of the rocket’s position at a given instant are (4, 4, 2) km, determine the direction cosines of the vectors rAR and rBR that the observers would measure at that instant. y rAR Solution: The vector rAR is given by rBR A rAR D 4i C 4j C 2k km x and the magnitude of rAR is given by jrAR j D  z 42 C 42 C 22 B (5, 0, 2) km km D 6 km. The unit vector along AR is given by uAR D rAR /jrAR j. Thus, uAR D 0.667i C 0.667j C 0.333k and the direction cosines are cos x D 0.667, cos y D 0.667, and cos z D 0.333. The vector rBR is given by rBR D xR  xB i C yR  yB j C zR  zB k km D 4  5i C 4  0j C 2  2k km and the magnitude of rBR is given by jrBR j D  12 C 42 C 02 km D 4.12 km. The unit vector along BR is given by eBR D rBR /jrBR j. Thus, uBR D 0.242i C 0.970j C 0k and the direction cosines are cos x D 0.242, cos y D 0.970, and cos z D 0.0. 44 c 2008 Pearson Education, Inc., Upper Saddle River, NJ. All rights reserved. This material is protected under all copyright laws as they  currently exist. No portion of this material may be reproduced, in any form or by any means, without permission in writing from the publisher. Problem 2.81 In Problem 2.80, suppose that the coordinates of the rocket’s position are unknown. At a given instant, the person at A determines that the direction cosines of rAR are cos x D 0.535, cos y D 0.802, and cos z D 0.267, and the person at B determines that the direction cosines of rBR are cos x D 0.576, cos y D 0.798, and cos z D 0.177. What are the coordinates of the rocket’s position at that instant. Solution: The vector from A to B is given by rAB D xB  xA i C yB  yA j C zB  zA k or Similarly, the vector along BR, uBR D 0.576i C 0.798  0.177k. From the diagram in the problem statement, we see that rAR D rAB C rBR . Using the unit vectors, the vectors rAR and rBR can be written as rAB D 5  0i C 0  0j C 2  0k D 5i C 2k km. The magnitude of rAB is given by jrAB j D The unit vector along AB, uAB , is given by  rAR D 0.535rAR i C 0.802rAR j C 0.267rAR k, and 52 C 22 D 5.39 km. uAB D rAB /jrAB j D 0.928i C 0j C 0.371k km. The unit vector along the line AR, uAR D cos x i C cos y j C cos z k D 0.535i C 0.802j C 0.267k. rBR D 0.576rBR i C 0.798rBR j  0.177rBR k. Substituting into the vector addition rAR D rAB C rBR and equating components, we get, in the x direction, 0.535rAR D 0.576rBR , and in the y direction, 0.802rAR D 0.798rBR . Solving, we get that rAR D 4.489 km. Calculating the components, we get rAR D rAR eAR D 0.5354.489i C 0.8024.489j C 0.2674.489k. Hence, the coordinates of the rocket, R, are (2.40, 3.60, 1.20) km. Problem 2.82* The height of Mount Everest was originally measured by a surveyor in the following way. He first measured the altitudes of two points and the horizontal distance between them. For example, suppose that the points A and B are 3000 m above sea level and are 10,000 m apart. He then used a theodolite to measure the direction cosines of the vector rAP from point A to the top of the mountain P and the vector rBP from point B to P. Suppose that the direction cosines of rAP are cos x D 0.5179, cos y D 0.6906, and cos z D 0.5048, and the direction cosines of rBP are cos x D 0.3743, cos y D 0.7486, and cos z D 0.5472. Using this data, determine the height of Mount Everest above sea level. z P y B x A Solution: We have the following coordinates A0, 0, 3000 m, B10, 000, 0, 3000 m, Px, y, z Then rAP D xi C yj C z  3000 mk D rAP 0.5179i C 0.6906j C 0.5048k rBP D x  10,000 mi C yj C z  3000 mk D rBP 0.3743i C 0.7486j C 0.5472k Equating components gives us five equations (one redundant) which we can solve for the five unknowns. x D rAP 0.5179 y D rAP 0.6906 z  3000 m D rAP 0.5048 ) z D 8848 m x  10000 m D rBP  0.7486 y D rBP 0.5472 c 2008 Pearson Education, Inc., Upper Saddle River, NJ. All rights reserved. This material is protected under all copyright laws as they  currently exist. No portion of this material may be reproduced, in any form or by any means, without permission in writing from the publisher. 45 y Problem 2.83 The distance from point O to point A is 20 ft. The straight line AB is parallel to the y axis, and point B is in the x-z plane. Express the vector rOA in terms of scalar components. A rOA Strategy: You can resolve rOA into a vector from O to B and a vector from B to A. You can then resolve the vector form O to B into vector components parallel to the x and z axes. See Example 2.8. O x 30° 60° B z Solution: See Example 2.8. The length BA is, from the right triangle The vector rOA is given by rOA D rOB C rBA , from which OAB, rOA D 15i C 10j C 8.66k (ft) jrAB j D jrOA j sin 30° D 200.5 D 10 ft. Similarly, the length OB is A jrOB j D jrOA j cos 30° D 200.866 D 17.32 ft The vector rOB can be resolved into components along the axes by the right triangles OBP and OBQ and the condition that it lies in the x-z plane. Hence, rOB D jrOB ji cos 30° C j cos 90° C k cos 60°  rOA y 30° O x Q z P 60° B or rOB D 15i C 0j C 8.66k. The vector rBA can be resolved into components from the condition that it is parallel to the y-axis. This vector is rBA D jrBA ji cos 90° C j cos 0° C k cos 90°  D 0i C 10j C 0k. y Problem 2.84 The magnitudes of the two force vectors are jFA j D 140 lb and jFB j D 100 lb. Determine the magFB nitude of the sum of the forces FA C FB . FA Solution: We have the vectors 60⬚ FA D 140 lb[cos 40° sin 50° ]i C [sin 40° ]j C [cos 40° cos 50° ]k 30⬚ 40⬚ x 50⬚ FA D 82.2i C 90.0j C 68.9k lb z FB D 100 lb[ cos 60° sin 30° ]i C [sin 60° ]j C [cos 60° cos 30° ]k FB D 25.0i C 86.6j C 43.3k lb Adding and taking the magnitude we have FA C FB D 57.2i C 176.6j C 112.2k lb jFA C FB j D  57.2 lb2 C 176.6 lb2 C 112.2 lb2 D 217 lb jFA C FB j D 217 lb 46 c 2008 Pearson Education, Inc., Upper Saddle River, NJ. All rights reserved. This material is protected under all copyright laws as they  currently exist. No portion of this material may be reproduced, in any form or by any means, without permission in writing from the publisher. y Problem 2.85 Determine the direction cosines of the vectors FA and FB . FB Solution: We have the vectors FA FA D 140 lb[cos 40° sin 50° ]i C [sin 40° ]j C [cos 40° cos 50° ]k FA D 82.2i C 90.0j C 68.9k lb 60⬚ FB D 100 lb[ cos 60° sin 30° ]i C [sin 60° ]j C [cos 60° cos 30° ]k 30⬚ FB D 25.0i C 86.6j C 43.3k lb 40⬚ x 50⬚ z The direction cosines for FA are cos x D 82.2 lb 90.0 lb D 0.587, cos y D D 0.643, 140 lb 140 lb cos z D 68.9 lb D 0.492 140 lb The direction cosines for FB are cos x D 25.0 lb 86.6 lb D 0.250, cos y D D 0.866, 100 lb 100 lb cos z D 43.3 lb D 0.433 100 lb FA : cos x D 0.587, cos y D 0.643, cos z D 0.492 FB : cos x D 0.250, cos y D 0.866, cos z D 0.433 Problem 2.86 In Example 2.8, suppose that a change in the wind causes a change in the position of the balloon and increases the magnitude of the force F exerted on the hook at O to 900 N. In the new position, the angle between the vector component Fh and F is 35° , and the angle between the vector components Fh and Fz is 40° . Draw a sketch showing the relationship of these angles to the components of F. Express F in terms of its components. y B F O O z x A Solution: We have jFy j D 900 N sin 35° D 516 N jFh j D 900 N cos 35° D 737 N jFx j D jFh j sin 40° D 474 N jFz j D jFh j cos 40° D 565 N Thus F D 474i C 516j C 565k N c 2008 Pearson Education, Inc., Upper Saddle River, NJ. All rights reserved. This material is protected under all copyright laws as they  currently exist. No portion of this material may be reproduced, in any form or by any means, without permission in writing from the publisher. 47 Problem 2.87 An engineer calculates that the magnitude of the axial force in one of the beams of a geodesic dome is jPj D 7.65 kN. The cartesian coordinates of the endpoints A and B of the straight beam are (12.4, 22.0, 18.4) m and (9.2, 24.4, 15.6) m, respectively. Express the force P in terms of scalar components. Solution: The components of the position vector from B to A are rBA D xA  xB i C yA  yB j C zA  zB k D 12.4 C 9.2i C 22.0  24.4j C 18.4 C 15.6k D 3.2i  2.4j  2.8k m. B P A Dividing this vector by its magnitude, we obtain a unit vector that points from B toward A: eBA D 0.655i  0.492j  0.573k. Therefore P D jPjeBA D 7.65 eBA D 5.01i  3.76j  4.39k kN. y Problem 2.88 The cable BC exerts an 8-kN force F on the bar AB at B. B (5, 6, 1) m (a) Determine the components of a unit vector that points from B toward point C. (b) Express F in terms of components. F A Solution: (a) eBC D x xC  xB i C yC  yB j C zC  zB k rBC D  jrBC j xC  xB 2 C yC  yB 2 C zC  zB 2 C (3, 0, 4) m z 2i  6j C 3k 2 6 3 eBC D p D i jC k 7 7 7 22 C 62 C 32 eBC D 0.286i  0.857j C 0.429k (b) 48 F D jFjeBC D 8eBC D 2.29i  6.86j C 3.43k kN c 2008 Pearson Education, Inc., Upper Saddle River, NJ. All rights reserved. This material is protected under all copyright laws as they  currently exist. No portion of this material may be reproduced, in any form or by any means, without permission in writing from the publisher. y Problem 2.89 A cable extends from point C to point E. It exerts a 50-lb force T on plate C that is directed along the line from C to E. Express T in terms of components. 6 ft E A D z 4 ft T 2 ft 20⬚ B x C 4 ft y Solution: Find the unit vector eCE and multiply it times the magnitude of the force to get the vector in component form, eCE D 6 ft xE  xC i C yE  yC j C zE  zC k rCE D  jrCE j xE  xC 2 C yE  yC 2 C zE  zC 2 A E The coordinates of point C are 4, 4 sin 20° , 4 cos 20°  or 4, 1.37, 3.76 (ft) The coordinates of point E are (0, 2, 6) (ft) D T 2 ft eCE 0  4i C 2  1.37j C 6  3.76k p D 42 C 3.372 C 2.242 x 4 ft T z B 20° C 4 ft eCE D 0.703i C 0.592j C 0.394k T D 50eCE lb T D 35.2i C 29.6j C 19.7k lb Problem 2.90 In Example 2.9, suppose that the metal loop at A is moved upward so that the vertical distance to A increases from 7 ft to 8 ft. As a result, the magnitudes of the forces FAB and FAC increase to jFAB j D jFAC j D 240 lb. What is the magnitude of the total force F D FAB C FAC exerted on the loop by the rope? Solution: The new coordinates of point A are (6, 8, 0) ft. The position vectors are rAB D 4i  8j C 4k ft rAC D 4i  8j C 6k ft The forces are 6 ft A FAB D 240 lb rAB D 98.0i  196j C 98.0k lb jrAB j FAC D 240 lb rAC D 89.1i  178j C 134.0k lb jrAC j 7 ft 4 ft 6 ft B The sum of the forces is C 2 ft F D FAB C FAC D 8.85i  374j C 232k lb 10 ft The magnitude is jFj D 440 lb y 6 ft A FAB FAC 7 ft 4 ft 2 ft x B 6 ft C 10 ft z c 2008 Pearson Education, Inc., Upper Saddle River, NJ. All rights reserved. This material is protected under all copyright laws as they  currently exist. No portion of this material may be reproduced, in any form or by any means, without permission in writing from the publisher. 49 y Problem 2.91 The cable AB exerts a 200-lb force FAB at point A that is directed along the line from A to B. Express FAB in terms of components. 8 ft C 8 ft Solution: The coordinates of B are B(0,6,8). The position vector from A to B is 6 ft B x rAB D 0  6i C 6  0j C 8  10k D 6i C 6j  2k The magnitude is jrAB j D p 62 C 62 C 22 D 8.718 ft. FAB The unit vector is uAB D 6 2 6 iC j k 8.718 8.718 8.718 z FAC A (6, 0, 10) ft or uAB D 0.6882i C 0.6882j  0.2294k. FAB D jFAB juAB D 2000.6882i C 0.6882j  0.2294k The components of the force are FAB D jFAB juAB D 2000.6882i C 0.6882j  0.2294k or FAB D 137.6i C 137.6j  45.9k Problem 2.92 Consider the cables and wall described in Problem 2.91. Cable AB exerts a 200-lb force FAB at point A that is directed along the line from A to B. The cable AC exerts a 100-lb force FAC at point A that is directed along the line from A to C. Determine the magnitude of the total force exerted at point A by the two cables. Solution: Refer to the figure in Problem 2.91. From Problem 2.91 the force FAB is The force is FAC D jFAC juAC D 100uAC D 16.9i C 50.7j  84.5k. FAB D 137.6i C 137.6j  45.9k The resultant of the two forces is The coordinates of C are C(8,6,0). The position vector from A to C is FR D FAB C FAC D 137.6 C 16.9i C 137.6 C 50.7j rAC D 8  6i C 6  0j C 0  10k D 2i C 6j  10k. The magnitude is jrAC j D The unit vector is uAC D C 84.5  45.9k. p 22 C 62 C 102 D 11.83 ft. 6 10 2 iC j k D 0.1691i C 0.5072j  0.8453k. 11.83 11.83 11.83 FR D 120.7i C 188.3j  130.4k. The magnitude is jFR j D 50 p 120.72 C 188.32 C 130.42 D 259.0 lb c 2008 Pearson Education, Inc., Upper Saddle River, NJ. All rights reserved. This material is protected under all copyright laws as they  currently exist. No portion of this material may be reproduced, in any form or by any means, without permission in writing from the publisher. Problem 2.93 The 70-m-tall tower is supported by three cables that exert forces FAB , FAC , and FAD on it. The magnitude of each force is 2 kN. Express the total force exerted on the tower by the three cables in terms of components. A y FAD A FAB FAC D Solution: The coordinates of the points are A (0, 70, 0), B (40, 0, 0), C (40, 0, 40) D (60, 0, 60). 60 m 60 m B The position vectors corresponding to the cables are: x 40 m rAD D 60  0i C 0  70j C 60  0k C 40 m 40 m z rAD D 60i  70k  60k rAC D 40  0i C 0  70j C 40  0k rAC D 40i  70j C 40k rAB D 40  0i C 0  70j C 0  0k rAB D 40i  70j C 0k The unit vectors corresponding to these position vectors are: uAD D 60 70 60 rAD D i j k jrAD j 110 110 110 D 0.5455i  0.6364j  0.5455k uAC D 40 70 40 rAC D i jC k jrAC j 90 90 90 D 0.4444i  0.7778j C 0.4444k uAB D 40 70 rAB D i j C 0k D 0.4963i  0.8685j C 0k jrAB j 80.6 80.6 The forces are: FAB D jFAB juAB D 0.9926i  1.737j C 0k FAC D jFAC juAC D 0.8888i  1.5556j C 0.8888 FAD D jFAD juAD D 1.0910i  1.2728j  1.0910k The resultant force exerted on the tower by the cables is: FR D FAB C FAC C FAD D 0.9875i  4.5648j  0.2020k kN c 2008 Pearson Education, Inc., Upper Saddle River, NJ. All rights reserved. This material is protected under all copyright laws as they  currently exist. No portion of this material may be reproduced, in any form or by any means, without permission in writing from the publisher. 51 Problem 2.94 Consider the tower described in Problem 2.93. The magnitude of the force FAB is 2 kN. The x and z components of the vector sum of the forces exerted on the tower by the three cables are zero. What are the magnitudes of FAC and FAD ? Solution: From the solution of Problem 2.93, the unit vectors are: uAC D 40 70 40 rAC D i jC k jrAC j 90 90 90 Taking the sum of the forces: FR D FAB C FAC C FAD D 0.9926  0.4444jFAC j  0.5455jFAD ji C 1.737  0.7778jFAC j  0.6364jFAD jj D 0.4444i  0.7778j C 0.4444k uAD D 60 70 60 rAD D i j jrAD j 110 110 110 C 0.4444jFAC j  0.5455jFAD jk The sum of the x- and z-components vanishes, hence the set of simultaneous equations: D 0.5455i  0.6364j  0.5455k From the solution of Problem 2.93 the force FAB is FAB D jFAB juAB D 0.9926i  1.737j C 0k The forces FAC and FAD are: FAC D jFAC juAC D jFAC j0.4444i  0.7778j C 0.4444k FAD D jFAD juAD D jFAD j0.5455i  0.6364j  0.5455k 0.4444jFAC j C 0.5455jFAD j D 0.9926 and 0.4444jFAC j  0.5455jFAD j D 0 These can be solved by means of standard algorithms, or by the use of commercial packages such as TK Solver Plus  or Mathcad. Here a hand held calculator was used to obtain the solution: jFAC j D 1.1163 kN Problem 2.95 In Example 2.10, suppose that the distance from point C to the collar A is increased from 0.2 m to 0.3 m, and the magnitude of the force T increases to 60 N. Express T in terms of its components. jFAD j D 0.9096 kN y 0.15 m 0.4 m B C Solution: The position vector from C to A is now rCA D 0.3 meCD D 0.137i  0.205j C 0.171km The position vector form the origin to A is T A 0.2 m 0.5 m rOA D rOC C rCA D 0.4i C 0.3j m C 0.137i  0.205j C 0.171k m O rOA D 0.263i C 0.0949j C 0.171k m The coordinates of A are (0.263, 0.0949, 0.171) m. The position vector from A to B is 0.3 m x D 0.25 m 0.2 m z rAB D [0  0.263]i C [0.5  0.0949]j C [0.15  0.171]k m rAB D 0.263i C 0.405j  0.209k m The force T is T D 60 N rAB D 32.7i C 50.3j  2.60k N jrAB j T D 32.7i C 50.3j  2.60k N 52 c 2008 Pearson Education, Inc., Upper Saddle River, NJ. All rights reserved. This material is protected under all copyright laws as they  currently exist. No portion of this material may be reproduced, in any form or by any means, without permission in writing from the publisher. y Problem 2.96 The cable AB exerts a 32-lb force T on the collar at A. Express T in terms of components. 4 ft B Solution: The coordinates of point B are B (0, 7, 4). The vector T 6 ft position of B is rOB D 0i C 7j C 4k. The vector from point A to point B is given by 7 ft rAB D rOB  rOA . A x From Problem 2.95, rOA D 2.67i C 2.33j C 2.67k. Thus 4 ft rAB D 0  2.67i C 7  2.33j C 4  2.67j 4 ft z rAB D 2.67i C 4.67j C 1.33k. The magnitude is jrAB j D p 2.672 C 4.672 C 1.332 D 5.54 ft. The unit vector pointing from A to B is uAB D rAB D 0.4819i C 0.8429j C 0.2401k jrAB j The force T is given by TAB D jTAB juAB D 32uAB D 15.4i C 27.0j C 7.7k (lb) Problem 2.97 The circular bar has a 4-m radius and lies in the x-y plane. Express the position vector from point B to the collar at A in terms of components. y Solution: From the figure, the point B is at (0, 4, 3) m. The coordinates of point A are determined by the radius of the circular bar and the angle shown in the figure. The vector from the origin to A is rOA D 4 cos20° i C 4 sin20° j m. Thus, the coordinates of point A are (3.76, 1.37, 0) m. The vector from B to A is given by rBA D xA  xB i C yA  yB j C zA  zB k D 3.76i  2.63j  3k m. Finally, the scalar components of the vector from B to A are (3.76, 2.63, 3) m. 3m B A 4m 20° 4m x z c 2008 Pearson Education, Inc., Upper Saddle River, NJ. All rights reserved. This material is protected under all copyright laws as they  currently exist. No portion of this material may be reproduced, in any form or by any means, without permission in writing from the publisher. 53 Problem 2.98 The cable AB in Problem 2.97 exerts a 60-N force T on the collar at A that is directed along the line from A toward B. Express T in terms of components. Solution: We know rBA D 3.76i  2.63j  3k m from Problem 2.97. The unit vector uAB D rBA /jrBA j. The unit vector is uAB D 0.686i C 0.480j C 0.547k. Hence, the force vector T is given by T D jTj0.686iC 0.480jC 0.547k N D 41.1i C 28.8j C 32.8k N Problem 2.99 In Active Example 2.11, suppose that the vector V is changed to V D 4i  6j  10k. (a) (b) What is the value of UžV? What is the angle between U and V when they are placed tail to tail? Solution: From Active Example 2.4 we have the expression for U. Thus U D 6i  5j  3k, V D 4i  6k  10k U Ð V D 64 C 56 C 310 D 84 cos  D 84 UÐV  D  D 0.814 jVjjVj 62 C 52 C 32 42 C 62 C 102  D cos1 0.814 D 35.5° a U Ð V D 84, b  D 35.5° Problem 2.100 In Example 2.12, suppose that the coordinates of point B are changed to (6, 4, 4) m. What is the angle  between the lines AB and AC? Solution: Using the new coordinates we have rAB D 2i C j C 2k m, jrAB j D 3 m rAC D 4i C 5j C 2k m, jrAC j D 6.71 m y C (8, 8, 4) m u A (4, 3, 2) m B (6, 1, ⫺2) m x cos  D 24 C 15 C 22 m2 rAB Ð rAC D D 0.845 jrAB jjrAC j 3 m6.71 m  D cos1 0.845 D 32.4°  D 32.4° z Problem 2.101 What is the dot product of the position vector r D 10i C 25j (m) and the force vector Solution: Use Eq. (2.23). F Ð r D 30010 C 25025 C 3000 D 3250 N-m F D 300i C 250j C 300k N? Problem 2.102 Suppose that the dot product of two vectors U and V is U Ð V D 0. If jUj 6D 0, what do you know about the vector V? 54 Solution: Either jVj D 0 or V ? U c 2008 Pearson Education, Inc., Upper Saddle River, NJ. All rights reserved. This material is protected under all copyright laws as they  currently exist. No portion of this material may be reproduced, in any form or by any means, without permission in writing from the publisher. Problem 2.103 Two perpendicular vectors are given in terms of their components by Solution: When the vectors are perpendicular, U Ð V  0. Thus U D Ux i  4j C 6k U Ð V D Ux Vx C Uy Vy C Uz Vz D 0 and V D 3i C 2j  3k. Use the dot product to determine the component Ux . D 3Ux C 42 C 63 D 0 3Ux D 26 Ux D 8.67 Problem 2.104 Three vectors Solution: For mutually perpendicular vectors, we have three equations, i.e., U D Ux i C 3j C 2k UÐVD0 V D 3i C Vy j C 3k W D 2i C 4j C Wz k are mutually perpendicular. Use the dot product to determine the components Ux , Vy , and Wz . UÐWD0 VÐWD0 Thus  3Ux C 3Vy C 6 D 0  3 Eqns 2Ux C 12 C 2Wz D 0   3 Unknowns C6 C 4Vy C 3Wz D 0 Solving, we get Ux Vy Wz Problem 2.105 The magnitudes jUj D 10 and jVj D 20. (a) (b) Use the definition of the dot product to determine U Ð V. Use Eq. (2.23) to obtain U Ð V. Solution: (a) D 2.857 D 0.857 D 3.143 y V U 45⬚ The definition of the dot product (Eq. (2.18)) is 30⬚ x U Ð V D jUjjVj cos . Thus U Ð V D 1020 cos45°  30°  D 193.2 (b) The components of U and V are U D 10i cos 45° C j sin 45°  D 7.07i C 7.07j V D 20i cos 30° C j sin 30°  D 17.32i C 10j From Eq. (2.23) U Ð V D 7.0717.32 C 7.0710 D 193.2 c 2008 Pearson Education, Inc., Upper Saddle River, NJ. All rights reserved. This material is protected under all copyright laws as they  currently exist. No portion of this material may be reproduced, in any form or by any means, without permission in writing from the publisher. 55 Problem 2.106 By evaluating the dot product U Ð V, prove the identity cos1  2  D cos 1 cos 2 C sin 1 sin 2 . y Strategy: Evaluate the dot product both by using Eq. (2.18) and by using Eq. (2.23). U V u1 Solution: The strategy is to use the definition Eq. (2.18) and the u2 x Eq. (2.23). From Eq. (2.18) and the figure, U Ð V D jUjjVj cos1  2 . From Eq. (2.23) and the figure, U D jUji cos 1 C j sin 2 , V D jVji cos 2 C j sin 2 , and the dot product is U Ð V D jUjjVjcos 1 cos 2 C sin 1 sin 2 . Equating the two results: U Ð V D jUjjVj cos1  2  D jUjjVjcos 1 cos 2 C sin 1 sin 2 , from which if jUj 6D 0 and jVj 6D 0, it follows that cos1  2  D cos 1 cos 2 C sin 1 sin 2 , Q.E.D. Problem 2.107 Use the dot product to determine the angle between the forestay (cable AB) and the backstay (cable BC). y B (4, 13) m Solution: The unit vector from B to A is eBA D rBA D 0.321i  0.947j jrBA j The unit vector from B to C is eBC D rBC D 0.385i  0.923j jrBC j From the definition of the dot product, eBA Ð eBC D 1 Ð 1 Ð cos , where  is the angle between BA and BC. Thus A (0, 1.2) m C (9, 1) m x cos  D 0.3210.385 C 0.9470.923 cos  D 0.750  D 41.3° 56 c 2008 Pearson Education, Inc., Upper Saddle River, NJ. All rights reserved. This material is protected under all copyright laws as they  currently exist. No portion of this material may be reproduced, in any form or by any means, without permission in writing from the publisher. y Problem 2.108 Determine the angle  between the lines AB and AC (a) by using the law of cosines (see Appendix A); (b) by using the dot product. B (4, 3, ⫺1) m Solution: (a) A We have the distances: AB D AC D BC D    42 C 32 C 12 m D x u p 26 m (5, ⫺1, 3) m C z p 52 C 12 C 32 m D 35 m 5  42 C 1  32 C 3 C 12 m D p 33 m The law of cosines gives BC2 D AB2 C AC2  2ABAC cos  cos  D (b) AB2 C AC2  BC2 D 0.464 2ABAC )  D 62.3° Using the dot product rAB D 4i C 3j  k m, rAC D 5i  j C 3k m rAB Ð rAC D 4 m5 m C 3 m1 m C 1 m3 m D 14 m2 rAB Ð rAC D ABAC cos  Therefore 14 m2 p D 0.464 )  D 62.3° cos  D p 26 m 35 m Problem 2.109 The ship O measures the positions of the ship A and the airplane B and obtains the coordinates shown. What is the angle  between the lines of sight OA and OB? y B (4, 4, ⫺4) km Solution: From the coordinates, the position vectors are: u x rOA D 6i C 0j C 3k and rOB D 4i C 4j  4k O The dot product: rOA Ð rOB D 64 C 04 C 34 D 12 The magnitudes: jrOA j D p 62 C 02 C 32 D 6.71 km and A z (6, 0, 3) km p jrOA j D 42 C 42 C 42 D 6.93 km. rOA Ð rOB D 0.2581, from which  D š75° . jrOA jjrOB j From the problem and the construction, only the positive angle makes sense, hence  D 75° From Eq. (2.24) cos  D c 2008 Pearson Education, Inc., Upper Saddle River, NJ. All rights reserved. This material is protected under all copyright laws as they  currently exist. No portion of this material may be reproduced, in any form or by any means, without permission in writing from the publisher. 57 Problem 2.110 Astronauts on the space shuttle use radar to determine the magnitudes and direction cosines of the position vectors of two satellites A and B. The vector rA from the shuttle to satellite A has magnitude 2 km and direction cosines cos x D 0.768, cos y D 0.384, cos z D 0.512. The vector rB from the shuttle to satellite B has magnitude 4 km and direction cosines cos x D 0.743, cos y D 0.557, cos z D 0.371. What is the angle  between the vectors rA and rB ? B rB x θ Solution: The direction cosines of the vectors along rA and rB are the components of the unit vectors in these directions (i.e., uA D cos x i C cos y j C cos z k, where the direction cosines are those for rA ). Thus, through the definition of the dot product, we can find an expression for the cosine of the angle between rA and rB . y rA A z cos  D cos xA cos xB C cos yA cos yB C cos zA cos zB . Evaluation of the relation yields cos  D 0.594 )  D 53.5° . Problem 2.111 In Example 2.13, if you shift your position and the coordinates of point A where you apply the 50-N force become (8, 3, 3) m, what is the vector component of F parallel to the cable OB? y A (6, 6, –3) m F Solution: We use the following vectors to define the force F. O rOA D 8i C 3j  3k m eOA z x (10, ⫺2, 3) m B rOA D 0.833i C 0.331j  0.331k D jrOA j F D 50 NeOA D 44.2i C 16.6j  16.6k N Now we need the unit vector eOB . rOB D 10i  2j C 3k m eOB D rOB D 0.941i  0.188j C 0.282k jrOB j To find the vector component parallel to OB we use the dot product in the following manner F Ð eOB D 44.2 N0.941 C 16.6 N0.188 C 16.6 N0.282 D 33.8 N Fp D F Ð eOB eOB D 33.8 N0.941i  0.188j C 0.282k Fp D 31.8i  6.35j C 9.53k N 58 c 2008 Pearson Education, Inc., Upper Saddle River, NJ. All rights reserved. This material is protected under all copyright laws as they  currently exist. No portion of this material may be reproduced, in any form or by any means, without permission in writing from the publisher. Problem 2.112 The person exerts a force F D 60i  40j (N) on the handle of the exercise machine. Use Eq. (2.26) to determine the vector component of F that is parallel to the line from the origin O to where the person grips the handle. r D 250i C 200j  150k mm, jrj D 354 mm To produce the unit vector that is parallel to this line we divide by the magnitude 150 mm y Solution: The vector r from the O to where the person grips the handle is eD r 250i C 200j  150k mm D D 0.707i C 0.566j  0.424k jrj 354 mm Using Eq. (2.26), we find that the vector component parallel to the line is F O Fp D e Ð Fe D [0.70760 N C 0.56640 N]0.707i 200 mm z C 0.566j  0.424k 250 mm Fp D 14.0i C 11.2j C 8.4k N x Problem 2.113 At the instant shown, the Harrier’s thrust vector is T D 17,000i C 68,000j  8,000k (N) and its velocity vector is v D 7.3i C 1.8j  0.6k (m/s). The quantity P D jTp jjvj, where Tp is the vector component of T parallel to v, is the power currently being transferred to the airplane by its engine. Determine the value of P. Solution: y v T T D 17,000i C 68,000j  8,000k N v D 7.3i C 1.8j  0.6k m/s x Power D T Ð v D 17,000 N7.3 m/s C 68,000 N1.8 m/s C 8,000 N0.6 m/s Power D 251,000 Nm/s D 251 kW c 2008 Pearson Education, Inc., Upper Saddle River, NJ. All rights reserved. This material is protected under all copyright laws as they  currently exist. No portion of this material may be reproduced, in any form or by any means, without permission in writing from the publisher. 59 Problem 2.114 Cables extend from A to B and from A to C. The cable AC exerts a 1000-lb force F at A. y A (a) What is the angle between the cables AB and AC? (b) Determine the vector component of F parallel to the cable AB. (0, 7, 0) ft F x Solution: Use Eq. (2.24) to solve. (a) From the coordinates of the points, the position vectors are: rAB D 0  0i C 0  7j C 10  0k B (0, 0, 10) ft z C (14, 0, 14) ft rAB D 0i  7j C 10k rAC D 14  0i C 0  7j C 14  0k rAC D 14i  7j C 14k The magnitudes are: jrAB j D p 72 C 102 D 12.2 (ft) and jrAB j D p 142 C 72 C 142 D 21. The dot product is given by rAB Ð rAC D 140 C 77 C 1014 D 189. The angle is given by cos  D 189 D 0.7377, 12.221 from which  D š42.5° . From the construction:  D C42.5° (b) The unit vector associated with AB is eAB D rAB D 0i  0.5738j C 0.8197k. jrAB j The unit vector associated with AC is eAC D rAC jrAC j D 0.6667i  0.3333j C 0.6667k. Thus the force vector along AC is FAC D jFjeAC D 666.7i  333.3j C 666.7k. The component of this force parallel to AB is FAC Ð eAB eAB D 737.5eAB D 0i  422.8j C 604.5k (lb) 60 c 2008 Pearson Education, Inc., Upper Saddle River, NJ. All rights reserved. This material is protected under all copyright laws as they  currently exist. No portion of this material may be reproduced, in any form or by any means, without permission in writing from the publisher. Problem 2.115 Consider the cables AB and AC shown in Problem 2.114. Let rAB be the position vector from point A to point B. Determine the vector component of rAB parallel to the cable AC. Solution: From Problem 2.114, rAB D 0i  7j C 10k, and eAC D 0.6667i  0.3333j C 0.6667k. Thus rAB Ð eAC D 9, and rAB Ð eAC eAC D 6i  3j C 6k ft. Problem 2.116 The force F D 10i C 12j  6k N. Determine the vector components of F parallel and normal to line OA. y A (0, 6, 4) m Solution: Find eOA F O rOA D jrOA j x Then z FP D F Ð eOA eOA and FN D F  FP eOA D 0i C 6j C 4k 6j C 4k p D p 52 62 C 42 eOA D 6 4 jC k D 0.832j C 0.555k 7.21 7.21 FP D [10i C 12j  6k Ð 0.832j C 0.555k]eOA FP D [6.656]eOA D 0i C 5.54j C 3.69k N FN D F  FP FN D 10i C 12  5.54j C 6  3.69k FN D 10i C 6.46j  9.69k N c 2008 Pearson Education, Inc., Upper Saddle River, NJ. All rights reserved. This material is protected under all copyright laws as they  currently exist. No portion of this material may be reproduced, in any form or by any means, without permission in writing from the publisher. 61 Problem 2.117 The rope AB exerts a 50-N force T on collar A. Determine the vector component of T parallel to the bar CD. y 0.15 m Solution: We have the following vectors 0.4 m B C rCD D 0.2i  0.3j C 0.25k m eCD D T rCD D 0.456i  0.684j C 0.570k jrCD j 0.2 m 0.3 m A 0.5 m rOB D 0.5j C 0.15k m O x 0.25 m D rOC D 0.4i C 0.3j m 0.2 m rOA D rOC C 0.2 meCD D 0.309i C 0.163j C 0.114k m z rAB D rOB  rOA D 0.309i C 0.337j C 0.036k m eAB D rAB D 0.674i C 0.735j C 0.079k jrAB j We can now write the force T and determine the vector component parallel to CD. T D 50 NeAB D 33.7i C 36.7j C 3.93k N Tp D eCD Ð TeCD D 3.43i C 5.14j  4.29k N Tp D 3.43i C 5.14j  4.29k N Problem 2.118 In Problem 2.117, determine the vector component of T normal to the bar CD. y 0.15 m Solution: From Problem 2.117 we have 0.4 m B C T D 33.7i C 36.7j C 3.93k N T Tp D 3.43i C 5.14j  4.29k N A The normal component is then 0.5 m O Tn D T  T p x D Tn D 37.1i C 31.6j C 8.22k N 62 0.2 m 0.3 m 0.25 m 0.2 m z c 2008 Pearson Education, Inc., Upper Saddle River, NJ. All rights reserved. This material is protected under all copyright laws as they  currently exist. No portion of this material may be reproduced, in any form or by any means, without permission in writing from the publisher. y Problem 2.119 The disk A is at the midpoint of the sloped surface. The string from A to B exerts a 0.2-lb force F on the disk. If you express F in terms of vector components parallel and normal to the sloped surface, what is the component normal to the surface? B (0, 6, 0) ft F 2 ft A x 8 ft 10 ft z 2 Solution: Consider a line on the sloped surface from A perpendicular to the surface. (see the diagram above) By SIMILAR triangles we see that one such vector is rN D 8j C 2k. Let us find the component of F parallel to this line. The unit vector in the direction normal to the surface is eN D y 8 rN 8j C 2k D 0.970j C 0.243k D p jrN j 82 C 22 2 The unit vector eAB can be found by z xB  xA i C yB  yA j C zB  zA h eAB D  xB  xA 2 C yB  yA 2 C zB  zA 2 8 Point B is at (0, 6, 0) (ft) and A is at (5, 1, 4) (ft). Substituting, we get eAB D 0.615i C 0.615j  0.492k Now F D jFjeAB D 0.2eAB F D 0.123i C 0.123j  0.0984k lb The component of F normal to the surface is the component parallel to the unit vector eN . FNORMAL D F Ð eN eN D 0.955eN FNORMAL D 0i C 0.0927j C 0.0232k lb Problem 2.120 In Problem 2.119, what is the vector component of F parallel to the surface? Solution: From the solution to Problem 2.119, Thus F D 0.123i C 0.123j  0.0984k lb and Fparallel D F  FNORMAL FNORMAL D 0i C 0.0927j C 0.0232k lb Substituting, we get The component parallel to the surface and the component normal to the surface add to give FF D FNORMAL C Fparallel . Fparallel D 0.1231i C 0.0304j  0.1216k lb c 2008 Pearson Education, Inc., Upper Saddle River, NJ. All rights reserved. This material is protected under all copyright laws as they  currently exist. No portion of this material may be reproduced, in any form or by any means, without permission in writing from the publisher. 63 Problem 2.121 An astronaut in a maneuvering unit approaches a space station. At the present instant, the station informs him that his position relative to the origin of the station’s coordinate system is rG D 50i C 80j C 180k (m) and his velocity is v D 2.2j  3.6k (m/s). The position of the airlock is rA D 12i C 20k (m). Determine the angle between his velocity vector and the line from his position to the airlock’s position. Solution: Points G and A are located at G: (50, 80, 180) m and A: (12, 0, 20) m. The vector rGA is rGA D xA  xG i C yA  yG j C zA  zG k D 12  50i C 0  80j C 20  180k m. The dot product between v and rGA is v ž rGA D jvjjrGA j cos  D vx xGA C vy yGA C vz zGA , where  is the angle between v and rGA . Substituting in the numerical values, we get  D 19.7° . y G A z x Problem 2.122 In Problem 2.121, determine the vector component of the astronaut’s velocity parallel to the line from his position to the airlock’s position. Solution: The coordinates are A (12, 0, 20) m, G (50, 80, 180) m. Therefore rGA D 62i  80j  160k m eGA D rGA D 0.327i  0.423j  0.845k jrGA j The velocity is given as v D 2.2j  3.6k m/s The vector component parallel to the line is now vp D eGA Ð veGA D [0.4232.2 C 0.8453.6]eGA vp D 1.30i  1.68j  3.36k m/s 64 c 2008 Pearson Education, Inc., Upper Saddle River, NJ. All rights reserved. This material is protected under all copyright laws as they  currently exist. No portion of this material may be reproduced, in any form or by any means, without permission in writing from the publisher. Problem 2.123 Point P is at longitude 30° W and latitude 45° N on the Atlantic Ocean between Nova Scotia and France. Point Q is at longitude 60° E and latitude 20° N in the Arabian Sea. Use the dot product to determine the shortest distance along the surface of the earth from P to Q in terms of the radius of the earth RE . y N P Q Strategy: Use the dot product to detrmine the angle between the lines OP and OQ; then use the definition of an angle in radians to determine the distance along the surface of the earth from P to Q. 45⬚ z 20⬚ O 30⬚ 60⬚ G Equator x Solution: The distance is the product of the angle and the radius of the sphere, d D RE , where  is in radian measure. From Eqs. (2.18) and (2.24), the angular separation of P and Q is given by  cos  D PÐQ jPjjQj  . The strategy is to determine the angle  in terms of the latitude and longitude of the two points. Drop a vertical line from each point P and Q to b and c on the equatorial plane. The vector position of P is the sum of the two vectors: P D rOB C rBP . The vector rOB D jrOB ji cos P C 0j C k sin P . From geometry, the magnitude is jrOB j D RE cos P . The vector rBP D jrBP j0i C 1j C 0k. From geometry, the magnitude is jrBP j D RE sin P . Substitute and reduce to obtain: The dot product is P Ð Q D RE2 cosP  Q  cos P cos Q C sin P sin Q  Substitute: cos  D PÐQ D cosP  Q  cos P cos Q C sin P sin Q jPjjQj Substitute P D C30° , Q D 60° , p D C45° , Q D C20° , to obtain cos  D 0.2418, or  D 1.326 radians. Thus the distance is d D 1.326RE y P D rOB C rBP D RE i cos P cos P C j sin P C k sin P cos P . N P θ A similar argument for the point Q yields 45° Q D rOC C rCQ D RE i cos Q cos Q C j sin Q C k sin Q cos Q  Q RE b 30° 60° Using the identity cos2 ˇ C sin2 ˇ D 1, the magnitudes are x 20° c G jPj D jQj D RE Problem 2.124 In Active Example 2.14, suppose that the vector V is changed to V D 4i  6j  10k. (a) Determine the cross product U ð V. (b) Use the dot product to prove that U ð V is perpendicular to V. Solution: We have U D 6i  5j  k, V D 4k  6j  10k (a)  i  U ð V D  6 4 j 5 6  k  1  D 44i C 56j  16k 10  U ð V D 44i C 56j  16k (b) U ð V Ð V D 444 C 566 C 1610 D 0 ) U ð V ? V c 2008 Pearson Education, Inc., Upper Saddle River, NJ. All rights reserved. This material is protected under all copyright laws as they  currently exist. No portion of this material may be reproduced, in any form or by any means, without permission in writing from the publisher. 65 Problem 2.125 Two vectors U D 3i C 2j and V D 2i C 4j. (a) What is the cross product U ð V? (b) What is the cross product V ð U? Solution: Use Eq. (2.34) and expand into 2 by 2 determinants.  i  U ð V D  3 2 j 2 4  k  0  D i20  40  j30  20 0 j 4 2  k  0  D i40  20  j20  30 0 C k34  22 D 8k  i  V ð U D  2 3 C k22  34 D 8k y Problem 2.126 The two segments of the L-shaped bar are parallel to the x and z axes. The rope AB exerts a force of magnitude jFj D 500 lb on the bar at A. Determine the cross product rCA ð F, where rCA is the position vector form point C to point A. 4 ft C 5 ft Solution: We need to determine the force F in terms of its components. The vector from A to B is used to define F. 4 ft rAB D 2i  4j  k ft A x F 2i  4j  k rAB D 500 lb  F D 500 lb jrAB j 22 C 42 C 12 F D 218i  436j  109k lb B z (6, 0, 4) ft Also we have rCA D 4i C 5k ft Therefore    i j k    rCA ð F D  4 0 5  D 2180i C 1530j  1750k ft-lb  218 436 109  rCA ð F D 2180i C 1530j  1750k ft-lb 66 c 2008 Pearson Education, Inc., Upper Saddle River, NJ. All rights reserved. This material is protected under all copyright laws as they  currently exist. No portion of this material may be reproduced, in any form or by any means, without permission in writing from the publisher. y Problem 2.127 The two segments of the L-shaped bar are parallel to the x and z axes. The rope AB exerts a force of magnitude jFj D 500 lb on the bar at A. Determine the cross product rCB ð F, where rCB is the position vector form point C to point B. Compare your answers to the answer to Problem 2.126. 4 ft C 5 ft Solution: We need to determine the force F in terms of its compo4 ft nents. The vector from A to B is used to define F. A rAB D 2i  4j  k ft F D 500 lb x F 2i  4j  k rAB D 500 lb  jrAB j 22 C 42 C 12 B z F D 218i  436j  109k lb (6, 0, 4) ft Also we have rCB D 6i  4j C 4k ft Therefore    i j k   rCB ð F D  6 4 4  D 2180i C 1530j  1750k ft-lb  218 436 109  rCB ð F D 2180i C 1530j  1750k ft-lb The answer is the same for 2.126 and 2.127 because the position vectors just point to different points along the line of action of the force. Problem 2.128 Suppose that the cross product of two vectors U and V is U ð V D 0. If jUj 6D 0, what do you know about the vector V? Solution: Either V D 0 or VjjU Problem 2.129 The cross product of two vectors U and V is U ð V D 30i C 40k. The vector V D 4i  2j C 3k. The vector U D 4i C Uy j C Uz k. Determine Uy and Uz . Solution: From the given information we have   i  U ð V D  4  4 j Uy 2  k  Uz  3 D 3Uy C 2Uz i C 4Uz  12j C 8  4Uy k U ð V D 30i C 40k Equating the components we have 3Uy C 2Uz D 30, 4Uz  12 D 0, 8  4Uy D 40. Solving any two of these three redundant equations gives Uy D 12, Uz D 3. c 2008 Pearson Education, Inc., Upper Saddle River, NJ. All rights reserved. This material is protected under all copyright laws as they  currently exist. No portion of this material may be reproduced, in any form or by any means, without permission in writing from the publisher. 67 Problem 2.130 The magnitudes jUj D 10 and jVj D 20. y V (a) Use the definition of the cross product to determine U ð V. (b) Use the definition of the cross product to determine V ð U. (c) Use Eq. (2.34) to determine U ð V. (d) U 30° 45° x Use Eq. (2.34) to determine V ð U. Solution: From Eq. (228) U ð V D jUjjVj sin e. From the sketch, the positive z-axis is out of the paper. For U ð V, e D 1k (points into the paper); for V ð U, e D C1k (points out of the paper). The angle  D 15° , hence (a) U ð V D 10200.2588e D 51.8e D 51.8k. Similarly, (b) V ð U D 51.8e D 51.8k (c) The two vectors are: U D 10i cos 45° C j sin 45 D 7.07i C 0.707j, V D 20i cos 30° C j sin 30°  D 17.32i C 10j   i  U ð V D  7.07  17.32  j k  7.07 0  D i0  j0 C k70.7  122.45 10 0  D 51.8k   i  (d) V ð U D  17.32  7.07  j k  10 0  D i0  j0 C k122.45  70.7 7.07 0  D 51.8k Problem 2.131 The force F D 10i  4j (N). Determine the cross product rAB ð F. y (6, 3, 0) m A rA B x z (6, 0, 4) m B F Solution: The position vector is y A (6, 3, 0) rAB D 6  6i C 0  3j C 4  0k D 0i  3j C 4k The cross product:    i j k   rAB ð F D  0 3 4  D i16  j40 C k30  10 4 0  rA B x z B (6, 0, 4) F D 16i C 40j C 30k (N-m) 68 c 2008 Pearson Education, Inc., Upper Saddle River, NJ. All rights reserved. This material is protected under all copyright laws as they  currently exist. No portion of this material may be reproduced, in any form or by any means, without permission in writing from the publisher. Problem 2.132 By evaluating the cross product U ð V, prove the identity sin1  2  D sin 1 cos 2  cos 1 sin 2 . y U V θ1 θ2 Solution: Assume that both U and V lie in the x-y plane. The strategy is to use the definition of the cross product (Eq. 2.28) and the Eq. (2.34), and equate the two. From Eq. (2.28) U ð V D jUjjVj sin1  2 e. Since the positive z-axis is out of the paper, and e points into the paper, then e D k. Take the dot product of both sides with e, and note that k Ð k D 1. Thus  sin1  2  D  U ð V Ð k jUjjVj x y U V θ1 θ2  x The vectors are: U D jUji cos 1 C j sin 2 , and V D jVji cos 2 C j sin 2 . The cross product is   i  U ð V D  jUj cos 1  jVj cos 2 j jUj sin 1 jVj sin 2  k  0  0 D i0  j0 C kjUjjVjcos 1 sin 2  cos 2 sin 1  Substitute into the definition to obtain: sin1  2  D sin 1 cos 2  cos 1 sin 2 . Q.E.D. Problem 2.133 In Example 2.15, what is the minimum distance from point B to the line OA? y Solution: Let  be the angle between rOA and rOB . Then the minimum distance is d D jrOB j sin  Using the cross product, we have B (6, 6, ⫺3) m jrOA ð rOB j D jrOA jjrOB j sin  D jrOA jd ) d D jrOA ð rOB j jrOA j We have O z x A (10, ⫺2, 3) m rOA D 10i  2j C 3k m rOB D 6i C 6j  3k m rOA ð rOB   i  D  10  6  j k  2 3  D 12i C 48j C 72k m2 6 3  Thus  dD 12 m2  C 48 m2 2 C 72 m2 2  D 8.22 m 10 m2 C 2 m2 C 3 m2 d D 8.22 m c 2008 Pearson Education, Inc., Upper Saddle River, NJ. All rights reserved. This material is protected under all copyright laws as they  currently exist. No portion of this material may be reproduced, in any form or by any means, without permission in writing from the publisher. 69 Problem 2.134 (a) What is the cross product rOA ð rOB ? (b) Determine a unit vector e that is perpendicular to rOA and rOB . y B ( 4, 4, –4) m Solution: The two radius vectors are rOB rOB D 4i C 4j  4k, rOA D 6i  2j C 3k (a) The cross product is  i  D  6 4 rOA ð rOB O j 2 4  k  3  D i8  12  j24  12 4  x rOA A (6, –2, 3) m z C k24 C 8 D 4i C 36j C 32k m2  The magnitude is jrOA ð rOB j D (b) p 42 C 362 C 322 D 48.33 m2 The unit vector is  eDš rOA ð rOB jrOA ð rOB j  D š0.0828i C 0.7448j C 0.6621k (Two vectors.) Problem 2.135 For the points O, A, and B in Problem 2.134, use the cross product to determine the length of the shortest straight line from point B to the straight line that passes through points O and A. Solution: (The magnitude of C is 338.3) rOA D 6i  2j C 3k (m) rOB D 4i C 4j  4k m We now want to find the length of the projection, P, of line OB in direction ec . P D rOB Ð eC rOA ð rOB D C D 4i C 4j  4k Ð eC (C is ? to both rOA and rOB )  i  C D  6 4 j 2 4  C8  12i k  3  D C12 C 24j C24 C 8k 4  P D 6.90 m y B ( 4, 4, –4) m C D 4i C 36j C 32k C is ? to both rOA and rOB . Any line ? to the plane formed by C and rOA will be parallel to the line BP on the diagram. C ð rOA is such a line. We then need to find the component of rOB in this direction and compute its magnitude. C ð rOA   i  D  4  6 j C36 2 rOB  k  32  3 C D 172i C 204j  208k O x rOA P z A(6, –2, 3) m The unit vector in the direction of C is eC D 70 C D 0.508i C 0.603j  0.614k jCj c 2008 Pearson Education, Inc., Upper Saddle River, NJ. All rights reserved. This material is protected under all copyright laws as they  currently exist. No portion of this material may be reproduced, in any form or by any means, without permission in writing from the publisher. Problem 2.136 The cable BC exerts a 1000-lb force F on the hook at B. Determine rAB ð F. y Solution: The coordinates of points A, B, and C are A (16, 0, 12), B (4, 6, 0), C (4, 0, 8). The position vectors are B F 6 ft rAB rOA D 16i C 0j C 12k, rOB D 4i C 6j C 0k, rOC D 4i C 0j C 8k. x 8 ft The force F acts along the unit vector eBC D rAC 4 ft rBC rOC  rOB rAB D D jrBC j jrOC  rOB j jrAB j 4 ft p 62 C 82 D 10. Thus y eBC D 0i  0.6j C 0.8k, and F D jFjeBC D 0i  600j C 800k (lb). B The vector 6 ft 8 ft C Thus the cross product is  k  12  D 2400i C 9600j C 7200k (ft-lb) 800  4 ft 4 ft A 12 ft y Problem 2.137 The force vector F points along the straight line from point A to point B. Its magnitude is jFj D 20 N. The coordinates of points A and B are xA D 6 m, yA D 8 m, zA D 4 m and xB D 8 m, yB D 1 m, zB D 2 m. (a) (b) r x rAB D 4  16i C 6  0j C 0  12k D 12i C 6j  12k   i j  6 rAB ð F D  12  0 600 A 12 ft z Noting rOC  rOB D 4  4i C 0  6j C 8  0k D 0i  6j C 8k jrOC  rOB j D C A F B rA Express the vector F in terms of its components. Use Eq. (2.34) to determine the cross products rA ð F and rB ð F. rB x z Solution: We have rA D 6i C 8j C 4k m, rB D 8i C j  2k m, F D 20 N (a) 8  6 mi C 1  8 mj C 2  4 mk  2 m2 C 7 m2 C 6 m2 20 N D p 2i  7j  6k 89 20 N rA ð F D p 89 (b)   i   6 m   2 j 8m 7  k   4 m  6  D 42.4i C 93.3j  123.0k Nm    i j k  20 N   rB ð F D p  8 m 1 m 2 m   89  2 7 6  D 42.4i C 93.3j  123.0k Nm Note that both cross products give the same result (as they must). c 2008 Pearson Education, Inc., Upper Saddle River, NJ. All rights reserved. This material is protected under all copyright laws as they  currently exist. No portion of this material may be reproduced, in any form or by any means, without permission in writing from the publisher. 71 Problem 2.138 The rope AB exerts a 50-N force T on the collar at A. Let rCA be the position vector from point C to point A. Determine the cross product rCA ð T. y 0.15 m 0.4 m B C Solution: We define the appropriate vectors. T rCD D 0.2i  0.3j C 0.25k m A 0.2 m 0.3 m 0.5 m rCA rCD D 0.091i  0.137j C 0.114k m D 0.2 m jrCD j O x D 0.25 m 0.2 m rOB D 0.5j C 0.15k m z rOC D 0.4i C 0.3j m rAB D rOB  rOC C rCA  D 0.61i  1.22j  0.305k m T D 50 N rAB D 33.7i C 36.7j C 3.93k N jrAB j Now take the cross product   i j  rCA ð T D  0.091 0.137  33.7 36.7  l  0.114  D 4.72i  3.48j C 7.96k N-m 3.93  rCA ð T D 47.2i  3.48j C 7.96k N-m Problem 2.139 In Example 2.16, suppose that the attachment point E is moved to the location (0.3, 0.3, 0) m and the magnitude of T increases to 600 N. What is the magnitude of the component of T perpendicular to the door? Solution: We first develop the force T. rCE D 0.3i C 0.1j m T D 600 N E (0.2, 0.4, ⫺0.1) m From Example 2.16 we know that the unit vector perpendicular to the door is e D 0.358i C 0.894j C 0.268k y T D The magnitude of the force perpendicular to the door (parallel to e) is then C (0, 0.2, 0) m A (0.5, 0, 0) m B (0.35, 0, 0.2) m rCE D 569i C 190j N jrCE j x jTn j D T Ð e D 569 N0.358 C 190 N0.894 D 373 N jTn j D 373 N z 72 c 2008 Pearson Education, Inc., Upper Saddle River, NJ. All rights reserved. This material is protected under all copyright laws as they  currently exist. No portion of this material may be reproduced, in any form or by any means, without permission in writing from the publisher. y Problem 2.140 The bar AB is 6 m long and is perpendicular to the bars AC and AD. Use the cross product to determine the coordinates xB , yB , zB of point B. B Solution: The strategy is to determine the unit vector perpendicular to both AC and AD, and then determine the coordinates that will agree with the magnitude of AB. The position vectors are: (0, 3, 0) m (xB, yB, zB) A rOA D 0i C 3j C 0k, rOD D 0i C 0j C 3k, and rOC D 4i C 0j C 0k. The vectors collinear with the bars are: rAD D 0  0i C 0  3j C 3  0k D 0i  3j C 3k, rAC D 4  0i C 0  3j C 0  0k D 4i  3j C 0k. The vector collinear with rAB is   i j k   R D rAD ð rAC D  0 3 3  D 9i C 12j C 12k  4 3 0  C D (0, 0, 3) m x (4, 0, 0) m z The magnitude jRj D 19.21 (m). The unit vector is eAB D R D 0.4685i C 0.6247j C 0.6247k. jRj Thus the vector collinear with AB is rAB D 6eAB D C2.811i C 3.75j C 3.75k. Using the coordinates of point A: xB D 2.81 C 0 D 2.81 (m) yB D 3.75 C 3 D 6.75 (m) zB D 3.75 C 0 D 3.75 (m) Problem 2.141* Determine the minimum distance from point P to the plane defined by the three points A, B, and C. y B (0, 5, 0) m P (9, 6, 5) m Solution: The strategy is to find the unit vector perpendicular to the plane. The projection of this unit vector on the vector OP: rOP Ð e is the distance from the origin to P along the perpendicular to the plane. The projection on e of any vector into the plane (rOA Ð e, rOB Ð e, or rOC Ð e) is the distance from the origin to the plane along this same perpendicular. Thus the distance of P from the plane is A (3, 0, 0) m C d D rOP Ð e  rOA Ð e. The position vectors are: rOA D 3i, rOB D 5j, rOC D 4k and rOP D 9i C 6j C 5k. The unit vector perpendicular to the plane is found from the cross product of any two vectors lying in the plane. Noting: rBC D rOC  rOB D 5j C 4k, and rBA D rOA  rOB D 3i  5j. The cross product: rBC ð rBA  i  D  0 3 (0, 0, 4) m z y P[9,6,5]  j k  5 4  D 20i C 12j C 15k. 5 0  The magnitude is jrBC ð rBA j D 27.73, thus the unit vector is e D 0.7212i C 0.4327j C 0.5409k. The distance of point P from the plane is d D rOP Ð e  rOA Ð e D 11.792  2.164 D 9.63 m. The second term is the distance of the plane from the origin; the vectors rOB , or rOC could have been used instead of rOA . x B[0,5,0] x O A[3,0,0] z C[0,0,4] c 2008 Pearson Education, Inc., Upper Saddle River, NJ. All rights reserved. This material is protected under all copyright laws as they  currently exist. No portion of this material may be reproduced, in any form or by any means, without permission in writing from the publisher. 73 y Problem 2.142* The force vector F points along the straight line from point A to point B. Use Eqs. (2.28)–(2.31) to prove that A F rB ð F D rA ð F. rA Strategy: Let rAB be the position vector from point A to point B. Express rB in terms of of rA and rAB . Notice that the vectors rAB and F are parallel. B rB x z Solution: We have rB D rA C rAB . Therefore rB ð F D rA C rAB  ð F D rA ð F C rAB ð F The last term is zero since rAB jjF. Therefore rB ð F D rA ð F Problem 2.143 For the vectors U D 6i C 2j  4k, V D 2i C 7j, and W D 3i C 2k, evaluate the following mixed triple products: (a) U Ð V ð W; (b) W Ð V ð U; (c) V Ð W ð U. Solution: Use Eq. (2.36).  6  (a) U Ð V ð W D  2 3 2 7 0  4  0  2 D 614  24 C 421 D 160  3  (b) W Ð V ð U D  2 6 0 7 2  2  0  4  D 328  0 C 24  42 D 160  2  (c) V Ð W ð U D  3 6 7 0 2  0  2  4  D 24  712  12 C 0 D 160 74 c 2008 Pearson Education, Inc., Upper Saddle River, NJ. All rights reserved. This material is protected under all copyright laws as they  currently exist. No portion of this material may be reproduced, in any form or by any means, without permission in writing from the publisher. Problem 2.144 Use the mixed triple product to calculate the volume of the parallelepiped. y (140, 90, 30) mm (200, 0, 0) mm x (160, 0, 100) mm z Solution: We are given the coordinates of point D. From the geometry, we need to locate points A and C. The key to doing this is to note that the length of side OD is 200 mm and that side OD is the x axis. Sides OD, AE, and CG are parallel to the x axis and the coordinates of the point pairs (O and D), (A and E), and (C and D) differ only by 200 mm in the x coordinate. Thus, the coordinates of point A are (60, 90, 30) mm and the coordinates of point C are (40, 0, 100) mm. Thus, the vectors rOA , rOD , and rOC are rOD D 200i mm, rOA D 60i C 90j C 30k mm, and rOC D 40i C 0j C 100k mm. The mixed triple product of the three vectors is the volume of the parallelepiped. The volume is   60  rOA Ð rOC ð rOD  D  40  200 90 0 0  30  100  0  y (140, 90, 30) mm E A B F O D G C x (200, 0, 0) mm (160, 0, 100) mm z D 600 C 90200100 C 300 mm3 D 1,800,000 mm3 Problem 2.145 By using Eqs. (2.23) and (2.34), show that    Ux Uy Uz    U Ð V ð W D  Vx Vy Vz  W W W  x y z . Solution: One strategy is to expand the determinant in terms of its components, take the dot product, and then collapse the expansion. Eq. (2.23) is an expansion of the dot product: Eq. (2.23): U Ð V D UX VX C UY VY C UZ VZ . Eq. (2.34) is the determinant representation of the cross product:   i  Eq. (2.34) U ð V D  UX  VX j UY VY  k  UZ  VZ   U Q Ð P D QX  Y VY For notational convenience, write P D U ð V. Expand the determinant about its first row:  U P D i  Y VY    UX UZ    j  VX VZ     UX UZ   C k  VX VZ  Since the two-by-two determinants are scalars, this can be written in the form: P D iPX C jPY C kPZ where the scalars PX , PY , and PZ are the two-by-two determinants. Apply Eq. (2.23) to the dot product of a vector Q with P. Thus Q Ð P D QX PX C QY PY C QZ PZ . Substitute PX , PY , and PZ into this dot product  UZ  VZ     UX UZ    Q Y VZ  VX    UX UZ   C Q z VZ  VX  UZ  VZ  But this expression can be collapsed into a three-by-three determinant directly, thus:   QX  Q Ð U ð V D  UX  VX QY UY VY  QZ  UZ . This completes the demonstration. VZ  c 2008 Pearson Education, Inc., Upper Saddle River, NJ. All rights reserved. This material is protected under all copyright laws as they  currently exist. No portion of this material may be reproduced, in any form or by any means, without permission in writing from the publisher. 75 Problem 2.146 The vectors U D i C UY j C 4k, V D 2i C j  2k, and W D 3i C j  2k are coplanar (they lie in the same plane). What is the component Uy ? Solution: Since the non-zero vectors are coplanar, the cross product of any two will produce a vector perpendicular to the plane, and the dot product with the third will vanish, by definition of the dot product. Thus U Ð V ð W D 0, for example.   1 UY  1 U Ð V ð W D  2  3 1  4  2  2  D 12 C 2  UY 4  6 C 42 C 3 D C10UY C 20 D 0 Thus UY D 2 Problem 2.147 The magnitude of F is 8 kN. Express F in terms of scalar components. Solution: The unit vector collinear with the force F is developed as follows: The collinear vector is r D 7  3i C 2  7j D 4i  5j y The magnitude: jrj D (3, 7) m eD F p 42 C 52 D 6.403 m. The unit vector is r D 0.6247i  0.7809j. The force vector is jrj F D jFje D 4.998i  6.247j D 5i  6.25j (kN) (7, 2) m x Problem 2.148 The magnitude of the vertical force W is 600 lb, and the magnitude of the force B is 1500 lb. Given that A C B C W D 0, determine the magnitude of the force A and the angle ˛. B Solution: The strategy is to use the condition of force balance to determine the unknowns. The weight vector is W D 600j. The vector B is W 50° α A B D 1500i cos 50° C j sin 50°  D 964.2i C 1149.1j The vector A is A D jAji cos180 C ˛ C j sin180 C ˛ A D jAji cos ˛  j sin ˛. The forces balance, hence A C B C W D 0, or 964.2  jAj cos ˛i D 0, and 1149.1  600  jAj sin ˛j D 0. Thus jAj cos ˛ D 964.2, and jAj sin ˛ D 549.1. Take the ratio of the two equations to obtain tan ˛ D 0.5695, or ˛ D 29.7° . Substitute this angle to solve: jAj D 1110 lb 76 c 2008 Pearson Education, Inc., Upper Saddle River, NJ. All rights reserved. This material is protected under all copyright laws as they  currently exist. No portion of this material may be reproduced, in any form or by any means, without permission in writing from the publisher. Problem 2.149 The magnitude of the vertical force vector A is 200 lb. If A C B C C D 0, what are the magnitudes of the force vectors B and C? Solution: The strategy is to express the forces in terms of scalar components, and then solve the force balance equations for the unknowns. C D jCji cos ˛  j sin ˛, where tan ˛ D 100 in. 70 in. 50 in. 50 D 0.7143, or ˛ D 35.5° . 70 C E B D A F Thus C D jCj0.8137i  0.5812j. Similarly, B D CjBji, and A D C200j. The force balance equation is A C B C C D 0. Substituting, 0.8137jCj C jBji D 0, and 0.5812jCj C 200j D 0. Solving, jCj D 344.1 lb, jBj D 280 lb Problem 2.150 The magnitude of the horizontal force vector D in Problem 2.149 is 280 lb. If D C E C F D 0, what are the magnitudes of the force vectors E and F? Solution: The strategy is to express the force vectors in terms of scalar components, and then solve the force balance equation for the unknowns. The force vectors are: E D jEji cos ˇ  j sin ˇ, where tan ˇ D 50 D 0.5, or ˇ D 26.6° . 100 Thus E D jEj0.8944i  0.4472j D D 280i, and F D jFjj. The force balance equation is D C E C F D 0. Substitute and resolve into two equations: 0.8944jEj  280i D 0, and 0.4472jEj C jFjj D 0. Solve: jEj D 313.1 lb, jFj D 140 lb Problem 2.151 What are the direction cosines of F? y Refer to this diagram when solving Problems 2.151– 2.157. A (4, 4, 2) ft Solution: Use the definition of the direction cosines and the ensuing discussion. p The magnitude of F: jFj D 202 C 102 C 102 D 24.5. F ⫽ 20i ⫹ 10j ⫺ 10k (lb) u B (8, 1, ⫺2) ft x z Fx 20 D D 0.8165, The direction cosines are cos x D jFj 24.5 cos y D Fy 10 D D 0.4082 jFj 24.5 cos z D 10 Fz D D 0.4082 jFj 24.5 c 2008 Pearson Education, Inc., Upper Saddle River, NJ. All rights reserved. This material is protected under all copyright laws as they  currently exist. No portion of this material may be reproduced, in any form or by any means, without permission in writing from the publisher. 77 Problem 2.152 Determine the scalar components of a unit vector parallel to line AB that points from A toward B. Solution: Use the definition of the unit vector, we get The position vectors are: rA D 4i C 4j C 2k, rB D 8i C 1j  2k. The vector from A to B is rAB D 8 p4i C 1  4j C 2  2k D 4i  3j  4k. The magnitude: jrAB j D 42 C 32 C 42 D 6.4. The unit vector is eAB D Problem 2.153 What is the angle  between the line AB and the force F? rAB 4 3 4 D i j k D 0.6247i  0.4685j  0.6247k jrAB j 6.4 6.4 6.4 Solution: Use the definition of the dot product Eq. (2.18), and Eq. (2.24): cos  D rAB Ð F . jrAB jjFj From the solution to Problem 2.130, the vector parallel to AB is rAB D 4i  3j  4k, with a magnitude jrAB j D 6.4. From Problem 2.151, the force is F D 20i C 10j  10k, with a magnitude of jFj D 24.5. The dot product is rAB Ð F D 420 C 310 C 410 D 90. Substi90 tuting, cos  D D 0.574,  D 55° 6.424.5 Problem 2.154 Determine the vector component of F that is parallel to the line AB. Solution: Use the definition in Eq. (2.26): UP D e Ð Ue, where e is parallel to a line L. From Problem 2.152 the unit vector parallel to line AB is eAB D 0.6247i  0.4688j  0.6247k. The dot product is e Ð F D 0.624720 C 0.468810 C 0.624710 D 14.053. The parallel vector is e Ð Fe D 14.053e D 8.78i  6.59j  8.78k (lb) Problem 2.155 Determine the vector component of F that is normal to the line AB. Solution: Use the Eq. (2.27) and the solution to Problem 2.154. FN D F  FP D 20  8.78i C 10 C 6.59j C 10 C 8.78k D 11.22i C 16.59j  1.22k (lb) Problem 2.156 Determine the vector rBA ð F, where rBA is the position vector from B to A. Solution: Use the definition in Eq. (2.34). Noting rBA D rAB , from Problem 2.155 rBA D 4i C 3j C 4k. The cross product is   i  rBA ð F D  4  20 j 3 10  k  4  D 30  40i  40  80j 10  C 40  60 D 70i C 40j  100k (ft-lb) 78 c 2008 Pearson Education, Inc., Upper Saddle River, NJ. All rights reserved. This material is protected under all copyright laws as they  currently exist. No portion of this material may be reproduced, in any form or by any means, without permission in writing from the publisher. Problem 2.157 (a) Write the position vector rAB from point A to point B in terms of components. y A (4, 4, 2) ft (b) A vector R has magnitude jRj D 200 lb and is parallel to the line from A to B. Write R in terms of components. F ⫽ 20i ⫹ 10j ⫺ 10k (lb) u B (8, 1, ⫺2) ft x Solution: (a) rAB D [8  4]i C [1  4]j C [2  2]k ft z rAB D 4i  3j  4k ft (b) R D 200 N rAB D 125i  93.7j  125k N jrAB j R D 125i  96.3j  125k N y Problem 2.158 The rope exerts a force of magnitude jFj D 200 lb on the top of the pole at B. (a) (b) Determine the position vector Determine the position vector vector rAB ð F, where rAB is the from A to B. vector rAC ð F, where rAC is the from A to C. F A Solution: The strategy is to define the unit vector pointing from B to A, express the force in terms of this unit vector, and take the cross product of the position vectors with this force. The position vectors rAB D 5i C 6j C 1k, rAC D 3i C 0j C 4k, B (5, 6, 1) ft x C (3, 0, 4) ft z rBC D 3  5i C 0  6j C 4  1k D 2i  6j C 3k. The magnitude jrBC j D eBC D p 22 C 62 C 32 D 7. The unit vector is rBC D 0.2857i  0.8571j C 0.4286k. jrBC j The force vector is F D jFjeBC D 200eBC D 57.14i  171.42j C 85.72k. The cross products:   i  rAB ð F D  5  57.14 j 6 171.42  k  1  85.72  D 685.74i  485.74j  514.26k D 685.7i  485.7j  514.3k (ft-lb)   i  rAC ð F D  3  57.14 j 0 171.42  k  4  85.72  D 685.68i  485.72j  514.26k D 685.7i  485.7j  514.3k (ft-lb) c 2008 Pearson Education, Inc., Upper Saddle River, NJ. All rights reserved. This material is protected under all copyright laws as they  currently exist. No portion of this material may be reproduced, in any form or by any means, without permission in writing from the publisher. 79 Problem 2.159 The pole supporting the sign is parallel to the x axis and is 6 ft long. Point A is contained in the y –z plane. (a) Express the vector r in terms of components. (b) What are the direction cosines of r? Solution: The vector r is r D jrjsin 45° i C cos 45° sin 60° j C cos 45° cos 60° k The length of the pole is the x component of r. Therefore y jrj sin 45° D 6 ft ) jrj D A (a) Bedford Falls (b) r D 6.00i C 5.20j C 3.00k ft The direction cosines are cos x D r 45⬚ 6 ft D 8.49 ft sin 45° rx ry rz D 0.707, cos y D D 0.612, cos z D D 0.354 jrj jrj jrj cos x D 0.707, cos y D 0.612, cos z D 0.354 60⬚ x O z Problem 2.160 The z component of the force F is 80 lb. (a) Express F in terms of components. (b) what are the angles x , y , and z between F and the positive coordinate axes? y F Solution: We can write the force as x 60⬚ We know that the z component is 80 lb. Therefore jFj cos 20° cos 60° (a) (b) 20⬚ O F D jFjcos 20° sin 60° i C sin 20° j C cos 20° cos 60° k A D 80 lb ) jFj D 170 lb F D 139i C 58.2j C 80k lb z The direction cosines can be found:   139 D 35.5° x D cos1 170  y D cos1  z D cos1 58.2 170 80 170  D 70.0°  D 62.0° x D 35.5° , y D 70.0° , z D 62.0° 80 c 2008 Pearson Education, Inc., Upper Saddle River, NJ. All rights reserved. This material is protected under all copyright laws as they  currently exist. No portion of this material may be reproduced, in any form or by any means, without permission in writing from the publisher. y Problem 2.161 The magnitude of the force vector FB is 2 kN. Express it in terms of scalar components. F D (4, 3, 1) m FC FA FB A z C x (6, 0, 0) m B Solution: The strategy is to determine the unit vector collinear with FB and then express the force in terms of this unit vector. F y The radius vector collinear with FB is D (4,3,1) rBD D 4  5i C 3  0j C 1  3k or rBD D 1i C 3j  2k. FA The magnitude is p jrBD j D 12 C 32 C 22 D 3.74. The unit vector is eBD (5, 0, 3) m FC A z x C(6,0,0) FB B (5,0,3) rBD D D 0.2673i C 0.8018j  0.5345k jrBD j The force is FB D jFB jeBD D 2eBD (kN) FB D 0.5345i C 1.6036j  1.0693k D 0.53i C 1.60j  1.07k (kN) Problem 2.162 The magnitude of the vertical force vector F in Problem 2.161 is 6 kN. Determine the vector components of F parallel and normal to the line from B to D. Solution: The projection of the force F onto the line from B to D is FP D F Ð eBD eBD . The vertical force has the component F D 6j (kN). From Problem 2.139, the unit vector pointing from B to D is eBD D 0.2673i C 0.8018j  0.5345k. The dot product is F Ð eBD D 4.813. Thus the component parallel to the line BD is FP D 4.813eBD D C1.29i  3.86j C 2.57k (kN). The component perpendicular to the line is: FN D F  FP . Thus FN D 1.29i  2.14j  2.57k (kN) c 2008 Pearson Education, Inc., Upper Saddle River, NJ. All rights reserved. This material is protected under all copyright laws as they  currently exist. No portion of this material may be reproduced, in any form or by any means, without permission in writing from the publisher. 81 Problem 2.163 The magnitude of the vertical force vector F in Problem 2.161 is 6 kN. Given that F C FA C FB C FC D 0, what are the magnitudes of FA , FB , and FC ? Solution: The strategy is to expand the forces into scalar components, and then use the force balance equation to solve for the unknowns. The unit vectors are used to expand the forces into scalar components. The position vectors, magnitudes, and unit vectors are: rAD D 4i C 3j C 1k, jrAD j D The forces are: FA D jFA jeAD , FB D jFB jeBD , FC D jFC jeCD , F D 6j (kN). Substituting into the force balance equation p 26 D 5.1, F C FA C FB C FC D 0, eAD D 0.7845i C 0.5883j C 0.1961k. rBD D 1i C 3j  2k, jrBD j D 0.7843jFA j  0.2674jFB j  0.5348jFC ji D 0 p 14 D 3.74, 0.5882jFA j C 0.8021jFB j C 0.8021jFC j  6j eBD D 0.2673i C 0.8018j  0.5345k. rCD D 2i C 3j C 1k, jrCD j D p D 00.1961jFA j  0.5348jFB j C 0.2674jFC jk D 0 14 D 3.74, These simple simultaneous equations can be solved a standard method (e.g., Gauss elimination) or, conveniently, by using a commercial package, such as TK Solver, Mathcad, or other. An HP-28S hand held calculator was used here: jFA j D 2.83 (kN), jFB j D 2.49 (kN), jFC j D 2.91 (kN) eCD D 0.5345i C 0.8018j C 0.2673k Problem 2.164 The magnitude of the vertical force W is 160 N. The direction cosines of the position vector from A to B are cos x D 0.500, cos y D 0.866, and cos z D 0, and the direction cosines of the position vector from B to C are cos x D 0.707, cos y D 0.619, and cos z D 0.342. Point G is the midpoint of the line from B to C. Determine the vector rAG ð W, where rAG is the position vector from A to G. Solution: Express the position vectors in terms of scalar components, calculate rAG , and take the cross product. The position vectors are: rAB D 0.6.5i C 0.866j C 0k rAB D 0.3i C 0.5196j C 0k, rBG D 0.30.707i C 0.619j  0.342k, rBG D 0.2121i C 0.1857j  0.1026k. rAG D rAB C rBG D 0.5121i C 0.7053j  0.1026k. W D 160j y 0 60 mm C   i  rAG ð W D  0.5121  0 j 0.7053 160   k  0.1026   0 G D 16.44i C 0j  81.95k D 16.4i C 0j  82k (N m) B W 600 mm 600 mm C 600 mm G B W A A z 82 x x c 2008 Pearson Education, Inc., Upper Saddle River, NJ. All rights reserved. This material is protected under all copyright laws as they  currently exist. No portion of this material may be reproduced, in any form or by any means, without permission in writing from the publisher. Problem 2.165 The rope CE exerts a 500-N force T on the hinged door. E (0.2, 0.4, ⫺0.1) m y (a) (b) Express T in terms of components. Determine the vector component of T parallel to the line from point A to point B. T D C (0, 0.2, 0) m Solution: We have A (0.5, 0, 0) m rCE D 0.2i C 0.2j  0.1k m T D 500 N (a) (b) x B (0.35, 0, 0.2) m rCE D 333i C 333j  167k N jrCE j z T D 333i C 333j  167k N We define the unit vector in the direction of AB and then use this vector to find the component parallel to AB. rAB D 0.15i C 0.2k m eAB D rAB D 0.6i C 0.8k jrAB j Tp D eAB Ð TeAB D [0.6][333 N] C [0.8][167 N]0.6i C 0.8k Tp D 200i  267k N Problem 2.166 In Problem 2.165, let rBC be the position vector from point B to point C. Determine the cross product rBC ð T. E (0.2, 0.4, ⫺0.1) m y T Solution: From Problem 2.165 we know that D C (0, 0.2, 0) m A (0.5, 0, 0) m T D 333i C 333j  167k N x B (0.35, 0, 0.2) m The vector rBC is rBC D 035i C 0.2j  0.2k m The cross product is   i j  rBC ð T D  0.35 0.2  333 333 z  k  0.2  D 33.3i  125j  183k Nm 137  rBC ð T D 33.3i  125j  183k Nm c 2008 Pearson Education, Inc., Upper Saddle River, NJ. All rights reserved. This material is protected under all copyright laws as they  currently exist. No portion of this material may be reproduced, in any form or by any means, without permission in writing from the publisher. 83 Problem 3.1 In Active Example 3.1, suppose that the angle between the ramp supporting the car is increased from 20° to 30° . Draw the free-body diagram of the car showing the new geometry. Suppose that the cable from A to B must exert a 1900-lb horizontal force on the car to hold it in place. Determine the car’s weight in pounds. A B 20⬚ Solution: The free-body diagram is shown to the right. Applying the equilibrium equations   Fx : T  N sin 30° D 0, Fy : N cos 30°  mg D 0 Setting T D 1900 lb and solving yields N D 3800 lb, mg D 3290 lb Problem 3.2 The ring weighs 5 lb and is in equilibrium. The force F1 D 4.5 lb. Determine the force F2 and the angle ˛. y F2 a F1 30⬚ x Solution: The free-body diagram is shown below the drawing. The equilibrium equations are   Fx : F1 cos 30°  F2 cos ˛ D 0 Fy : F1 sin 30° C F2 sin ˛  5 lb D 0 We can write these equations as F2 sin ˛ D 5 lb  F1 sin 30° F2 cos ˛ D F1 cos 30° Dividing these equations and using the known value for F1 we have. tan ˛ D F2 D 5 lb  4.5 lb sin 30° D 0.706 ) ˛ D 35.2° 4.5 lb cos 30° 4.5 lb cos 30° D 4.77 lb cos ˛ F2 D 4.77 lb, ˛ D 35.2° 84 c 2008 Pearson Education, Inc., Upper Saddle River, NJ. All rights reserved. This material is protected under all copyright laws as they  currently exist. No portion of this material may be reproduced, in any form or by any means, without permission in writing from the publisher. Problem 3.3 In Example 3.2, suppose that the attachment point C is moved to the right and cable AC is extended so that the angle between cable AC and the ceiling decreases from 45° to 35° . The angle between cable AB and the ceiling remains 60° . What are the tensions in cables AB and AC? B 60⬚ 45⬚ C A Solution: The free-body diagram is shown below the picture. The equilibrium equations are:   Fx : TAC cos 35°  TAB cos 60° D 0 Fy : TAC sin 35° C TAB sin 60°  1962 N D 0 Solving we find TAB D 1610 N, TAC D 985 N Problem 3.4 The 200-kg engine block is suspended by the cables AB and AC. The angle ˛ D 40° . The freebody diagram obtained by isolating the part of the system within the dashed line is shown. Determine the forces TAB and TAC . y TAB B TAC C a A a A x (200 kg) (9.81 m/s2) Solution: TAB TAC ˛ D 40°   Fx : TAC cos ˛  TAB cos ˛ D 0 α α Fy : TAC sin ˛ C TAB sin ˛  1962 N D 0 Solving: TAB D TAC D 1.526 kN 1962 Ν c 2008 Pearson Education, Inc., Upper Saddle River, NJ. All rights reserved. This material is protected under all copyright laws as they  currently exist. No portion of this material may be reproduced, in any form or by any means, without permission in writing from the publisher. 85 Problem 3.5 A heavy rope used as a mooring line for a cruise ship sags as shown. If the mass of the rope is 90 kg, what are the tensions in the rope at A and B? 55⬚ A B 40⬚ Solution: The free-body diagram is shown. The equilibrium equations are   Fx : TB cos 40°  TA cos 55° D 0 Fy : TB sin 40° C TA sin 55°  909.81 N D 0 Solving: TA D 679 N, TB D 508 N Problem 3.6 A physiologist estimates that the masseter muscle of a predator, Martes, is capable of exerting a force M as large as 900 N. Assume that the jaw is in equilibrium and determine the necessary force T that the temporalis muscle exerts and the force P exerted on the object being bitten. 22⬚ T P M Solution: The equilibrium equations are   Fx : T cos 22°  M cos 36° D 0 36⬚ Fy : T sin 22° C M sin 36°  P D 0 Setting M D 900 N, and solving, we find T D 785 N, P D 823 N 86 c 2008 Pearson Education, Inc., Upper Saddle River, NJ. All rights reserved. This material is protected under all copyright laws as they  currently exist. No portion of this material may be reproduced, in any form or by any means, without permission in writing from the publisher. Problem 3.7 The two springs are identical, with unstretched lengths 250 mm and spring constants k D 1200 N/m. (a) Draw the free-body diagram of block A. (b) Draw the free-body diagram of block B. (c) What are the masses of the two blocks? 300 mm A 280 mm B Solution: The tension in the upper spring acts on block A in the positive Y direction, Solve the spring force-deflection equation for the tension in the upper spring. Apply the equilibrium conditions to block A. Repeat the steps for block B. 300 mm   N 0.3 m  0.25 mj D 0i C 60j N TUA D 0i C 1200 m Similarly, the tension in the lower spring acts on block A in the negative Y direction  TLA D 0i  1200 A 280 mm  N m 0.28 m  0.25 mj D 0i  36j N B The weight is WA D 0i  jWA jj The equilibrium conditions are  FD  Fx C  Fy D 0,  Tension, upper spring F D WA C TUA C TLA D 0 A Collect and combine like terms in i, j  Solve Fy D jWA j C 60  36j D 0 Tension, lower spring Weight, mass A jWA j D 60  36 D 24 N The mass of A is mA D 24 N jWL j D D 2.45 kg jgj 9.81 m/s2 The free body diagram for block B is shown. The tension in the lower spring TLB D 0i C 36j The weight: WB D 0i  jWB jj Apply the equilibrium conditions to block B.  Tension, lower spring y B x Weight, mass B F D WB C TLB D 0 Collect and combine like terms in i, j:  Solve: Fy D jWB j C 36j D 0 jWB j D 36 N The mass of B is given by mB D 36 N jWB j D D 3.67 kg jgj 9.81 m/s2 c 2008 Pearson Education, Inc., Upper Saddle River, NJ. All rights reserved. This material is protected under all copyright laws as they  currently exist. No portion of this material may be reproduced, in any form or by any means, without permission in writing from the publisher. 87 Problem 3.8 The two springs in Problem 3.7 are identical, with unstretched lengths of 250 mm. Suppose that their spring constant k is unknown and the sum of the masses of blocks A and B is 10 kg. Determine the value of k and the masses of the two blocks. Solution: All of the forces are in the vertical direction so we will use scalar equations. First, consider the upper spring supporting both masses (10 kg total mass). The equation of equilibrium for block the entire assembly supported by the upper spring is A is TUA  mA C mB g D 0, where TUA D kU  0.25 N. The equation of equilibrium for block B is TUB  mB g D 0, where TUB D kL  0.25 N. The equation of equilibrium for block A alone is TUA C TLA  mA g D 0 where TLA D TUB . Using g D 9.81 m/s2 , and solving simultaneously, we get k D 1962 N/m, mA D 4 kg, and mB D 6 kg . Problem 3.9 The inclined surface is smooth (Remember that “smooth” means that friction is negligble). The two springs are identical, with unstretched lengths of 250 mm and spring constants k D 1200 N/m. What are the masses of blocks A and B? 300 mm A 280 mm B 30⬚ Solution: F1 D 1200 N/m0.3  0.25m D 60 N mAg F1 F2 D 1200 N/m0.28  0.25m D 36 N   F2 FB &: F2 C mB g sin 30° D 0 m Bg F2 FA &: F1 C F2 C mA g sin 30° D 0 NA Solving: mA D 4.89 kg, mB D 7.34 kg NB 88 c 2008 Pearson Education, Inc., Upper Saddle River, NJ. All rights reserved. This material is protected under all copyright laws as they  currently exist. No portion of this material may be reproduced, in any form or by any means, without permission in writing from the publisher. Problem 3.10 The mass of the crane is 20,000 kg. The crane’s cable is attached to a caisson whose mass is 400 kg. The tension in the cable is 1 kN. (a) (b) Determine the friction forces level ground. Determine the friction forces level ground. magnitudes of the normal and exerted on the crane by the 45° magnitudes of the normal and exerted on the caisson by the Strategy: To do part (a), draw the free-body diagram of the crane and the part of its cable within the dashed line. Solution: (a)   45° Fy : Ncrane  196.2 kN  1 kN sin 45° D0 196.2 kN 1 kN Fx : Fcrane C 1 kN cos 45° D 0 y Ncrane D 196.9 kN, Fcrane D 0.707 kN (b)   Fy : Ncaisson  3.924 kN C 1 kN sin 45° D 0 x Fcrane Fx : 1 kN cos 45° C Fcaisson D 0 Ncrane Ncaisson D 3.22 kN, Fcaisson D 0.707 kN 1 kN 3.924 kN 45° Fcaisson Ncaisson Problem 3.11 The inclined surface is smooth. The 100-kg crate is held stationary by a force T applied to the cable. Solution: (a) The FBD T (a) (b) Draw the free-body diagram of the crate. Determine the force T. 981 Ν T Ν 60° 60⬚ (b)  F -: T  981 N sin 60° D 0 T D 850 N c 2008 Pearson Education, Inc., Upper Saddle River, NJ. All rights reserved. This material is protected under all copyright laws as they  currently exist. No portion of this material may be reproduced, in any form or by any means, without permission in writing from the publisher. 89 Problem 3.12 The 1200-kg car is stationary on the sloping road. If ˛ D 20° , what are the magnitudes of the total normal and friction forces exerted on the car’s tires by the road? (b) The car can remain stationary only if the total friction force necessary for equilibrium is not greater than 0.6 times the total normal force. What is the largest angle ˛ for which the car can remain stationary? (a) a Solution: 11.772 kN (a) ˛D   20° F% : N  11.772 kN cos ˛ D 0 F- : F  11.772 kN sin ˛ D 0 N D 11.06 kN, F D 4.03 kN (b) α F D 0.6 N   F F% : N  11.772 kN cos ˛ D 0 ) ˛ D 31.0° N F- : F  11.772 kN sin ˛ D 0 Problem 3.13 The 100-lb crate is in equilibrium on the smooth surface. The spring constant is k D 400 lb/ft. Let S be the stretch of the spring. Obtain an equation for S (in feet) as a function of the angle ˛. a Solution: The free-body diagram is shown. The equilibrium equation in the direction parallel to the inclined surface is kS  100 lb sin ˛ D 0 Solving for S and using the given value for k we find S D 0.25 ft sin ˛ 90 c 2008 Pearson Education, Inc., Upper Saddle River, NJ. All rights reserved. This material is protected under all copyright laws as they  currently exist. No portion of this material may be reproduced, in any form or by any means, without permission in writing from the publisher. Problem 3.14 A 600-lb box is held in place on the smooth bed of the dump truck by the rope AB. (a) If ˛ D 25° , what is the tension in the rope? (b) If the rope will safely support a tension of 400 lb, what is the maximum allowable value of ˛? B A α Solution: Isolate the box. Resolve the forces into scalar components, and solve the equilibrium equations. A B The external forces are the weight, the tension in the rope, and the normal force exerted by the surface. The angle between the x axis and the weight vector is 90  ˛ (or 270 C ˛). The weight vector is α W D jWji sin ˛  j cos ˛ D 600i sin ˛  j cos ˛ The projections of the rope tension and the normal force are y T D jTx ji C 0j N D 0i C jNy jj T The equilibrium conditions are  x FDWCNCTD0 Substitute, and collect like terms   N W α Fx D 600 sin ˛  jTx ji D 0 Fy D 600 cos ˛ C jNy jj D 0 Solve for the unknown tension when For ˛ D 25° jTx j D 600 sin ˛ D 253.6 lb. For a tension of 400 lb, (600 sin ˛  400 D 0. Solve for the unknown angle sin ˛ D 400 D 0.667 or ˛ D 41.84° 600 c 2008 Pearson Education, Inc., Upper Saddle River, NJ. All rights reserved. This material is protected under all copyright laws as they  currently exist. No portion of this material may be reproduced, in any form or by any means, without permission in writing from the publisher. 91 A Problem 3.15 The 80-lb box is held in place on the smooth inclined surface by the rope AB. Determine the tension in the rope and the normal force exerted on the box by the inclined surface. 30⬚ B 50⬚ Solution: The equilibrium equations (in terms of a coordinate system with the x axis parallel to the inclined surface) are   Fx : 80 lb sin 50°  T cos 50 D 0 Fx : N  80 lb cos 50°  T sin 50 D 0 Solving: T D 95.34 lb, N D 124 lb Problem 3.16 The 1360-kg car and the 2100-kg tow truck are stationary. The muddy surface on which the car’s tires rest exerts negligible friction forces on them. What is the tension in the tow cable? 18⬚ 10⬚ 26⬚ Solution: FBD of the car being towed  F- : T cos 8°  13.34 kN sin 26° D 0 13.34 kN T 18° T D 5.91 kN 26° N 92 c 2008 Pearson Education, Inc., Upper Saddle River, NJ. All rights reserved. This material is protected under all copyright laws as they  currently exist. No portion of this material may be reproduced, in any form or by any means, without permission in writing from the publisher. Problem 3.17 Each box weighs 40 lb. The angles are measured relative to the horizontal. The surfaces are smooth. Determine the tension in the rope A and the normal force exerted on box B by the inclined surface. A B C 70⬚ 45⬚ D 20⬚ Solution: The free-body diagrams are shown. The equilibrium equations for box D are   Fx : 40 lb sin 20°  TC cos 25° D 0 Fy : ND  40 lb cos 20° C TC sin 25° D 0 The equilibrium equations for box B are   Fx : 40 lb sin 70° C TC cos 25°  TA D 0 Fy : NB  40 lb cos 70° C TC sin 25° D 0 Solving these four equations yields: TA D 51.2 lb, TC D 15.1 lb, NB D 7.30 lb, ND D 31.2 lb Thus TA D 51.2 lb, NB D 7.30 lb Problem 3.18 A 10-kg painting is hung with a wire supported by a nail. The length of the wire is 1.3 m. (a) (b) What is the tension in the wire? What is the magnitude of the force exerted on the nail by the wire? 1.2 m Solution: (a)  Fy : 98.1 N  2 98.1 N 5 TD0 13 T D 128 N T (b) T 5 Force D 98.1 N 12 12 c 2008 Pearson Education, Inc., Upper Saddle River, NJ. All rights reserved. This material is protected under all copyright laws as they  currently exist. No portion of this material may be reproduced, in any form or by any means, without permission in writing from the publisher. 93 Problem 3.19 A 10-kg painting is hung with a wire supported by two nails. The length of the wire is 1.3 m. (a) What is the tension in the wire? (b) What is the magnitude of the force exerted on each nail by the wire? (Assume that the tension is the same in each part of the wire.) 0.4 m 0.4 m 0.4 m Compare your answers to the answers to Problem 3.18. T Solution: (a) Examine the point on the left where the wire is attached to the picture. This point supports half of the weight  R 27.3° Fy : T sin 27.3°  49.05 N D 0 T D 107 N (b) 49.05 N Examine one of the nails  Fx : Rx  T cos 27.3° C T D 0  Ry Fy : Ry  T sin 27.3° D 0 Rx  27.3° RD Rx 2 C Ry 2 T T R D 50.5 N Problem 3.20 Assume that the 150-lb climber is in equilibrium. What are the tensions in the rope on the left and right sides? 15⬚ 14⬚ y Solution:   Fx D TR cos15°   TL cos14°  D 0   Fy D TR sin15°  C TL sin14°   150 D 0 14° TR TL 15° Solving, we get TL D 299 lb, TR D 300 lb x 150 lb 94 c 2008 Pearson Education, Inc., Upper Saddle River, NJ. All rights reserved. This material is protected under all copyright laws as they  currently exist. No portion of this material may be reproduced, in any form or by any means, without permission in writing from the publisher. Problem 3.21 If the mass of the climber shown in Problem 3.20 is 80 kg, what are the tensions in the rope on the left and right sides? y Solution:   Fx D TR cos15°   TL cos14°  D 0   Fy D TR sin15°  C TL sin14°   mg D 0 TR TL 14° 15° Solving, we get x TL D 1.56 kN, TR D 1.57 kN mg = (80) (9.81) N Problem 3.22 The construction worker exerts a 20-lb force on the rope to hold the crate in equilibrium in the position shown. What is the weight of the crate? 5⬚ 30⬚ Solution: The free-body diagram is shown. The equilibrium equations for the part of the rope system where the three ropes are joined are   Fx : 20 lb cos 30°  T sin 5° D 0 Fy : 20 lb sin 30° C T cos 5°  W D 0 Solving yields W D 188 lb c 2008 Pearson Education, Inc., Upper Saddle River, NJ. All rights reserved. This material is protected under all copyright laws as they  currently exist. No portion of this material may be reproduced, in any form or by any means, without permission in writing from the publisher. 95 Problem 3.23 A construction worker on the moon, where the acceleration due to gravity is 1.62 m/s2 , holds the same crate described in Problem 3.22 in the position shown. What force must she exert on the cable to hold the crate ub equilibrium (a) in newtons; (b) in pounds? 5⬚ 30⬚ Solution: The free-body diagram is shown. From Problem 3.22 we know that the weight is W D 188 lb. Therefore its mass is mD 188 lb D 5.84 slug 32.2 ft/s2  m D 5.84 slug 14.59 kg slug  D 85.2 kg The equilibrium equations for the part of the rope system where the three ropes are joined are   Fx : F cos 30°  T sin 5° D 0 Fy : F sin 30° C T cos 5°  mgm D 0 where gm D 1.62 m/s2 . Solving yields F D 3.30 lb D 14.7 N a F D 14.7 N, b F D 3.30 lb 96 c 2008 Pearson Education, Inc., Upper Saddle River, NJ. All rights reserved. This material is protected under all copyright laws as they  currently exist. No portion of this material may be reproduced, in any form or by any means, without permission in writing from the publisher. Problem 3.24 The person wants to cause the 200-lb crate to start sliding toward the right. To achieve this, the horizontal component of the force exerted on the crate by the rope must equal 0.35 times the normal force exerted on the crate by the floor. In Fig.a, the person pulls on the rope in the direction shown. In Fig.b, the person attaches the rope to a support as shown and pulls upward on the rope. What is the magnitude of the force he must exert on the rope in each case? 20⬚ (a) 10⬚ (b) Solution: The friction force Ffr is given by Ffr D 0.35N (a) For equilibrium we have   Fx : T cos 20°  0.35N D 0 Fy : T sin 20°  200 lb C N D 0 Solving: T D 66.1 lb (b) The person exerts the force F. Using the free-body diagram of the crate and of the point on the rope where the person grabs the rope, we find     Fx : TL  0.35N D 0 Fy : N  200 lb D 0 Fx : TL C TR cos 10° D 0 Fy : F  TR sin 10° D 0 Solving we find F D 12.3 lb c 2008 Pearson Education, Inc., Upper Saddle River, NJ. All rights reserved. This material is protected under all copyright laws as they  currently exist. No portion of this material may be reproduced, in any form or by any means, without permission in writing from the publisher. 97 Problem 3.25 A traffic engineer wants to suspend a 200-lb traffic light above the center of the two right lanes of a four-lane thoroughfare as shown. Determine the tensions in the cables AB and BC. Solution:   80 ft 20 ft A C 6 2 Fx :  p TAB C p TBC D 0 37 5 1 1 Fy : p TAB C p TBC  200 lb D 0 37 5 Solving: TAB D 304 lb, TBC D 335 lb 10 ft B TBC 30 ft 6 1 1 TAB 2 200 lb Problem 3.26 Cable AB is 3 m long and cable BC is 4 m long. The mass of the suspended object is 350 kg. Determine the tensions in cables AB and BC. 5m A C B Solution:   TAB TAC 3 4 Fx :  TAB C TBC D 0 5 5 4 4 3 Fy : TAB C TBC  3.43 kN D 0 5 5 4 3 3 TAB D 2.75 kN, TBC D 2.06 kN 3.43 kN 98 c 2008 Pearson Education, Inc., Upper Saddle River, NJ. All rights reserved. This material is protected under all copyright laws as they  currently exist. No portion of this material may be reproduced, in any form or by any means, without permission in writing from the publisher. Problem 3.27 In Problem 3.26, the length of cable AB is adjustable. If you don’t want the tension in either cable AB or cable BC to exceed 3 kN, what is the minimum acceptable length of cable AB? Solution: Consider the geometry: x 5−x We have the constraints LAB 2 D x 2 C y 2 , 4 m2 D 5 m  x2 C y 2 y LAB 4m These constraint imply yD  10 mx  x2  9 m2 TAB  L D 10 mx  9 m2 Now draw the FBD and write the equations in terms of x   x 5x TBC D 0 Fx :  p TAB C 4 10x  9 Fy : TBC y 4 y x 5−x p p 10x  x2  9 10x  x2  9 p TBC  3.43 kN D 0 TAB C 4 10x  9 If we set TAB D 3 kN and solve for x we find x D 1.535, TBC D 2.11 kN < 3 kN 3.43 kN Using this value for x we find that LAB D 2.52 m c 2008 Pearson Education, Inc., Upper Saddle River, NJ. All rights reserved. This material is protected under all copyright laws as they  currently exist. No portion of this material may be reproduced, in any form or by any means, without permission in writing from the publisher. 99 Problem 3.28 What are the tensions in the upper and lower cables? (Your answers will be in terms of W. Neglect the weight of the pulley.) 45° 30° W Solution: Isolate the weight. The frictionless pulley changes the TU direction but not the magnitude of the tension. The angle between the right hand upper cable and the x axis is ˛, hence TU β TUR D jTU ji cos ˛ C j sin ˛. α y The angle between the positive x and the left hand upper pulley is 180°  ˇ, hence TUL D jTU ji cos180  ˇ C j sin180  ˇ TL W x D jTU ji cos ˇ C j sin ˇ. The lower cable exerts a force: TL D jTL ji C 0j The weight: W D 0i  jWjj The equilibrium conditions are  F D W C TUL C TUR C TL D 0 Substitute and collect like terms,   Fx D jTU j cos ˇ C jTU j cos ˛  jTL ji D 0 Fy D jTU j sin ˛ C jTU j sin ˇ  jWjj D 0.  Solve: jTU j D jWj sin ˛ C sin ˇ  , jTL j D jTU jcos ˛  cos ˇ.  From which For jTL j D jWj cos ˛  cos ˇ sin ˛ C sin ˇ  . ˛ D 30° and ˇ D 45° jTU j D 0.828jWj, jTL j D 0.132jWj 100 c 2008 Pearson Education, Inc., Upper Saddle River, NJ. All rights reserved. This material is protected under all copyright laws as they  currently exist. No portion of this material may be reproduced, in any form or by any means, without permission in writing from the publisher. Problem 3.29 Two tow trucks lift a 660-lb motorcycle out of a ravine following an accident. If the motorcycle is in equilibrium in the position shown, what are the tensions in cables AB and AC? (36, 36) ft y (12, 32) ft C B (26, 16) ft A x Solution: The angles are  ˛ D tan1  ˇ D tan1 32  16 26  12 36  16 36  26  D 48.8°  D 63.4° Now from equilibrium we have   Fx : TAB cos ˛ C TAC cos ˇ D 0 Fy : TAB sin ˛ C TAC sin ˇ  660 lb D 0 Solving yields TAB D 319 lb, TAC D 470 lb c 2008 Pearson Education, Inc., Upper Saddle River, NJ. All rights reserved. This material is protected under all copyright laws as they  currently exist. No portion of this material may be reproduced, in any form or by any means, without permission in writing from the publisher. 101 Problem 3.30 An astronaut candidate conducts experiments on an airbearing platform. While she carries out calibrations, the platform is held in place by the horizontal tethers AB, AC, and AD. The forces exerted by the tethers are the only horizontal forces acting on the platform. If the tension in tether AC is 2 N, what are the tensions in the other two tethers? TOP VIEW D 4.0 m A 3.5 m B C 3.0 m Solution: Isolate the platform. The angles ˛ and ˇ are  tan ˛ D  Also, tan ˇ D 1.5 3.5 3.0 3.5  D 0.429, ˛ D 23.2° . B 3.0 m A  D 0.857, ˇ D 40.6° . C 3.5 m 4.0 m y B x TAB D jTAB ji cos180°  ˇ C j sin180°  ˇ β α TAB D jTAB ji cos ˇ C j sin ˇ. The angle between the tether AC and the positive x axis is 180° C ˛. The tension is D jTAC ji cos ˛  j sin ˛. C  Solve: jTAB j D  The tether AD is aligned with the positive x axis, TAD D jTAD ji C 0j. The equilibrium condition: F D TAD C TAB C TAC D 0. Substitute and collect like terms,  102 D A TAC D jTAC ji cos180° C ˛ C j sin180° C ˛  D 1.5 m The angle between the tether AB and the positive x axis is 180°  ˇ, hence  1.5 m jTAD j D sin ˛ sin ˇ  jTAC j, jTAC j sin˛ C ˇ sin ˇ  . For jTAC j D 2 N, ˛ D 23.2° and ˇ D 40.6° , jTAB j D 1.21 N, jTAD j D 2.76 N Fx D jTAB j cos ˇ  jTAC j cos ˛ C jTAD ji D 0, Fy D jTAB j sin ˇ  jTAC j sin ˛j D 0. c 2008 Pearson Education, Inc., Upper Saddle River, NJ. All rights reserved. This material is protected under all copyright laws as they  currently exist. No portion of this material may be reproduced, in any form or by any means, without permission in writing from the publisher. Problem 3.31 The bucket contains concrete and weighs 5800 lb. What are the tensions in the cables AB and AC? (5, 34) ft y B C (12, 16) ft (20, 34) ft A x Solution: The angles are  ˛ D tan1  ˇ D tan1 34  16 12  5 34  16 20  12  D 68.7°  D 66.0° Now from equilibrium we have   Fx : TAB cos ˛ C TAC cos ˇ D 0 Fy : TAB sin ˛ C TAC sin ˇ  660 lb D 0 Solving yields TAB D 319 lb, TAC D 470 lb Problem 3.32 The slider A is in equilibrium and the bar is smooth. What is the mass of the slider? 20⬚ 200 N A 45⬚ Solution: The pulley does not change the tension in the rope that passes over it. There is no friction between the slider and the bar. y T = 200 N 20° Eqns. of Equilibrium:   Fx D T sin 20° C N cos 45° D 0 T D 200 N   Fy D N sin 45° C T cos 20°  mg D 0 g D 9.81 m/s2 x Substituting for T and g, we have two eqns in two unknowns (N and m). Solving, we get N D 96.7 N, m D 12.2 kg. N 45° mg = (9.81) g c 2008 Pearson Education, Inc., Upper Saddle River, NJ. All rights reserved. This material is protected under all copyright laws as they  currently exist. No portion of this material may be reproduced, in any form or by any means, without permission in writing from the publisher. 103 Problem 3.33 The 20-kg mass is suspended from three cables. Cable AC is equipped with a turnbuckle so that its tension can be adjusted and a strain gauge that allows its tension to be measured. If the tension in cable AC is 40 N, what are the tensions in cables AB and AD? 0.4 m 0.4 m B 0.48 m C D 0.64 m A Solution: TAC TAB 5 TAC D 40 N  5 5 11 TAD D 0 Fx :  p TAB C p TAC C p 89 89 185 8 11 5  TAD 8 8 8 8 8 TAD  196.2 N D 0 Fy : p TAB C p TAC C p 89 89 185 Solving: TAB D 144.1 N, TAD D 68.2 N 196.2 N Problem 3.34 The structural joint is in equilibrium. If FA D 1000 lb and FD D 5000 lb, what are FB and FC ? FC 80⬚ FB 65⬚ 35⬚ FA FD Solution: The equilibrium equations are   Fx : FD  FC cos 65°  FB cos 35°  FA D 0 Fy : FC sin 65° C FB sin 35° D 0 Solving yields FB D 3680 lb, FC D 2330 lb 104 c 2008 Pearson Education, Inc., Upper Saddle River, NJ. All rights reserved. This material is protected under all copyright laws as they  currently exist. No portion of this material may be reproduced, in any form or by any means, without permission in writing from the publisher. Problem 3.35 The collar A slides on the smooth vertical bar. The masses mA D 20 kg and mB D 10 kg. When h D 0.1 m, the spring is unstretched. When the system is in equilibrium, h D 0.3 m. Determine the spring constant k. 0.25 m h A B k Solution: The triangles formed by the rope segments and the horizontal line level with A can be used to determine the lengths Lu and Ls . The equations are Lu D  0.252 C 0.12 and Ls D  0.252 C 0.32 . The stretch in the spring when in equilibrium is given by υ D Ls  Lu . Carrying out the calculations, we get Lu D 0.269 m, Ls D 0.391 m, and υ D 0.121 m. The angle, , between the rope at A and the horizontal when the system is in equilibrium is given by tan  D 0.3/0.25, or  D 50.2° . From the free body diagram for mass A, we get two equilibrium equations. They are  and  T NA A mA g Fx D NA C T cos  D 0 T Fy D T sin   mA g D 0. We have two equations in two unknowns and can solve. We get NA D 163.5 N and T D 255.4 N. Now we go to the free body diagram for B, where the equation of equilibrium is T  mB g  kυ D 0. This equation has only one unknown. Solving, we get k D 1297 N/m Lu 0.1 m B mBg Kδ 0.25 m Ls Lu 0.3 m 0.25 m c 2008 Pearson Education, Inc., Upper Saddle River, NJ. All rights reserved. This material is protected under all copyright laws as they  currently exist. No portion of this material may be reproduced, in any form or by any means, without permission in writing from the publisher. 105 Problem 3.36* Suppose that you want to design a cable system to suspend an object of weight W from the ceiling. The two wires must be identical, and the dimension b is fixed. The ratio of the tension T in each wire to its cross-sectional area A must equal a specified value T/A D . The “cost” of your design ispthe total volume of material in the two wires, V D 2A b2 C h2 . Determine the value of h that minimizes the cost. b b h W Solution: From the equation  T T θ θ Fy D 2T sin   W D 0, we obtain T D p W W b2 C h2 D . 2 sin  2h Since T/A D , A D p W b2 C h2 T D  2h W p Wb2 C h2  . and the “cost” is V D 2A b2 C h2 D h To determine the value of h that minimizes V, we set dV W b2 C h2  D C2 D0  dh  h2 and solve for h, obtaining h D b. 106 c 2008 Pearson Education, Inc., Upper Saddle River, NJ. All rights reserved. This material is protected under all copyright laws as they  currently exist. No portion of this material may be reproduced, in any form or by any means, without permission in writing from the publisher. Problem 3.37 The system of cables suspends a 1000-lb bank of lights above a movie set. Determine the tensions in cables AB, CD, and CE. 20 ft 18 ft B D Solution: Isolate juncture A, and solve the equilibrium equations. C Repeat for the cable juncture C. E The angle between the cable AC and the positive x axis is ˛. The tension in AC is TAC D jTAC ji cos ˛ C j sin ˛ 45° 30° A The angle between the x axis and AB is 180°  ˇ. The tension is TAB D jTAB ji cos180  ˇ C j sin180  ˇ TAB D i cos ˇ C j sin ˇ. The weight is W D 0i  jWjj. The equilibrium conditions are  Solve: jTCE j D jTCA j cos ˛, F D 0 D W C TAB C TAC D 0. jTCD j D jTCA j sin ˛; Substitute and collect like terms,   for jTCA j D 732 lb and ˛ D 30° , Fx D jTAC j cos ˛  jTAB j cos ˇi D 0 jTAB j D 896.6 lb, Fy D jTAB j sin ˇ C jTAC j sin ˛  jWjj D 0. jTCE j D 634 lb, Solving, we get  jTAB j D cos ˛ cos ˇ   and jTAC j jTAC j D jWj cos ˇ sin˛ C ˇ jTCD j D 366 lb  , jWj D 1000 lb, and ˛ D 30° , ˇ D 45°  jTAC j D 1000  jTAB j D 732 0.7071 0.9659 0.866 0.7071 B C A β  α D 732.05 lb y  x W D 896.5 lb Isolate juncture C. The angle between the positive x axis and the cable CA is 180°  ˛. The tension is D TCA D jTCA ji cos180° C ˛ C j sin180° C ˛, C 90° E or TCA D jTCA ji cos ˛  j sin ˛. The tension in the cable CE is α y A TCE D ijTCE j C 0j. x The tension in the cable CD is TCD D 0i C jjTCD j. The equilibrium conditions are  F D 0 D TCA C TCE C TCD D 0 Substitute t and collect like terms,   Fx D jTCE j  jTCA j cos ˛i D 0, Fy D jTCD j  jTCA j sin ˛j D 0. c 2008 Pearson Education, Inc., Upper Saddle River, NJ. All rights reserved. This material is protected under all copyright laws as they  currently exist. No portion of this material may be reproduced, in any form or by any means, without permission in writing from the publisher. 107 Problem 3.38 Consider the 1000-lb bank of lights in Problem 3.37. A technician changes the position of the lights by removing the cable CE. What is the tension in cable AB after the change? Solution: The original configuration in Problem 3.35 is used to solve for the dimensions and the angles. Isolate the juncture A, and solve the equilibrium conditions. 18 ft 20 ft D B C The lengths are calculated as follows: The vertical interior distance in the triangle is 20 ft, since the angle is 45 deg. and the base and altitude of a 45 deg triangle are equal. The length AB is given by α β A AB D 20 ft D 28.284 ft. cos 45° The length AC is given by AC D 18 ft D 20.785 ft. cos 30° 38 B The altitude of the triangle for which AC is the hypotenuse is 18 tan 30° D 10.392 ft. The distance CD is given by 20  10.392 D 9.608 ft. D β α β 20.784+9.608 = 30.392 α 28.284 The distance AD is given by A AD D AC C CD D 20.784 C 9.608 D 30.392 B The new angles are given by the cosine law AB2 D 382 C AD2 D β  238AD cos ˛. A α Reduce and solve:  cos ˛ D C 30.3922   28.2842 23830.392  cos ˇ D 382 28.2842 C 382  30.3922 228.28438 y D 0.6787, ˛ D 47.23° .  D 0.6142, ˇ D 52.1° . Isolate the juncture A. The angle between the cable AD and the positive x axis is ˛. The tension is:  Solve: jTAB j D and jTAD j D  TAD D jTAD ji cos ˛ C j sin ˛. The angle between x and the cable AB is 180°  ˇ. The tension is TAB D jTAB ji cos ˇ C j sin ˇ. The weight is W D 0i  jWjj F D 0 D W C TAB C TAD D 0. cos ˛ cos ˇ  jTAD j, jWj cos ˇ sin˛ C ˇ  . For jWj D 1000 lb, and ˛ D 51.2° , ˇ D 47.2°  jTAD j D 1000 The equilibrium conditions are  x W 0.6142 0.989  jTAB j D 622.3  0.6787 0.6142 D 621.03 lb,  D 687.9 lb Substitute and collect like terms,   108 Fx D jTAD j cos ˛  jTAB j cos ˇi D 0, Fy D jTAB j sin ˇ C jTAD j sin ˛  jWjj D 0. c 2008 Pearson Education, Inc., Upper Saddle River, NJ. All rights reserved. This material is protected under all copyright laws as they  currently exist. No portion of this material may be reproduced, in any form or by any means, without permission in writing from the publisher. Problem 3.39 While working on another exhibit, a curator at the Smithsonian Institution pulls the suspended Voyager aircraft to one side by attaching three horizontal cables as shown. The mass of the aircraft is 1250 kg. Determine the tensions in the cable segments AB, BC, and CD. D C B 30° 50° Solution: Isolate each cable juncture, beginning with A and solve the equilibrium equations at each juncture. The angle between the cable AB and the positive x axis is ˛ D 70° ; the tension in cable AB is TAB D jTAB ji cos ˛ C j sin ˛. The weight is W D 0i  jWjj. The tension in cable AT is T D jTji C 0j. The equilibrium conditions are  A 70° F D W C T C TAB D 0. Substitute and collect like terms   Fx jTAB j cos ˛  jTji D 0, Fy D jTAB j sin ˛  jWjj D 0. y  Solve: the tension in cable AB is jTAB j D For jWj D 1250 kg 9.81  jTAB j D 12262.5 0.94 m s2  jWj . sin ˛ D 12262.5 N and ˛ D 70° B x α A T  D 13049.5 N W Isolate juncture B. The angles are ˛ D 50° , ˇ D 70° , and the tension cable BC is TBC D jTBC ji cos ˛ C j sin ˛. The angle between the cable BA and the positive x axis is 180 C ˇ; the tension is y C x TBA D jTBA ji cos180 C ˇ C j sin180 C ˇ The tension in the left horizontal cable is T D jTji C 0j. The equilibrium conditions are  β A F D TBA C TBC C T D 0. Substitute and collect like terms   α B T D jTBA ji cos ˇ  j sin ˇ y T Fy D jTBC j sin ˛  jTBA j sin ˇj D 0.  Solve: jTBC j D sin ˇ sin ˛ D x Fx D jTBC j cos ˛  jTBA j cos ˇ  jTji D 0 α C  β jTBA j. B For jTBA j D 13049.5 N, and ˛ D 50° , ˇ D 70° ,  jTBC j D 13049.5 0.9397 0.7660  D 16007.6 N Isolate the cable juncture C. The angles are ˛ D 30° , ˇ D 50° . By symmetry with the cable juncture B above, the tension in cable CD is  jTCD j D sin ˇ sin ˛  jTCB j.  Substitute: jTCD j D 16007.6 0.7660 0.5  D 24525.0 N. This completes the problem solution. c 2008 Pearson Education, Inc., Upper Saddle River, NJ. All rights reserved. This material is protected under all copyright laws as they  currently exist. No portion of this material may be reproduced, in any form or by any means, without permission in writing from the publisher. 109 Problem 3.40 A truck dealer wants to suspend a 4000kg truck as shown for advertising. The distance b D 15 m, and the sum of the lengths of the cables AB and BC is 42 m. Points A and C are at the same height. What are the tensions in the cables? 40 m b A C B Solution: Determine the dimensions and angles of the cables. Iso- 15 m late the cable juncture B, and solve the equilibrium conditions. The dimensions of the triangles formed by the cables: L D 25 m, L b A b D 15 m, 25 m β AB C BC D S D 42 m. C α Subdivide into two right triangles with a common side of unknown length. Let the unknown length of this common side be d, then by the Pythagorean Theorem b2 C d2 D AB2 , L2 C d2 D BC2 . B y Subtract the first equation from the second to eliminate the unknown d, L 2  b2 D BC2  AB2 . A α B β C Note that BC2  AB2 D BC  ABBC C AB. W Substitute and reduce to the pair of simultaneous equations in the unknowns   x BC  AB D L 2  b2 S Solve:   2  1 L  b2 CS 2 S BC D D , BC C AB D S Substitute and collect like terms    2  1 25  152 C 42 D 25.762 m 2 42  Fx D jTBC j cos ˛  jTBA j cos ˇi D 0, Fy D jTBC j sin ˛ C jTBA j sin ˇ  jWjj D 0  and AB D S  BC D 42  25.762 D 16.238 m. Solve: jTBC j D The interior angles are found from the cosine law:  cos ˛ D  cos ˇ D L C b2 C BC2  2L C bBC  AB2 L C b2 C AB2  BC2 2L C bAB  and jTBA j D D 0.9704 ˛ D 13.97° cos ˇ cos ˛  jTBA j, jWj cos ˛ sin˛ C ˇ  . For jWj D 40009.81 D 39240 N,  D 0.9238 ˇ D 22.52° Isolate cable juncture B. The angle between BC and the positive x axis is ˛; the tension is and ˛ D 13.97° , ˇ D 22.52° , jTBA j D 64033 D 64 kN, jTBC j D 60953 D 61 kN TBC D jTBC ji cos ˛ C j sin ˛ The angle between BA and the positive x axis is 180°  ˇ; the tension is TBA D jTBA ji cos180  ˇ C j sin180  ˇ D jTBA ji cos ˇ C j sin ˇ. The weight is W D 0i  jWjj. The equilibrium conditions are  110 F D W C TBA C TBC D 0. c 2008 Pearson Education, Inc., Upper Saddle River, NJ. All rights reserved. This material is protected under all copyright laws as they  currently exist. No portion of this material may be reproduced, in any form or by any means, without permission in writing from the publisher. Problem 3.41 The distance h D 12 in, and the tension in cable AD is 200 lb. What are the tensions in cables AB and AC? B 12 in. A D C 12 in. h 8 in. 12 in. Solution: Isolated the cable juncture. From the sketch, the angles are found from  tan ˛ D  tan ˇ D 8 12 4 12  D 0.667 8 in. y B 12 in α ˛ D 33.7° 8 in A D  D 0.333 ˇ D 18.4° β 4 in C The angle between the cable AB and the positive x axis is 180°  ˛, the tension in AB is: x TAB D jTAB ji cos180  ˛ C j sin180  ˛ TAB D jTAB ji cos ˛ C j sin ˛. The angle between AC and the positive x axis is 180 C ˇ. The tension is TAC D jTAC ji cos180 C ˇ C j sin180 C ˇ TAC D jTAC ji cos ˇ  j sin ˇ. The tension in the cable AD is TAD D jTAD ji C 0j. The equilibrium conditions are  F D TAC C TAB C TAD D 0. Substitute and collect like terms,   Fx D jTAB j cos ˛  jTAC j cos ˇ C jTAD ji D 0 Fy D jTAB j sin ˛  jTAC j sin ˇj D 0.  Solve: jTAB j D  and jTAC j D sin ˇ sin ˛  jTAC j, sin ˛ sin˛ C ˇ  jTAD j. For jTAD j D 200 lb, ˛ D 33.7° , ˇ D 18.4° jTAC j D 140.6 lb, jTAB j D 80.1 lb c 2008 Pearson Education, Inc., Upper Saddle River, NJ. All rights reserved. This material is protected under all copyright laws as they  currently exist. No portion of this material may be reproduced, in any form or by any means, without permission in writing from the publisher. 111 Problem 3.42 You are designing a cable system to support a suspended object of weight W. Because your design requires points A and B to be placed as shown, you have no control over the angle ˛, but you can choose the angle ˇ by placing point C wherever you wish. Show that to minimize the tensions in cables AB and BC, you must choose ˇ D ˛ if the angle ˛ ½ 45° . B A Strategy: Draw a diagram of the sum of the forces exerted by the three cables at A. W Solution: Draw the free body diagram of the knot at point A. Then y TAC draw the force triangle involving the three forces. Remember that ˛ is fixed and the force W has both fixed magnitude and direction. From the force triangle, we see that the force TAC can be smaller than TAB for a large range of values for ˇ. By inspection, we see that the minimum simultaneous values for TAC and TAB occur when the two forces are equal. This occurs when ˛ D ˇ. Note: this does not happen when ˛ < 45° . TAB and  B α x A W In this case, we solved the problem without writing the equations of equilibrium. For reference, these equations are:  C β α B Possible locations for C lie on line C? C? α TAB Fx D TAB cos ˛ C TAC cos ˇ D 0 Fy D TAB sin ˛ C TAC sin ˇ  W D 0. Candidate β W Candidate values for TAC Fixed direction for line AB Problem 3.43* The length of the cable ABC is 1.4 m. The 2-kN force is applied to a small pulley. The system is stationary. What is the tension in the cable? 1m A C B 0.75 m 15⬚ Solution: Examine the geometry  h2 C 0.75 m2 C tan ˛ D )  0.75 m 2 kN 0.25 m β α h2 C 0.25 m2 D 1.4 m h h h , tan ˇ D 0.75 m 0.25 m h D 0.458 m, ˛ D 31.39° , ˇ D 61.35° Now draw a FBD and solve for the tension. We can use either of the equilibrium equations  Fx : T cos ˛ C T cos ˇ C 2 kN sin 15° D 0 T T β α T D 1.38 kN 2 kN 15° 112 c 2008 Pearson Education, Inc., Upper Saddle River, NJ. All rights reserved. This material is protected under all copyright laws as they  currently exist. No portion of this material may be reproduced, in any form or by any means, without permission in writing from the publisher. Problem 3.44 The masses m1 D 12 kg and m2 D 6 kg are suspended by the cable system shown. The cable BC is horizontal. Determine the angle ˛ and the tensions in the cables AB, BC, and CD. TAB A D α TBC α B B C 70⬚ m1 m2 117.7 N Solution: We have 4 unknowns and 4 equations     TCD FBx : TAB cos ˛ C TBC D 0 FBy : TAB sin ˛  117.7 N D 0 FCx : TBC C TCD cos 70° D 0 70° TBC C FCy : TCD sin 70°  58.86 N D 0 Solving we find ˛ D 79.7° , TAB D 119.7 N, TBC D 21.4 N, TCD D 62.6 N Problem 3.45 The weights W1 D 50 lb and W2 are suspended by the cable system shown. Determine the weight W2 and the tensions in the cables AB, BC, and CD. 58.86 N 30 in 30 in 30 in A D 16 in 20 in C B W2 W1 Solution: We have 4 unknowns and 4 equilibrium equations to use     3 15 TBC D 0 FBx :  p TAB C p 229 13 TAB 2 15 2 3 2 2 TBC  50 lb D 0 FBy : p TAB C p 229 13 TBC B 15 15 TBC C TCD D 0 FCx :  p 17 229 50 lb 2 8 TBC C TCD  W2 D 0 FCy :  p 17 229 TCD W2 D 25 lb, TAB D 75.1 lb 8 ) C TBC D 63.1 lb, TCD D 70.8 lb TBC 15 15 2 W2 c 2008 Pearson Education, Inc., Upper Saddle River, NJ. All rights reserved. This material is protected under all copyright laws as they  currently exist. No portion of this material may be reproduced, in any form or by any means, without permission in writing from the publisher. 113 Problem 3.46 In the system shown in Problem 3.45, assume that W2 D W1 /2. If you don’t want the tension anywhere in the supporting cable to exceed 200 lb, what is the largest acceptable value of W1 ? TAB Solution:     3 15 TBC D 0 FBx :  p TAB C p 229 13 2 15 2 3 2 2 TBC  W1 D 0 FBy : p TAB C p 229 13 TBC B 15 15 TBC C TCD D 0 FCx :  p 17 229 W1 2 8 W1 TCD  D0 TBC C FCy :  p 17 2 229 TCD TAB D 1.502W1 , TBC D 1.262W1 , TCD D 1.417W1 8 C AB is the critical cable 200 lb D 1.502W1 ) W1 D 133.2 lb TBC 15 2 15 W2 = W1/2 114 c 2008 Pearson Education, Inc., Upper Saddle River, NJ. All rights reserved. This material is protected under all copyright laws as they  currently exist. No portion of this material may be reproduced, in any form or by any means, without permission in writing from the publisher. Problem 3.47 The hydraulic cylinder is subjected to three forces. An 8-kN force is exerted on the cylinder at B that is parallel to the cylinder and points from B toward C. The link AC exerts a force at C that is parallel to the line from A to C. The link CD exerts a force at C that is parallel to the line from C to D. (a) (b) Draw the free-body diagram of the cylinder. (The cylinder’s weight is negligible). Determine the magnitudes of the forces exerted by the links AC and CD. Solution: From the figure, if C is at the origin, then points A, B, and D are located at 1m D C Hydraulic cylinder 1m 0.6 m B A 0.15 m 0.6 m Scoop y A0.15, 0.6 B0.75, 0.6 D FCD D1.00, 0.4 and forces FCA , FBC , and FCD are parallel to CA, BC, and CD, respectively. C x FBC We need to write unit vectors in the three force directions and express the forces in terms of magnitudes and unit vectors. The unit vectors are given by eCA D rCA D 0.243i  0.970j jrCA j eCB D rCB D 0.781i  0.625j jrCB j eCD D rCD D 0.928i C 0.371j jrCD j FCA A B Now we write the forces in terms of magnitudes and unit vectors. We can write FBC as FCB D 8eCB kN or as FCB D 8eCB  kN (because we were told it was directed from B toward C and had a magnitude of 8 kN. Either way, we must end up with FCB D 6.25i C 5.00j kN Similarly, FCA D 0.243FCA i  0.970FCA j FCD D 0.928FCD i C 0.371FCD j For equilibrium, FCA C FCB C FCD D 0 In component form, this gives   Fx D 0.243FCA C 0.928FCD  6.25 (kN) D 0   F D 0.970F C 0.371F C 5.00 (kN) D 0 y CA CD Solving, we get FCA D 7.02 kN, FCD D 4.89 kN c 2008 Pearson Education, Inc., Upper Saddle River, NJ. All rights reserved. This material is protected under all copyright laws as they  currently exist. No portion of this material may be reproduced, in any form or by any means, without permission in writing from the publisher. 115 Problem 3.48 The 50-lb cylinder rests on two smooth surfaces. (a) (b) Draw the free-body diagram of the cylinder. If ˛ D 30° , what are the magnitudes of the forces exerted on the cylinder by the left and right surfaces? α Solution: Isolate the cylinder. (a) The free body diagram of the 45° y isolated cylinder is shown. (b) The forces acting are the weight and the normal forces exerted by the surfaces. The angle between the normal force on the right and the x axis is 90 C ˇ. The normal force is β α NL NR D jNR ji cos90 C ˇ C j sin90 C ˇ NR W x NR D jNR ji sin ˇ C j cos ˇ. The angle between the positive x axis and the left hand force is normal 90  ˛; the normal force is NL D jNL ji sin ˛ C j cos ˛. The weight is W D 0i  jWjj. The equilibrium conditions are   jNR j D  F D W C NR C NL D 0. and jNL j D Substitute and collect like terms,   Solve: sin ˛ sin ˇ  jNL j, jWj sin ˇ sin˛ C ˇ  . For jWj D 50 lb, and ˛ D 30° , ˇ D 45° , the normal forces are Fx D jNR j sin ˇ C jNL j sin ˛i D 0, jNL j D 36.6 lb, jNR j D 25.9 lb Fy D jNR j cos ˇ C jNL j cos ˛  jWjj D 0. Problem 3.49 For the 50-lb cylinder in Problem 3.48, obtain an equation for the force exerted on the cylinder by the left surface in terms of the angle ˛ in two ways: (a) using a coordinate system with the y axis vertical, (b) using a coordinate system with the y axis parallel to the right surface. y Solution: The solution for Part (a) is given in Problem 3.48 (see free body diagram).  jNR j D sin ˛ sin ˇ   jNL j jNL j D jWj sin ˇ sin˛ C ˇ . Part (b): The isolated cylinder with the coordinate system is shown. The angle between the right hand normal force and the positive x axis is 180° . The normal force: NR D jNR ji C 0j. The angle between the left hand normal force and the positive x is 180  ˛ C ˇ. The normal force is NL D jNL ji cos˛ C ˇ C j sin˛ C ˇ. The angle between the weight vector and the positive x axis is ˇ. The weight vector is W D jWji cos ˇ  j sin ˇ. The equilibrium conditions are  NR NL W x Substitute and collect like terms,   Fx D jNR j  jNL j cos˛ C ˇ C jWj cos ˇi D 0, Fy D jNL j sin˛ C ˇ  jWj sin ˇj D 0.  F D W C NR C NL D 0. Solve: 116 β α  jNL j D jWj sin ˇ sin˛ C ˇ  c 2008 Pearson Education, Inc., Upper Saddle River, NJ. All rights reserved. This material is protected under all copyright laws as they  currently exist. No portion of this material may be reproduced, in any form or by any means, without permission in writing from the publisher. Problem 3.50 The two springs are identical, with unstretched length 0.4 m. When the 50-kg mass is suspended at B, the length of each spring increases to 0.6 m. What is the spring constant k? 0.6 m A C k k B Solution: F F F D k0.6 m  0.4 m  Fy : 2F sin 60°  490.5 N D 0 60° 60° k D 1416 N/m 490.5 N Problem 3.51 The cable AB is 0.5 m in length. The unstretched length of the spring is 0.4 m. When the 50-kg mass is suspended at B, the length of the spring increases to 0.45 m. What is the spring constant k? 0.7 m A C k B Solution: The Geometry 0.7 m Law of Cosines and Law of Sines φ θ 0.72 D 0.52 C 0.452  20.50.45 cos ˇ sin  sin  sin ˇ D D 0.45 m 0.5 m 0.7 m 0.5 m 0.45 m β ˇ D 94.8° ,  D 39.8°  D 45.4° Now do the statics TAB F F D k0.45 m  0.4 m   Fx : TAB cos  C F cos  D 0 θ φ Fy : TAB sin  C F sin   490.5 N D 0 Solving: k D 7560 N/m 490.5 N c 2008 Pearson Education, Inc., Upper Saddle River, NJ. All rights reserved. This material is protected under all copyright laws as they  currently exist. No portion of this material may be reproduced, in any form or by any means, without permission in writing from the publisher. 117 Problem 3.52 The small sphere of mass m is attached to a string of length L and rests on the smooth surface of a fixed sphere of radius R. The center of the sphere is directly below the point where the string is attached. Obtain an equation for the tension in the string in terms of m, L, h, and R. L h m R Solution: From the geometry we have cos  D x RChy , sin  D L L cos  D x y , sin  D R R Thus the equilibrium equations can be written   x x Fx :  T C N D 0 L R Fy : RChy y T C N  mg D 0 L R Solving, we find T D mgL RCh Problem 3.53 The inclined surface is smooth. Determine the force T that must be exerted on the cable to hold the 100-kg crate in equilibrium and compare your answer to the answer of Problem 3.11. T 60⬚ 3T Solution:  981 N F- : 3 T  981 N sin 60° D 0 T D 283 N N 60° 118 c 2008 Pearson Education, Inc., Upper Saddle River, NJ. All rights reserved. This material is protected under all copyright laws as they  currently exist. No portion of this material may be reproduced, in any form or by any means, without permission in writing from the publisher. Problem 3.54 In Example 3.3, suppose that the mass of the suspended object is mA and the masses of the pulleys are mB D 0.3mA , mC D 0.2mA , and mD D 0.2mA . Show that the force T necessary for the system to be in equilibrium is 0.275mA g. D C B T A Solution: From the free-body diagram of pulley C TD TD  2T  mC g D 0 ) TD D 2T C mC g Then from the free-body diagram of pulley B D C mg T C T C 2T C mC g  mB g  mA g D 0 T Thus TD TD C 1 mA C mB  mC g 4 T D 0.275mA g TD ⫽ 2T ⫹ mg B 1 mA C 0.3mA  0.2mA g D 0.275mA g 4 T (a) T T T A B mg mAg (b) c 2008 Pearson Education, Inc., Upper Saddle River, NJ. All rights reserved. This material is protected under all copyright laws as they  currently exist. No portion of this material may be reproduced, in any form or by any means, without permission in writing from the publisher. 119 Problem 3.55 The mass of each pulley of the system is m and the mass of the suspended object A is mA . Determine the force T necessary for the system to be in equilibrium. Solution: Draw free body diagrams of each pulley and the object A. Each pulley and the object A must be in equilibrium. The weights of the pulleys and object A are W D mg and WA D mA g. The equilibrium equations for the weight A, the lower pulley, second pulley, third pulley, and the top pulley are, respectively, B  WA D 0, 2C  B  W D 0, 2D  C  W D 0, 2T  D  W D 0, and FS  2T  W D 0. Begin with the first equation and solve for B, substitute for B in the second equation and solve for C, substitute for C in the third equation and solve for D, and substitute for D in the fourth equation and solve for T, to get T in terms of W and WA . The result is T A Fs W T W B D WA , DD CD WA W C , 2 2 3W WA 7W WA C , and T D C , 4 4 8 8 or in terms of the masses, TD 120 g mA C 7m. 8 T T T W D D D C W C C B B WA c 2008 Pearson Education, Inc., Upper Saddle River, NJ. All rights reserved. This material is protected under all copyright laws as they  currently exist. No portion of this material may be reproduced, in any form or by any means, without permission in writing from the publisher. Problem 3.56 The suspended mass m1 D 50 kg. Neglecting the masses of the pulleys, determine the value of the mass m2 necessary for the system to be in equilibrium. A B C m2 m1 Solution:   T1 T T FC : T1 C 2m2 g  m1 g D 0 FB : T1  2m2 g D 0 C m2 D m1 D 12.5 kg 4 T1 m1 g B T = m2 g T c 2008 Pearson Education, Inc., Upper Saddle River, NJ. All rights reserved. This material is protected under all copyright laws as they  currently exist. No portion of this material may be reproduced, in any form or by any means, without permission in writing from the publisher. 121 Problem 3.57 The boy is lifting himself using the block and tackle shown. If the weight of the block and tackle is negligible, and the combined weight of the boy and the beam he is sitting on is 120 lb, what force does he have to exert on the rope to raise himself at a constant rate? (Neglect the deviation of the ropes from the vertical.) Solution: A free-body diagram can be obtained by cutting the four ropes between the two pulleys of the block and tackle and the rope the boy is holding. The tension has the same value T in all five of these ropes. So the upward force on the free-body diagram is 5T and the downward force is the 120-lb weight. Therefore the force the boy must exert is T D 120 lb/5 D 24 lb T D 24 lb 122 c 2008 Pearson Education, Inc., Upper Saddle River, NJ. All rights reserved. This material is protected under all copyright laws as they  currently exist. No portion of this material may be reproduced, in any form or by any means, without permission in writing from the publisher. Problem 3.58 Pulley systems containing one, two, and three pulleys are shown. Neglecting the weights of the pulleys, determine the force T required to support the weight W in each case. T T T W (a) One pulley W (b) Two pulleys W (c) Three pulleys Solution: (a) (b)    (b) For two pulleys T Fy : 2T  W D 0 ) T D Fupper : 2T  T1 D 0 Flower : 2T1  W D 0 T1 W TD 4 (c)    T W 2 T1 Fupper : 2T  T1 D 0 Fmiddle : 2T1  T2 D 0 W Flower : 2T2  W D 0 (c) For three pulleys TD T W 8 T (a) For one pulleys T T T1 T1 W T2 T2 T2 W c 2008 Pearson Education, Inc., Upper Saddle River, NJ. All rights reserved. This material is protected under all copyright laws as they  currently exist. No portion of this material may be reproduced, in any form or by any means, without permission in writing from the publisher. 123 Problem 3.59 Problem 3.58 shows pulley systems containing one, two, and three pulleys. The number of pulleys in the type of system shown could obviously be extended to an arbitrary number N. Solution: By extrapolation of the previous problem Neglecting the weights of the pulleys, determine the force T required to support the weight W as a function of the number of pulleys N in the system. (b) Using the result of part (a), determine the force T required to support the weight W for a system with 10 pulleys. (a) TD W 2N (b) TD W 1024 (a) Problem 3.60 A 14,000-kg airplane is in steady flight in the vertical plane. The flight path angle is  D 10° , the angle of attack is ˛ D 4° , and the thrust force exerted by the engine is T D 60 kN. What are the magnitudes of the lift and drag forces acting on the airplane? (See Example 3.4). Solution: Let us draw a more detailed free body diagram to see the angles involved more clearly. Then we will write the equations of equilibrium and solve them. y L W D mg D 14,0009.81 N The equilibrium equations are x   Fx D T cos ˛  D  W sin  D 0   Fy D T sin ˛ C L  W cos  D 0 α α = 4° γ = 10° T γ D W T D 60 kN D 60000 N Solving, we get γ D D 36.0 kN, L D 131.1 kN y Path x T γ L α D Horizon W 124 c 2008 Pearson Education, Inc., Upper Saddle River, NJ. All rights reserved. This material is protected under all copyright laws as they  currently exist. No portion of this material may be reproduced, in any form or by any means, without permission in writing from the publisher. Problem 3.61 An airplane is in steady flight, the angle of attack ˛ D 0, the thrust-to-drag ratio T/D D 2, and the lift-to-drag ratio L/D D 4. What is the flight path angle ? (See Example 3.4). Solution: Use the same strategy as in Problem 3.52. The angle between the thrust vector and the positive x axis is ˛, T D jTji cos ˛ C j sin ˛ The lift vector: L D 0i C jLjj The drag: D D jDji C 0j. The angle between the weight vector and the positive x axis is 270  ; W D jWji sin   j cos . The equilibrium conditions are  F D T C L C D C W D 0. Substitute and collect like terms  and  Fx D jTj cos ˛  jDj  jWj sin i D 0, Fy D jTj sin ˛ C jLj  jWj cos j D 0 Solve the equations for the terms in : jWj sin  D jTj cos ˛  jDj, and jWj cos  D jTj sin ˛ C jLj. Take the ratio of the two equations  tan  D jTj cos ˛  jDj jTj sin ˛ C jLj  . Divide top and bottom on the right by jDj. For ˛ D 0, jLj jTj D 2, D 4, jDj jDj  tan  D 21 4  D 1 or  D 14° 4 y Path T L α D x γ Horizontal W c 2008 Pearson Education, Inc., Upper Saddle River, NJ. All rights reserved. This material is protected under all copyright laws as they  currently exist. No portion of this material may be reproduced, in any form or by any means, without permission in writing from the publisher. 125 Problem 3.62 An airplane glides in steady flight (T D 0), and its lift-to-drag ratio is L/D D 4. (a) What is the flight path angle ? (b) If the airplane glides from an altitude of 1000 m to zero altitude, what horizontal distance does it travel? (See Example 3.4.) Solution: See Example 3.4. The angle between the thrust vector y and the positive x axis is ˛: Path x T T D jTji cos ˛ C j sin ˛. L α The lift vector: L D 0i C jLjj. The drag: D D jDji C 0j. The angle between the weight vector and the positive x axis is 270  : γ Horizontal D W W D jWji sin   j cos . The equilibrium conditions are  F D T C L C D C W D 0. γ Substitute and collect like terms:   1 km Fx D jTj cos ˛  jDj  jWj sin i D 0 γ Fy D jTj sin ˛ C jLj  jWj cos j D 0 h Solve the equations for the terms in , jWj sin  D jTj cos ˛  jDj, and jWj cos  D jTj sin ˛ C jLj Part (a): Take the ratio of the two equilibrium equations:  tan  D jTj cos ˛  jDj jTj sin ˛ C jLj  . Divide top and bottom on the right by jDj. For ˛ D 0, jTj D 0, jLj D 4, jDj  tan  D 1 4   D 14° Part (b): The flight path angle is a negative angle measured from the horizontal, hence from the equality of opposite interior angles the angle  is also the positive elevation angle of the airplane measured at the point of landing. tan  D 126 1 , h hD 1 1 D   D 4 km 1 tan  4 c 2008 Pearson Education, Inc., Upper Saddle River, NJ. All rights reserved. This material is protected under all copyright laws as they  currently exist. No portion of this material may be reproduced, in any form or by any means, without permission in writing from the publisher. Problem 3.63 In Active Example 3.5, suppose that the attachment point B is moved to the point (5,0,0) m. What are the tensions in cables AB, AC, and AD? C (⫺2, 0, ⫺2) m y x B (4, 0, 2) m (⫺3, 0, 3) m D Solution: The position vector from point A to point B can be used to write the force TAB . A (0, ⫺4, 0) m z rAB D 5i C 4j m TAB D TAB rAB D TAB 0.781i C 0.625j jrAB j 100 kg Using the other forces from Active Example 3.5, we have    Fx : 0.781TAB  0.408TAC  0.514TAD D 0 Fy : 0.625TAB C 0.816TAC C 0.686TAD  981 N D 0 Fz : 0.408TAC C 0.514TAD D 0 Solving yields TAB D 509 N, TAC D 487 N, TAD D 386 N y Problem 3.64 The force F D 800i C 200j (lb) acts at point A where the cables AB, AC, and AD are joined. What are the tensions in the three cables? F D (0, 6, 0) ft A (12, 4, 2) ft C B (0, 4, 6) ft (6, 0, 0) ft x z Solution: We first write the position vectors rAB D 6i  4j  2k ft rAC D 12i C 6k ft rAD D 12i C 2j  2k ft Now we can use these vectors to define the force vectors TAB D TAB rAB D TAB 0.802i  0.535j  0.267k jrAB j TAC D TAC rAC D TAC 0.949i C 0.316k jrAC j TAD D TAD rAD D TAD 0.973i C 0.162j  0.162k jrAD j The equilibrium equations are then    Fx : 0.802TAB  0.949TAC  0.973TAD C 800 lb D 0 Fy : 0.535TAB C 0.162TAD C 200 lb D 0 Fz : 0.267TAB C 0.316TAC  0.162TAD D 0 Solving, we find TAB D 405 lb, TAC D 395 lb, TAD D 103 lb c 2008 Pearson Education, Inc., Upper Saddle River, NJ. All rights reserved. This material is protected under all copyright laws as they  currently exist. No portion of this material may be reproduced, in any form or by any means, without permission in writing from the publisher. 127 y Problem 3.65 Suppose that you want to apply a 1000-lb force F at point A in a direction such that the resulting tensions in cables AB, AC, and AD are equal. Determine the components of F. F D (0, 6, 0) ft A (12, 4, 2) ft C B (0, 4, 6) ft (6, 0, 0) ft x z Solution: We first write the position vectors rAB D 6i  4j  2k ft rAC D 12i C 6k ft rAD D 12i C 2j  2k ft Now we can use these vectors to define the force vectors TAB D T rAB D T0.802i  0.535j  0.267k jrAB j TAC D T rAC D T0.949i C 0.316k jrAC j TAD D T rAD D T0.973i C 0.162j  0.162k jrAD j The force F can be written F D Fx i C Fy j C Fz k The equilibrium equations are then    Fx : 0.802T  0.949T  0.973T C Fx D 0 ) Fx D 2.72T Fy : 0.535T C 0.162T C Fy D 0 ) Fy D 0.732T Fz : 0.267T C 0.316T  0.162T C Fz D 0 ) Fz D 0.113T  We also have the constraint equation ) T D 363 lb Fx 2 C Fy 2 C Fz 2 D 1000 lb Solving, we find Fx D 990 lb, Fy D 135 lb, Fz D 41.2 lb 128 c 2008 Pearson Education, Inc., Upper Saddle River, NJ. All rights reserved. This material is protected under all copyright laws as they  currently exist. No portion of this material may be reproduced, in any form or by any means, without permission in writing from the publisher. Problem 3.66 The 10-lb metal disk A is supported by the smooth inclined surface and the strings AB and AC. The disk is located at coordinates (5,1,4) ft. What are the tensions in the strings? y B (0, 6, 0) ft C (8, 4, 0) ft Solution: The position vectors are 2 ft rAB D 5i C 5j  4k ft A x 8 ft rAC D 3i C 3j  4k ft z 10 ft The angle ˛ between the inclined surface the horizontal is ˛ D tan1 2/8 D 14.0° We identify the following force: TAB D TAB rAB D TAB 0.615i C 0.615j  0.492k jrAB j TAC D TAC rAC D TAC 0.514i C 0.514j  0.686k jrAC j N D Ncos ˛j C sin ˛k D N0.970j C 0.243k W D 10 lbj The equilibrium equations are then    Fx : 0.615TAB C 0.514TAC D 0 Fy : 0.615TAB C 0.514TAC C 0.970N  10 lb D 0 Fz : 0.492TAB  0.686TAC C 0.243N D 0 Solving, we find N D 8.35 lb TAB D 1.54 lb, TAC D 1.85 lb c 2008 Pearson Education, Inc., Upper Saddle River, NJ. All rights reserved. This material is protected under all copyright laws as they  currently exist. No portion of this material may be reproduced, in any form or by any means, without permission in writing from the publisher. 129 Problem 3.67 The bulldozer exerts a force F D 2i (kip) at A. What are the tensions in cables AB, AC, and AD? y 6 ft C 8 ft 2 ft Solution: Isolate the cable juncture. Express the tensions in terms of unit vectors. Solve the equilibrium equations. The coordinates of points A, B, C, D are: A8, 0, 0, B0, 3, 8, C0, 2, 6, B A 3 ft D0, 4, 0. D z The radius vectors for these points are 4 ft rA D 8i C 0j C 0k, rB D 0i C 3j C 8k, rC D 0i C 2j  6k, rD D 0i C 4j C 0k. 8 ft x By definition, the unit vector parallel to the tension in cable AB is eAB D rB  r A . jrB  rA j Carrying out the operations for each of the cables, the results are: eAB D 0.6835i C 0.2563j C 0.6835k, eAC D 0.7845i C 0.1961j  0.5883k, eAD D 0.8944i  0.4472j C 0k. The tensions in the cables are expressed in terms of the unit vectors, TAB D jTAB jeAB , TAC D jTAC jeAC , TAD D jTAD jeAD . The external force acting on the juncture is F D 2000i C 0j C 0k. The equilibrium conditions are  F D 0 D TAB C TAC C TAD C F D 0. Substitute the vectors into the equilibrium conditions:    Fx D 0.6835jTAB j 0.7845jTAC j 0.8944jTAD jC2000i D 0 Fy D 0.2563jTAB j C 0.1961jTAC j  0.4472jTAD jj D 0 Fz D 0.6835jTAB j  0.5883jTAC j C 0jTAD jk D 0 The commercial program TK Solver Plus was used to solve these equations. The results are jTAB j D 780.31 lb , 130 jTAC j D 906.49 lb , jTAD j D 844.74 lb . c 2008 Pearson Education, Inc., Upper Saddle River, NJ. All rights reserved. This material is protected under all copyright laws as they  currently exist. No portion of this material may be reproduced, in any form or by any means, without permission in writing from the publisher. y Problem 3.68 Prior to its launch, a balloon carrying a set of experiments to high altitude is held in place by groups of student volunteers holding the tethers at B, C, and D. The mass of the balloon, experiments package, and the gas it contains is 90 kg, and the buoyancy force on the balloon is 1000 N. The supervising professor conservatively estimates that each student can exert at least a 40-N tension on the tether for the necessary length of time. Based on this estimate, what minimum numbers of students are needed at B, C, and D? A (0, 8, 0) m C (10,0, –12) m D (–16, 0, 4) m x B (16, 0, 16) m z Solution:  1000 N Fy D 1000  909.81  T D 0 T D 117.1 N (90) g A0, 8, 0 B16, 0, 16 T C10, 0, 12 D16, 0, 4 We need to write unit vectors eAB , eAC , and eAD . y T eAB D 0.667i  0.333j C 0.667k (0, 8, 0) eAC D 0.570i  0.456j  0.684k A FAC eAD D 0.873i  0.436j C 0.218k FAD We now write the forces in terms of magnitudes and unit vectors  FAB D 0.667FAB i  0.333FAB j C 0.667FAB k      FAC D 0.570FAC i  0.456FAC j  0.684FAC k  FAD D 0.873FAD i  0.436FAC j C 0.218FAC k     T D 117.1j (N) C (10, 0, −12) m D x (−16, 0, 4) z B (16, 0, 16) m The equations of equilibrium are    Fx D 0.667FAB C 0.570FAC  0.873FAD D 0 Fy D 0.333FAB  0.456FAC  0.436FAC C 117.1 D 0 Fz D 0.667FAB  0.684FAC C 0.218FAC D 0 Solving, we get FAB D 64.8 N ¾ 2 students FAC D 99.8 N ¾ 3 students FAD D 114.6 N ¾ 3 students c 2008 Pearson Education, Inc., Upper Saddle River, NJ. All rights reserved. This material is protected under all copyright laws as they  currently exist. No portion of this material may be reproduced, in any form or by any means, without permission in writing from the publisher. 131 Problem 3.69 The 20-kg mass is suspended by cables attached to three vertical 2-m posts. Point A is at (0, 1.2, 0) m. Determine the tensions in cables AB, AC, and AD. y C B D A 1m 1m 2m 0.3 m x z Solution: Points A, B, C, and D are located at A0, 1.2, 0, B0.3, 2, 1, C0, 2, 1, D2, 2, 0 y C FAC B FAB FAD Write the unit vectors eAB , eAC , eAD D A eAB D 0.228i C 0.608j C 0.760k W eAC D 0i C 0.625j  0.781k eAD D 0.928i C 0.371j C 0k z (20) (9.81) N x The forces are FAB D 0.228FAB i C 0.608FAB j C 0.760FAB k FAC D 0FAC i C 0.625FAC j  0.781FAC k FAD D 0.928FAD i C 0.371FAD j C 0k W D 209.81j The equations of equilibrium are   Fx D 0.228FAB C 0 C 0.928FAD D 0     Fy D 0.608FAB C 0.625FAC C 0.371FAD  209.81 D 0       Fz D 0.760FAB  0.781FAC C 0 D 0 We have 3 eqns in 3 unknowns solving, we get FAB D 150.0 N FAC D 146.1 N FAD D 36.9 N 132 c 2008 Pearson Education, Inc., Upper Saddle River, NJ. All rights reserved. This material is protected under all copyright laws as they  currently exist. No portion of this material may be reproduced, in any form or by any means, without permission in writing from the publisher. Problem 3.70 The weight of the horizontal wall section is W D 20,000 lb. Determine the tensions in the cables AB, AC, and AD. Solution: Set the coordinate origin at A with axes as shown. The upward force, T, at point A will be equal to the weight, W, since the cable at A supports the entire wall. The upward force at A is T D W k. From the figure, the coordinates of the points in feet are A D 10 ft A4, 6, 10, B0, 0, 0, C12, 0, 0, and 7 ft D4, 14, 0. The three unit vectors are of the form 6 ft C B 4 ft xI  xA i C yI  yA j C zI  zA k , eAI D  xI  xA 2 C yI  yA 2 C zI  zA 2 14 ft 8 ft W where I takes on the values B, C, and D. The denominators of the unit vectors are the distances AB, AC, and AD, respectively. Substitution of the coordinates of the points yields the following unit vectors: T z eAB D 0.324i  0.487j  0.811k, A y TD eAC D 0.566i  0.424j  0.707k, 10 ft TB D 7 ft TC and eAD D 0i C 0.625j  0.781k. 6 ft The forces are TAB D TAB eAB , 14 ft 4 ft TAC D TAC eAC , and TAD D TAD eAD . C B X 8 ft W The equilibrium equation for the knot at point A is T C TAB C TAC C TAD D 0. From the vector equilibrium equation, write the scalar equilibrium equations in the x, y, and z directions. We get three linear equations in three unknowns. Solving these equations simultaneously, we get TAB D 9393 lb, TAC D 5387 lb, and TAD D 10,977 lb A D 10 ft 6 ft C B 4 ft 8 ft 7 ft 14 ft W c 2008 Pearson Education, Inc., Upper Saddle River, NJ. All rights reserved. This material is protected under all copyright laws as they  currently exist. No portion of this material may be reproduced, in any form or by any means, without permission in writing from the publisher. 133 Problem 3.71 The car in Fig. a and the pallet supporting it weigh 3000 lb. They are supported by four cables AB, AC, AD, and AE. The locations of the attachment points on the pallet are shown in Fig. b. The tensions in cables AB and AE are equal. Determine the tensions in the cables. y A (0, 10, 0) ft Solution: Isolate the knot at A. Let TAB , TAC , TAD and TAE be the forces exerted by the tensions in the cables. The force exerted by the vertical cable is (3000 lb)j. We first find the position vectors and then express all of the forces as vectors. rAB D 5i  10j C 5k ft rAC D 6i  10j  5k ft E C rAD D 8i  10j  4k ft z rAE D 6i  10j C 5k ft B TAB rAB D TAB 0.408i  0.816j C 0.408k D TAB jrAB j x (a) TAC D TAC rAC D TAC 0.473i  0.788j  0.394k jrAC j TAD rAD D TAD D TAD 0.596i  0.745j  0.298k jrAD j TAE rAE D TAE 0.473i  0.788j C 0.394k D TAE jrAE j 8 ft 6 ft C D 5 ft 4 ft x The equilibrium equations are    5 ft Fx : 0.408TAB C 0.473TAC  0.596TAD  0.473TAE D 0 E Fy : 0.816TAB  0.788TAC  0.745TAD  0.788TAE C 3000 lb D 0 B 6 ft 5 ft z Fz : 0.408TAB  0.394TAC  0.298TAD C 0.394TAE D 0 (b) Solving, we find TAB D 896 lb, TAC D 1186 lb, TAD D 843 lb, TAE D 896 lb 134 c 2008 Pearson Education, Inc., Upper Saddle River, NJ. All rights reserved. This material is protected under all copyright laws as they  currently exist. No portion of this material may be reproduced, in any form or by any means, without permission in writing from the publisher. Problem 3.72 The 680-kg load suspended from the helicopter is in equilibrium. The aerodynamic drag force on the load is horizontal. The y axis is vertical, and cable OA lies in the x-y plane. Determine the magnitude of the drag force and the tension in cable OA. y A 10° O x B C D y Solution:   TOA Fx D TOA sin 10°  D D 0, Fy D TOA cos 10°  6809.81 D 0. Solving, we obtain D D 1176 N, TOA D 6774 N. 10° D x (680) (9.81) N c 2008 Pearson Education, Inc., Upper Saddle River, NJ. All rights reserved. This material is protected under all copyright laws as they  currently exist. No portion of this material may be reproduced, in any form or by any means, without permission in writing from the publisher. 135 Problem 3.73 In Problem 3.72, the coordinates of the three cable attachment points B, C, and D are (3.3, 4.5, 0) m, (1.1, 5.3, 1) m, and (1.6, 5.4, 1) m, respectively. What are the tensions in cables OB, OC, and OD? Solution: The position vectors from O to pts B, C, and D are rOB D 3.3i  4.5j (m), rOC D 1.1i  5.3j C k (m), rOD D 1.6i  5.4j  k (m). Dividing by the magnitudes, we obtain the unit vectors eOB D 0.591i  0.806j, eOC D 0.200i  0.963j C 0.182k, eOD D 0.280i  0.944j  0.175k. Using these unit vectors, we obtain the equilibrium equations    Fx D TOA sin 10°  0.591TOB C 0.200TOC C 0.280TOD D 0, Fy D TOA cos 10°  0.806TOB  0.963TOC  0.944TOD D 0, Fz D 0.182TOC  0.175TOD D 0. From the solution of Problem 3.72, TOA D 6774 N. Solving these equations, we obtain TOB D 3.60 kN, TOC D 1.94 kN, TOD D 2.02 kN. y TOA 10° x TOB TOC 136 TOD c 2008 Pearson Education, Inc., Upper Saddle River, NJ. All rights reserved. This material is protected under all copyright laws as they  currently exist. No portion of this material may be reproduced, in any form or by any means, without permission in writing from the publisher. Problem 3.74 If the mass of the bar AB is negligible compared to the mass of the suspended object E, the bar exerts a force on the “ball” at B that points from A toward B. The mass of the object E is 200 kg. The y-axis points upward. Determine the tensions in the cables BC and CD. y (0, 4, ⫺3) m C B (4, 3, 1) m D (0, 5, 5) m Strategy: Draw a free-body diagram of the ball at B. (The weight of the ball is negligible.) x A E z Solution:  FAB D FAB 4i  3j  k p 26   , TBC D TBC 4i C j  4k p 33  , The forces  TBD D TBD 4i C 2j C 4k 6  , W D 200 kg9.81 m/s2 j The equilibrium equations    4 4 4 Fx :  p FAB  p TBC  TBD D 0 6 26 33 3 1 2 Fy :  p FAB C p TBC C TBD  1962 N D 0 6 26 33 1 4 4 Fz :  p FAB  p TBC C TBD D 0 6 26 33 TBC D 1610 N ) TBD D 1009 N c 2008 Pearson Education, Inc., Upper Saddle River, NJ. All rights reserved. This material is protected under all copyright laws as they  currently exist. No portion of this material may be reproduced, in any form or by any means, without permission in writing from the publisher. 137 Problem 3.75* The 3400-lb car is at rest on the plane surface. The unit vector en D 0.456i C 0.570j C 0.684k is perpendicular to the surface. Determine the magnitudes of the total normal force N and the total friction force f exerted on the surface by the car’s wheels. y en Solution: The forces on the car are its weight, the normal force, and the friction force. The normal force is in the direction of the unit vector, so it can be written N D Nen D N0.456i C 0.570j C 0.684k The equilibrium equation is x z Nen C f  3400 lbj D 0 The friction force f is perpendicular to N, so we can eliminate the friction force from the equilibrium equation by taking the dot product of the equation with en . Nen C f  3400 lbj Ð en D N  3400 lbj Ð en  D 0 N D 3400 lb0.57 D 1940 lb Now we can solve for the friction force f. f D 3400 lbj  Nen D 3400 lbj  1940 lb0.456i C 0.570j C 0.684k f D 884i C 2300j  1330k lb jfj D  884 lb2 C 2300 lb2 C 1330 lb2 D 2790 lb jNj D 1940 lb, jfj D 2790 lb 138 c 2008 Pearson Education, Inc., Upper Saddle River, NJ. All rights reserved. This material is protected under all copyright laws as they  currently exist. No portion of this material may be reproduced, in any form or by any means, without permission in writing from the publisher. y Problem 3.76 The system shown anchors a stanchion of a cable-suspended roof. If the tension in cable AB is 900 kN, what are the tensions in cables EF and EG? G (0, 1.4, ⫺1.2) m Solution: Using the coordinates for the points we find E F rBA D [3.4  2i C 1  1j C 0  0k] m (2, 1, 0) m (1, 1.2, 0) m rBA D 1.4 mi eBA rBA Di D jrBA j (3.4, 1, 0) m B (0, 1.4, 1.2) m A (2.2, 0, ⫺1) m Using the same procedure we find the other unit vectors that we need. D eBC D 0.140i  0.700j C 0.700k z x (2.2, 0, 1) m C eBD D 0.140i  0.700j C 0.700k eBE D 0.981i C 0.196j eEG D 0.635i C 0.127j  0.762k eEF D 0.635i C 0.127j C 0.762k We can now write the equilibrium equations for the connections at B and E. 900 kNeBA C TBC eBC C TBD eBD C TBE eBE D 0, TBE eBE  C TEF eEF C TEG eEG 0 Breaking these equations into components, we have the following six equations to solve for five unknows (one of the equations is redundant). 900 kN C TBC 0.140 C TBD 0.140 C TBE 0.981 D 0 TBC 0.700 C TBD 0.700 C TBE 0.196 D 0 TBC 0.700 C TBD 0.700 D 0 TBE 0.981 C TEG 0.635 C TEF 0.635 D 0 TBE 0.196 C TEG 0.127 C TEF 0.127 D 0 TEG 0.726 C TEF 0.726 D 0 Solving, we find TBC D TBD D 134 kN, TBE D 956 kN TEF D TEG D 738 kN y Problem 3.77* The cables of the system will each safely support a tension of 1500 kN. Based on this criterion, what is the largest safe value of the tension in cable AB? Solution: From Problem 3.76 we know that if the tension in AB is 900 kN, then the largest force in the system occurs in cable BE and that tension is 956 kN. To solve this problem, we can just scale the results from Problem 3.76   1500 kN 900 kN TAB D 1410 kN TAB D 956 kN G (0, 1.4, ⫺1.2) m F E (2, 1, 0) m (1, 1.2, 0) m (3.4, 1, 0) m B (0, 1.4, 1.2) m A (2.2, 0, ⫺1) m D z C x (2.2, 0, 1) m c 2008 Pearson Education, Inc., Upper Saddle River, NJ. All rights reserved. This material is protected under all copyright laws as they  currently exist. No portion of this material may be reproduced, in any form or by any means, without permission in writing from the publisher. 139 y Problem 3.78 The 200-kg slider at A is held in place on the smooth vertical bar by the cable AB. 2m (a) Determine the tension in the cable. (b) Determine the force exerted on the slider by the bar. B A 5m 2m x 2m z Solution: The coordinates of the points A, B are A2, 2, 0, B0, 5, 2. The vector positions rA D 2i C 2j C 0k, T rB D 0i C 5j C 2k N The equilibrium conditions are:  F D T C N C W D 0. W Eliminate the slider bar normal force as follows: The bar is parallel to the y axis, hence the unit vector parallel to the bar is eB D 0i C 1j C 0k. The dot product of the unit vector and the normal force vanishes: eB Ð N D 0. Take the dot product of eB with the equilibrium conditions: eB Ð N D 0.  eB Ð F D eB Ð T C eB Ð W D 0. The weight is eB Ð W D 1j Ð jjWj D jWj D 2009.81 D 1962 N. Note: For this specific configuration, the problem can be solved without eliminating the slider bar normal force, since it does not appear in the y-component of the equilibrium equation (the slider bar is parallel to the y-axis). However, in the general case, the slider bar will not be parallel to an axis, and the unknown normal force will be projected onto all components of the equilibrium equations (see Problem 3.79 below). In this general situation, it will be necessary to eliminate the slider bar normal force by some procedure equivalent to that used above. End Note. The unit vector parallel to the cable is by definition, eAB D rB  r A . jrB  rA j Substitute the vectors and carry out the operation: eAB D 0.4851i C 0.7278j C 0.4851k. (a) The tension in the cable is T D jTjeAB . Substitute into the modified equilibrium condition  eB F D 0.7276jTj  1962 D 0. Solve: jTj D 2696.5 N from which the tension vector is T D jTjeAB D 1308i C 1962j C 1308k. (b) The equilibrium conditions are  F D 0 D T C N C W D 1308i C 1308k C N D 0. Solve for the normal force: N D 1308i  1308k. The magnitude is jNj D 1850 N. 140 c 2008 Pearson Education, Inc., Upper Saddle River, NJ. All rights reserved. This material is protected under all copyright laws as they  currently exist. No portion of this material may be reproduced, in any form or by any means, without permission in writing from the publisher. Problem 3.79 In Example 3.6, suppose that the cable AC is replaced by a longer one so that the distance from point B to the slider C increases from 6 ft to 8 ft. Determine the tension in the cable. Solution: The vector from B to C is now y B 4 ft A 6 ft rBC D 8 ft eBD  rBC D 8 ft 7 4 4 i jC k 9 9 9  7 ft C rBC D 3.56i  6.22j C 3.56k ft O x We can now find the unit vector form C to A. rCA D rOA  rOB C rBC  D [7j C 4k z 4 ft D 4 ft  f7j C 3.56i  6.22j C 3.56kg] ft rCA D 3.56i C 6.22j C 0.444k ft eCA D rCA D 0.495i C 0.867j C 0.0619k jrCA j Using N to stand for the normal force between the bar and the slider, we can write the equilibrium equation: TeCA C N  100 lbj D 0 We can use the dot product to eliminate N from the equation [TeCA C N  100 lbj] Ð eBD D TeCA Ð eBD   100 lbj Ð eBD  D 0  T  7 4 4 [0.495] C  [0.867] C [0.0619]  100 lb0.778 D 0 9 9 9 T0.867 C 77.8 lb D 0 ) T D 89.8 lb T D 89.8 lb c 2008 Pearson Education, Inc., Upper Saddle River, NJ. All rights reserved. This material is protected under all copyright laws as they  currently exist. No portion of this material may be reproduced, in any form or by any means, without permission in writing from the publisher. 141 Problem 3.80 The cable AB keeps the 8-kg collar A in place on the smooth bar CD. The y axis points upward. What is the tension in the cable? y 0.15 m 0.4 m B Solution: We develop the following position vectors and unit C vectors rCD D 0.2i  0.3j C 0.25k m 0.2 m 0.3 m A 0.5 m eCD D rCD D 0.456i  0.684j C 0.570k jrCD j O x 0.25 m D 0.2 m rCA D 0.2 meCD D 0.091i  0.137j C 0.114k m z rAB D rOB  rOC C rCA  rAB D [0.5j C 0.15k  f0.4i C 0.3jg C f0.091i  0.137j C 0.114kg] m rAB D 0.309i C 0.337j C 0.036k m eAB D rAB D 0.674i C 0.735j C 0.079k jrAB j We can now write the equilibrium equation for the slider using N to stand for the normal force between the slider and the bar CD. TeAB C N  8 kg9.81 m/s2 j D 0 To eliminate the normal force N we take a dot product with eCD . [TeAB C N  8 kg9.81 m/s2 j] Ð eCD D 0 TeAB Ð eCD   78.5 Nj Ð eCD  D 0 T[0.674][0.456] C [0.735][0.684] C [0.079][0.570]  78.5 N0.684 D 0 T0.150 C 53.6 N D 0 T D 357 N Problem 3.81 Determine the magnitude of the normal force exerted on the collar A by the smooth bar. y 0.15 m Solution: From Problem 3.81 we have 0.4 m B C eAB D 0.674i C 0.735j C 0.079k T D 357 N 0.2 m 0.3 m A 0.5 m The equilibrium equation is O TeAB C N  78.5 Nj D 0 D We can now solve for the normal force N. x 0.25 m 0.2 m z N D 78.5 Nj  357 N0.674i C 0.735j C 0.079k N D 240i  184j  28.1k N The magnitude of N is jNj D  240 N2 C 184 N2 C 28.1 N2 jNj D 304 N 142 c 2008 Pearson Education, Inc., Upper Saddle River, NJ. All rights reserved. This material is protected under all copyright laws as they  currently exist. No portion of this material may be reproduced, in any form or by any means, without permission in writing from the publisher. y Problem 3.82* The 10-kg collar A and 20-kg collar B are held in place on the smooth bars by the 3-m cable from A to B and the force F acting on A. The force F is parallel to the bar. Determine F. (0, 5, 0) m (0, 3, 0) m F Solution: The geometry is the first part of the Problem. To ease our work, let us name the points C, D, E, and G as shown in the figure. The unit vectors from C to D and from E to G are essential to the location of points A and B. The diagram shown contains two free bodies plus the pertinent geometry. The unit vectors from C to D and from E to G are given by A 3m B (4, 0, 0) m eCD D erCDx i C eCDy j C eCDz k, (0, 0, 4) m z and eEG D erEGx i C eEGy j C eEGz k. y Using the coordinates of points C, D, E, and G from the picture, the unit vectors are D (0, 5, 0) m eCD D 0.625i C 0.781j C 0k, yA D yC C CAeCDy , B mBg z yB D yA C ABeABy , A TAB NB and zA D zC C CAeCDz , xB D xA C ABeABx , NA TAB The location of point A is given by where CA D 3 m. From these equations, we find that the location of point A is given by A (2.13, 2.34, 0) m. Once we know the location of point A, we can proceed to find the location of point B. We have two ways to determine the location of B. First, B is 3 m from point A along the line AB (which we do not know). Also, B lies on the line EG. The equations for the location of point B based on line AB are: F G (0, 3, 0) m and eEG D 0i C 0.6j C 0.8k. xA D xC C CAeCDx , mAg 3m C (4, 0, 0) m We now have two fewer equation than unknowns. Fortunately, there are two conditions we have not yet invoked. The bars at A and B are smooth. This means that the normal force on each bar can have no component along that bar. This can be expressed by using the dot product of the normal force and the unit vector along the bar. The two conditions are NA Ð eCD D NAx eCDx C NAy eCDy C NAz eCDz D 0 The equations based on line EG are: for slider A and yB D yE C EBeEGy , and zB D zE C EBeEGz . We have six new equations in the three coordinates of B and the distance EB. Some of the information in the equations is redundant. However, we can solve for EB (and the coordinates of B). We get that the length EB is 2.56 m and that point B is located at (0, 1.53, 1.96) m. We next write equilibrium equations for bodies A and B. From the free body diagram for A, we get NAx C TAB eABx C FeCDx D 0, x E (0, 0, 4) m and zB D zA C ABeABz . xB D xE C EBeEGx , x NB Ð eEG D NBx eEGx C NBy eEGy C NBz eEGz D 0. Solving the eight equations in the eight unknowns, we obtain F D 36.6 N . Other values obtained in the solution are EB D 2.56 m, NAx D 145 N, NBx D 122 N, NAy D 116 N, NBy D 150 N, NAz D 112 N, and NBz D 112 N. NAy C TAB eABy C FeCDy  mA g D 0, and NAz C TAB eABz C FeCDz D 0. From the free body diagram for B, we get NBx  TAB eABx D 0, Nby  TAB eABy  mB g D 0, and NBz  TAB eABz D 0. c 2008 Pearson Education, Inc., Upper Saddle River, NJ. All rights reserved. This material is protected under all copyright laws as they  currently exist. No portion of this material may be reproduced, in any form or by any means, without permission in writing from the publisher. 143 Problem 3.83 The 100-lb crate is held in place on the smooth surface by the rope AB. Determine the tension in the rope and the magnitude of the normal force exerted on the crate by the surface. A 45⬚ B 30⬚ Solution: The free-body diagram is sketched. The equilibrium equations are   Fx - : T cos45°  30°   100 lb sin 30° D 0 F% : T sin45°  30°   100 lb cos 30° C N D 0 Solving, we find T D 51.8 lb, N D 73.2 lb Problem 3.84 The system shown is called Russell’s traction. If the sum of the downward forces exerted at A and B by the patient’s leg is 32.2 lb, what is the weight W? y 60⬚ 20⬚ 25⬚ B A W x Solution: The force in the cable is W everywhere. The free-body diagram of the leg is shown. The downward force is given, but the horizontal force FH is unknown. The equilibrium equation in the vertical direction is  Fy : W sin 25° C W sin 60°  32.2 lb D 0 Thus WD 32.2 lb sin 25° C sin 60° W D 25.0 lb 144 c 2008 Pearson Education, Inc., Upper Saddle River, NJ. All rights reserved. This material is protected under all copyright laws as they  currently exist. No portion of this material may be reproduced, in any form or by any means, without permission in writing from the publisher. Problem 3.85 The 400-lb engine block is suspended by the cables AB and AC. If you don’t want either TAB or TAC to exceed 400 lb, what is the smallest acceptable value of the angle ˛? y TAB B TAC C a A a A x 400 lb Solution: The equilibrium equations are   Fx : TAB cos ˛ C TAC cos ˛ D 0 Fy : TAB sin ˛ C TAC sin ˛  400 lb D 0 Solving, we find TAB D TAC D 400 lb 2 sin ˛ If we limit the tensions to 400 lb, we have 400 lb D 400 lb 1 ) sin ˛ D ) ˛ D 30° 2 sin ˛ 2 Problem 3.86 The cable AB is horizontal, and the box on the right weighs 100 lb. The surface are smooth. (a) What is the tension in the cable? (b) What is the weight of the box on the left? A B 20⬚ 40⬚ Solution: We have the following equilibrium equations     FyB : NB cos 40°  100 lb D 0 FxB : NB sin 40°  T D 0 FxA : T  NA sin 20° D 0 FyA : NA cos 20°  WA D 0 Solving these equations sequentially, we find NB D 131 lb, T D 83.9 lb NA D 245 lb, WA D 230.5 lb Thus we have T D 83.9 lb, WA D 230.5 lb c 2008 Pearson Education, Inc., Upper Saddle River, NJ. All rights reserved. This material is protected under all copyright laws as they  currently exist. No portion of this material may be reproduced, in any form or by any means, without permission in writing from the publisher. 145 Problem 3.87 Assume that the forces exerted on the 170-lb climber by the slanted walls of the “chimney” are perpendicular to the walls. If he is in equilibrium and is exerting a 160-lb force on the rope, what are the magnitudes of the forces exerted on him by the left and right walls? 10⬚ 4⬚ 3⬚ Solution: The forces in the free-body diagram are in the directions shown on the figure. The equilibrium equations are:   Fx : T sin 10° C NL cos 4°  NR cos 3° D 0 Fy : T cos 10°  170 lb C NL sin 40° C NR sin 3° D 0 where T D 160 lb. Solving we find NL D 114 lb, NR D 85.8 lb Left Wall: 114 lb Right Wall: 85.8 lb 146 c 2008 Pearson Education, Inc., Upper Saddle River, NJ. All rights reserved. This material is protected under all copyright laws as they  currently exist. No portion of this material may be reproduced, in any form or by any means, without permission in writing from the publisher. Problem 3.88 The mass of the suspended object A is mA and the masses of the pulleys are negligible. Determine the force T necessary for the system to be in equilibrium. T A Solution: Break the system into four free body diagrams as shown. Carefully label the forces to ensure that the tension in any single cord is uniform. The equations of equilibrium for the four objects, starting with the leftmost pulley and moving clockwise, are: S  3T D 0, R  3S D 0, F F  3R D 0, R and 2T C 2S C 2R  mA g D 0. We want to eliminate S, R, and F from our result and find T in terms of mA and g. From the first two equations, we get S D 3T, and R D 3S D 9T. Substituting these into the last equilibrium equation results in 2T C 23T C 29T D mA g. R R R S S S Solving, we get T D mA g/26 . S T T S S R R T T T A mAg Note: We did not have to solve for F to find the appropriate value of T. The final equation would give us the value of F in terms of mA and g. We would get F D 27mA g/26. If we then drew a free body diagram of the entire assembly, the equation of equilibrium would be F  T  mA g D 0. Substituting in the known values for T and F, we see that this equation is also satisfied. Checking the equilibrium solution by using the “extra” free body diagram is often a good procedure. c 2008 Pearson Education, Inc., Upper Saddle River, NJ. All rights reserved. This material is protected under all copyright laws as they  currently exist. No portion of this material may be reproduced, in any form or by any means, without permission in writing from the publisher. 147 Problem 3.89 The assembly A, including the pulley, weighs 60 lb. What force F is necessary for the system to be in equilibrium? F A Solution: From the free body diagram of the assembly A, we have 3F  60 D 0, or F D 20 lb F F F F F F F 60 lb. 148 c 2008 Pearson Education, Inc., Upper Saddle River, NJ. All rights reserved. This material is protected under all copyright laws as they  currently exist. No portion of this material may be reproduced, in any form or by any means, without permission in writing from the publisher. Problem 3.90 The mass of block A is 42 kg, and the mass of block B is 50 kg. The surfaces are smooth. If the blocks are in equilibrium, what is the force F? B F 45⬚ A 20⬚ Solution: Isolate the top block. Solve the equilibrium equations. The weight is. The angle between the normal force N1 and the positive x axis is. The normal force is. The force N2 is. The equilibrium conditions are  from which   Solve: y B N2 F D N1 C N2 C W D 0 N1 Fx D 0.7071jN1 j  jN2 ji D 0 Fy D 0.7071jN1 j  490.5j D 0. N1 D 693.7 N, W α x y N1 jN2 j D 490.5 N Isolate the bottom block. The weight is F β α A W D 0i  jWjj D 0i  429.81j D 0i  412.02j (N). The angle between the normal force N1 and the positive x axis is 270°  45°  D 225° . x N3 W The normal force: N1 D jN1 ji cos 225° C j sin 225°  D jN1 j0.7071i  0.7071j. The angle between the normal force N3 and the positive x-axis is 90°  20°  D 70° . The normal force is N1 D jN3 ji cos 70° C j sin 70°  D jN3 j0.3420i C 0.9397j. The force is . . . F D jFji C 0j. The equilibrium conditions are  F D W C N1 C N3 C F D 0, from which:   Fx D 0.7071jN1 j C 0.3420jN3 j C jFji D 0 Fy D 0.7071jN1 j C 0.9397jN3 j  412j D 0 For jN1 j D 693.7 N from above: jFj D 162 N c 2008 Pearson Education, Inc., Upper Saddle River, NJ. All rights reserved. This material is protected under all copyright laws as they  currently exist. No portion of this material may be reproduced, in any form or by any means, without permission in writing from the publisher. 149 Problem 3.91 The climber A is being helped up an icy slope by two friends. His mass is 80 kg, and the direction cosines of the force exerted on him by the slope are cos x D 0.286, cos y D 0.429, cos z D 0.857. The y axis is vertical. If the climber is in equilibrium in the position shown, what are the tensions in the ropes AB and AC and the magnitude of the force exerted on him by the slope? y B (2, 2, 0) m B dinates of the end points. Express the tensions in terms of these unit vectors, and solve the equilibrium conditions. The rope tensions, the normal force, and the weight act on the climber. The coordinates of points A, B, C are given by the problem, A3, 0, 4, B2, 2, 0, C5, 2, 1. rB D 2i C 2j C 0k, C A The vector locations of the points A, B, C are: rA D 3i C 0j C 4k, x A (3, 0, 4) m z Solution: Get the unit vectors parallel to the ropes using the coor- C (5, 2, ⫺1) m W N rC D 5i C 2j  1k. Substitute and collect like terms, The unit vector parallel to the tension acting between the points A, B in the direction of B is eAB D rB  r A jrB  rA j    The unit vectors are eAB D 0.2182i C 0.4364j  0.8729k, Fx D 0.2182jTAB j C 0.3482jTAC j  0.286jNji D 0 Fy D 0.4364jTAB j C 0.3482jTAC j C 0.429jNj  784.8j D 0 Fz D 0.8729jTAB j C 0.8704jTAC j  0.857jNjk D 0 We have three linear equations in three unknowns. The solution is: eAC D 0.3482i C 0.3482j  0.8704k, jTAB j D 100.7 N , jTAC j D 889.0 N , jNj D 1005.5 N . and eN D 0.286i C 0.429j C 0.857k. where the last was given by the problem statement. The forces are expressed in terms of the unit vectors, TAB D jTAB jeAB , TAC D jTAC jeAC , N D jNjeN . The weight is W D 0i  jWjj C 0k D 0i  809.81j C 0k  0i  784.8j C 0k. The equilibrium conditions are  150 F D 0 D TAB C TAC C N C W D 0. c 2008 Pearson Education, Inc., Upper Saddle River, NJ. All rights reserved. This material is protected under all copyright laws as they  currently exist. No portion of this material may be reproduced, in any form or by any means, without permission in writing from the publisher. Problem 3.92 Consider the climber A being helped by his friends in Problem 3.91. To try to make the tensions in the ropes more equal, the friend at B moves to the position (4, 2, 0) m. What are the new tensions in the ropes AB and AC and the magnitude of the force exerted on the climber by the slope? Solution: Get the unit vectors parallel to the ropes using the coordinates of the end points. Express the tensions in terms of these unit vectors, and solve the equilibrium conditions. The coordinates of points A, B, C are A3, 0, 4, B4, 2, 0, C5, 2, 1. The vector locations of the points A, B, C are: rA D 3i C 0j C 4k, rB D 4i C 2j C 0k, rC D 5i C 2j  1k. The unit vectors are eAB D C0.2182i C 0.4364j  0.8729k, eAC D C0.3482i C 0.3482j  0.8704k, eN D 0.286i C 0.429j C 0.857k. where the last was given by the problem statement. The forces are expressed in terms of the unit vectors, TAB D jTAB jeAB , TAC D jTAC jeAC , N D jNjeN . The weight is W D 0i  jWjj C 0k D 0i  809.81j C 0k  0i  784.8j C 0k. The equilibrium conditions are  F D 0 D TAB C TAC C N C W D 0. Substitute and collect like terms,    Fx D C0.281jTAB j C 0.3482jTAC j  0.286jNji D 0 Fy D 0.4364jTAB j C 0.3482jTAC j C 0.429jNj  784.8j D 0 Fz D 0.8729jTAB j C 0.8704jTAC j  0.857jNjk D 0 The HP-28S hand held calculator was used to solve these simultaneous equations. The solution is: jTAB j D 420.5 N , jTAC j D 532.7 N , jNj D 969.3 N . c 2008 Pearson Education, Inc., Upper Saddle River, NJ. All rights reserved. This material is protected under all copyright laws as they  currently exist. No portion of this material may be reproduced, in any form or by any means, without permission in writing from the publisher. 151 Problem 3.93 A climber helps his friend up an icy slope. His friend is hauling a box of supplies. If the mass of the friend is 90 kg and the mass of the supplies is 22 kg, what are the tensions in the ropes AB and CD? Assume that the slope is smooth. That is, only normal forces are exerted on the man and the box by the slope. A 20⬚ B C 40⬚ Solution: Isolate the box. The weight vector is 75 W2 D 229.81j D 215.8j (N). D The angle between the normal force and the positive x axis is 90°  60°  D 30° . 60⬚ The normal force is NB D jNB j0.866i  0.5j. The angle between the rope CD and the positive x axis is 180°  75°  D 105° ; the tension is: T y β T2 D jT2 ji cos 105° C j sin 105°  D jT2 j0.2588i C 0.9659j The equilibrium conditions are  N Fx D 0.866jNB j C 0.2588jT2 ji D 0, α x  W Fy D 0.5jNB j C 0.9659jT2 j  215.8j D 0. Solve: NB D 57.8 N, jT2 j D 193.5 N. y T1 Isolate the friend. The weight is 20° W D 909.81j D 882.9j (N). The angle between the normal force and the positive x axis is 40°  D 50° . The normal force is: 90° 40°  N 75° T2 W x NF D jNF j0.6428i C 0.7660j. The angle between the lower rope and the x axis is 75° ; the tension is T2 D jT2 j0.2588i C 0.9659j. The angle between the tension in the upper rope and the positive x axis is 180°  20°  D 160° , the tension is T1 D jT1 j0.9397i C 0.3420j. The equilibrium conditions are  F D W C T1 C T2 C NF D 0. From which:   Fx D 0.6428jNF j C 0.2588jT2 j  0.9397jT1 ji D 0 Fy D 0.7660jNF j  0.9659jT2 j C 0.3420jT1 j  882.9j D 0 Solve, for jT2 j D 193.5 N. The result: jNF j D 1051.6 N , 152 jT1 j D 772.6 N . c 2008 Pearson Education, Inc., Upper Saddle River, NJ. All rights reserved. This material is protected under all copyright laws as they  currently exist. No portion of this material may be reproduced, in any form or by any means, without permission in writing from the publisher. Problem 3.94 The 2800-lb car is moving at constant speed on a road with the slope shown. The aerodynamic forces on the car the drag D D 270 lb, which is parallel to the road, and the lift L D 120 lb, which is perpendicular to the road. Determine the magnitudes of the total normal and friction forces exerted on the car by the road. L D 15⬚ Solution: The free-body diagram is shown. If we write the equilibrium equations parallel and perpendicular to the road, we have:   F- : N  2800 lb cos 15° C 120 lb D 0 F% : f  270 lb  2800 lb sin 15° D 0 Solving, we find N D 2580 lb, f D 995 lb c 2008 Pearson Education, Inc., Upper Saddle River, NJ. All rights reserved. This material is protected under all copyright laws as they  currently exist. No portion of this material may be reproduced, in any form or by any means, without permission in writing from the publisher. 153 Problem 3.95 An engineer doing preliminary design studies for a new radio telescope envisions a triangular receiving platform suspended by cables from three equally spaced 40-m towers. The receiving platform has a mass of 20 Mg (megagrams) and is 10 m below the tops of the towers. What tension would the cables be subjected to? TOP VIEW Solution: Isolate the platform. Choose a coordinate system with the origin at the center of the platform, with the z axis vertical, and the x,y axes as shown. Express the tensions in terms of unit vectors, and solve the equilibrium conditions. The cable connections at the platform are labeled a, b, c, and the cable connections at the towers are labeled A, B, C. The horizontal distance from the origin (center of the platform) to any tower is given by LD 20 m 65 m C 65 D 37.5 m. 2 sin60 z c The coordinates of points A, B, C are b a A37.5, 0, 10, B A y x B37.5 cos120° , 37.5 sin120° .10, C37.5 cos240° , 37.5 sin240° , 10, The vector locations are: The tensions in the cables are expressed in terms of the unit vectors, rA D 37.5i C 0j C 10k, rB D 18.764i C 32.5j C 10k, rC D 18.764i C 32.5j C 10k. The distance from the origin to any cable connection on the platform is dD 20 D 11.547 m. 2 sin60°  The coordinates of the cable connections are a11.547, 0, 0, b11.547 cos120° , 11547 sin120° , 0, TaA D jTaA jeaA , TcC D jTcC jecC . The weight is W D 0i  0j  200009.81k D 0i C 0j  196200k. The equilibrium conditions are  F D 0 D TaA C TbB C TcC C W D 0, from which:  c11.547 cos240° , 11.547 sin240° , 0.  The vector locations of these points are,  ra D 11.547i C 0j C 0k, TbB D jTbB jebB , Fx D 0.9333jTaA j  0.4666jTbB j  0.4666jTcC ji D 0 Fy D 0jTaA j C 0.8082jTbB j  0.8082jTcC jj D 0 Fz D 0.3592jTaA j  0.3592jTbB j rb D 5.774i C 10j C 0k, C 0.3592jTcC  196200jk D 0 rc D 5.774i C 10j C 0k. The unit vector parallel to the tension acting between the points A, a in the direction of A is by definition eaA r A  ra D . jrA  ra Perform this operation for each of the unit vectors to obtain eaA D C0.9333i C 0j  0.3592k The commercial package TK Solver Plus was used to solve these equations. The results: jTaA j D 182.1 kN ,  Fz D 3jTj sin   196200 D 0,  where  D tan1 10 37.5  11.547  D 21.07° , from which each tension is jTj D 182.1 kN. 154 jTcC j D 182.1 kN . Check: For this geometry, where from symmetry all cable tensions may be assumed to be the same, only the z-component of the equilibrium equations is required: ebB D 0.4667i C 0.8082j  0.3592k ecC D 0.4667i C 0.8082j C 0.3592k jTbB j D 182.1 kN , check. c 2008 Pearson Education, Inc., Upper Saddle River, NJ. All rights reserved. This material is protected under all copyright laws as they  currently exist. No portion of this material may be reproduced, in any form or by any means, without permission in writing from the publisher. Problem 3.96 To support the tent, the tension in the rope AB must be 35 lb. What are the tensions in the ropes AC, AD, and AE? y Solution: We develop the following position vectors rAB D 2i ft rAC D 6i C j  3k ft (0, 5, 0) ft rAD D 6i C 2j C 3k ft C (0, 6, 6) ft rAE D 3i  4j ft (6, 4, 3) ft A B (8, 4, 3) ft D x If we divide by the respective magnitudes we can develop the unit vectors that are parallel to these position vectors. E (3, 0, 3) ft eAB D 1.00i eAC D 0.885i C 0.147j  0.442k z eAD D 0.857i C 0.286j C 0.429k eAE D 6.00i  0.800j The equilibrium equation is TAB eAB C TAC eAC C TAD eAD C TAE eAE D 0. If we break this up into components, we have    Fx : TAB  0.885TAC  0.857TAD  0.600TAE D 0 Fy : 0.147TAC C 0.286TAD  0.800TAE D 0 Fz : 0.442TAC C 0.429TAD D 0 If we set TAB D 35 lb, we cans solve for the other tensions. The result is TAC D 16.7 lb, TAD D 17.2 lb, TAE D 9.21 lb c 2008 Pearson Education, Inc., Upper Saddle River, NJ. All rights reserved. This material is protected under all copyright laws as they  currently exist. No portion of this material may be reproduced, in any form or by any means, without permission in writing from the publisher. 155 y Problem 3.97 Cable AB is attached to the top of the vertical 3-m post, and its tension is 50 kN. What are the tensions in cables AO, AC, and AD? 5m 5m C D Solution: Get the unit vectors parallel to the cables using the coordinates of the end points. Express the tensions in terms of these unit vectors, and solve the equilibrium conditions. The coordinates of points A, B, C, D, O are found from the problem sketch: The coordinates of the points are A6, 2, 0, B12, 3, 0, C0, 8, 5, D0, 4, 5, O0, 0, 0. 4m 8m (6, 2, 0) m The vector locations of these points are: rA D 6i C 2j C 0k, rB D 12i C 3j C 0k, O rC D 0i C 8j C 5k, B A z 3m 12 m rD D 0i C 4j  5k, rO D 0i C 0j C 0k. x The unit vector parallel to the tension acting between the points A, B in the direction of B is by definition eAB D y rB  r A . jrB  rA j 5m 5m Perform this for each of the unit vectors D 4m C eAB D C0.9864i C 0.1644j C 0k eAC D 0.6092i C 0.6092j C 0.5077k 8m O (6, 2, 0) m eAD D 0.7442i C 0.2481j  0.6202k A eAO D 0.9487i  0.3162j C 0k The tensions in the cables are expressed in terms of the unit vectors, TAB D jTAB jeAB D 50eAB , TAD D jTAD jeAD , TAC D jTAC jeAC , TAO D jTAO jeAO . The equilibrium conditions are  F D 0 D TAB C TAC C TAD C TAO D 0. Substitute and collect like terms,  Fx D 0.986450  0.6092jTAC j  0.7422jTAD j  0.9487jTAO ji D 0  Fy D 0.164450 C 0.6092jTAC j C 0.2481jTAD j  0.3162jTAO jj D 0  Fz D C0.5077jTAC j  0.6202jTAD jk D 0. This set of simultaneous equations in the unknown forces may be solved using any of several standard algorithms. The results are: jTAO j D 43.3 kN, 156 jTAC j D 6.8 kN, jTAD j D 5.5 kN. c 2008 Pearson Education, Inc., Upper Saddle River, NJ. All rights reserved. This material is protected under all copyright laws as they  currently exist. No portion of this material may be reproduced, in any form or by any means, without permission in writing from the publisher. Problem 3.98* The 1350-kg car is at rest on a plane surface with its brakes locked. The unit vector en D 0.231i C 0.923j C 0.308k is perpendicular to the surface. The y axis points upward. The direction cosines of the cable from A to B are cos x D 0.816, cos y D 0.408, cos z D 0.408, and the tension in the cable is 1.2 kN. Determine the magnitudes of the normal and friction forces the car’s wheels exert on the surface. y en B ep x z Solution: Assume that all forces act at the center of mass of the car. The vector equation of equilibrium for the car is y en "car" FN TAB FS C TAB C W D 0. Writing these forces in terms of components, we have x W D mgj D 13509.81 D 13240j N, FS D FSx i C FSy j C FSz k, FS F W z and TAB D TAB eAB , where eAB D cos x i C cos y j C cos z k D 0.816i C 0.408j  0.408k. Substituting these values into the equations of equilibrium and solving for the unknown components of FS , we get three scalar equations of equilibrium. These are: FSx  TABx D 0, FSy  TABy  W D 0, and FSz  TABz D 0. Substituting in the numbers and solving, we get FSx D 979.2 N, FSy D 12, 754 N, and FSz D 489.6 N. The next step is to find the component of FS normal to the surface. This component is given by FN D FN Ð en D FSx eny C FSx eny C FSz enz . Substitution yields FN D 12149 N . From its components, the magnitude of FS is FS D 12800 N. Using the Pythagorean theorem, the friction force is  fD F2S  F2N D 4033 N. c 2008 Pearson Education, Inc., Upper Saddle River, NJ. All rights reserved. This material is protected under all copyright laws as they  currently exist. No portion of this material may be reproduced, in any form or by any means, without permission in writing from the publisher. 157 Problem 3.99* The brakes of the car in Problem 3.98 are released, and the car is held in place on the plane surface by the cable AB. The car’s front wheels are aligned so that the tires exert no friction forces parallel to the car’s longitudinal axis. The unit vector ep D 0.941i C 0.131j C 0.314k is parallel to the plane surface and aligned with the car’s longitudinal axis. What is the tension in the cable? Solution: Only the cable and the car’s weight exert forces in the direction parallel to ep . Therefore ep Ð T  mgj D 0: 0.941i C 0.131j C 0.314k Ð [T0.816i C 0.408j  0.408k  mgj] D 0, 0.9410.816T C 0.1310.408T  mg C 0.3140.408T D 0. Solving, we obtain T D 2.50 kN. 158 c 2008 Pearson Education, Inc., Upper Saddle River, NJ. All rights reserved. This material is protected under all copyright laws as they  currently exist. No portion of this material may be reproduced, in any form or by any means, without permission in writing from the publisher. Problem 4.1 In Active Example 4.1, the 40-kN force points 30° above the horizontal. Suppose that the force points 30° below the horizontal instead. Draw a sketch of the beam with the new orientation of the force. What is the moment of the force about point A? 40 kN 30° A 6m Solution: The perpendicular distance from A to the line of action of the force is unchanged D D 6 m sin 30° D 3 m The magnitude of the moment is therefore unchanged M D 3 m40 kN D 120 kN-m However, with its new orientation, the force would tend to cause the beam to rotate about A in the clockwise direction. The moment is clockwise M D 120 kN-m clockwise Problem 4.2 The mass m1 D 20 kg. The magnitude of the total moment about B due to the forces exerted on bar AB by the weights of the two suspended masses is 170 N-m. What is the magnitude of the total moment due to the forces about point A? 0.35 m 0.35 m 0.35 m A B m1 m2 Solution: The total moment about B is MB D m2 9.81 m/s2 0.35 m C 20 kg9.81 m/s2 0.7 m D 170 N-m Solving, we find m2 D 9.51 kg The moment about A is then jMA j D 20 kg9.81 m/s2 0.35 m C 9.51 kg9.81 m/s2 0.7 m jMA j D 134 N-m c 2008 Pearson Education, Inc., Upper Saddle River, NJ. All rights reserved. This material is protected under all copyright laws as they  currently exist. No portion of this material may be reproduced, in any form or by any means, without permission in writing from the publisher. 159 Problem 4.3 The wheels of the overhead crane exert downward forces on the horizontal I-beam at B and C. If the force at B is 40 kip and the force at C is 44 kip, determine the sum of the moments of the forces on the beam about (a) point A, (b) point D. 10 ft 25 ft B A 15 ft C D Solution: Use 2-dimensional moment strategy: determine normal distance to line of action D; calculate magnitude DF; determine sign. Add moments. (a) The normal distances from A to the lines of action are DAB D 10 ft, and DAC D 35 ft. The moments are clockwise (negative). Hence,  (b) 10 ft 25 ft 15 ft A D B C MA D 1040  3544 D 1940 ft-kip . The normal distances from D to the lines of action are DDB D 40 ft, and DDC D 15 ft. The actions are positive; hence  MD D C4040 C 1544 D 2260 ft-kip Problem 4.4 What force F applied to the pliers is required to exert a 4 N-m moment about the center of the bolt at P? Solution: MP D 4 N-m D F0.165 m sin 42°  ) F D 4 N-m 0.165 m sin 42° D 36.2 N P F 165 mm 42⬚ 160 c 2008 Pearson Education, Inc., Upper Saddle River, NJ. All rights reserved. This material is protected under all copyright laws as they  currently exist. No portion of this material may be reproduced, in any form or by any means, without permission in writing from the publisher. Problem 4.5 Two forces of equal magnitude F are applied to the wrench as shown. If a 50 N-m moment is required to loosen the nut, what is the necessary value of F? Solution:  300 mm 380 mm F Mnut center D F cos 30° 0.3 m C F cos 20° 0.38 m F 30⬚ D 50 N-m 50 N-m FD D 81.1 N 0.3 m cos 30° C 0.38 m cos 20° Problem 4.6 The force F D 8 kN. What is the moment of the force about point P? 20⬚ F F y (3, 7) m Solution: The angle between the force F and the x axis is F ˛ D tan1 5/4 D 51.3° Q (8, 5) m The force can then be written P (3, 2) m F D 8 kNcos ˛i  sin ˛j D 5.00i  6.25j kN (7, 2) m x The line of action of the j component passes through P, so it exerts no moment about P. The moment of the i component about P is clockwise, and its magnitude is MP D 5 m5.00 kN D 25.0 kN-m MP D 25.0 kN-m clockwise Problem 4.7 If the magnitude of the moment due to the force F about Q is 30 kN-m, what is F? y (3, 7) m F Solution: The angle between the force F and the x axis is Q (8, 5) m ˛ D tan1 5/4 D 51.3° The force can then be written F D Fcos ˛i  sin ˛j D F0.625i  0.781j P (3, 2) m (7, 2) m x Treating counterclockwise moment as positive, the total moment about point Q is MQ D 0.781F5 m  0.625F2 m D 30 kN-m Solving, we find F D 11.3 kN c 2008 Pearson Education, Inc., Upper Saddle River, NJ. All rights reserved. This material is protected under all copyright laws as they  currently exist. No portion of this material may be reproduced, in any form or by any means, without permission in writing from the publisher. 161 Problem 4.8 The support at the left end of the beam will fail if the moment about A of the 15-kN force F exceeds 18 kN-m. Based on this criterion, what is the largest allowable length of the beam? F 30° B A 25° Solution:  MA D L Ð F sin 30° D L 15 2  30° F = 15 kN 30° MA D 7.5 L kN Ð m L set MA D MAmax D 18 kN Ð m D 7.5 Lmax Lmax D 2.4 m 25° Problem 4.9 The length of the bar AP is 650 mm. The radius of the pulley is 120 mm. Equal forces T D 50 N are applied to the ends of the cable. What is the sum of the moments of the forces (a) about A; (b) about P. 45⬚ A 30⬚ T T Solution: (a)  MA D 50 N0.12 m  50 N0.12 m D 0 MA D 0 (b)  P 45⬚ MP D 50 N0.12 m  50 N cos 30° 0.65 m sin 45° C 0.12 m cos 30°   50 N sin 30° 0.65 m cos 45° C 0.12 m sin 30°  MP D 31.4 N-m 162 or MP D 31.4 N-m CW c 2008 Pearson Education, Inc., Upper Saddle River, NJ. All rights reserved. This material is protected under all copyright laws as they  currently exist. No portion of this material may be reproduced, in any form or by any means, without permission in writing from the publisher. Problem 4.10 The force F D 12 kN. A structural engineer determines that the magnitude of the moment due to F about P should not exceed 5 kN-m. What is the acceptable range of the angle ˛? Assume that 0  ˛  90° . F α 1m P 2m Solution: We have the moment about P 12 kN MP D 12 kN sin ˛2 m  12 kN cos ˛1 m α MP D 122 sin ˛  cos ˛ kN-m The moment must not exceed 5 kN-m 1m Thus 5 kN-m ½ j122 sin ˛  cos ˛jkN-m P The limits occur when 122 sin ˛  cos ˛ D 5 ) ˛ D 37.3 2m 122 sin ˛  cos ˛ D 5 ) ˛ D 15.83° So we must have 15.83°  ˛  37.3° c 2008 Pearson Education, Inc., Upper Saddle River, NJ. All rights reserved. This material is protected under all copyright laws as they  currently exist. No portion of this material may be reproduced, in any form or by any means, without permission in writing from the publisher. 163 Problem 4.11 The length of bar AB is 350 mm. The moments exerted about points B and C by the vertical force F are MB D 1.75 kN-m and MC D 4.20 kN-m. Determine the force F and the length of bar AC. B 30° C 20° A Solution: We have 1.75 kN-m D F0.35 m sin 30° ) F D 10 kN F 4.20 kN-m D FLAC  cos 20° ) LAC D 0.447 m In summary F D 10 kN, LAC D 447 mm B C 30° 20° F d1 30° 50 0.3 m 0.450 m 20° d2 164 600 N c 2008 Pearson Education, Inc., Upper Saddle River, NJ. All rights reserved. This material is protected under all copyright laws as they  currently exist. No portion of this material may be reproduced, in any form or by any means, without permission in writing from the publisher. Problem 4.12 In Example 4.2, suppose that the 2-kN force points upward instead of downward. Draw a sketch of the machine part showing the orientations of the forces. What is the sum of the moments of the forces about the origin O? 4 kN 30⬚ 2 kN 300 mm O 3 kN 5 kN 300 mm 400 mm Solution: If the 2-kN force points upward, the magnitude of its moment about O does not change, but the direction of the moment changes from clockwise to counterclockwise. Treating counterclockwise moments as positive, the moment due to the 2-kN force is 0.3 m2 kN D 0.6 kN-m The moments due to the other forces do not change, so the sum of the moments of the four forces is  MO D 0.6  1.039 C 1.400 kN-m  MO D 0.961 kN-m Problem 4.13 Two equal and opposite forces act on the beam. Determine the sum of the moments of the two forces (a) about point P; (b) about point Q; (c) about the point with coordinates x D 7 m, y D 5 m. y 40 N P 30⬚ 40 N 2m Solution: 30⬚ Q 2m y 40 N 40 N MP D 40 N cos 30° 2 m C 40 N cos 30° 4 m x (a) D 69.3 N-m CCW 30° 30° (b) MQ D 40 N cos 30° 2 m D 69.3 N-m CCW M D 40 N sin 30° 5 m C 40 N cos 30° 5 m (c) x P 2m 2m Q  40 N sin 30° 5 m  40 N cos 30° 3 m D 69.3 N-m CCW c 2008 Pearson Education, Inc., Upper Saddle River, NJ. All rights reserved. This material is protected under all copyright laws as they  currently exist. No portion of this material may be reproduced, in any form or by any means, without permission in writing from the publisher. 165 Problem 4.14 The moment exerted about point E by the weight is 299 in-lb. What moment does the weight exert about point S? S 13 in. 30° 12 E 40° in. Solution: The key is the geometry From trigonometry, cos 40° D Thus d1 D 12 in cos 30° d1 D and d2 d1 , cos 30° D 13 in 12 in d1 S 30° 13 in 12 i 10.3900 n W 40° d2 D 13 in cos 40° d2 E d2 D 9.9600 We are given that 299 in-lb D d2 W D 9.96 W W D 30.0 lb Now, Ms D d1 C d2 W Ms D 20.3530.0 Ms D 611 in-lb clockwise Problem 4.15 The magnitudes of the forces exerted on the pillar at D by the cables A, B, and C are equal: FA D FB D FC . The magnitude of the total moment about E due to the forces exerted by the three cables at D is 1350kN-m. What is FA ? D FC D FA FB 6m A Solution: The angles between the three cables and the pillar are ˛A D tan1 4/6 D 33.7° B C E 4m 4m 4m ˛B D tan1 8/6 D 53.1° ˛C D tan1 12/6 D 63.4° The vertical components of each force at point D exert no moment about E. Noting that FA D FB D FC , the magnitude of the moment about E due to the horizontal components is  ME D FA sin ˛A C sin ˛B C sin ˛C 6 m D 1350 kN-m Solving for FA yields FA D 100 kN 166 c 2008 Pearson Education, Inc., Upper Saddle River, NJ. All rights reserved. This material is protected under all copyright laws as they  currently exist. No portion of this material may be reproduced, in any form or by any means, without permission in writing from the publisher. Problem 4.16 Three forces act on the piping. Determine the sum of the moments of the three forces about point P. 2 kN Solution:  MP D 4 kN0.2 m C 2 kN0.6 m  2 kN cos 20° 0.2 m C 2 kN sin 20° 0.4 m D 10.18 kN-m 20⬚ MP D 0.298 kN-m CCW 2 kN 4 kN 0.2 m 2 kN 20° 4 kN P 0.2 m 0.2 m 0.2 m P 0.2 m Problem 4.17 The forces F1 D 30 N, F2 D 80 N, and F3 D 40 N. What is the sum of the moments of the forces about point A? Solution: The moment about point A due to F1 is zero. Treating counterclockwise moments as positive the sum of the moments is   2 kN 0.2 m 0.2 m 0.2 m y F3 A C 30⬚ F1 2m MA D F3 sin 30° 8 m C F2 cos 45° 2 m B 45⬚ F2 MA D 273 N-m counterclockwise 8m Problem 4.18 The force F1 D 30 N. The vector sum of the forces is zero. What is the sum of the moments of the forces about point A? x y F3 A 30⬚ C F1 2m Solution: The sums of the forces in the x and y directions equal zero:  Fx : F1 C F2 cos 45°  F3 cos 30° D 0  B 45⬚ F2 8m Fy : F2 sin 45° C F3 sin 30° x D0 Setting F1 D 30 N and solving yields F2 D 58.0 N, F3 D 82.0 N. The sum of the moments about point A is   MA D F2 sin 30° 8 m C F2 cos 45° 2 m D 410 N-m MA D 410 N-m counterclockwise c 2008 Pearson Education, Inc., Upper Saddle River, NJ. All rights reserved. This material is protected under all copyright laws as they  currently exist. No portion of this material may be reproduced, in any form or by any means, without permission in writing from the publisher. 167 Problem 4.19 The forces FA D 30 lb, FB D 40 lb, FC D 20 lb, and FD D 30 lb. What is the sum of the moments of the forces about the origin of the coordinate system? y FD 30⬚ FA FB Solution: The moment about the origin due to FA and FD is zero. Treating counterclockwise moments as positive, the sum of the moments is  M D FB 6 ft C FC 10 ft 6 ft x FC 4 ft D 40 lb6 ft C 20 lb10 ft D 40 ft-lb  M D 40 ft-lb clockwise y Problem 4.20 The force FA D 30 lb. The vector sum of the forces on the beam is zero, and the sum of the moments of the forces about the origin of the coordinate system is zero. (a) (b) FD 30⬚ FA FB Determine the forces FB , FC , and FD . Determine the sum of the moments of the forces about the right end of the beam. 6 ft x FC 4 ft Solution: (a) The sum of the forces and the sum of the moments equals zero    Fx : FA cos 30°  FD D 0 Fy : FA sin 30°  FB C FC D 0 Morigin : FB 6 ft C FC 10 ft D 0 Setting FA D 30 lb and solving yields (b) FB D 37.5 lb, FC D 22.5 lb, FD D 26.0 lb The sum of the moments about the right end is  MRight End : FB 4 ft  FA sin 30° 10 ft D 37.5 lb4 ft  30 lb10 ft D0  168 MRight End D0 c 2008 Pearson Education, Inc., Upper Saddle River, NJ. All rights reserved. This material is protected under all copyright laws as they  currently exist. No portion of this material may be reproduced, in any form or by any means, without permission in writing from the publisher. y Problem 4.21 Three forces act on the car. The sum of the forces is zero and the sum of the moments of the forces about point P is zero. (a) (b) 3 ft 6 ft Determine the forces A and B. Determine the sum of the moments of the forces about point Q. x B P Q 2800 lb A Solution: 6 ft   (a) Fy : A C B  2800 lb D 0 MP : 2800 lb6 ft C A9 ft D 0 Solving we find A D 1867 lb, B D 933 lb (b) 3 ft  Q P 2800 lb MQ D 2800 lb3 ft  B9 ft D 0 B A MQ D 0 80 lb Problem 4.22 Five forces act on the piping. The vector sum of the forces is zero and the sum of the moments of the forces about point P is zero. (a) (b) 45⬚ y Determine the forces A, B, and C. Determine the sum of the moments of the forces about point Q. 2 ft 20 lb Q x A P C 2 ft B 2 ft 2 ft Solution: 80 lb The conditions given in the problem are:   Fx : A C 80 lb cos 45° D 0 45° y 20 lb 2 ft (a) Fy : B  C  20 lb C 80 lb sin 45° D 0 P  MP : 20 lb2 ft  C6 ft  80 lb cos 45° 2 ft Q x A 2 ft C 80 lb sin 45° 4 ft D 0 Solving we have B 2 ft 2 ft C A D 56.6 lb, B D 24.4 lb, C D 12.19 lb  MQ : 80 lb cos 45° 2 ft  80 lb sin 45° 2 ft (b) C20 lb4 ft C B6 ft D 0 c 2008 Pearson Education, Inc., Upper Saddle River, NJ. All rights reserved. This material is protected under all copyright laws as they  currently exist. No portion of this material may be reproduced, in any form or by any means, without permission in writing from the publisher. 169 Problem 4.23 In Example 4.3, suppose that the attachment point B is moved upward and the cable is lengthened so that the vertical distance from C to B is 9 ft. (the positions of points C and A are unchanged.) Draw a sketch of the system with the cable in its new position. What is the tension in the cable? B A 7 ft W 4 ft C 2 ft 2 ft Solution: The angle ˛ between the cable AB and the horizontal is. ˛ D tan1 5/4 D 51.3° The sum of the moments about C is  MC : W2 ft C T cos ˛4 ft C T sin ˛4 ft D 0 Solving yields T D 106.7 lb Problem 4.24 The tension in the cable is the same on both sides of the pulley. The sum of the moments about point A due to the 800-lb force and the forces exerted on the bar by the cable at B and C is zero. What is the tension in the cable? Solution: Let T be the tension in the cable. The sum of the moments about A is  MA : T30 in C T sin 30° 90 in  800 lb60in D 0 A B 30 30 in 30 in C 800 lb 30 in Solving yields T D 640 lb 170 c 2008 Pearson Education, Inc., Upper Saddle River, NJ. All rights reserved. This material is protected under all copyright laws as they  currently exist. No portion of this material may be reproduced, in any form or by any means, without permission in writing from the publisher. Problem 4.25 The 160-N weights of the arms AB and BC of the robotic manipulator act at their midpoints. Determine the sum of the moments of the three weights about A. 150 600 mm mm 40 N C 20° B m 0m Solution: The strategy is to find the perpendicular distance from 40° 60 160 N the points to the line of action of the forces, and determine the sum of the moments, using the appropriate sign of the action. The distance from A to the action line of the weight of the arm AB is: A 160 N dAB D 0.300 cos 40° D 0.2298 m The distance from A to the action line of the weight of the arm BC is dBC D 0.600cos 40°  C 0.300cos 20°  D 0.7415 m. The distance from A to the line of action of the force is dF D 0.600cos 40°  C 0.600cos 20°  C 0.150cos 20°  D 1.1644 m. The sum of the moments about A is  MA D dAB 160  dBC 160  dF 40 D 202 N-m Problem 4.26 The space shuttle’s attitude thrusters exert two forces of magnitude F D 7.70 kN. What moment do the thrusters exert about the center of mass G? 2.2 m 2.2 m F F G 5° 18 m Solution: The key to this problem is getting the geometry correct. The simplest way to do this is to break each force into components parallel and perpendicular to the axis of the shuttle and then to sum the moments of the components. (This will become much easier in the next section) 6° 12 m F sin 6° F sin 5° 5˚ 6°c 2.2 m 18 m F cos 5° FRONT 2.2 m 12 m F cos 6° REAR CMFRONTý D 18F sin 5°  2.2F cos 5° CMREARý D 2.2F cos 6°  12F sin 6° CMTOTAL D MFRONT C MREAR CMTOTAL D 4.80 C 7.19 N-m CMTOTAL D 2.39 N-m c 2008 Pearson Education, Inc., Upper Saddle River, NJ. All rights reserved. This material is protected under all copyright laws as they  currently exist. No portion of this material may be reproduced, in any form or by any means, without permission in writing from the publisher. 171 Problem 4.27 The force F exerts a 200 ft-lb counterclockwise moment about A and a 100 ft-lb clockwise moment about B. What are F and ? y A (–5, 5) ft F θ (4, 3) ft x B (3, – 4) ft Solution: The strategy is to resolve F into x- and y-components, and compute the perpendicular distance to each component from A and B. The components of F are: F D iFX C jFY . The vector from A to the point of application is: rAF D 4  5i C 3  5j D 9i  2j. The perpendicular distances are dAX D 9 ft, and dAY D 2 ft, and the actions are positive. The moment about A is MA D 9FY C 2FX D 200 ft-lb. The vector from B to the point of application is rBF D 4  3i C 3  4j D 1i C 7j; the distances dBX D 1 ft and dBY D 7 ft, the action of FY is positive and the action of FX is negative. The moment about B is MB D 1FY  7FX D 100 ft-lb. The two simultaneous equations have solution: FY D 18.46 lb and FX D 16.92 lb. Take the ratio to find the angle:   D tan1 FY FX   D tan1 18.46 16.92 y A (–5, 5) ft F θ (4, 3) ft x B (3, –4) ft  D tan1 1.091 D 47.5° . From the Pythagorean theorem  jFj D 172 F2Y C F2X D p 18.462 C 16.922 D 25.04 lb c 2008 Pearson Education, Inc., Upper Saddle River, NJ. All rights reserved. This material is protected under all copyright laws as they  currently exist. No portion of this material may be reproduced, in any form or by any means, without permission in writing from the publisher. Problem 4.28 Five forces act on a link in the gearshifting mechanism of a lawn mower. The vector sum of the five forces on the bar is zero. The sum of their moments about the point where the forces Ax and Ay act is zero. (a) Determine the forces Ax , Ay , and B. (b) Determine the sum of the moments of the forces about the point where the force B acts. Ay Ax 25 kN 20° 650 mm 450 mm 30 kN 45° B 650 mm 350 mm Solution: The strategy is to resolve the forces into x- and y-components, determine the perpendicular distances from B to the line of action, determine the sign of the action, and compute the moments. The angles are measured counterclockwise from the x axis. The forces are F2 D 30i cos 135° C j sin 135°  D 21.21i C 21.21j F1 D 25i cos 20° C j sin 20°  D 23.50i C 8.55j. (a) The sum of the forces is  F D A C B C F1 C F2 D 0. Substituting:  and FX  D AX C BX C 23.5  21.2i D 0, FY D AY C 21.2 C 8.55j D 0. Solve the second equation: AY D 29.76 kN. The distances of the forces from A are: the triangle has equal base and altitude, hence the angle is 45° , so that the line of action of F1 passes through A. The distance to the line of action of B is 0.65 m, with a positive action. The distance to the line of action of the y-component of F2 is 0.650 C 0.350 D 1 m, and the action is positive. The distance to the line of action of the x-component of F2 is 0.650  0.450 D 0.200 m, and the action is positive. The moment about A is  MA D 8.551 C 23.50.2 C BX 0.65 D 0. Solve: BX D 20.38 kN. Substitute into the force equation to obtain AX D 18.09 kN (b) The distance from B to the line of action of the y-component of F1 is 0.350 m, and the action is negative. The distance from B to the line of action of AX is 0.650 m and the action is negative. The distance from B to the line of action of AY is 1 m and the action is positive. The distance from B to the line of action of the x-component of F2 is 0.450 m and the action is negative. The sum of the moments about B:  MB D 0.35021.21  0.65018.09 C 129.76  0.45023.5 D 0 c 2008 Pearson Education, Inc., Upper Saddle River, NJ. All rights reserved. This material is protected under all copyright laws as they  currently exist. No portion of this material may be reproduced, in any form or by any means, without permission in writing from the publisher. 173 F Problem 4.29 Five forces act on a model truss built by a civil engineering student as part of a design project. The dimensions are b D 300 mm and h D 400 mm; F D 100 N. The sum of the moments of the forces about the point where Ax and Ay act is zero. If the weight of the truss is negligible, what is the force B? F 60° 60° h Ax Ay b b b b b b B Solution: The x- and y-components of the force F are F D jFji cos 60° C j sin 60°  D jFj0.5i C 0.866j. The distance from A to the x-component is h and the action is positive. The distances to the y-component are 3b and 5b. The distance to B is 6b. The sum of the moments about A is  MA D 2jFj0.5h  3bjFj0.866  5bjFj0.866 C 6bB D 0. Substitute and solve: B D 1.6784jFj D 93.2 N 1.8 Problem 4.30 Consider the truss shown in Problem 4.29. The dimensions are b D 3 ft and h D 4 ft; F D 300 lb. The vector sum of the forces acting on the truss is zero, and the sum of the moments of the forces about the point where Ax and Ay act is zero. (a) (b) Determine the forces Ax , Ay , and B. Determine the sum of the moments of the forces about the point where the force B acts. Solution: The forces are resolved into x- and y-components: Solve the first: Ax D 300 lb. The distance from point A to the x-components of the forces is h, and the action is positive. The distances between the point A and the lines of action of the ycomponents of the forces are 3b and 5b. The actions are negative. The distance to the line of action of the force B is 6b. The action is positive. The sum of moments about point A is F D 300i cos 60° C j sin 60°  D 150i  259.8j. (a) The sum of the forces:  F D 2F C A C B D 0.  MA D 2150 h  3b259.8  5b259.8 C 6b B D 0. The x- and y-components:   Substitute and solve: B D 279.7 lb. Substitute this value into the force equation and solve: Ax D 519.6  279.7 D 239.9 lb Fx D Ax  300i D 0, (b) Fy D 519.6 C Ay C Bj D 0. The distances from B and the line of action of AY is 6b and the action is negative. The distance between B and the x-component of the forces is h and the action is positive. The distance between B and the y-components of the forces is b and 3b, and the action is positive. The sum of the moments about B:  174 MB D 6b239.9 C 2150 h C b259.8 C 3b259.8 D 0 c 2008 Pearson Education, Inc., Upper Saddle River, NJ. All rights reserved. This material is protected under all copyright laws as they  currently exist. No portion of this material may be reproduced, in any form or by any means, without permission in writing from the publisher. Problem 4.31 The mass m D 70 kg. What is the moment about A due to the force exerted on the beam at B by the cable? B A 45° 30° 3m m Solution: The strategy is to resolve the force at B into components parallel to and normal to the beam, and solve for the moment using the normal component of the force. The force at B is to be determined from the equilibrium conditions on the cable juncture O. Angles are measured from the positive x axis. The forces at the cable juncture are: FOB FOC O FOB D jFOB ji cos 150° C j sin 150°  D jFOB j0.866i C 0.5j W FOC D jFOC ji cos 45° C j sin 45°  D jFOC j0.707i C 0.707j. W D 709.810i  1j D 686.7j (N). The equilibrium conditions are:   Fx D 0.866jFOB j C 0.7070jFOC ji D 0 FY D 0.500jFOB j C .707jFOC j  686.7j D 0. Solve: jFOB j D 502.70 N. This is used to resolve the cable tension at B: FB D 502.7i cos 330° C j sin 330°  D 435.4i  251.4j. The distance from A to the action line of the y-component at B is 3 m, and the action is negative. The x-component at passes through A, so that the action line distance is zero. The moment at A is MA D 3251.4 D 754.0 N-m Problem 4.32 The weights W1 and W2 are suspended by the cable system shown. The weight W1 D 12 lb. The cable BC is horizontal. Determine the moment about point P due to the force exerted on the vertical post at D by the cable CD. A D 50⬚ W1 B 6 ft C W2 P Solution: Isolate part of the cable system near point B. The equilibrium equations are   Fx : TBC  TAC cos 50° D 0 Fy : TAB  12 lb D 0 Solving yields TAB D 15.7 lb, TBC D 10.1 lb Let ˛ be the angle between the cable CD and the horizontal. The magnitude of the moment about P due to the force exerted at D by cable CD is M D TCD cos ˛6 ft Isolate part of the cable system near point C. From the equilibrium equation  Fx : TCD cos ˛  TBC D 0 ) TCD cos ˛ D TBC D 10.1 lb Thus M D 10.1 lb6 ft M D 60.4 lb c 2008 Pearson Education, Inc., Upper Saddle River, NJ. All rights reserved. This material is protected under all copyright laws as they  currently exist. No portion of this material may be reproduced, in any form or by any means, without permission in writing from the publisher. 175 Problem 4.33 The bar AB exerts a force at B that helps support the vertical retaining wall. The force is parallel to the bar. The civil engineer wants the bar to exert a 38 kN-m moment about O. What is the magnitude of the force the bar must exert? B 4m A 1m O 1m 3m Solution: The strategy is to resolve the force at B into components FB parallel to and normal to the wall, determine the perpendicular distance from O to the line of action, and compute the moment about O in terms of the magnitude of the force exerted by the bar. B By inspection, the bar forms a 3, 4, 5 triangle. The angle the bar makes with the horizontal is cos  D 35 D 0.600, and sin  D 45 D 0.800. The force at B is FB D jFB j0.600i C 0.800j. The perpendicular distance from O to the line of action of the x-component is 4 C 1 D 5 m, and the action is positive. The distance from O to the line of action of the y-component is 1 m, and the action is positive. The moment about  O is MO D 50.600jFB j C 10.800jFB j D 3.8jFB j D 38 kN, from which jFB j D 10 kN 4m A θ O 1m 1m 3m Problem 4.34 A contestant in a fly-casting contest snags his line in some grass. If the tension in the line is 5 lb, what moment does the force exerted on the rod by the line exert about point H, where he holds the rod? H Solution: The strategy is to resolve the line tension into a compo- 6 ft nent normal to the rod; use the length from H to tip as the perpendicular distance; determine the sign of the action, and compute the moment. 4 ft The line and rod form two right triangles, as shown in the sketch. The angles are: ˛ D tan1   2 D 15.95° 7  ˇ D tan1 7 ft 6 15 α  D 21.8° . β α 2 ft 7 ft The angle between the perpendicular distance line and the fishing line is  D ˛ C ˇ D 37.7° . The force normal to the p distance line is F D 5sin 37.7°  D 3.061 lb. The distance is d D 22 C 72 D 7.28 ft, and the action is negative. The moment about H is MH D 7.283.061 D 22.3 ft-lb Check: The tension can be resolved into x and y components, 15 ft 6 ft 15 ft β Fx D F cos ˇ D 4.642 lb, Fy D F sin ˇ D 1.857 lb. The moment is M D 2Fx C 7Fy D 22.28 D 22.3 ft-lb. check. 176 c 2008 Pearson Education, Inc., Upper Saddle River, NJ. All rights reserved. This material is protected under all copyright laws as they  currently exist. No portion of this material may be reproduced, in any form or by any means, without permission in writing from the publisher. A Problem 4.35 The cables AB and AC help support the tower. The tension in cable AB is 5 kN. The points A, B, C, and O are contained in the same vertical plane. (a) What is the moment about O due to the force exerted on the tower by cable AB? (b) If the sum of the moments about O due to the forces exerted on the tower by the two cables is zero, what is the tension in cable AC? Solution: The strategy is to resolve the cable tensions into components normal to the vertical line through OA; use the height of the tower as the perpendicular distance; determine the sign of the action, and compute the moments. (a) (b) 20 m 60° 45° C O A B FN 60° FN A 45° The component normal to the line OA is FBN D 5cos 60°  D 2.5 kN. The action is negative. The moment about O is MOA D 2.520 D 50 kN-m By a similar process, the normal component of the tension in the cable AC is FCN D jFC j cos 45° D 0.707jFC j. The action is positive. If the sum of the moments is zero,  MO D 0.70720jFC j  50 D 0, from which jFC j D 50 kN m D 3.54 kN 0.70720 m c 2008 Pearson Education, Inc., Upper Saddle River, NJ. All rights reserved. This material is protected under all copyright laws as they  currently exist. No portion of this material may be reproduced, in any form or by any means, without permission in writing from the publisher. 177 Problem 4.36 The cable from B to A (the sailboat’s forestay) exerts a 230-N force at B. The cable from B to C (the backstay) exerts a 660-N force at B. The bottom of the sailboat’s mast is located at x D 4 m, y D 0. What is the sum of the moments about the bottom of the mast due to the forces exerted at B by the forestay and backstay? Solution: Triangle ABP tan ˛ D Triangle BCQ tan ˇ D y 4 , ˛ D 18.73° 11.8 5 , ˇ D 22.62° 12 CMO D 13230 sin ˛  13660 sin ˇ B (4,13) m CMO D 2340 N-m B (4,13) 230 N 660 N β α A (0,1.2) m C (9,1) m x P A (0,1.2) C (9,1) O (4,0) Q 660 sin β 230 sin α α β 13 m O 178 c 2008 Pearson Education, Inc., Upper Saddle River, NJ. All rights reserved. This material is protected under all copyright laws as they  currently exist. No portion of this material may be reproduced, in any form or by any means, without permission in writing from the publisher. Problem 4.37 The cable AB exerts a 290-kN force on the building crane’s boom at B. The cable AC exerts a 148-kN force on the boom at C. Determine the sum of the moments about P due to the forces the cables exert on the boom. A B 8m C G Boom P 16 m 38 m 56 m Solution: A 56 8  290 8 8 290 kN56 m  p 148 kN16 m MP D  p 3200 320  kN 8 kN 16 14 40 m B D 3.36 MNm 8 C 16 m P MP D 3.36 MN-m CW Problem 4.38 The mass of the building crane’s boom in Problem 4.37 is 9000 kg. Its weight acts at G. The sum of the moments about P due to the boom’s weight, the force exerted at B by the cable AB, and the force exerted at C by the cable AC is zero. Assume that the tensions in cables AB and AC are equal. Determine the tension in the cables. Solution: A 56 8 8  8 8 TAB 56 m  p TAC 16 m MP D  p 3200 320 C 9000 kg9.81 m/s2 38 m D 0 using TAB D TAC we solve and find T AB B 18 m C 16 TA 22 m C 16 m P 9000 kg (9.81 m/s2) TAB D TAC D 223 kN c 2008 Pearson Education, Inc., Upper Saddle River, NJ. All rights reserved. This material is protected under all copyright laws as they  currently exist. No portion of this material may be reproduced, in any form or by any means, without permission in writing from the publisher. 179 Problem 4.39 The mass of the luggage carrier and the suitcase combined is 12 kg. Their weight acts at A. The sum of the moments about the origin of the coordinate system due to the weight acting at A and the vertical force F applied to the handle of the luggage carrier is zero. Determine the force F (a) if ˛ D 30° ; (b) if ˛ D 50° . F y 0.28 m Solution: O is the origin of the coordinate system  MO D F1.2 m cos ˛  12 kg9.81 m/s2 0.28 cos ˛  0.14 sin ˛ D 0 Solving we find x FD 12 kg9.81 m/s2 0.28 cos ˛  0.14 sin ˛ 1.2 m cos ˛ (a) For ˛ D 30° We find F D 19.54 N (b) For ˛ D 50° We find F D 11.10 N 0.14 m 1.2 m A a C Problem 4.40 The hydraulic cylinder BC exerts a 300-kN force on the boom of the crane at C. The force is parallel to the cylinder. What is the moment of the force about A? Solution: The strategy is to resolve the force exerted by the hydraulic cylinder into the normal component about the crane; determine the distance; determine the sign of the action, and compute the moment. Two right triangles are constructed: The angle formed by the hydraulic cylinder with the horizontal is  ˇ D tan1 2.4 1.2  D 63.43° . The angle formed by the crane with the horizontal is C  ˛ D tan1 1.4 3  D 25.02° . A 2.4 m 1m B 1.8 m 1.2 m 7m The angle between the hydraulic cylinder and the crane is  D ˇ  ˛ D 38.42° . The normal component of the force is: Fp N D 300sin 38.42°  D 186.42 kN. The distance from point A is d D 1.42 C 32 D 3.31 m. The action is positive. The moment about A is MO D C3.31186.42 D 617.15 kN-m Check: The force exerted by the actuator can be resolved into x- and y-components, Fx D F cos ˇ D 134.16 kN, Fy D F sin ˇ D 268.33 kN. The moment about the point A is M D 1.4Fx C 3.0 Fy D 617.15 kN m. check. β α 1.2 m α 3m 2.4 m β 1.4 m 180 c 2008 Pearson Education, Inc., Upper Saddle River, NJ. All rights reserved. This material is protected under all copyright laws as they  currently exist. No portion of this material may be reproduced, in any form or by any means, without permission in writing from the publisher. Problem 4.41 The hydraulic piston AB exerts a 400-lb force on the ladder at B in the direction parallel to the piston. The sum of the moments about C due to the force exerted on the ladder by the piston and the weight W of the ladder is zero. What is the weight of the ladder? 6 ft W Solution: The angle between the piston AB and the horizontal is 3 ft ˛ D tan1 3/6 D 26.6° A B C The sum of the counterclockwise moment about C is  MC : W6 ft  400 lb cos ˛3 ft  400 lb sin ˛3 ft D 0 6 ft 3 ft Solving yields W D 268 lb Problem 4.42 The hydraulic cylinder exerts an 8-kN force at B that is parallel to the cylinder and points from C toward B. Determine the moments of the force about points A and D. 1m D C Hydraulic cylinder 1m 0.6 m B A 0.6 m Scoop 0.15 m Solution: Use x, y coords with origin A. We need the unit vector from C to B, eCB . From the geometry, eCB D 0.780i  0.625j 5.00 kN 6.25 kN C (−0.15, + 0.6) The force FCB is given by FCB D 0.7808i  0.6258j kN 0.6 m FCB D 6.25i  5.00j kN For the moments about A and D, treat the components of FCB as two separate forces. 0.15 m CMA D 5, 000.15  0.66.25 kN Ð m A (0 , 0) CMA D 3.00 kN Ð m 5.0 kN m D For the moment about D C  0,4 m MD D 5 kN1 m C 6.25 kN0.4 m C 6.25 kN CMD D 7.5 kN Ð m c 2008 Pearson Education, Inc., Upper Saddle River, NJ. All rights reserved. This material is protected under all copyright laws as they  currently exist. No portion of this material may be reproduced, in any form or by any means, without permission in writing from the publisher. 181 Problem 4.43 The structure shown in the diagram is one of the two identical structures that support the scoop of the excavator. The bar BC exerts a 700-N force at C that points from C toward B. What is the moment of this force about K? 320 mm C Shaft 100 mm Scoop 260 mm H B 180 260 mm mm J D 160 mm L K 380 mm 1040 mm 1120 mm Solution: 320 320 700 N0.52 m D 353 Nm MK D  p 108800 80 700 N 520 mm MK D 353 Nm CW K Problem 4.44 In the structure shown in Problem 4.43, the bar BC exerts a force at C that points from C toward B. The hydraulic cylinder DH exerts a 1550-N force at D that points from D toward H. The sum of the moments of these two forces about K is zero. What is the magnitude of the force that bar BC exerts at C? Solution: 320 80 260 mm 320 1120 1550 N0.26 m  p F0.52 D 0 MK D p 1264400 108800 Solving we find BC 1120 F D 796 N 100 1550 N 260 mm  K 182 c 2008 Pearson Education, Inc., Upper Saddle River, NJ. All rights reserved. This material is protected under all copyright laws as they  currently exist. No portion of this material may be reproduced, in any form or by any means, without permission in writing from the publisher. y Problem 4.45 In Active Example 4.4, what is the moment of F about the origin of the coordinate system? C (7, 7, 0) ft A (0, 6, 5) ft Solution: The vector from the origin to point B is F x r D 11i C 4k ft B (11, 0, 4) ft From Active Example 4.4 we know that the force F is z F D 40i C 70j  40k lb The moment of F about the origin is    i j k   M D r ð F D  11 0 4  D 280i C 280j C 770k ft-lb  40 70 40  M D 280i C 280j C 770k ft-lb Problem 4.46 Use Eq. (4.2) to determine the moment of the 80-N force about the origin O letting r be the vector (a) from O to A; (b) from O to B. Solution: (a) MO D rOA ð F y D 6i ð 80j D 480k N-m. 80j (N) B MO D rOB ð F (b) (6, 4, 0) m D 6i C 4j ð 80j D 480k N-m. O x A (6, 0, 0) m Problem 4.47 A bioengineer studying an injury sustained in throwing the javelin estimates that the magnitude of the maximum force exerted was jFj D 360 N and the perpendicular distance from O to the line of action of F was 550 mm. The vector F and point O are contained in the xy plane. Express the moment of F about the shoulder joint at O as a vector. Solution: The magnitude of the moment is jFj0.55 m D 360 N 0.55 m D 198 N-m. The moment vector is perpendicular to the xy plane, and the right-hand rule indicates it points in the positive z direction. Therefore MO D 198k N-m. y F y 550 mm F O O x x c 2008 Pearson Education, Inc., Upper Saddle River, NJ. All rights reserved. This material is protected under all copyright laws as they  currently exist. No portion of this material may be reproduced, in any form or by any means, without permission in writing from the publisher. 183 y Problem 4.48 Use Eq. (4.2) to determine the moment of the 100-kN force (a) about A, (b) about B. A Solution: (a) The coordinates of A are (0,6,0). The coordinates of the point of application of the force are (8,0,0). The position vector from A to the point of application of the force is rAF D 8  0i C 0  6j D 8i  6j. The force is F D 100j (kN). The cross product is  i  rAF ð F D  8 0 100j (kN) 6m  j k  6 0  D 800k (kN-m) 100 0  B x 8m 12 m (b) The coordinates of B are (12,0,0). The position vector from B to the point of application of the force is rBF D 8  12 i D 4i. The cross product is:   i  rBF ð F D  4  0  j k  0 0  D 400k (kN-m) 100 0  Problem 4.49 The cable AB exerts a 200-N force on the support at A that points from A toward B. Use Eq. (4.2) to determine the moment of this force about point P in two ways: (a) letting r be the vector from P to A; (b) letting r be the vector from P to B. y P (0.9, 0.8) m (0.3, 0.5) m Solution: First we express the force as a vector. The force points in the same direction as the position vector AB. AB D 1  0.3 mi C 0.2  0.5 mj D 0.7i  0.3j m jABj D  p 0.7 m2 C 0.3 m2 D 0.58 m A B (1, 0.2) m x 200 N 0.7i  0.3j FD p 0.58 (a) 200 N 0.7i  0.3j MP D PA ð F D 0.6 mi  0.3 mj ð p 0.58 Carrying out the cross product we find MP D 102.4 N-mk (b) 200 N 0.7i  0.3j MP D PB ð F D 0.1 mi  0.6 mj ð p 0.58 Carrying out the cross product we find MP D 102.4 N-mk 184 c 2008 Pearson Education, Inc., Upper Saddle River, NJ. All rights reserved. This material is protected under all copyright laws as they  currently exist. No portion of this material may be reproduced, in any form or by any means, without permission in writing from the publisher. Problem 4.50 The line of action of F is contained in the xy plane. The moment of F about O is 140k (Nm), and the moment of F about A is 280k (N-m). What are the components of F? y A (0, 7, 0) m F (5, 3, 0) m Solution: The strategy is to find the moments in terms of the components of F and solve the resulting simultaneous equations. The position vector from O to the point of application is rOF D 5i C 3j. The position vector from A to the point of application is rAF D 5  0i C 3  7j D 5i  4j. The cross products:   i  rOF ð F D  5  FX j 3 FY  k  0  D 5FY  3FX k D 140k, and 0   i  rAF ð F D  5  FX j 4 FY  k  0  D 5FY C 4FX k D 280k. 0 Take the dot product of both sides with k to eliminate k. The simultaneous equations are: x O 5FY  3FX D 140, 5FY C 4FX D 280. Solving: FY D 40, FX D 20, from which F D 20i C 40j (N) y A (0,7,0) F (5,3,0) x O Problem 4.51 Use Eq. (4.2) to determine the sum of the moments of the three forces (a) about A, (b) about B. y 6 kN 3 kN Solution: (a) 3 kN B A x MA D 0.2i ð 3j C 0.4i ð 6j C 0.6i ð 3j 0.2 m 0.2 m 0.2 m 0.2 m D O. (b) MB D 0.2i ð 3j C 0.4i ð 6j C 0.6i ð 3j D O. Problem 4.52 Three forces are applied to the plate. Use Eq. (4.2) to determine the sum of the moments of the three forces about the origin O. Solution: The position vectors from O to the points of application of the forces are: rO1 D 3j, F1 D 200i; rO2 D 10i, F2 D 500j; rO3 D 6i C 6j, F3 D 200i. y 200 lb 3 ft 200 lb 3 ft The sum of the moments about O is    i  MO D  0  200   j k   i 3 0  C  10 0 0  0   j k   i 0 0  C  6 500 0   200  j k  6 0  lb 0 0 O x 6 ft 4 ft 500 lb D 600k  5000k  1200k D 5600k ft-lb c 2008 Pearson Education, Inc., Upper Saddle River, NJ. All rights reserved. This material is protected under all copyright laws as they  currently exist. No portion of this material may be reproduced, in any form or by any means, without permission in writing from the publisher. 185 Problem 4.53 Three forces act on the plate. Use Eq. (4.2) to determine the sum of the moments of the three forces about point P. y 4 kN 45⬚ Solution: 3 kN r1 D 0.12i C 0.08j m, F1 D 4 cos 45° i C 4 sin 45° j kN 30⬚ 0.18 m P 0.10 m r2 D 0.16i m, F2 D 3 cos 30° i C 3 sin 30° j kN r3 D 0.16i  0.1j m, F3 D 12 cos 20° i  12 sin 20° j kN 20⬚ 0.12 m 12 kN 0.28 m MP D r1 ð F1 C r2 ð F2 C r3 ð F3 x MP D 0.145 kN-mk D 145 N-mk Problem 4.54 (a) Determine the magnitude of the moment of the 150-N force about A by calculating the perpendicular distance from A to the line of action of the force. y (0, 6, 0) m 150k (N) (b) Use Eq. (4.2) to determine the magnitude of the moment of the 150-N force about A. A Solution: (a) x (6, 0, 0) m The perpendicular from A to the line of action of the force lies in the xy plane d p D z 62 C 62 D 8.485 m jMj D dF D 8.485150 D 1270 N-m (b) M D 6i C 6j ð 150k D 900j C 900i N-m p jMj D 186 9002 C 9002 D 1270 N-m c 2008 Pearson Education, Inc., Upper Saddle River, NJ. All rights reserved. This material is protected under all copyright laws as they  currently exist. No portion of this material may be reproduced, in any form or by any means, without permission in writing from the publisher. y Problem 4.55 (a) Determine the magnitude of the moment of the 600-N force about A by calculating the perpendicular distance from A to the line of action of the force. A (0.6, 0.5, 0.4) m (b) Use Eq. (4.2) to determine the magnitude of the moment of the 600-N force about A. Solution: (a) x 0.8 m Choose some point Px, 0, 0.8 m. on the line of action of the force. The distance from A to P is then  d D x  0.6 m2 C 0  0.5 m2 C 0.8 m  0.4 m2 600i (N) z The perpendicular distance is the shortest distance d which occurs when x D 0.6 m. We have d D 0.6403 m. Thus the magnitude of the moment is M D 600 N0.6403 m D 384 N-m (b) Define the point on the end of the rod to be B. Then AB D 0.6i  0.5j C 0.4k m we have M D AB ð F D 0.6i  0.5j C 0.4k m ð 600 Ni M D 240j C 300k N-m Thus the magnitude is MD  240 Nm2 C 300 Nm2 D 384 N-m y Problem 4.56 what is the magnitude of the moment of F about point B? A (4, 4, 2) ft Solution: The position vector from B to A is B (8, 1, ⫺2) ft x rBA D [4  8i C 4  1j C 2  2k] ft z rBA D 4i C 3j C 4k ft The moment of F about B   i  MB D rBA ð F D  4  20 F ⫽ 20i ⫹ 10j ⫺ 10k (lb) is j 3 10  k  4  D 70i C 40j  100k ft-lb 10  Its magnitude is jMB j D  70 ft-lb2 C 40 ft-lb2 C 100 ft-lb2 D 128 ft-lb jMB j D 128 ft-lb c 2008 Pearson Education, Inc., Upper Saddle River, NJ. All rights reserved. This material is protected under all copyright laws as they  currently exist. No portion of this material may be reproduced, in any form or by any means, without permission in writing from the publisher. 187 Problem 4.57 In Example 4.5, suppose that the attachment point C is moved to the location (8,2,0) m and the tension in cable AC changes to 25 kN. What is the sum of the moments about O due to the forces excerted on the attachment point A by the two cables? Solution: The position vector from A to C is y C (6, 3, 0) m B (0, 4, 8) m O x A (4, 0, 6) m rAC D [8  4i C 2  0j C 0  6k] m z rAC D 4i C 2j  6k The force exerted at A by cable AC can be written FAC D 25 kN rAC D 13.4i C 6.68j  20.0k kN jrAC j The total force exerted at A by the two cables is F D FAB C FAC D 6.70i C 13.3j  16.7k kN The moment about O is   i j  0 MO D rAB ð F D  4  6.70 13.3  k  6  D 80.1i C 107j C 53.4k kN-m 16.7  MO D 80.1i C 107j C 53.4k kN-m y Problem 4.58 The rope exerts a force of magnitude jFj D 200 lb on the top of the pole at B. Determine the magnitude of the moment of F about A. Solution: The position vector from B to C is B (5, 6, 1) ft F rBC D [3  5i C 0  6j C 4  1k] ft A x rBC D 2i  6j C 3k ft C (3, 0, 4) ft The force F can be written F D 200 lb rBC D 57.1i  171j C 85.7k lb jrBC j The moment of F about A is   i j  MA D rAB ð F D  5 6  57.1 171 z  k  1  85.7  D 686i  486j  514k ft-lb Its magnitude is jMA j D  686 ft-lb2 C 486 ft-lb2 C 514 ft-lb2 D 985 ft-lb jMA j D 985 ft-lb 188 c 2008 Pearson Education, Inc., Upper Saddle River, NJ. All rights reserved. This material is protected under all copyright laws as they  currently exist. No portion of this material may be reproduced, in any form or by any means, without permission in writing from the publisher. y Problem 4.59 The force F D 30i C 20j  10k (N). (a) Determine the magnitude of the moment of F about A. (b) Suppose that you can change the direction of F while keeping its magnitude constant, and you want to choose a direction that maximizes the moment of F about A. What is the magnitude of the resulting maximum moment? Solution: The vector from A to the point of application of F is F A (8, 2, – 4) m (4, 3, 3) m x z r D 4i  1j  7k m and p jrj D (a) 42 C 12 C 72 D 8.12 m The moment of F about A is MA   i j  D r ð F D  4 1  30 20 jMA j D (b)  k  7  D 150i  170j C 110k N-m 10  p 1502 C 1702 C 1102 D 252 N-m The maximum moment occurs when r ? F. In this case jMAmax j D jrjjFj Hence, we need jFj. p jFj D 302 C 202 C 102 D 37.4 N Thus, jMAmax j D 8.1237.4 D 304 N-m Problem 4.60 The direction cosines of the force F are cos x D 0.818, cos y D 0.182, and cos z D 0.545. The support of the beam at O will fail if the magnitude of the moment of F about O exceeds 100 kN-m. Determine the magnitude of the largest force F that can safely be applied to the beam. y z O F 3m Solution: The strategy is to determine the perpendicular distance x from O to the action line of F, and to calculate the largest magnitude of F from MO D DjFj. The position vector from O to the point of application of F is rOF D 3i (m). Resolve the position vector into components parallel and normal to F. The component parallel to F is rP D rOF Ð eF eF , where the unit vector eF parallel to F is eF D i cos X C j cos Y C k cos Z D 0.818i C 0.182j  0.545k. The dot product is rOF Ð eF D 2.454. The parallel component is rP D 2.007i C 0.4466j  1.3374k. The component normal to F is rN D rOF  rP D 3  2i  0.4466j C 1.3374k. The magnitude p of the normal component is the perpendicular distance: D D 12 C 0.44662 C 1.3372 D 1.7283 m. The maximum moment allowed is MO D 1.7283jFj D 100 kN-m, from which jFj D 100 kN-m D 57.86 ¾ D 58 kN 1.7283 m c 2008 Pearson Education, Inc., Upper Saddle River, NJ. All rights reserved. This material is protected under all copyright laws as they  currently exist. No portion of this material may be reproduced, in any form or by any means, without permission in writing from the publisher. 189 Problem 4.61 The force F exerted on the grip of the exercise machine points in the direction of the unit vector e D 23 i  23 j C 13 k and its magnitude is 120 N. Determine the magnitude of the moment of F about the origin O. 150 mm y F Solution: The vector from O to the point of application of the O 200 mm force is z r D 0.25i C 0.2j  0.15k m 250 mm x and the force is F D jFje or F D 80i  80j C 40k N. The moment of F about O is   i j  MO D r ð F D  0.25 0.2  80 80  k  0.15  N-m 40  or MO D 4i  22j  36k N-m and p jMO j D 42 C 222 C 362 N-m jMO j D 42.4 N-m Problem 4.62 The force F in Problem 4.61 points in the direction of the unit vector e D 23 i  23 j C 13 k. The support at O will safely support a moment of 560 N-m magnitude. (a) Based on this criterion, what is the largest safe magnitude of F? (b) If the force F may be exerted in any direction, what is its largest safe magnitude? Solution: See the figure of Problem 4.61. The moment in Problem 4.61 can be written as   i j  MO D  0.25 0.2  2F 2F 3 3  k  0.15  where F D jFj C 31 F  MO D 0.0333i  0.1833j  0.3kF And the magnitude of MO is p jMO j D  0.03332 C 0.18332 C 0.32 F jMO j D 0.353 F 190 If we set jMO j D 560 N-m, we can solve for jFmax j 560 D 0.353jFmax j jFmax j D 1586 N (b) If F can be in any direction, then the worst case is when r ? F. The moment in this case is jMO j D jrjjFworst j jrj D p 0.252 C 0.22 C 0.152 D 0.3536 m 560 D 0.3536jFWORST j jFworst j D 1584 N c 2008 Pearson Education, Inc., Upper Saddle River, NJ. All rights reserved. This material is protected under all copyright laws as they  currently exist. No portion of this material may be reproduced, in any form or by any means, without permission in writing from the publisher. Problem 4.63 A civil engineer in Boulder, Colorado estimates that under the severest expected Chinook winds, the total force on the highway sign will be F D 2.8i  1.8j (kN). Let MO be the moment due to F about the base O of the cylindrical column supporting the sign. The y component of MO is called the torsion exerted on the cylindrical column at the base, and the component of MO parallel to the xz plane is called the bending moment. Determine the magnitudes of the torsion and bending moment. y F 8m 8m O x Solution: The total moment is z M D 8j C 8k m ð 2.8i  1.8j kN D 14.4i C 22.4j  22.4k kN-m We now identify Torsion D My D 22.4 kN-m  Bending moment D Mx 2 C Mz 2  D 14.4 kNm2 C 22.4 kNm2 D 26.6 kN-m Problem 4.64 The weights of the arms OA and AB of the robotic manipulator act at their midpoints. The direction cosines of the centerline of arm OA are cos x D 0.500, cos y D 0.866, and cos z D 0, and the direction cosines of the centerline of arm AB are cos x D 0.707, cos y D 0.619, and cos z D 0.342. What is the sum of the moments about O due to the two forces? Solution: By definition, the direction cosines are the scalar components of the unit vectors. Thus the unit vectors are e1 D 0.5i C 0.866j, and e2 D 0.707i C 0.619j  0.342k. The position vectors of the midpoints of the arms are r1 D 0.3e1 D 0.30.5i C 0.866j D 0.15i C 0.2598j r2 D 0.6e1 C 0.3e2 D 0.512i C 0.7053j  0.1026k. The sum of moments is y 0 60 mm B 160 N M D r1 ð W 1 C r 2 ð W 2     i j k   i   D  0.15 0.2598 0  C  0.512  0 200 0   0 j 0.7053 160   k  0.1026   0 D 16.42i  111.92k (N-m) 600 mm A 200 N O z x c 2008 Pearson Education, Inc., Upper Saddle River, NJ. All rights reserved. This material is protected under all copyright laws as they  currently exist. No portion of this material may be reproduced, in any form or by any means, without permission in writing from the publisher. 191 Problem 4.65 The tension in cable AB is 100 lb. If you want the magnitude of the moment about the base O of the tree due to the forces exerted on the tree by the two ropes to be 1500 ft-lb, what is the necessary tension in rope AC ? Solution: We have the forces 100 lb TAC 8j C 10k, F2 D p 14i  8j C 14k F1 D p 164 456 Thus the total moment is M D 8 ftj ð F1 C F2  D 625 ft lb C 5.24 ft TAC i y  5.24 ftTAC K The magnitude squared is then 625 ft lb C 5.24 ft TAC 2 C 5.24 ft TAC 2 D 1500 ft lb2 Solving we find TAC D 134 lb (0, 8, 0) ft A x O B (0, 0, 10) ft (14, 0, 14) ft C z Problem 4.66* A force F acts at the top end A of the pole. Its magnitude is jFj D 6 kN and its x component is Fx D 4 kN. The coordinates of point A are shown. Determine the components of F so that the magnitude of the moment due to F about the base P of the pole is as large as possible. (There are two answers.) Solution: The force is given by F D 4 kNi C Fy j C Fz k. Since the magnitude is constrained we must have 4 kN2 C Fy 2 C Fz 2 D 6 kN2 ) Fz D  20 kN2  Fy 2 Thus we will use (suppressing the units)  y FD F   4i C Fy j C 20  Fy 2 k A (4, 3, ⫺2) m The moment is now given by M D 4i C 3j  2k ð F     M D 2Fy C 3 20  Fy 2 i  8 C 4 20  Fy 2 j C 12 C 4Fy k The magnitude is P x     M2 D 708  5Fy 2 C 64 20  Fy 2 C 12Fy 8 C 20  Fy 2 To maximize this quantity we solve z dM2 D 0 for the critical values dFy of Fy . There are three solutions Fy D 4.00, 3.72, 3.38. The first and third solutions produce the same maximum moment. The second answer corresponds to a local minimum and is therefore discarded. So the force that produces the largest moment is F D 4i  4j C 2k 192 or F D 4i  3.38j C 2.92k c 2008 Pearson Education, Inc., Upper Saddle River, NJ. All rights reserved. This material is protected under all copyright laws as they  currently exist. No portion of this material may be reproduced, in any form or by any means, without permission in writing from the publisher. y Problem 4.67 The force F D 5i (kN) acts on the ring A where the cables AB, AC, and AD are joined. What is the sum of the moments about point D due to the force F and the three forces exerted on the ring by the cables? D (0, 6, 0) m A Strategy: The ring is in equilibrium. Use what you know about the four forces acting on it. C B Solution: The vector from D to A is rDA D 12i  2j C 2k m. The sum of the moments about point D is given by   F (12, 4, 2) m (6, 0, 0) m x (0, 4, 6) m z FAD A F MD D rDA ð FAD C rDA ð FAC C rDA ð FAB C rDA ð F MD D rDA ð FAD C FAC C FAB C F FAC FAB However, we are given that ring A is in equilibrium and this implies that FAD C FAC C FAB C F D O D 0 Thus,  MD D rDA ð O D 0 c 2008 Pearson Education, Inc., Upper Saddle River, NJ. All rights reserved. This material is protected under all copyright laws as they  currently exist. No portion of this material may be reproduced, in any form or by any means, without permission in writing from the publisher. 193 Problem 4.68 In Problem 4.67, determine the moment about point D due to the force exerted on the ring A by the cable AB. Solution: We need to write the forces as magnitudes times the D(0, 6, 0) appropriate unit vectors, write the equilibrium equations for A in component form, and then solve the resulting three equations for the three unknown magnitudes. The unit vectors are of the form FAD FAC eAP D A(12, 4, 2) m F = 5i (kN) xP  xA i C yP  yA j C zP  zA k jrAP j C(0, 4, 6) m Where P takes on values B, C, and D B(6, 0, 0) m Calculating the unit vectors, we get  e D 0.802i  0.535j  0.267k   AB eAC D 0.949i C 0j C 0.316k   eAD D 0.973i C 0.162j  0.162k From equilibrium, we have FAB eAB C FAC eAC C FAD eAD C 5i kN D 0 In component form, we get  i: 0.802FAB  0.949FAC  0.973FAD C 5 D 0   j: 0.535FAB C 0FAC C 0.162FAD D 0   k: 0.267FAB C 0.316FAC  0.162FAD D 0 Solving, we get FAB D 779.5 N, FAC D 1976 N FAD D 2569 N The vector from D to A is rDA D 12i  2j C 2k m The force FAB is given by FAB D FAB eAB FAB D 0.625i  0.417j  0.208k kN The moment about D is given by MD D rDA ð FAB   i  D  12  0.625 j 2 0.417  k  2  0.208  MD D 1.25i C 1.25j  6.25k kN-m 194 c 2008 Pearson Education, Inc., Upper Saddle River, NJ. All rights reserved. This material is protected under all copyright laws as they  currently exist. No portion of this material may be reproduced, in any form or by any means, without permission in writing from the publisher. Problem 4.69 The tower is 70 m tall. The tensions in cables AB, AC, and AD are 4 kN, 2 kN, and 2 kN, respectively. Determine the sum of the moments about the origin O due to the forces exerted by the cables at point A. y A D 35 m B 35 m 40 m C O x 40 m 40 m z Solution: The coordinates of the points are A (0, 70, 0), B (40, 0, 0), C (40, 0, 40) D(35, 0, 35). The position vectors corresponding to the cables are: rAD D 35  0i C 0  70j C 35  0k rAD D 35i  70k  35k rAC D 40  0i C 0  70j C 40  0k rAC D 40i  70j C 40k The sum of the forces acting at A are TA D 0.2792i  6.6615j C 0.07239k (kN-m) The position vector of A is rOA D 70j. The moment about O is M D rOA ð TA   i  M D  0  0.2792 j 70 6.6615   k   0  0.07239  D 700.07239i  j0  k700.2792 D 5.067i  19.54k rAB D 40  0i C 0  70j C 0  0k rAB D 40i  70j C 0k The unit vectors corresponding to these position vectors are: eAD D rAD 35 70 35 D i j jrAD j 85.73 85.73 85.73 D 0.4082i  0.8165j  0.4082k eAC D rAC 40 70 40 D i jC k jrAC j 90 90 90 D 0.4444i  0.7778j C 0.4444k eAB D 40 70 rAB D i j C 0k D 0.4962i  0.8682j C 0k jrAB j 80.6 80.6 The forces at point A are TAB D 4eAB D 1.9846i  3.4729j C 0k TAC D 2eAB D 0.8889i  1.5556j C 0.8889k TAD D 2eAD D 0.8165i  1.6330j  0.8165k. c 2008 Pearson Education, Inc., Upper Saddle River, NJ. All rights reserved. This material is protected under all copyright laws as they  currently exist. No portion of this material may be reproduced, in any form or by any means, without permission in writing from the publisher. 195 Problem 4.70 Consider the 70-m tower in Problem 4.69. Suppose that the tension in cable AB is 4 kN, and you want to adjust the tensions in cables AC and AD so that the sum of the moments about the origin O due to the forces exerted by the cables at point A is zero. Determine the tensions. Solution: From Varignon’s theorem, the moment is zero only if the resultant of the forces normal to the vector rOA is zero. From Problem 4.69 the unit vectors are: eAD 35 70 35 rAD D i j D jrAD j 85.73 85.73 85.73 The tensions are TAB D 4eAB , TAC D jTAC jeAC , and TAD D jTAD jeAD . The components normal to rOA are   D 0.4082i  0.8165j  0.4082k eAC D 40 70 40 rAC D i jC k jrAC j 90 90 90 D 0.4444i  0.7778j C 0.4444k eAB D FX D 0.4082jTAD j  0.4444jTAC j C 1.9846i D 0 FZ D 0.4082jTAD j C 0.4444jTAC jk D 0. The HP-28S calculator was used to solve these equations: jTAC j D 2.23 kN, jTAD j D 2.43 kN rAB 40 70 D i j C 0k D 0.4963i  0.8685j C 0k jrAB j 80.6 80.6 Problem 4.71 The tension in cable AB is 150 N. The tension in cable AC is 100 N. Determine the sum of the moments about D due to the forces exerted on the wall by the cables. y 5m 5m B C Solution: The coordinates of the points A, B, C are A (8, 0, 0), B (0, 4, 5), C (0, 8, 5), D(0, 0, 5). The point A is the intersection of the lines of action of the forces. The position vector DA is 4m 8m 8m D rDA D 8i C 0j  5k. A z x The position vectors AB and AC are rAB D 8i C 4j  5k, rAB D rAC D 8i C 8j C 5k, rAC D p 82 C 42 C 52 D 10.247 m. p 82 C 82 C 52 D 12.369 m. The unit vectors parallel to the cables are: eAB D 0.7807i C 0.3904j  0.4879k, eAC D 0.6468i C 0.6468j C 0.4042k.   i  MD D  8  181.79 j 0 123.24  k  5  D 123.245i C32.77   8C32.77  5181.79j C 8123.24k MD D 616.2i  117.11j  985.9k (N-m) (Note: An alternate method of solution is to express the moment in terms of the sum: MD D rDC ð TC C rDB ð TB . The tensions are y TAB D 150eAB D 117.11i C 58.56j  73.19k, 5m TAC D 100eAC D 64.68i C 64.68j C 40.42k. The sum of the forces exerted by the wall on A is 5m B 4m C TA D 181.79i C 123.24j  32.77k. The force exerted on the wall by the cables is TA . The moment about D is MD D rDA ð TA , 196 A 8m z D 8m F x c 2008 Pearson Education, Inc., Upper Saddle River, NJ. All rights reserved. This material is protected under all copyright laws as they  currently exist. No portion of this material may be reproduced, in any form or by any means, without permission in writing from the publisher. Problem 4.72 Consider the wall shown in Problem 4.71. The total force exerted by the two cables in the direction perpendicular to the wall is 2 kN. The magnitude of the sum of the moments about D due to the forces exerted on the wall by the cables is 18 kN-m. What are the tensions in the cables? Solution: From the solution of Problem 4.71, we have rDA D 8i C 0j  5k. Forces in both cables pass through point A and we can use this vector to determine moments of both forces about D. The position vectors AB and AC are p rAB D 8i C 4j  5k, jrAB j D rAC D 8i C 8j C 5k, jrAC j D 82 C 42 C 52 D 10.247 m. p 82 C 82 C 52 D 12.369 m. The unit vectors parallel to the cables are: eAB D 0.7807i C 0.3904j  0.4879k, eAC D 0.6468i C 0.6468j C 0.4042k. The tensions are TBA D TBA eAB D TBA 0.7807i C 0.3904j  0.4879k, and TCA D TCA eAC D TCA 0.6468i C 0.6468j C 0.4042k. The sum of the forces exerted by the cables perpendicular to the wall is given by TPerpendicular D TAB 0.7807 C TAC 0.6468 D 2 kN. The moments of these two forces about D are given by MD D rDA ð TCA  C rDA ð TBA  D rDA ð TCA C TBA . The sum of the two forces is given by   i  8 MD D   TCA C TCB X j 0 TCA C TCB Y   k  . 5  TCA C TCB Z  This expression can be expanded to yield MD D 5TCA C TCB Y i C [8TCA C TCB Z  5TCA C TCB X ]j C 8TCA C TCB Y k. The magnitude of this vector is given as 18 kN-m. Thus, we obtain the relation  jMD j D 25TCA C TCB 2Y C [8TCA C TCB Z D 18 kN-m. 5TCA C TCB X ]2 C 64TCA C TCB 2Y We now have two equations in the two tensions in the cables. Either algebraic substitution or a numerical solver can be used to give TBA D 1.596 kN, and TCA D 1.166 kN. c 2008 Pearson Education, Inc., Upper Saddle River, NJ. All rights reserved. This material is protected under all copyright laws as they  currently exist. No portion of this material may be reproduced, in any form or by any means, without permission in writing from the publisher. 197 Problem 4.73 The tension in the cable BD is 1 kN. As a result, cable BD exerts a 1-kN force on the “ball” at B that points from B toward D. Determine the moment of this force about point A. Solution: We have the force and position vectors FD 1 kN 4i C 2j C 4k, r D AB D 4i C 3j C k m 6 The moment is then y M D r ð F D 1.667i  3.33j C 3.33k kN-m C (0, 4, ⫺3) m B (4, 3, 1) m D (0, 5, 5) m x A E z Problem 4.74* Suppose that the mass of the suspended object E in Problem 4.73 is 100 kg and the mass of the bar AB is 20 kg. Assume that the weight of the bar acts at its midpoint. By using the fact that the sum of the moments about point A due to the weight of the bar and the forces exerted on the “ball” at B by the three cables BC, BD, and BE is zero, determine the tensions in the cables BC and BD. Solution: We have the following forces applied at point B. F1 D 100 kg9.81 m/s2 j, F3 D TBC F2 D p 4i C j  4k, 33 TBD 4i C 2j C 4k 6 In addition we have the weight of the bar F4 D 20 kg9.81 m/s2 j The moment around point A is MA D 4i C 3j C k m ð F1 C F2 C F3  C 2i C 1.5j C 0.5k m ð F4 D 0 Carrying out the cross products and breaking into components we find Mx D 1079  2.26TBC C 1.667TBD D 0 My D 2.089TBC  3.333TBD D 0 Mz D 4316 C 2.785TBC C 3.333TBD D 0 Only two of these three equations are independent. Solving we find TBC D 886 N, TBD D 555 N 198 c 2008 Pearson Education, Inc., Upper Saddle River, NJ. All rights reserved. This material is protected under all copyright laws as they  currently exist. No portion of this material may be reproduced, in any form or by any means, without permission in writing from the publisher. Problem 4.75 The 200-kg slider at A is held in place on the smooth vertical bar by the cable AB. Determine the moment about the bottom of the bar (point C with coordinates x D 2 m, y D z D 0) due to the force exerted on the slider by the cable. y 2m B A 5m 2m 2m C x z Solution: The slider is in equilibrium. The smooth bar exerts no vertical forces on the slider. Hence, the vertical component of FAB supports the weight of the slider. FAB The unit vector from A to B is determined from the coordinates of points A and B A2, 2, 0, B0, 5, 2 m Thus, H rAB D 2i C 3j C 2k m eAB D 0.485i C 0.728j C 0.485k and −mg j FAB D FAB eAB The horizontal force exerted by the bar on the slider is H D Hx i C H z k Equilibrium requires H C FAB  mgj D 0 i: Hx  0.485FAB D 0 m D 200 kg j: 0.728FAB  mg D 0 g D 9.81 m/s2 k: Hz C 0.485FAB D 0 Solving, we get FAB D 2697N D 2, 70 kN Hx D 1308N D 1.31 kN Hz D 1308N D 1.31 kN rCA D 2j m FAB D FAB eAB FAB D 1308i C 1962j C 1308k N   i  Mc D  0  1308 j 2 1962  k  0  1308  Mc D 2616i C 0j C 2616k N-m Mc D 2.62i C 2.62i kN-m c 2008 Pearson Education, Inc., Upper Saddle River, NJ. All rights reserved. This material is protected under all copyright laws as they  currently exist. No portion of this material may be reproduced, in any form or by any means, without permission in writing from the publisher. 199 y Problem 4.76 To evaluate the adequacy of the design of the vertical steel post, you must determine the moment about the bottom of the post due to the force exerted on the post at B by the cable AB. A calibrated strain gauge mounted on cable AC indicates that the tension in cable AC is 22 kN. What is the moment? 5m 5m C D 4m 8m (6, 2, 0) m B A O z 3m 12 m x D(0, 4, −5) m Solution: To find the moment, we must find the force in cable AB. In order to do this, we must find the forces in cables AO and AD also. This requires that we solve the equilibrium problem at A. Our first task is to write unit vectors eAB , eAO , eAC , and eAD . Each will be of the form eAi xi  xA i C yi  yA j C zi  zA k D  xi  xA 2 C yi  yA 2 C zi  zA 2 where i takes on the values B, C, D, and O. We get eAB D 0.986i C 0.164j C 0k eAC D 0.609i C 0.609j C 0.508k eAD D 0.744i C 0.248j  0.620k C (0, 8, 5) m TAD TAC A (6, 2, 0) m TAO B(12, 3, 0) m TAB O(0, 0, 0) m In component form,  T e C TAC eACx C TAD eADx C TAO eAOx D 0   AB ABx TAB eABy C TAC eACy C TAD eADy C TAO eAOy D 0   TAB eABz C TAC eACz C TAD eADz C TAO eAOz D 0 We know TAC D 22 kN. Substituting this in, we have 3 eqns in 3 unknowns. Solving, we get eAO D 0.949i  0.316j C 0k We now write the forces as TAB D 163.05 kN, TAD D 18.01 kN TAO D 141.28 kN We now know that TAB is given as TAB D TAB eAB TAB D TAB eAB D 160.8i C 26.8j kN TAC D TAC eAC and that the force acting at B is TAB . TAD D TAD eAD The moment about the bottom of the post is given by TAO D TAO eAO MBOTTOM D r ð TAB  D 3j ð TAB  We then sum the forces and write the equilibrium equations in component form. For equilibrium at A,  200  FA D 0 Solving, we get MBOTTOM D 482k kN-m FA D TAB C TAC C TAD C TAO D 0. c 2008 Pearson Education, Inc., Upper Saddle River, NJ. All rights reserved. This material is protected under all copyright laws as they  currently exist. No portion of this material may be reproduced, in any form or by any means, without permission in writing from the publisher. Problem 4.77 The force F D 20i C 40j  10k (N). Use both of the procedures described in Example 4.7 to determine the moment due to F about the z axis. y F Solution: First Method: We can use Eqs. (4.5) and (4.6) r D 8i m x (8, 0, 0) m F D 20i C 40j  10k N Mzaxis D [k Ð r ð F]k   0  jMzaxis j D k Ð r ð F D  8 m  20 N 01 0 40 N    0  D 320 N-m 10 N  z Mzaxis D 320 N-mk Second Method: The y-component of the force is perpendicular to the plane containing the z axis and the position vector r. The perpendicular distance from the z axis to the y-component of the force is 8 m. Therefore jMzaxis j D 40 N8 m D 320 N-m Using the right-hand rule we see that the moment is about the Cz axis. Thus Mzaxis D 320 N-mk Problem 4.78 Use Eqs. (4.5) and (4.6) to determine the moment of the 20-N force about (a) the x axis, (b) the y axis, (c) the z axis. (First see if you can write down the results without using the equations.) Solution: The force is parallel to the z axis. The perpendicular distance from the x axis to the line of action of the force is 4 m. The perpendicular distance frompthe y axis p is 7 m and the perpendicular distance from the z axis is 42 C 72 D 65 m. By inspection, the moment about the x axis is y Mx D 420i (N-m) (7, 4, 0) m Mx D 80i N-m By inspection, the moment about the y axis is My D 720j N-m 20 k (N) x My D 140j (N-m) z By inspection, the moment about the z axis is zero since F is parallel to the z axis. Mz D 0 N-m Now for the calculations using (4.5) and (4.6) ML D [e Ð r ð F]e  1  Mx D  7 0 0 4 0  0  0  i D 80i N-m 20   0  My D  7 0 1 4 0  0  0  j D 140j N-m 20   0  Mz D  7 0 0 4 0  1  0  k D 0k N-m 20  c 2008 Pearson Education, Inc., Upper Saddle River, NJ. All rights reserved. This material is protected under all copyright laws as they  currently exist. No portion of this material may be reproduced, in any form or by any means, without permission in writing from the publisher. 201 y Problem 4.79 Three forces parallel to the y axis act on the rectangular plate. Use Eqs. (4.5) and (4.6) to determine the sum of the moments of the forces about the x axis. (First see if you can write down the result without using the equations.) 3 kN x 2 kN 6 kN 600 mm 900 mm z Solution: By inspection, the 3 kN force has no moment about the M6 x axis since it acts through the x axis. The perpendicular distances of the other two forces from the x axis is 0.6 m. The H 2 kN force has a positive moment and the 6 kN force has a negative about the x axis.    Mx D [20.6  60.6]i kN  Mx D 2.4i kN  kN  1  D  0 0 Mx D M3 0 0 6 kN  0  .6  i D 3.6i kN 0 C M2 kN C M6 kN Mx D 0 C 1.2i  3.6i kN Mx D 2.4i kN Calculating the result: M3 M2 kN kN  1  D  0 0 0 0 3  0  0  i D 0i kN 0  1  D  0 0 0 0 2  0  .6  i D 1.2i kN 0 Problem 4.80 Consider the rectangular plate shown in Problem 4.79. The three forces are parallel to the y axis. Determine the sum of the moments of the forces (a) about the y axis, (b) about the z axis. Solution: (a) The magnitude of the moments about the y axis is M D eY Ð r ð F. The position vectors of the three forces are given in the solution to Problem 4.79. The magnitude for each force is:   0 1 0   eY Ð r ð F D  0.9 0 0  D 0, 0 3 0   0 1  eY Ð r ð F D  0.9 0 0 6  0  0.6  D 0, 0   0  eY Ð r ð F D  0 0  0  0.6  D 0 0  1 0 2 Thus the moment about the y axis is zero, since the magnitude of each moment is zero. 202 (b) The magnitude of each moment about the z axis is  0 1  eZ Ð r ð F D  0.9 0 0 3  0  0  D 2.7, 0   0 0  eZ Ð r ð F D  0.9 0 0 C 6  1  0.6  D 5.4, 0   0  eZ Ð r ð F D  0 0 0 0 2  1  0.6  D 0. 0  Thus the moment about the z axis is  MZ D 2.7eZ C 5.4eZ D 2.7k (kN-m) c 2008 Pearson Education, Inc., Upper Saddle River, NJ. All rights reserved. This material is protected under all copyright laws as they  currently exist. No portion of this material may be reproduced, in any form or by any means, without permission in writing from the publisher. Problem 4.81 The person exerts a force F D 0.2i  0.4j C 1.2k (lb) on the gate at C. Point C lies in the xy plane. What moment does the person exert about the gate’s hinge axis, which is coincident with the y axis? y A C Solution: 3.5 ft M D [e Ð r ð F]e e D j,  0  MY D  2  .2 r D 2i ft, F is given x  1 0  0 0  j D 2.4j ft-lb .4 1.2  B 2 ft y Problem 4.82 Four forces act on the plate. Their components are FB FA D 2i C 4j C 2k (kN), FA FB D 3j  3k (kN), x FD FC FC D 2j C 3k (kN), FD D 2i C 6j C 4k (kN). Determine the sum of the moments of the forces (a) about the x axis; (b) about the z axis. z 2m 3m Solution: Note that FA acts at the origin so no moment is generated about the origin. For the other forces we have      i j k  j k   i      0 2m 0 0 C3 m MO D  3 m      0 2 kN 3 kN  3 kN 3 kN   0   i   C 0   2 kN j 0 6 kN  k   2m  4 kN  MO D 16i C 4j C 15k kN-m Now we find Mx D MO Ð i D 16 kN-m, Mz D MO Ð k D 15 kN-m c 2008 Pearson Education, Inc., Upper Saddle River, NJ. All rights reserved. This material is protected under all copyright laws as they  currently exist. No portion of this material may be reproduced, in any form or by any means, without permission in writing from the publisher. 203 y Problem 4.83 The force F D 30i C 20j  10k (lb). (a) What is the moment of F about the y axis? (b) Suppose that you keep the magnitude of F fixed, but you change its direction so as to make the moment of F about the y axis as large as possible. What is the magnitude of the resulting moment? F (4, 2, 2) ft Solution: x z (a) My D j Ð [4i C 2j C 2k ft ð 30i C 20j  10k lb]   0   My D  4 ft   30 lb     2 ft 2 ft  D 100 ft lb  20 lb 10 lb  1 0 ) My D 100 ft-lbj (b) p p Mymax D Fd D  302 C 202 C 102 lb 42 C 22 ft D 167.3 ft-lb Note that d is the distance from the y axis, not the distance from the origin. Problem 4.84 The moment of the force F shown in Problem 4.83 about the x axis is 80i (ft-lb), the moment about the y axis is zero, and the moment about the z axis is 160k (ft-lb). If Fy D 80 lb, what are Fx and Fz ? Solution: The magnitudes of the moments:   eX  e ž r ð F D  rX  FX eY rY FY  eZ  rZ  , FZ    0  eZ Ð r ð F D  4  FX 0 2 80  1  2  D 320  2FX D 160 FZ  Solve: FX D 80 lb, FZ D 40 lb, from which the force vector is F D 80i C 80j C 40k 204 c 2008 Pearson Education, Inc., Upper Saddle River, NJ. All rights reserved. This material is protected under all copyright laws as they  currently exist. No portion of this material may be reproduced, in any form or by any means, without permission in writing from the publisher. Problem 4.85 The robotic manipulator is stationary. The weights of the arms AB and BC act at their midpoints. The direction cosines of the centerline of arm AB are cos x D 0.500, cos y D 0.866, cos z D 0, and the direction cosines of the centerline of arm BC are cos x D 0.707, cos y D 0.619, cos z D 0.342. What total moment is exerted about the z axis by the weights of the arms? m y 0m C 60 160 N 600 mm B Solution: The unit vectors along AB and AC are of the form 200 N e D cos x i C cos y j C cos z k. The unit vectors are A eAB D 0.500i C 0.866j C 0k and eBC D 0.707i C 0.619j  0.342k. z The vector to point G at the center of arm AB is x rAG D 3000.500i C 0.866j C 0k D 150i C 259.8j C 0k mm, and the vector from A to the point H at the center of arm BC is given by rAH D rAB C rBH D 600eAB C 300eBC D 512.1i C 705.3j  102.6k mm. The weight vectors acting at G and H are WG D 200j N, and WH D 160j N. The moment vectors of these forces about the z axis are of the form   eX  e ž r ð F D  rX  FX ey rY FY  ez  rZ  . FZ  Here, WG and WH take on the role of F, and e D k. Substituting into the form for the moment of the force at G, we get   0  e ž r ð F D  0.150  0  0 1  0.260 0  D 30 N-m. 200 0  Similarly, for the moment of the force at H, we get   0 0  e ž r ð F D  0.512 0.705  0 160  1  0.103  D 81.9 N-m. 0  The total moment about the z axis is the sum of the two moments. Hence, Mz axis D 111.9 N-m c 2008 Pearson Education, Inc., Upper Saddle River, NJ. All rights reserved. This material is protected under all copyright laws as they  currently exist. No portion of this material may be reproduced, in any form or by any means, without permission in writing from the publisher. 205 Problem 4.86 In Problem 4.85, what total moment is exerted about the x axis by the weights of the arms? Solution: The solution is identical to that of Problem 4.85 except that e D i. Substituting into the form for the moment of the force at G, we get    1 0 0   e Ð r ð F D  0.150 0.260 0  D 0 N-m.  0 200 0  Similarly, for the moment of the force at H, we get   1 0  e Ð r ð F D  0.512 0.705  0 160  0  0.103  D 16.4 N-m. 0  The total moment about the x axis is the sum of the two moments. Hence, Mx axis D 16.4 N-m Problem 4.87 In Active Example 4.6, suppose that the force changes to F D 2i C 3j C 6k (kN). Determine the magnitude of the moment of the force about the axis of the bar BC. y C (0, 4, 0) m F ⫽ ⫺2i ⫹ 6j ⫹ 3k (kN) A (4, 2, 2) m Solution: We have the following vectors rBA D 4i C 2j  1k m x B z (0, 0, 3) m F D 2i C 3j C 6k kN rBC D 4j  3k m eBC D rBC D 0.8j  0.6k jrBC j The moment of F about the axis of the bar is    0 0.8 0.6    jMBC j D eBC Ð r ð F D  4 2 1  D 27.2 kN-m  2 3 6  Thus MBC D 27.2 kN-meBC , jMBC j D 27.2 kN-m 206 c 2008 Pearson Education, Inc., Upper Saddle River, NJ. All rights reserved. This material is protected under all copyright laws as they  currently exist. No portion of this material may be reproduced, in any form or by any means, without permission in writing from the publisher. Problem 4.88 Determine the moment of the 20-N force about the line AB. Use Eqs. (4.5) and (4.6), letting the unit vector e point (a) from A toward B, (b) from B toward A. y A (0, 5, 0) m (7, 4, 0) m 20k (N) B (– 4, 0, 0) m x z Solution: First, we need the unit vector Using eAB xB  xA i C yB  yA j C zB  zA k eAB D  xB  xA 2 C yB  yA 2 C zB  zA 2   0.625  ML D  7  0  0.781 0  1 0  0.625i  0.781j 0 20  eAB D 0.625i  0.781j D eBA ML D 76.1i  95.1j N-m Now, the moment of the 20k (N) force about AB is given as   ex  ML D  rx  Fx ey ry Fy  ez  rz  e Fz  Using eBA   0.625  ML D  7  0 where e is eAB or eBA  0.781 0  1 0  0.625i C 0.781j 0 20  For this problem, r must go from line AB to the point of application of the force. Let us use point A. ML D 76.1i  95.1j N-m r D 7  0i C 4  5j C 0  0k m Ł Results are the same r D 7i  1j C 0k m Problem 4.89 The force F D 10i C 5j  5k (kip). Determine the moment of F about the line AB. Draw a sketch to indicate the sense of the moment. y B (6, 6, 0) ft Solution: The moment of F about pt. A is MA D 6i ð F   i j  D  6 0  10 5 F  k  0  5  A x (6, 0, 0) ft z D 30j  30k ft-kip. y The unit vector j is parallel to line AB, so the moment about AB is (6, 6, 0) ft MAB D j Ð MA j B D 30j ft-kip. F y B z x A (6, 0, 0) ft −30j (ft-kip) Direction of moment x A z c 2008 Pearson Education, Inc., Upper Saddle River, NJ. All rights reserved. This material is protected under all copyright laws as they  currently exist. No portion of this material may be reproduced, in any form or by any means, without permission in writing from the publisher. 207 y Problem 4.90 The force F D 10i C 12j  6k (N). What is the moment of F about the line OA? Draw a sketch to indicate the sense of the moment. Solution: The strategy is to determine a unit vector parallel to OA and to use this to determine the moment about OA. The vector parallel to OA is rOA D 6j C 4k. The magnitude: F. The unit vector parallel to OA is eOA D 0.8321j C 0.5547k. The vector from O to the point of application of F is rOF D 8i C 6k. The magnitude of the moment about OA is   0  jMO j D eOA Ð rOF ð F D  8  10 A (0, 6, 4) m x (8, 0, 6) m  0.5547  6  6  0.8321 0 12 F O z D 89.8614 C 53.251 D 143.1 N-m. The moment about OA is MOA D jMOA jeOA D 119.1j C 79.4k (N-m). The sense of the moment is in the direction of the curled fingers of the right hand when the thumb is parallel to OA, pointing to A. Problem 4.91 The tension in the cable AB is 1 kN. Determine the moment about the x axis due to the force exerted on the hatch by the cable at point B. Draw a sketch to indicate the sense of the moment. Solution: The vector parallel to BA is y A (400, 300, 0) mm x rBA D 0.4  1i C 0.3j  0.6k D 0.6i C 0.3j  0.6k. 600 mm B The unit vector parallel to BA is 1000 mm z eBA D 0.6667i C 0.3333j  0.6667k. The moment about O is   i  MO D rOB ð T D  1  0.6667 j 0 0.3333   k   0.6  0.66667  MO D 0.2i C 0.2667j C 0.3333k. The magnitude is jMX j D eX Ð MO D 0.2 kN-m. The moment is MX D 0.2i kN-m. The sense is clockwise when viewed along the x axis toward the origin. 208 c 2008 Pearson Education, Inc., Upper Saddle River, NJ. All rights reserved. This material is protected under all copyright laws as they  currently exist. No portion of this material may be reproduced, in any form or by any means, without permission in writing from the publisher. y Problem 4.92 Determine the moment of the force applied at D about the straight line through the hinges A and B. (The line through A and B lies in the yz plane.) 6 ft 20i – 60j (lb) Solution: From the figure, we see that the unit vector along the line from A toward B is given by eAB D  sin 20° j C cos 20° k. The position vector is rAD D 4i ft, and the force vector is as shown in the figure. The moment vector of a force about an axis is of the form   eX  e ž r ð F D  rX  FX ey rY FY E A D 4 ft 2 ft  ez  rZ  . FZ  x B z 20° C 4 ft For this case,   0  sin 20°  e ž r ð F D  4 0  20 60  cos 20°  0  D 240 cos 20° ft-lb 0  D 225.5 ft-lb. The negative sign is because the moment is opposite in direction to the unit vector from A to B. Problem 4.93 In Problem 4.92, the tension in the cable CE is 160 lb. Determine the moment of the force exerted by the cable on the hatch at C about the straight line through the hinges A and B. Solution: From the figure, we see that the unit vector along the line from A toward B is given by eAB D  sin 20° j C cos 20° k. The position vector is rBC D 4i ft. The coordinates of point C are (4, 4 sin 20° , 4 cos 20° ). The unit vector along CE is 0.703i C 0.592j C 0.394k and the force vector is as shown acting at point D. The moment vector is a force about an axis is of the form   eX  e ž r ð F D  rX  FX ey rY FY  ez  rZ  . FZ  For this case, rCE D 4i C 3.368j C 2.242k TCE D 160eCE D 112.488i C 94.715j C 63.049k   0  sin 20°  e ž r ð F D  4 0  112.488 94.715  cos 20°  0  D 240 cos 20° ft-lb 63.049  D 701 ft-lbs. c 2008 Pearson Education, Inc., Upper Saddle River, NJ. All rights reserved. This material is protected under all copyright laws as they  currently exist. No portion of this material may be reproduced, in any form or by any means, without permission in writing from the publisher. 209 Problem 4.94 The coordinates of A are (2.4, 0, 0.6) m, and the coordinates of B are (2.2, 0.7, 1.2) m. The force exerted at B by the sailboat’s main sheet AB is 130 N. Determine the moment of the force about the centerline of the mast (the y axis). Draw a sketch to indicate the sense of the moment. y x B Solution: The position vectors: rOA D 2.4i  0.6k (m), rOB D 2.2i C 0.7j  1.2k (m), A rBA D 2.4 C 2.2i C 0  0.7j C 0.6 C 1.2k (m) z D 0.2i  0.7j C 0.6k (m). The magnitude is jrBA j D 0.9434 m. The unit vector parallel to BA is eBA D 0.2120i  0.7420j C 0.6360k. The tension is TBA D 130eBA . The moment of TBA about the origin is MO D rOB ð TBA or   i  D  2.2  27.56 j 0.7 96.46  k  1.2  , 82.68  MO D 57.88i C 214.97j C 231.5k. The magnitude of the moment about the y axis is jMY j D eY Ð MO D 214.97 N-m. The moment is MY D eY 214.97 D 214.97j N-m. 210 c 2008 Pearson Education, Inc., Upper Saddle River, NJ. All rights reserved. This material is protected under all copyright laws as they  currently exist. No portion of this material may be reproduced, in any form or by any means, without permission in writing from the publisher. Problem 4.95 The tension in cable AB is 200 lb. Determine the moments about each of the coordinate axes due to the force exerted on point B by the cable. Draw sketches to indicate the senses of the moments. y A (2, 5, –2) ft x z B (10, –2, 3) ft y Solution: The position vector from B to A is 443 ft-lb rBA D 2  10i C [5  2]j C 2  3k D 8i C 7j  5k ft, 187 ft-lb So the force exerted on B is F D 200 x rBA D 136.2i C 119.2j  85.1k lb. jrBA j The moment of F about the origin O is   i  rOB ð F D  10  136.2 j 2 119.2  k  3  85.1  919 ft-b z D 187i C 443j C 919k ft-lb. The moments about the x, y, and z axes are [rOB ð F Ð i]i D 187i ft-lb, [rOB ð F Ð j]j D 443j ft-lb, [rOB ð F Ð k]k D 919k ft-lb. Problem 4.96 The total force exerted on the blades of the turbine by the steam nozzle is F D 20i  120j C 100k (N), and it effectively acts at the point (100, 80, 300) mm. What moment is exerted about the axis of the turbine (the x axis)? y Fixed Rotating x Solution: The moment about the origin is   i  MO D  0.1  20 j 0.08 120  k  0.3  100  D 44.0i  4.0j  13.6k N-m. The moment about the x axis is z MO Ð ii D 44.0i N-m. c 2008 Pearson Education, Inc., Upper Saddle River, NJ. All rights reserved. This material is protected under all copyright laws as they  currently exist. No portion of this material may be reproduced, in any form or by any means, without permission in writing from the publisher. 211 Problem 4.97 The pneumatic support AB holds a trunk lid in place. It exerts a 35-N force on the fixture at B that points in the direction from A toward B. Determine the magnitude of the moment of the force about the hinge axis of the lid, which is the z axis. Solution: The vector from A to B is rAB D [60  480i C 100  40j C 30  40k] mm rAB D 420i C 140j  70k mm y The 35-N force can be written F D 35 N rAB D 32.8i C 10.9j  5.47k N jrAB j The moment about point O is   i j  MO D rOB ð F D  60 100  32.8 10.9 B (60, 100, ⫺30) mm  k  30  5.47  O D 219i C 1310j C 3940k N-mm The magnitude of the moment about the z axis is z Mz D MO Ð k D 3940 N-mm D 3.94 N-m x Mz D 3.94 N-m 212 A (480, ⫺40, 40) mm c 2008 Pearson Education, Inc., Upper Saddle River, NJ. All rights reserved. This material is protected under all copyright laws as they  currently exist. No portion of this material may be reproduced, in any form or by any means, without permission in writing from the publisher. Problem 4.98 The tension in cable AB is 80 lb. What is the moment about the line CD due to the force exerted by the cable on the wall at B? y 8 ft 3 ft Solution: The strategy is to find the moment about the point C exerted by the force at B, and then to find the component of that moment acting along the line CD. The coordinates of the points B, C, D are B (8, 6, 0), C (3, 6, 0), D(3, 0, 0). The position vectors are: rOB D 8i C 6j, rOC D 3i C 6j, rOD D 3i. The vector parallel to CD is rCD D rOD  rOC D 6j. The unit vector parallel to CD is eCD D 1j. The vector from point C to B is rCB D rOB  rOC D 5i. B The position vector of A is rOA D 6i C 10k. The vector parallel to BA is rBA D rOA  rOB D 2i  6j C 10k. The magnitude is jrBA j D 11.832 ft. The unit vector parallel to BA is C 6 ft eBA D 0.1690i  0.5071j C 0.8452k. The tension acting at B is x D TBA D 80eBA D 13.52i  40.57j C 67.62k. The magnitude of the moment about CD due to the tension acting at B is z A (6, 0, 10) ft   0  jMCD j D eCD Ð rCB ð TBA  D  5  13.52 1 0 40.57  0  0  67.62  D 338.1 (ft lb). The moment about CD is MCD D 338.1eCD D 338.1j (ft lb). The sense of the moment is along the curled fingers of the right hand when the thumb is parallel to CD, pointing toward D. c 2008 Pearson Education, Inc., Upper Saddle River, NJ. All rights reserved. This material is protected under all copyright laws as they  currently exist. No portion of this material may be reproduced, in any form or by any means, without permission in writing from the publisher. 213 Problem 4.99 The magnitude of the force F is 0.2 N and its direction cosines are cos x D 0.727, cos y D 0.364, and cos z D 0.582. Determine the magnitude of the moment of F about the axis AB of the spool. Solution: We have rAB D 0.3i  0.1j  0.4k m, rAB D  0.32 C 0.12  C 0.42 m D p 0.26 m y 1 0.3i  0.1j  0.4k eAB D p 0.26 B F D 0.2 N0.727i  0.364j C 0.582k (200, 400, 0) mm rAP D 0.26i  0.025j  0.11k m (160, 475, 290) mm Now the magnitude of the moment about the spool axis AB is P A F (⫺100, 500, 400) mm x MAB 0.2 N D p 0.26   0.3    0.26 m   0.727  0.4   0.11 m  D 0.0146 N-m  0.582  0.1 0.025 m 0.364 z Problem 4.100 A motorist applies the two forces shown to loosen a lug nut. The direction cosines of 4 3 F are cos x D 13 , cos y D 12 , and cos z D 13 . If the 13 magnitude of the moment about the x axis must be 32 ftlb to loosen the nut, what is the magnitude of the forces the motorist must apply? y Solution: The unit vectors for the forces are the direction cosines. The position vector of the force F is rOF D 1.333k ft. The magnitude of the moment due to F is   1  0 jMOF j D eX Ð rOF ð F D   0.3077F 0 0 0.9231F   0  1.333  0.2308F  jMOF j D 1.230F ft lb. The magnitude of the moment due to F is –F F jMOF j D eX Ð rOF ð F   1  D  0  .3077F z 0 0 0.9231F   0  1.333  D 1.230F ft lb. 0.2308F  The total moment about the x axis is  16 in 16 in x MX D 1.230Fi C 1.230Fi D 2.46Fi, from which, for a total magnitude of 32 ft lb, the force to be applied is FD 214 32 D 13 lb 2.46 c 2008 Pearson Education, Inc., Upper Saddle River, NJ. All rights reserved. This material is protected under all copyright laws as they  currently exist. No portion of this material may be reproduced, in any form or by any means, without permission in writing from the publisher. Problem 4.101 The tension in cable AB is 2 kN. What is the magnitude of the moment about the shaft CD due to the force exerted by the cable at A? Draw a sketch to indicate the sense of the moment about the shaft. 2m C A Solution: The strategy is to determine the moment about C due to A, and determine the component parallel to CD. The moment is determined from the distance CA and the components of the tension, which is to be found from the magnitude of the tension and the unit vector parallel to AB. The coordinates of the points A, B, C, and D are: A (2, 2, 0), B (3, 0, 1), C (0, 2, 0), and D (0,0,0). The unit vector parallel to CD is by inspection eCD D 1j. The position vectors parallel to DC, DA, and DB: rDC D 2j, rDA D 2i C 2j, rDB D 3i C 1k. 2m The vector parallel to CA is rCA D 2i. The vector parallel to AB is rAB D rDB  rDA D 1i  2j C 1k. D B 1m The magnitude: jrAB j D 2.4495 m. The unit vector parallel to AB is 3m eAB D 0.4082i  0.8165j C 0.4082k. The tension is TAB D 2eAB D 0.8165i  1.633j C 0.8165k. The magnitude of the moment about CD is   0  jMCD j D eCD Ð rCA ð TAB  D  2  0.8164 1 0 1.633  0  0  0.8165  D 1.633 kN-m. The moment about CD is MCD D eCD jMCD j D 1.633j (kN-m). The sense is in the direction of the curled fingers of the right hand when the thumb is parallel to DC, pointed toward D. Problem 4.102 The axis of the car’s wheel passes through the origin of the coordinate system and its direction cosines are cos x D 0.940, cos y D 0, cos z D 0.342. The force exerted on the tire by the road effectively acts at the point x D 0, y D 0.36 m, z D 0 and has components F D 720i C 3660j C 1240k (N). What is the moment of F about the wheel’s axis? Solution: We have to determine the moment about the axle where a unit vector along the axle is e D cos x i C cos y j C cos z k e D 0.940i C 0j C 0.342k The vector from the origin to the point of contact with the road is r D 0i  0.36j C 0k m The force exerted at the point of contact is F D 720i C 3660j C 1240k N y The moment of the force F about the axle is MAXLE D [e Ð r ð F]e x MAXLE z   0.940  D  0  720 0 0.36 C3660  0.342  0  0.940i C 0.342k N-m C1240  MAXLE D 508.260.940i C 0.342k N-m MAXLE D 478i  174k N-m c 2008 Pearson Education, Inc., Upper Saddle River, NJ. All rights reserved. This material is protected under all copyright laws as they  currently exist. No portion of this material may be reproduced, in any form or by any means, without permission in writing from the publisher. 215 Problem 4.103 The direction cosines of the centerline OA are cos x D 0.500, cos y D 0.866, and cos z D 0, and the direction cosines of the line AG are cos x D 0.707, cos y D 0.619, and cos z D 0.342. What is the moment about OA due to the 250-N weight? Draw a sketch to indicate the sense of the moment about the shaft. Solution: By definition, the direction cosines are the scalar components of the unit vectors. Thus the unit vectors are e1 D 0.5i C 0.866j, and e2 D 0.707i C 0.619j  0.341k. The force is W D 250j (N). The position vector of the 250 N weight is rW D 0.600e1 C 0.750e2 D 0.8303i C 0.9839j  0.2565k The moment about OA is G m MOA D eOA eOA Ð rW ð W m 50 y   0.5  D  0.8303  0 7 250 N 0.866 0.9839 250   0  0.2565  e1 D 32.06e1  0 D 16i  27.77j (N-m) A 600 mm The moment is anti parallel to the unit vector parallel to OA, with the sense of the moment in the direction of the curled fingers when the thumb of the right hand is directed oppositely to the direction of the unit vector. O z x Problem 4.104 The radius of the steering wheel is 200 mm. The distance from O to C is 1 m. The center C of the steering wheel lies in the x  y plane. The driver exerts a force F D 10i C 10j  5k (N) on the wheel at A. If the angle ˛ D 0, what is the magnitude of the moment about the shaft OC? Draw a sketch to indicate the sense of the moment about the shaft. y Solution: The strategy is to determine the moment about C, and then determine its component about OC. The radius vectors parallel to OC and CA are: rOC D 1i cos 20° C j sin 20° C 0k D 0.9397i C 0.3420j. The line from C to the x axis is perpendicular to OC since it lies in the plane of the steering wheel. The unit vector from C to the x axis is eCX D i cos20  90 C j sin20  90 D 0.3420i  0.9397j, where the angle is measured positive counterclockwise from the x axis. The vector parallel to CA is F C A O z rCA D 0.2eCX D C0.0684i  0.1879j (m). The magnitude of the moment about OC 20° α   0.9397  jMOC j D eOC Ð rCA ð F D  0.0684  10  0.3420 0  0.1879 0  10 5  x D 0.9998 D 1 N-m. The sense of the moment is in the direction of the curled fingers of the right hand if the thumb is parallel to OC, pointing from O to C. 216 c 2008 Pearson Education, Inc., Upper Saddle River, NJ. All rights reserved. This material is protected under all copyright laws as they  currently exist. No portion of this material may be reproduced, in any form or by any means, without permission in writing from the publisher. Problem 4.105* The magnitude of the force F is 10 N. Suppose that you want to choose the direction of the force F so that the magnitude of its moment about the line L is a maximum. Determine the components of F and the magnitude of its moment about L. (There are two solutions for F.) y Solution: The moment of the general force F D Fx i C Fy j C Fz k about the line is developed by eBA D 1 3i C 6j  6k D i C 2j  2k, 9 3 rBP D 12i C 2j  2k m, MBA D eBA Ð rBP ð F A (3, 8, 0) m This expression simplifies to MBA D  We also have the constraint that 10 N2 D Fx 2 C Fy 2 C Fz 2 L F Since Fx does not contribute to the moment we set it equal to zero. Solving the constraint equation for Fz and substituting this into the expression for the moment we find B (0, 2, 6) m P (12, 4, 4) m MBA D  22 Fy š 3  100  Fy 2 . ) x z 22 m Fy C Fz  3 dMBA D0 dFy p p ) Fy D š5 2N ) Fz D š5 2 We thus have two answers: F D 7.07j C 7.07k N or F D 7.07j C 7.07k c 2008 Pearson Education, Inc., Upper Saddle River, NJ. All rights reserved. This material is protected under all copyright laws as they  currently exist. No portion of this material may be reproduced, in any form or by any means, without permission in writing from the publisher. 217 Problem 4.106 The weight W causes a tension of 100 lb in cable CD. If d D 2 ft, what is the moment about the z axis due to the force exerted by the cable CD at point C? y (12, 10, 0) ft (0, 3, 0) ft W Solution: The strategy is to use the unit vector parallel to the bar to locate point C relative to the origin, and then use this location to find the unit vector parallel to the cable CD. With the tension resolved into components about the origin, the moment about the origin can be resolved into components along the z axis. Denote the top of the bar by T and the bottom of the bar by B. The position vectors of the ends of the bar are: D C d x z (3, 0, 10) ft rOB D 3i C 0j C 10k, rOT D 12i C 10j C 0k. The vector from the bottom to the top of the bar is rBT D rOT  rOB D 9i C 10j  10k. The magnitude: p jrBT j D 92 C 102 C 102 D 16.763 ft. The unit vector parallel to the bar, pointing toward the top, is eBT D 0.5369i C 0.5965j  0.5965k. The position vector of the point C relative to the bottom of the bar is rBC D 2eBT D 1.074i C 1.193j  1.193k. The position vector of point C relative to the origin is rOC D rOB C rBC D 4.074i C 1.193j C 8.807k. The position vector of point D is rOD D 0i C 3j C 0k. The vector parallel to CD is rCD D rOD  rOC D 4.074i C 1.807j  8.807k. The magnitude is jrCD j D p 4.0742 C 1.8072 C 8.8072 D 9.87 ft. The unit vector parallel to CD is eCD D 0.4127i C 0.1831j  0.8923k. The tension is TCD D 100eCD D 41.27i C 18.31j  89.23k lb. The magnitude of the moment about the z axis is   0  jMO j D eZ Ð rOC ð TCD  D  4.074  41.27 0 1.193 18.31  1  8.807  89.23  D 123.83 ft lb 218 c 2008 Pearson Education, Inc., Upper Saddle River, NJ. All rights reserved. This material is protected under all copyright laws as they  currently exist. No portion of this material may be reproduced, in any form or by any means, without permission in writing from the publisher. Problem 4.107* The y axis points upward. The weight of the 4-kg rectangular plate acts at the midpoint G of the plate. The sum of the moments about the straight line through the supports A and B due to the weight of the plate and the force exerted on the plate by the cable CD is zero. What is the tension in the cable? Solution: Note (150, 152.5, 195). that the coordinates of point G are We calculate the moment about the line BA due to the two forces as follows. eBA D 0.1i C 0.07j  0.36k p 0.1445 y r1 D 0.2i  0.125j C 0.03k m, A (100, 500, 700) mm (100, 250, 0) mm F1 D TCD D 0.1i C 0.445j C 0.31k p 0.304125 r2 D 0.15i  0.0275j  0.165k m, F2 D 4 kg9.81 m/s2 j G x B MBA D eBA Ð r1 ð F1 C r2 ð F2  The moment reduces to (0, 180, 360) mm C MBA D 3.871 N-m  0.17793 mTCD D 0 ) TCD D 21.8 N (200, 55, 390) mm z Problem 4.108 In Active Example 4.9, suppose that the point of application of the force F is moved from (8, 3, 0) m to (8, 8, 0) m. Draw a sketch showing the new position of the force. From your sketch, will the moment due to the couple be clockwise or counterclockwise? Calculate the moment due to the couple. Represent the moment by its magnitude and a circular arrow indicating its direction. y ⴚF (6, 6, 0) m (8, 3, 0) m F x Solution: From Active Example 4.9 we know that F D 10i  4j N From the sketch, it is evident that the moment will be clockwise. The moment due to the couple is the sum of the moments of the two forces about any point. If we determine the sum of the moments about the point of application of one of the forces, the moment due to that force is zero and we only need to determine the moment due to the other force. Let us determine the moment about the point of application of the force F. The vector from the point of application of F to the point of application of the force -F is r D [6  8i C 6  8j] m D 2i  2j m The sum of the moments of the two forces is    i j k   M D r ð F D  2 2 0  D 28k N-m  10 4 0  The magnitude of the moment is 28 N-m. Pointing the thumb of the right hand into the page, the right-hand rule indicates that the moment is clockwise. M D 28 N-m clockwise c 2008 Pearson Education, Inc., Upper Saddle River, NJ. All rights reserved. This material is protected under all copyright laws as they  currently exist. No portion of this material may be reproduced, in any form or by any means, without permission in writing from the publisher. 219 Problem 4.109 The forces are contained in the xy plane. Solution: The right hand force is F D [1000 lb]cos 60° i  sin 60° j (a) (b) Determine the moment of the couple and represent it as shown in Fig. 4.28c. What is the sum of the moments of the two forces about the point (10, 40, 20) ft? y F D C500i  867j lb. The vector from the x intercept of the left force to that of the right force is r D 40i ft. The moment is MC D r ð F 1000 lb 1000 lb 60° MC D 40i ð 500i  867j ft-lb 60° MC D 34700 ft-lb k x 20 ft 20 ft or Problem 4.110 The moment of the couple is 600 k (N-m). What is the angle ˛? MC D 34700 ft-lb) clockwise Solution: M D 100 N cos ˛4 m C 100 N sin ˛5 m D 600 N-m y Solving yields two answers: ˛ D 30.9° a or ˛ D 71.8° (0, 4) m 100 N 100 N a x (5, 0) m Problem 4.111 Point P is contained in the xy plane, jFj D 100 N, and the moment of the couple is 500k (N-m). What are the coordinates of P? Solution: The force is F D 100i cos30°  C j sin30°  D 86.6i  50j. Let r be the distance OP. The vector parallel to OP is y P r D ri cos 70° C j sin 70°  D r0.3420i C 0.9397j. 30° F The moment is –F 70° x   i  M D r ð F D  0.3420r  86.6 From which, r D j 0.9397r 50.0  k  0  D 98.48rk. 0 500 D 5.077 m. From above, 98.48 r D 5.0770.3420i C 0.9397j. The coordinates of P are x D 5.0770.3420 D 1.74 m, y D 5.0770.9397 D 4.77 m 220 c 2008 Pearson Education, Inc., Upper Saddle River, NJ. All rights reserved. This material is protected under all copyright laws as they  currently exist. No portion of this material may be reproduced, in any form or by any means, without permission in writing from the publisher. Problem 4.112 Three forces of equal magnitude are applied parallel to the sides of an equilateral triangle. (a) Show that the sum of the moments of the forces is the same about any point. (b) Determine the magnitude of the sum of the moments. F L F Solution: (a) (b) F Resolving one of the forces into vector components parallel to the other two forces results in two equal and opposite forces with the same line of action and one couple. Therefore the moment due to the forces is the same about any point. Determine the moment about one of the vertices of the triangle. A vertex lies on the line of action of two of the forces, so the moment due to them is zero. The perpendicular distance to the line of action of the third force is L cos 30° , so the magnitude of the moment due to the three force is M D FL cos 30° Problem 4.113 In Example 4.10, suppose that the 200 ft-lb couple is counterclockwise instead of clockwise. Draw a sketch of the beam showing the forces and couple acting on it. What are the forces A and B? y 200 ft-lb A B 4 ft 4 ft x Solution: In Example 4.10 we are given that the sum of the forces is zero and the sum of the moments is zero. Thus Fy D A C B D 0 MA D B 4 ft C 200 ft-lb D 0 Solving we find A D 50 lb, B D 50 lb Problem 4.114 The moments of two couples are shown. What is the sum of the moments about point P? y 50 ft-lb P x (– 4, 0, 0) ft 10 ft-lb Solution: The moment of a couple is the same anywhere in the plane. Hence the sum about the point P is  M D 50k C 10k D 40k ft lb c 2008 Pearson Education, Inc., Upper Saddle River, NJ. All rights reserved. This material is protected under all copyright laws as they  currently exist. No portion of this material may be reproduced, in any form or by any means, without permission in writing from the publisher. 221 Problem 4.115 Determine the sum of the moments exerted on the plate by the two couples. y Solution: The moment due to the 30 lb couple, which acts in a clockwise direction is M30 D 330k D 90k ft lb. 30 lb The moment due to the 20 lb couple, which acts in a counterclockwise direction, is 3 ft 30 lb M20 D 920k D 180k ft lb. 2 ft The sum of the moments is x  20 lb 20 lb 5 ft 4 ft M D 90k C 180k D C90k ft lb. The sum of the moments is the same anywhere on the plate. Problem 4.116 Determine the sum of the moments exerted about A by the couple and the two forces. 100 lb Solution: Let the x axis point to the right and the y axis point upward in the plane of the page. The moments of the forces are 400 lb M100 D 3i ð 100j D 300k (ft-lb), 900 ft-lb and A 3 ft B 4 ft 3 ft M400 D 7i ð 400j D 2800k (ft-lb). The moment of the couple is MC D 900k (ft-lb). Summing the moments, we get 4 ft MTotal D 2200k (ft-lb) Problem 4.117 Determine the sum of the moments exerted about A by the couple and the two forces. Solution:  MA D 0.2i ð 200j C 0.4i C 0.2j 100 N 30° ð 86.7i C 50j C 300k N-m  200 N 0.2 m  A MA D 40k C 2.66k C 300k N-m MA D 262.7k N-m ' 263k N-m 300 N-m 0.2 m 0.2 m 0.2 m Problem 4.118 The sum of the moments about point A due to the forces and couples acting on the bar is zero. Solution: (a)  MA D 20 kN-m  2 kN5 m  4 kN3 m (a) What is the magnitude of the couple C? (b) Determine the sum of the moments about point B due to the forces and couples acting on the bar.  3 kN8 C C D 0 C D 26 kN-m B 4 kN 3m 20 kN-m  MB D 3 kN3 m  4 kN3 m  5 kN5 m (b) C 20 kN-m C 26 kN-m D 0 C A 4 kN 2 kN 5 kN 5m 222 3 kN 3m c 2008 Pearson Education, Inc., Upper Saddle River, NJ. All rights reserved. This material is protected under all copyright laws as they  currently exist. No portion of this material may be reproduced, in any form or by any means, without permission in writing from the publisher. Problem 4.119 In Example 4.11, suppose that instead of acting in the positive z direction, the upper 20-N force acts in the positive x axis direction. Instead of acting in the negative z axis direction, let the lower 20-N force act in the negative x axis direction. Draw a sketch of the pipe showing the forces acting on it. Determine the sum of the moments exerted on the pipe by the two couples. y 20 N 30 N 30 N 2m 4m 4m x 60⬚ 20 N 60⬚ z Solution: The magnitude of the moment of the 20-N couple is unchanged, 2 m20 N D 40 N-m. The direction of the moment vector is perpendicular to eh x-y plane, and the right-hand rule indicates that it points in the negative z axis direction. The moment of the 20-N couple is (40 N-m) k. The sum of the moments exerted on the pipe by the two couples is M D 40 N-m k C 30 N cos 60° 4 m j  30 N sin 60° 4 m k M D 60j  144k N-m Problem 4.120 (a) What is the moment of the couple? (b) Determine the perpendicular distance between the lines of action of the two forces. y (0, 4, 0) m ⫺2i ⫹ 2j ⫹ k (kN) Solution: 2i ⫺ 2j ⫺ k (kN) x M D 4j  5k m ð 2i  2j  k kN (a) D 14i  10j  8k kN-m (b) MD FD   (0, 0, 5) m 142 C 102 C 82 kN-m D 18.97 kN-m z 22 C 22 C 12 kN D 3 kN M D Fd ) d D M 18.97 kN-m D D 6.32 m F 3 kN c 2008 Pearson Education, Inc., Upper Saddle River, NJ. All rights reserved. This material is protected under all copyright laws as they  currently exist. No portion of this material may be reproduced, in any form or by any means, without permission in writing from the publisher. 223 Problem 4.121 Determine the sum of the moments exerted on the plate by the three couples. (The 80-lb forces are contained in the xz plane.) y 3 ft 20 lb 3 ft 20 lb 40 lb x 8 ft 40 lb z 60° 60° 80 lb Solution: The moments of two of the couples can be determined The moment is from inspection:   i j  M3 D r3 ð F3 D  6 0  69.282 0 M1 D 320k D 60k ft lb. M2 D 840j D 320j ft lb The forces in the 3rd couple are resolved: 80 lb  k  0  D 240j. 40  The sum of the moments due to the couples:  M D 60k C 320j  240j D 80j  60k ft lb F D 80i sin 60° C k cos 60°  D 69.282i C 40k The two forces in the third couple are separated by the vector r3 D 6i C 8k  8k D 6i Problem 4.122 What is the magnitude of the sum of the moments exerted on the T-shaped structure by the two couples? y 3 ft 50i + 20j – 10k (lb) 3 ft 50j (lb) 3 ft z –50j (lb) 3 ft x –50i – 20j + 10k (lb) Solution: The moment of the 50 lb couple can be determined by y 3 ft inspection: F 3 ft 50j (lb) M1 D 503k D 150k ft lb. The vector separating the other two force is r D 6k. The moment is   i  M2 D r ð F D  0  50 j 0 20  k  6  D 120i C 300j. 10  3 ft x –F –50j (lb) 3 ft z The sum of the moments is  M D 120i C 300j  150k. The magnitude is p jMj D 224 1202 C 3002 C 1502 D 356.23 ft lb c 2008 Pearson Education, Inc., Upper Saddle River, NJ. All rights reserved. This material is protected under all copyright laws as they  currently exist. No portion of this material may be reproduced, in any form or by any means, without permission in writing from the publisher. y Problem 4.123 The tension in cables AB and CD is 500 N. (a) (b) A (0, 2, 0) m Show that the two forces exerted by the cables on the rectangular hatch at B and C form a couple. What is the moment exerted on the plate by the cables? 3m B z x 3m C D Solution: One condition for a couple is that the sum of a pair of forces vanish; another is for a non-zero moment to be the same anywhere. The first condition is demonstrated by determining the unit vectors parallel to the action lines of the forces. The vector position of point B is rB D 3i m. The vector position of point A is rA D 2j. The vector parallel to cable AB is rBA D rA  rB D 3i C 2j. (6, –2, 3) m The moment about the origin is MO D rB  rC  ð TAB D rCB ð TAB , which is identical with the above expression for the moment. Let rPC and rPB be the distances to points C and B from an arbitrary point P on the plate. Then MP D rPB  rPC  ð TAB D rCB ð TAB which is identical to the above expression. Thus the moment is the same everywhere on the plate, and the forces form a couple. The magnitude is: jrAB j D p 32 C 22 D 3.606 m. The unit vector: eAB D rAB D 0.8321i C 0.5547j. jrAB j The tension is TAB D jTAB jeAB D 416.05i C 277.35j. The vector position of points C and D are: rC D 3i C 3k, rD D 6i  2j C 3k. The vector parallel to the cable CD is rCD D rD  rC D 3i  2j. The magnitude is jrCD j D 3.606 m, and the unit vector parallel to the cable CD is eCD D C0.8321i  0.5547j. The magnitude of the tension in the two cables is the same, and eBA D eCD , hence the sum of the tensions vanish on the plate. The second condition is demonstrated by determining the moment at any point on the plate. By inspection, the distance between the action lines of the forces is rCB D rB  rC D 3i  3i  3k D 3k. The moment is M D rCB ð TAB   i  D  0  416.05  j k  0 3  277.35 0  D 832.05i  1248.15j (N-m). c 2008 Pearson Education, Inc., Upper Saddle River, NJ. All rights reserved. This material is protected under all copyright laws as they  currently exist. No portion of this material may be reproduced, in any form or by any means, without permission in writing from the publisher. 225 Problem 4.124 The cables AB and CD exert a couple on the vertical pipe. The tension in each cable is 8 kN. Determine the magnitude of the moment the cables exert on the pipe. (⫺1.6, 2.2, ⫺1.2) m y D C (0.2, 1.6, ⫺0.2) m Solution: FAB D 8 kN A 1.4i  0.6j C 1.0k p , rDB D 3.2i  2.2j C 2.4k m 3.32 (0.2, 0.6, 0.2) m M D rBD ð FAB D 3.34i C 0.702j C 5.09k kN-m x ) M D 6.13 kN-m B z Problem 4.125 The bar is loaded by the forces y FB FB D 2i C 6j C 3k (kN), A FC D i  2j C 2k (kN), and the couple (1.6, 0, 1.2) m B MC C x 1m z 1m FC MC D 2i C j  2k (kN-m). Determine the sum of the moments of the two forces and the couple about A. Solution: The moments of the two forces about A are given by MFB D 1i ð 2i C 6j C 3k (kN-m) D 0i  3j C 6k (kN-m) and MFC D 2i ð 1i  2j C 2k (kN-m) D 0i  4j  4k (kN-m). Adding these two moments and MC D 2i C 1j  2k (kN-m), we get MTOTAL D 2i  6j C 0k (kN-m) Problem 4.126 In Problem 4.125, the forces Solution: From the solution to Problem 4.125, the sum of the moments of the two forces about A is FB D 2i C 6j C 3k (kN), MForces D 0i  7j C 2k (kN-m). FC D i  2j C 2k (kN), The required moment, MC , must be the negative of this sum. and the couple Thus MCy D 7 (kN-m), and MCz D 2 (kN-m). MC D MCy j C MCz k (kN-m). Determine the values for MCy and MCz , so that the sum of the moments of the two forces and the couple about A is zero. 226 c 2008 Pearson Education, Inc., Upper Saddle River, NJ. All rights reserved. This material is protected under all copyright laws as they  currently exist. No portion of this material may be reproduced, in any form or by any means, without permission in writing from the publisher. y Problem 4.127 Two wrenches are used to tighten an elbow fitting. The force F D 10k (lb) on the right wrench is applied at (6, 5, 3) in, and the force F on the left wrench is applied at (4, 5, 3) in. (a) Determine the moment about the x axis due to the force exerted on the right wrench. (b) Determine the moment of the couple formed by the forces exerted on the two wrenches. (c) Based on the results of (a) and (b), explain why two wrenches are used. z x F –F Solution: The position vector of the force on the right wrench is rR D 6i  5j  3k. The magnitude of the moment about the x axis is  1  jMR j D eX Ð rR ð F D  6 0 (a)  0 0  5 3  D 50 in lb 0 10  from which MXL D 50i in lb, which is opposite in direction and equal in magnitude to the moment exerted on the x axis by the right wrench. The left wrench force is applied 2 in nearer the origin than the right wrench force, hence the moment must be absorbed by the space between, where it is wanted. The moment about the x axis is MR D jMR jeX D 50i (in lb). (b) The moment of the couple is  i  MC D rR  rL  ð FR D  2 0 (c) j 0 0  k  6  D 20j in lb 10  The objective is to apply a moment to the elbow relative to connecting pipe, and zero resultant moment to the pipe itself. A resultant moment about the x axis will affect the joint at the origin. However the use of two wrenches results in a net zero moment about the x axis the moment is absorbed at the juncture of the elbow and the pipe. This is demonstrated by calculating the moment about the x axis due to the left wrench:  1  jMX j D eX Ð rL ð FL  D  4 0 0 5 0  0  3  D 50 in lb 10  c 2008 Pearson Education, Inc., Upper Saddle River, NJ. All rights reserved. This material is protected under all copyright laws as they  currently exist. No portion of this material may be reproduced, in any form or by any means, without permission in writing from the publisher. 227 Problem 4.128 Two systems of forces act on the beam. Are they equivalent? Strategy: Check the two conditions for equivalence. The sums of the forces must be equal, and the sums of the moments about an arbitrary point must be equal. System 1 y 100 N x 50 N 1m 1m System 2 y 50 N x 2m Solution: The strategy is to check the two conditions for equivalence: (a) the sums of the forces must be equal and (b) the sums of the moments about an arbitrary point must be equal. The sums of the  forces of the two systems: FX D 0, (both systems) and   FY1 D 100j C 50j D 50j (N) FY2 D 50j (N). The sums of the forces are equal. The sums of the moments about the left end are:   M1 D 1100k D 100k (N-m) M2 D 250k D 100k (N-m). The sums of the moments about the left end are equal. Choose any point P at the same distance r D xi from the left end on each beam. The sums of the moments about the point P are   M1 D 50x C 100x  1k D 50x  100k (N-m) M2 D 502  xk D 50x  100k (N-m). Thus the sums of the moments about any point on the beam are equal for the two sets of forces; the systems are equivalent. Yes 228 c 2008 Pearson Education, Inc., Upper Saddle River, NJ. All rights reserved. This material is protected under all copyright laws as they  currently exist. No portion of this material may be reproduced, in any form or by any means, without permission in writing from the publisher. Problem 4.129 Two systems of forces and moments act on the beam. Are they equivalent? System 1 y 20 lb 50 ft-lb 10 lb x Solution: The sums of the forces are:    2 ft FX D 0 (both systems) FY1 D 10j  20j D 10j (lb) System 2 y FY2 D 20j C 10j D 10j (lb) Thus the sums of the forces are equal. The sums of the moments about the left end are:   2 ft 20 lb 30 ft-lb 10 lb x M1 D 204k C 50k D 30k (ft lb) 2 ft 2 ft M2 D C102k  30k D 10k (ft lb) The sums of the moments are not equal, hence the systems are not equivalent. No Problem 4.130 Four systems of forces and moments act on an 8-m beam. Which systems are equivalent? System 1 the moments about some point (the left end will be used) must be the same. System 2 10 kN 8m Solution: For equivalence, the sum of the forces and the sum of   F kN ML kN-m 10 kN 80 kN-m 8m System 1 10j 80k System 2 10j 80k Systems 1, 2, and 4 are equivalent. System 3 System 4 20 kN 10 kN 8m System 3 10j 160k System 4 10j 80k y x 20 kN 10 kN 80 kN-m 4m 4m c 2008 Pearson Education, Inc., Upper Saddle River, NJ. All rights reserved. This material is protected under all copyright laws as they  currently exist. No portion of this material may be reproduced, in any form or by any means, without permission in writing from the publisher. 229 Problem 4.131 The four systems shown in Problem 4.130 can be made equivalent by adding a couple to one of the systems. Which system is it, and what couple must be added? Solution: From the solution to 4.130, all systems have  F D 10j kN and systems 1, 2, and 4 have  ML D 80k kN-m system 3 has  ML D 160k kN-m. Thus, we need to add a couple M D 80k kN-m to system 3 (clockwise moment). Problem 4.132 System 1 is a force F acting at a point O. System 2 is the force F acting at a different point O0 along the same line of action. Explain why these systems are equivalent. (This simple result is called the principle of transmissibility.) System 2 System 1 F F O' O O Solution: The sum of forces is obviously equal for both systems. Let P be any point on the dashed line. The moment about P is the cross product of the distance from P to the line of action of a force times the force, that is, M D rPL ð F, where rPL is the distance from P to the line of action of F. Since both systems have the same line of action, and the forces are equal, the systems are equivalent. 230 c 2008 Pearson Education, Inc., Upper Saddle River, NJ. All rights reserved. This material is protected under all copyright laws as they  currently exist. No portion of this material may be reproduced, in any form or by any means, without permission in writing from the publisher. Problem 4.133 The vector sum of the forces exerted on the log by the cables is the same in the two cases. Show that the systems of forces exerted on the log are equivalent. A 12 m B 16 m C 12 m D 6m Solution: The angle formed by the single cable with the positive E 20 m Solve: x axis is   D 180°  tan1 12 16  jTL j D 0.3353jT1 j, and D 143.13° . jTR j D 0.7160jT1 j. The single cable tension is T1 D jTji cos 143.13° C j sin 143.13°  D jTj0.8i C 0.6j. The position vector to the center of the log from the left end is rc D 10i. The moment about the end of the log is   i  M D r ð T1 D jT1 j  10  0.8  j k  0 0  D jTj6k (N-m). 0.6 0  The tension in the right hand cable is TR D jT1 j0.71600.9079i C 0.4191j D jT1 j0.6500i C 0.3000. The position vector of the right end of the log is rR D 20i m relative to the left end. The moments about the left end of the log for the second system are   i  M2 D rR ð TR D jT1 j  20  0.6500  j k  0 0  D jT1 j6k (N-m). 0.3000 0  This is equal to the moment about the left end of the log for System 1, hence the systems are equivalent. For the two cables, the angles relative to the positive x axis are  1 D 180°  tan1  2 D 180  tan1 12 6 12 26  D 116.56° , and  D 155.22° . The two cable vectors are TL D jTL ji cos 116.56° C j sin 116.56°  D jTL j0.4472i C 0.8945j, TR D jTR ji cos 155.22° C j sin 155.22°  D jTR j0.9079i C 0.4191j. Since the vector sum of the forces in the two systems is equal, two simultaneous equations are obtained: 0.4472jTL j C 0.9079jTR j D 0.8jT1 j, and 0.8945jTL j C 0.4191jTR j D 0.6jT1 j c 2008 Pearson Education, Inc., Upper Saddle River, NJ. All rights reserved. This material is protected under all copyright laws as they  currently exist. No portion of this material may be reproduced, in any form or by any means, without permission in writing from the publisher. 231 Problem 4.134 Systems 1 and 2 each consist of a couple. If they are equivalent, what is F? System 1 y System 2 y 200 N F 20° 30° Solution: For couples, the sum of the forces vanish for both sys- (5, 4, 0) m 5m 200 N tems. For System 1, the two forces are located at r11 D 4i, and r12 D C5j. The forces are F1 D 200i cos 30° C j sin 30°  D 173.21i C 100j. The moment due to the couple in System 1 is   i  M1 D r11  r12  ð F1 D  4  173.21 30° 2m x  j k  5 0  D 1266.05k (N-m). 100 0  x 20° F 4m For System 2, the positions of the forces are r21 D 2i, and r22 D 5i C 4j. The forces are F2 D Fi cos20°  C j sin20°  D F0.9397i  0.3420j. The moment of the couple in System 2 is   i  M2 D r21  r22  ð F2 D F  3  0.9397  j k  4 0  D 4.7848Fk, 0.3420 0  from which, if the systems are to be equivalent, FD 1266 D 264.6 N 4.7848 Problem 4.135 Two equivalent systems of forces and moments act on the L-shaped bar. Determine the forces FA and FB and the couple M. System 2 System 1 120 N-m FB 40 N 60 N 3m FA M 3m 50 N 3m Solution: The sums of the forces for System 1 are   FX D 50, and The sums of the forces for System 2 are   6m The sum of the moments about the left end for System 2 is  FY D FA C 60. 3m M2 D 3FB C M D 150 C M N-m. Equating the sums of the moments, M D 150  180 D 30 N-m FX D FB , and FY D 40. For equivalent systems: FB D 50 N, and FA D 60  40 D 20 N. The sum of the moments about the left end for System 1 is  232 M1 D 3FA  120 D 180 N-m. c 2008 Pearson Education, Inc., Upper Saddle River, NJ. All rights reserved. This material is protected under all copyright laws as they  currently exist. No portion of this material may be reproduced, in any form or by any means, without permission in writing from the publisher. Problem 4.136 Two equivalent systems of forces and moments act on the plate. Determine the force F and the couple M. System 1 30 lb System 2 10 lb 30 lb 100 in-lb 5 in 5 in 8 in 8 in M 50 lb 30 lb F Solution: The sums of the forces for System 1 are   FX D 30 lb, FY D 50  10 D 40 lb. The sums of the forces for System 2 are   FX D 30 lb, FY D F  30 lb. For equivalent forces, F D 30 C 40 D 70 lb. The sum of the moments about the lower left corner for System 1 is  M1 D 530  810 C M D 230 C M in lb. The sum of the moments about the lower left corner for System 2 is  M2 D 100 in lb. Equating the sum of moments, M D 230  100 D 130 in lb c 2008 Pearson Education, Inc., Upper Saddle River, NJ. All rights reserved. This material is protected under all copyright laws as they  currently exist. No portion of this material may be reproduced, in any form or by any means, without permission in writing from the publisher. 233 System 1 Problem 4.137 In Example 4.13, suppose that the 30kN vertical force in system 1 is replaced by a 230-kN vertical force. Draw a sketch of the new system 1. If you represent system 1 by a single force F as in system 3, at what position D on the x axis must the force be placed? y 30j (kN) 20i ⫹ 20j (kN) x O 3m 2m 210 kN-m Solution: The first step is to represent system 1 by a single force F acting at the origin and a couple M (system 2). The force F must equal the sum of the forces in system 1: F2 D F1 F D 230 kN j C 20i C 20j kN F D 20i C 250j kN The moment about the origin in system 2 is M. Therefore M must equal the sum of the moments about the origin due to the forces and moments in system 1: M2 D M1 M D 230 kN3 m C 20 kN5 m C 210 kN-m D 1000 kN-m The next step is to represent system 2 by system 3. The sums of the forces in the two systems are equal. The sums of the moments about the origin must be equal. The j component of F is 250 kN, so M3 D M2 1000 kN-m D 250 kND DD 1000 kN-m D 4m 250 kN DD4m 234 c 2008 Pearson Education, Inc., Upper Saddle River, NJ. All rights reserved. This material is protected under all copyright laws as they  currently exist. No portion of this material may be reproduced, in any form or by any means, without permission in writing from the publisher. Problem 4.138 Three forces and a couple are applied to a beam (system 1). (a) If you represent system 1 by a force applied at A and a couple (system 2), what are F and M? If you represent system 1 by the force F (system 3), what is the distance D? (b) System 1 y 30 lb 40 lb 20 lb 30 ft-lb x A 2 ft 2 ft System 2 y F M x A System 3 y F x A D Solution: The sum of the forces in System 1 is   FX D 0i, FY D 20 C 40  30j D 10j lb. The sum of the moments about the left end for System 1 is  M1 D 240  430 C 30k D 10k ft lb. (a) For System 2, the force at A is F D 10j lb The moment at A is M2 D 10k ft lb (b) For System 3 the force at D is F D 10j lb. The distance D is the ratio of the magnitude of the moment to the magnitude of the force, where the magnitudes are those in System 1: DD 10 D 1 ft 10 c 2008 Pearson Education, Inc., Upper Saddle River, NJ. All rights reserved. This material is protected under all copyright laws as they  currently exist. No portion of this material may be reproduced, in any form or by any means, without permission in writing from the publisher. 235 y Problem 4.139 Represent the two forces and couple acting on the beam by a force F. Determine F and determine where its line of action intersects the x axis. 60i + 60j (N) 280 N-m x – 40 j (N) 3m 3m y Solution: We first represent the system by an equivalent system consisting of a force F at the origin and a couple M: F This system is equivalent if M F D 40j C 60i C 60j x D 60i C 20j N, M D 280 C 660 y D 80 N-m. F We then represent this system by an equivalent system consisting of F alone: x For equivalence, M D dFy , so dD d M 80 D D 4 m. Fy 20 y Problem 4.140 The bracket is subjected to three forces and a couple. If you represent this system by a force F, what is F, and where does its line of action intersect the x axis? 400 N 180 N 0.4 m 140 N-m 200 N 0.2 m x 0.65 m Solution: We locate a single equivalent force along the x axis a distance d to the right of the origin. We must satisfy the following three equations:    Fx D 400 N  200 N D Rx Fy D 180 N D Ry MO D 400 N0.6 m C 200 N0.2 m C 180 N0.65 m C 140 Nm D Ry d Solving we find Rx D 200 N, Ry D 180 N, d D 0.317 m 236 c 2008 Pearson Education, Inc., Upper Saddle River, NJ. All rights reserved. This material is protected under all copyright laws as they  currently exist. No portion of this material may be reproduced, in any form or by any means, without permission in writing from the publisher. Problem 4.141 The vector sum of the forces acting on the beam is zero, and the sum of the moments about the left end of the beam is zero. Solution: (a) The sum of the forces is (a)  (b) (c) Determine the forces Ax and Ay , and the couple MA . Determine the sum of the moments about the right end of the beam. If you represent the 600-N force, the 200-N force, and the 30 N-m couple by a force F acting at the left end of the beam and a couple M, what are F and M?  FY D AY  600 C 200j D 0, from which AY D 400 N. The sum of the moments is  ML D MA  0.38600  30 C 0.560200k D 0, from which MA D 146 N-m. (b) The sum of the moments about the right end of the beam is  y FX D AX i D 0 and MR D 0.18600  30 C 146  0.56400 D 0. 600 N MA (c) The sum of the forces for the new system is x Ax 30 N-m Ay  200 N 380 mm FY D AY C Fj D 0, from F D AY D 400 N, or F D 400j N. The sum of the moments for the new system is 180 mm  M D MA C M D 0, from which M D MA D 146 N-m Problem 4.142 The vector sum of the forces acting on the truss is zero, and the sum of the moments about the origin O is zero. Solution: (a) The sum of the forces is (a) (b) from which AX D 12 kip (c) Determine the forces Ax , Ay , and B. If you represent the 2-kip, 4-kip, and 6-kip forces by a force F, what is F, and where does its line of action intersect the y axis? If you replace the 2-kip, 4-kip, and 6-kip forces by the force you determined in (b), what are the vector sum of the forces acting on the truss and the sum of the moments about O?  FX D AX  2  4  6i D 0, FY D AY C Bj D 0. The sum of the moments about the origin is  MO D 36 C 64 C 92 C 6B D 0, from which B D 10j kip. (b) Substitute into the force balance equation to obtain AY D B D 10 kip. (b) The force in the new system will replace the 2, 4, and 6 kip forces, F D 2  4  6i D 12i kip. The force must match the moment due to these forces: FD D 36 C 60 D 5 ft, or the action 64 C 92 D 60 kip ft, from which D D 12 line intersects the y axis 5 ft above the origin. (c) The new system is equivalent to the old one, hence the sum of the forces vanish and the sum of the moments about O are zero. 2 kip y  3 ft 4 kip 3 ft 6 kip 3 ft Ax O x B Ay 6 ft c 2008 Pearson Education, Inc., Upper Saddle River, NJ. All rights reserved. This material is protected under all copyright laws as they  currently exist. No portion of this material may be reproduced, in any form or by any means, without permission in writing from the publisher. 237 Problem 4.143 The distributed force exerted on part of a building foundation by the soil is represented by five forces. If you represent them by a force F, what is F, and where does its line of action intersect the x axis? y Solution: The equivalent force must equal the sum of the forces x exerted by the soil: 80 kN 3m F D 80 C 35 C 30 C 40 C 85j D 270j kN 35 kN 30 kN 40 kN 3m 3m 3m 85 kN The sum of the moments about any point must be equal for the two systems. The sum of the moments are  M D 335 C 630 C 940 C 1285 D 1665 kN-m. Equating the moments for the two systems FD D 1665 kN-m from which DD 1665 kN-m D 6.167 m. 270 kN Thus the action line intersects the x axis at a distance D D 6.167 m to the right of the origin. Problem 4.144 At a particular instant, aerodynamic forces distributed over the airplane’s surface exert the 88-kN and 16-kN vertical forces and the 22 kN-m counterclockwise couple shown. If you represent these forces and couple by a system consisting of a force F acting at the center of mass G and a couple M, what are F and M? Solution:   y 88 kN 16 kN x G 5m 22 kN-m 5.7 m 9m Fy D 88 kN C 16 kN D Ry MG D 88 kN0.7 m C 16 kN3.3 m C 22 kN-m D M Solving we find Ry D 104 kN, M D 13.2 kN-m Problem 4.145 If you represent the two forces and couple acting on the airplane in Problem 4.144 by a force F, what is F, and where does its line of action intersect the x axis? Solution:   Fy D 88 kN C 16 kN D Ry MOrigin D 88 kN5 m C 16 kN9 m C 22 kN-m D Ry x Solving we find F D Ry j D 104 kNj, x D 5.83 m 238 c 2008 Pearson Education, Inc., Upper Saddle River, NJ. All rights reserved. This material is protected under all copyright laws as they  currently exist. No portion of this material may be reproduced, in any form or by any means, without permission in writing from the publisher. Problem 4.146 The system is in equilibrium. If you represent the forces FAB and FAC by a force F acting at A and a couple M, what are F and M? y 60° B 40° FAC FAB C A A 100 lb 100 lb x Solution: The sum of the forces acting at A is in opposition to the weight, or F D jWjj D 100j lb. The moment about point A is zero. Problem 4.147 Three forces act on a beam. (a) (b) y Represent the system by a force F acting at the origin O and a couple M. Represent the system by a single force. Where does the line of action of the force intersect the x axis? 30 N 5m x O Solution: (a) The sum of the forces is   30 N 6m 4m 50 N FX D 30i N, and FY D 30 C 50j D 80j N. The equivalent at O is F D 30i C 80j (N). The sum of the moments about O:  M D 530 C 1050 D 350 N-m (b) The solution of Part (a) is the single force. The intersection is the 350 D 4.375 m moment divided by the y-component of force: D D 80 c 2008 Pearson Education, Inc., Upper Saddle River, NJ. All rights reserved. This material is protected under all copyright laws as they  currently exist. No portion of this material may be reproduced, in any form or by any means, without permission in writing from the publisher. 239 y Problem 4.148 The tension in cable AB is 400 N, and the tension in cable CD is 600 N. (a) If you represent the forces exerted on the left post by the cables by a force F acting at the origin O and a couple M, what are F and M? (b) If you represent the forces exerted on the left post by the cables by the force F alone, where does its line of action intersect the y axis? A 400 mm B C 300 mm D O Solution: From the right triangle, the angle between the positive x axis and the cable AB is   D  tan1 400 800  D 26.6° . x 800 mm Check. (b) The equivalent single force retains the same scalar components, but must act at a point that duplicates the sum of the moments. The distance on the y axis is the ratio of the sum of the moments to the x-component of the equivalent force. Thus 419 D 0.456 m 919.6 The tension in AB is DD TAB D 400i cos26.6° Cj sin26.6°  D 357.77i  178.89j (N). Check: The moment is The angle between the positive x axis and the cable CD is   i  M D rF ð F D  0  919.6  ˛ D  tan1 300 800  D 20.6° . from which D D The tension in CD is 300 mm  j k  D 0  D 919.6Dk D 419k, 389.6 0  419 D 0.456 m, Check. 919.6 TCD D 600i cos20.6°  C j sin20.6°  D 561.8i  210.67j. The equivalent force acting at the origin O is the sum of the forces acting on the left post: F D 357.77 C 561.8i C 178.89  210.67j D 919.6i  389.6j (N). The sum of the moments acting on the left post is the product of the moment arm and the x-component of the tensions:  M D 0.7357.77k  0.3561.8k D 419k N-m Check: The position vectors at the point of application are rAB D 0.7j, and rCD D 0.3j. The sum of the moments is  M D rAB ð TAB  C rCD ð TCD    i  D  0  357.77   j k   i 0.7 0  C  0 178.89 0   561.8  j k  0.3 0  210.67 0  D 0.7357.77k  0.3561.8k D 419k 240 c 2008 Pearson Education, Inc., Upper Saddle River, NJ. All rights reserved. This material is protected under all copyright laws as they  currently exist. No portion of this material may be reproduced, in any form or by any means, without permission in writing from the publisher. Problem 4.149 Consider the system shown in Problem 4.148. The tension in each of the cables AB and CD is 400 N. If you represent the forces exerted on the right post by the cables by a force F, what is F, and where does its line of action intersect the y axis? Solution: From the solution of Problem 4.148, the tensions are TAB D 400i cos26.6° Cj sin26.6°  D 357.77i C 178.89j, and TCD D 400i cos20.6° Cj sin20.6°  D 374.42i C 140.74j. The equivalent force is equal to the sum of these forces: F D 357.77  374.42i C 178.77 C 140.74j D 732.19i C 319.5j (N). The sum of the moments about O is  M D 0.3357.77 C 0.8140.74 C 178.89k D 363k (N-m). The intersection is D D 363 D 0.496 m on the positive y axis. 732.19 Problem 4.150 If you represent the three forces acting on the beam cross section by a force F, what is F, and where does its line of action intersect the x axis? y 500 lb 800 lb 6 in Solution: The sum of the forces is   FX D 500  500i D 0. x 6 in z FY D 800j. Thus a force and a couple with moment M D 500k ft lb act on the cross section. The equivalent force is F D 800j which acts at a positive 500 D 0.625 ft D 7.5 in to the right of the x axis location of D D 800 origin. 500 lb c 2008 Pearson Education, Inc., Upper Saddle River, NJ. All rights reserved. This material is protected under all copyright laws as they  currently exist. No portion of this material may be reproduced, in any form or by any means, without permission in writing from the publisher. 241 Problem 4.151 In Active Example 4.12, suppose that the force FB is changed to FB D 20i  15j C 30k (kN), and you want to represent system 1 by an equivalent system consisting of a force F acting at the point P with coordinates (4, 3, 2) m and a couple M (system 2). Determine F and M. System 1 y (4, 3, ⫺2) m P FB FA Solution: From Active Example 4.12 we know that x (6, 0, 0) m FA D 10i C 10j  15k kN MC D 90i C 150j C 60k kN-m z MC The force F must equal the sum of the forces in system 1: F2 D F1 : F D FA C FB D 10i  5j C 15k kN In system 2, the sum of the moments about P is M. Therefore equivalence requires that M be equal to the sum of the moments about point P due to the forces and moments in system 1: MP 2 D MP 1 :   i  M D  4  10 j 3 10   k   i 2  C  2 15   20 j 3 15   k   2  C 90i C 150j C 60k kN-m 30  M D 125i C 50j C 20k kN-m Thus F D 10i  5j C 15k kN, M D 125i C 50j C 20k kN-m Problem 4.152 The wall bracket is subjected to the force shown. Determine the moment exerted by the force about the z axis. (b) Determine the moment exerted by the force about the y axis. (c) If you represent the force by a force F acting at O and a couple M, what are F and M? y O (a) z 10i – 30j + 3k (lb) 12 in x Solution: (a) The moment about the z axis is negative, MZ D 130 D 30 ft lb, (b) The moment about the y axis is negative, MY D 13 D 3 ft lb (c) The equivalent force at O must be equal to the force at x D 12 in, thus FEQ D 10i  30j C 3k (lb) The couple moment must equal the moment exerted by the force at x D 12 in. This moment is the product of the moment arm and the y- and zcomponents of the force: M D 130k  13j D 3j  30k (ft lb). 242 c 2008 Pearson Education, Inc., Upper Saddle River, NJ. All rights reserved. This material is protected under all copyright laws as they  currently exist. No portion of this material may be reproduced, in any form or by any means, without permission in writing from the publisher. Problem 4.153 A basketball player executes a “slam dunk” shot, then hangs momentarily on the rim, exerting the two 100-lb forces shown. The dimensions are h D 14 12 in, and r D 9 21 in, and the angle ˛ D 120° . the forces applied: FEQ D 200j. The position vectors of the points of application of the forces are r1 D h C ri, and r2 D ih C r cos ˛  kr sin ˛. The moments about the origin are (a) M D r1 ð F1  C r2 ð F2  D r1 C r2  ð F (b) If you represent the forces he exerts by a force F acting at O and a couple M, what are F and M? The glass backboard will shatter if jMj > 4000 inlb. Does it break? Solution: The equivalent force at the origin must equal the sum of   i j  D  2h C r1 C cos ˛ 0  0 100   k  r sin ˛   0 D 100r sin ˛i  1002h C r1 C cos ˛k. y –100j (lb) For the values of h, r, and ˛ given, the moment is M D 822.72i  3375k in lb. This is the p couple moment required. (b) The magnitude of the moment is jMj D 822.722 C 33752 D 3473.8 in lb. The backboard does not break. O α r –100j (lb) h x z Problem 4.154 In Example 4.14, suppose that the 30lb upward force in system 1 is changed to a 25-lb upward force. If you want to represent system 1 by a single force F (system 2), where does the line of action of F intersect the xz plane? System 1 y 20j (lb) 30j (lb) (⫺3, 0, –2) ft (6, 0, 2) ft (2, 0, 4) ft of the forces in system 1: z F2 D F1 x O Solution: The sum of the forces in system 2 must equal the sum ⫺10j (lb) System 2 y F D 20 C 25  10j lb F D 35j lb The sum of the moments about a point in system 2 must equal the sum of the moments about the same point is system 1. We sum moments about the origin. F M2 D M1  i  x  0   j k   i y z  D  6 35 0   0   j k   i 0 2  C  2 25 0   0   j j k   i 0 4  C  3 0 10 0   0 20  k  2  0  x O P z Expanding the determinants results in the equations 35z D 50 C 40 C 40 35x D 150  20  60 Solving yields x D 2.00 ft, z D 0.857 ft c 2008 Pearson Education, Inc., Upper Saddle River, NJ. All rights reserved. This material is protected under all copyright laws as they  currently exist. No portion of this material may be reproduced, in any form or by any means, without permission in writing from the publisher. 243 Problem 4.155 The normal forces exerted on the car’s tires by the road are C A 0.8 m NA D 5104j (N), x NB D 5027j (N), 0.8 m NC D 3613j (N), D 1.4 m 1.4 m B z ND D 3559j (N). y If you represent these forces by a single equivalent force N, what is N, and where does its line of action intersect the xz plane? Solution: We must satisfy the following three equations   Fy :5104 N C 5027 N C 3613 N C 3559 N D Ry x Mx :5104 N C 3613 N0.8 m  5027 N C 3559 N0.8 m D Ry z  Mz :5104 N C 5027 N1.4 m  3613 N C 3559 N1.4 m D Ry x Solving we find Ry D 17303 N, x D 0.239 m, z D 0.00606 m Problem 4.156 Two forces act on the beam. If you represent them by a force F acting at C and a couple M, what are F and M? Solution: The equivalent force must equal the sum of forces: F D 100j C 80k. The equivalent couple is equal to the moment about C:  M D 380j  3100k D 240j  300k y 100 N 80 N z C x 3m 244 c 2008 Pearson Education, Inc., Upper Saddle River, NJ. All rights reserved. This material is protected under all copyright laws as they  currently exist. No portion of this material may be reproduced, in any form or by any means, without permission in writing from the publisher. Problem 4.157 An axial force of magnitude P acts on the beam. If you represent it by a force F acting at the origin O and a couple M, what are F and M? b Pi z Solution: The equivalent force at the origin is equal to the applied force F D Pi. The position vector of the applied force is r D hj C bk. The moment is  i  M D r ð P D  0 P j h 0 h O  k  Cb  D bPj C hPk. 0  x This is the couple at the origin. y (Note that in the sketch the axis system has been rotated 180 about the x axis; so that up is negative and right is positive for y and z.) Problem 4.158 The brace is being used to remove a screw. (a) (b) If you represent the forces acting on the brace by a force F acting at the origin O and a couple M, what are F and M? If you represent the forces acting on the brace by a force F0 acting at a point P with coordinates xP , yP , zP  and a couple M0 , what are F0 and M0 ? Solution: (a) Equivalent force at the origin O has the same value as the sum of forces,   y h r h B z O 1 A 2 A B x 1 A 2 FX D B  Bi D 0,   FY D A C 12 A C 12 A j D 0, thus F D 0. The equivalent couple moment has the same value as the moment exerted on the brace by the forces,  MO D rAi. Thus the couple at O has the moment M D rAi. (b) The equivalent force at xP , yP , zP  has the same value as the sum of forces on the brace, and the equivalent couple at xP , yP , zP  has the same moment as the moment exerted on the brace by the forces: F D 0, M D rAi. c 2008 Pearson Education, Inc., Upper Saddle River, NJ. All rights reserved. This material is protected under all copyright laws as they  currently exist. No portion of this material may be reproduced, in any form or by any means, without permission in writing from the publisher. 245 y Problem 4.159 Two forces and a couple act on the cube. If you represent them by a force F acting at point P and a couple M, what are F and M? P FB = 2i – j (kN) Solution: The equivalent force at P has the value of the sum of forces,  F = (2 − 1)i + (1 − 1)j + k, FP = i + k (kN). FA = – i + j + k (kN) x MC = 4i – 4j + 4k (kN-m) The equivalentcouple at P has the moment exerted by the forces and moment about P. The position vectors of the forces relative to P are: 1m z rA D i  j C k, and rB D Ck. The moment of the couple:  M D rA ð FA  C rB ð FB  C MC   i  D  1  1   j k   i 1 1  C  0 1 1 2  j k  0 1  C MC 1 0  D 3i  2j C 2k (kN-m). Problem 4.160 The two shafts are subjected to the torques (couples) shown. If you represent the two couples by a force F acting at the origin O and a couple M, what are F and M? (b) What is the magnitude of the total moment exerted by the two couples? y (a) 6 kN-m 4 kN-m 40° Solution: The equivalent force at the origin is zero, F D 0 since there is no resultant force on the system. Represent the couples of 4 kN-m and 6 kN-m magnitudes by the vectors M1 and M2 . The couple at the origin must equal the sum:  30° x z M D M1 C M2 . The sense of M1 is (see sketch) negative with respect to both y and z, and the sense of M2 is positive with respect to both x and y. M1 D 4j sin 30°  k cos 30°  D 2j  3.464k, M2 D 6i cos 40° C j sin 40°  D 4.5963i C 3.8567j. Thus the couple at the origin is MO D 4.6i C 1.86j  3.46k (kN-m) (b) The magnitude of the total moment exerted by the two couples is p jMO j D 4.62 C 1.862 C 3.462 D 6.05 (kN-m) 246 c 2008 Pearson Education, Inc., Upper Saddle River, NJ. All rights reserved. This material is protected under all copyright laws as they  currently exist. No portion of this material may be reproduced, in any form or by any means, without permission in writing from the publisher. Problem 4.161 The two systems of forces and moments acting on the bar are equivalent. If y FA D 30i C 30j  20k (kN), FA FB D 40i  20j C 25k (kN), z A 2m MB D 10i C 40j  10k (kN-m), MB 2m B what are F and M? x FB System 1 y Solution: F F D FA C FB D 70i C 10j C 5k kN z M D 2 mi ð FA C 4 mi ð FB C MB x M D 10i  20j  30k kNm System 2 Problem 4.162 Point G is at the center of the block. The forces are FA D 20i C 10j C 20k (lb), Solution: The equivalent force is the sum of the forces:  F D 20i C 10 C 10j C 20  10k D 20i C 20j C 10k (lb). FB D 10j  10k (lb). The equivalent couple is the sum of the moments about G. The position vectors are: If you represent the two forces by a force F acting at G and a couple M, what are F and M? rA D 15i C 5j C 10k (in), rB D 15i C 5j  10k. y The sum of the moments:  MG D rA ð FA  C rB ð FB  FB FA 10 in x G   i  D  15  20   j k   i j 5 10  C  15 5 10 20   0 10  k  10  10  D 50i C 250j C 100k (in lb) 20 in z 30 in c 2008 Pearson Education, Inc., Upper Saddle River, NJ. All rights reserved. This material is protected under all copyright laws as they  currently exist. No portion of this material may be reproduced, in any form or by any means, without permission in writing from the publisher. 247 Problem 4.163 The engine above the airplane’s fuselage exerts a thrust T0 D 16 kip, and each of the engines under the wings exerts a thrust TU D 12 kip. The dimensions are h D 8 ft, c D 12 ft, and b D 16 ft. If you represent the three thrust forces by a force F acting at the origin O and a couple M, what are F and M? y T0 c O z h 2 TU y Solution: The equivalent thrust at the point G is equal to the sum of the thrusts:  T D 16 C 12 C 12 D 40 kip b The sum of the moments about the point G is  x O b M D r1U ð TU  C r2U ð TU  C rO ð TO  D r1U C r2U  ð TU C rO ð TO . The position vectors are r1U D Cbi  hj, r2U D bi  hj, and rO D Ccj. For h D 8 ft, c D 12 ft, and b D 16 ft, the sum of the moments is   i  M D  0 0   j k   i 16 0  C  0 0 12   0 j 12 0  k  0  D 192 C 192i D 0. 16  Thus the equivalent couple is M D 0 Problem 4.164 Consider the airplane described in Problem 4.163 and suppose that the engine under the wing to the pilot’s right loses thrust. Solution: The sum of the forces is now (a) The sum of the moments is now: If you represent the two remaining thrust forces by a force F acting at the origin O and a couple M, what are F and M? (b) If you represent the two remaining thrust forces by the force F alone, where does its line of action intersect the xy plane?   F D 12 C 16 D 28k (kip). M D r2U ð TU  C rO ð TO . For h D 8 ft, c D 12 ft, and b D 16 ft, using the position vectors for the engines given in Problem 4.163, the equivalent couple is   i  M D  16  0   j k   i 8 0  C  0 0 12   0  j k  12 0  D 96i  192j (ft kip) 0 16  (b) The moment of the single force is  i j  M D  x y 0 0  k  z  D 28yi  28xj D 96i  192j. 28  From which xD 96 192 D 6.86 ft, and y D D 3.43 ft. 28 28 As to be expected, z can have any value, corresponding to any point on the line of action. Arbitrarily choose z D 0, so that the coordinates of the point of action are (6.86, 3.43, 0). 248 c 2008 Pearson Education, Inc., Upper Saddle River, NJ. All rights reserved. This material is protected under all copyright laws as they  currently exist. No portion of this material may be reproduced, in any form or by any means, without permission in writing from the publisher. Problem 4.165 The tension in cable AB is 100 lb, and the tension in cable CD is 60 lb. Suppose that you want to replace these two cables by a single cable EF so that the force exerted on the wall at E is equivalent to the two forces exerted by cables AB and CD on the walls at A and C. What is the tension in cable EF, and what are the coordinates of points E and F? y y C (4, 6, 0) ft (0, 6, 6) ft E x A x D (7, 0, 2) ft B F (3, 0, 8) ft z z Solution: The position vectors of the points A, B, C, and D are For the systems to be equivalent, the moments about the origin must be the same. The moments about the origin are rA D 0i C 6j C 6k,  rB D 3i C 0j C 8k, MO D rA ð FA  C rC ð FC    i  D  0  42.86 rC D 4i C 6j C 0k, and j 6 85.71   k   i 6  C  4 28.57   25.72 j 6 51.43  k  0  17.14  rD D 7i C 0j C 2k. D 788.57i C 188.57j  617.14k. The unit vectors parallel to the cables are obtained as follows: rAB D rB  rA D 3i  6j C 2k, jrAB j D p 32 C 62 C 22 D 7, This result is used to establish the coordinates of the point E. For the one cable system, the end E is located at x D 0. The moment is M1 D r ð FEF from which   i  D  0  68.58 j y 137.14  k  z  45.71  eAB D 0.4286i  0.8571j C 0.2857k. D 45.71y C 137.14zi C 68.58zj  68.58yk rCD D rD  rC D 3i  6j C 2k, D 788.57i C 188.57j  617.14k, jrCD j D p 32 C 62 C 22 D 7, from which eCD D 0.4286i  0.8571j C 0.2857k. Since eAB D eCD , the cables are parallel . To duplicate the force, the single cable EF must have the same unit vector. The force on the wall at point A is FA D 100eAB D 42.86i  85.71j C 28.57k (lb). The force on the wall at point C is from above. From which yD 617.14 D 8.999 . . . D 9 ft 68.58 zD 188.57 D 2.75 ft. 68.58 Thus the coordinates of point E are E (0, 9, 2.75) ft. The coordinates of the point F are found as follows: Let L be the length of cable EF. Thus, from the definition of the unit vector, yF  yE D Ley with 9 D 10.5 ft. The other coordithe condition that yF D 0, L D 0.8571 nates are xF  xE D LeX , from which xF D 0 C 10.50.4286 D 4.5 ft zF  zE D LeZ , from which zF D 2.75 C 10.50.2857 D 5.75 ft The coordinates of F are F (4.5, 0, 5.75) ft FC D 60eCD D 25.72i  51.43j C 17.14k (lb). The total force is FEF D 68.58i  137.14j C 45.71k (lb), jFEF j D 160 lb. c 2008 Pearson Education, Inc., Upper Saddle River, NJ. All rights reserved. This material is protected under all copyright laws as they  currently exist. No portion of this material may be reproduced, in any form or by any means, without permission in writing from the publisher. 249 y Problem 4.166 The distance s D 4 m. If you represent the force and the 200-N-m couple by a force F acting at origin O and a couple M, what are F and M? (2, 6, 0) m s Solution: The equivalent force at the origin is F D 100i C 20j  20k. 100i ⫹ 20j ⫺ 20k (N) O The strategy is to establish the position vector of the action point of the force relative to the origin O for the purpose of determining the moment exerted by the force about the origin. The position of the top of the bar is 200 N-m x (4, 0, 3) m z rT D 2i C 6j C 0k. The vector parallel to the bar, pointing toward the base, is rTB D 2i  6j C 3k, with a magnitude of jrTB j D 7. The unit vector parallel to the bar is eTB D 0.2857i  0.8571j C 0.4286k. The vector from the top of the bar to the action point of the force is rTF D seTB D 4eTB D 1.1429i  3.4286j C 1.7143k. The position vector of the action point from the origin is rF D rT C rTF D 3.1429i C 2.5714j C 1.7143k. The moment of the force about the origin is   i  MF D r ð F D  3.1429  100 j 2.5714 20  k  1.7143  20  D 85.71i C 234.20j  194.3k. The couple is obtained from the unit vector and the magnitude. The sense of the moment is directed positively toward the top of the bar. MC D 200eTB D 57.14i C 171.42j  85.72k. The sum of the moments is M D MF C MC D 142.86i C 405.72j  280k. This is the moment of the equivalent couple at the origin. 250 c 2008 Pearson Education, Inc., Upper Saddle River, NJ. All rights reserved. This material is protected under all copyright laws as they  currently exist. No portion of this material may be reproduced, in any form or by any means, without permission in writing from the publisher. Problem 4.167 The force F and couple M in system 1 are System 1 System 2 y F D 12i C 4j  3k (lb), y M M D 4i C 7j C 4k (ft-lb). F Suppose you want to represent system 1 by a wrench (system 2). Determine the couple Mp and the coordinates x and z where the line of action of the force intersects the xz plane. O z O x z Mp F x (x, 0, z) Solution: The component of M that is parallel to F is found as follows: The unit vector parallel to F is eF D F D 0.9231i C 0.3077j  0.2308k. jFj The component of M parallel to F is MP D eF Ð MeF D 4.5444i C 1.5148j  1.1361k (ft-lb). The component of M normal to F is MN D M  MP D 0.5444i C 5.4858j C 5.1361k (ft-lb). The moment of F must produce a moment equal to the normal component of M. The moment is   i  MF D r ð F D  x  12  k  z  D 4zi C 3x C 12zj C 4xk, 3  j 0 4 from which zD 0.5444 D 0.1361 ft 4 xD 5.1362 D 1.2840 ft 4 Problem 4.168 A system consists of a force F acting at the origin O and a couple M, where F D 10i (lb), M D 20j (ft-lb). If you represent the system by a wrench consisting of the force F and a parallel couple Mp , what is Mp , and where does the line of action F intersect the yz plane? Solution: The component of M parallel to F is zero, since MP D eF Ð MeF D 0. The normal component is equal to M. The equivalent force must produce the same moment as the normal component   i j  M D r ð F D  0 y  10 0 from which z D  k  z  D 10zj  10yk D 20j, 0 20 D 2 ft and y D 0 10 c 2008 Pearson Education, Inc., Upper Saddle River, NJ. All rights reserved. This material is protected under all copyright laws as they  currently exist. No portion of this material may be reproduced, in any form or by any means, without permission in writing from the publisher. 251 Problem 4.169 A system consists of a force F acting at the origin O and a couple M, where F D i C 2j C 5k (N), M D 10i C 8j  4k (N-m). If you represent it by a wrench consisting of the force F and a parallel couple Mp , (a) determine Mp , and determine where the line of action of F intersects (b) the xz plane, (c) the yz plane. Solution: The unit vector parallel to F is eF D F D 0.1826i C 0.3651j C 0.9129k. jFj from which zD 9.8 5 D 4.9 m, and x D D 2.5 m 2 2 (a) The component of M parallel to F is (c) The intersection with the yz plane is MP D eF Ð MeF D 0.2i C 0.4j C 1.0k (N-m).  i  MN D r ð F D  0 1 The normal component is j y 2  k  z  D 5y  2zi C zj  yk 5 D 9.8i C 7.6j  5k, MN D M  MP D 9.8i C 7.6j  5k. The moment of the force about the origin must be equal to the normal component of the moment. (b) The intersection with the xz plane:  i j  MN D r ð F D  x 0 1 2  k  z  D 2zi  5x  zj C 2xk 5 from which y D 5 m and z D 7.6 m D 9.8i C 7.6j  5k, Problem 4.170 Consider the force F acting at the origin O and the couple M given in Example 4.15. If you represent this system by a wrench, where does the line of action of the force intersect the xy plane? 6j C 2k (N), and M D 12i C 4j C 6k (N-m). The normal component of the moment is MN D 7.592i  4.816j C 3.061k (N-m). y The moment produced by the force must equal the normal component: F  i j  MN D r ð F D  x y 3 6 M O Solution: From Example 4.15 the force and moment are F D 3i C  k  0  2 x D 2yi  2xj C 6x  3yk D 7.592i  4.816j C 3.061k, z from which xD 252 4.816 7.592 D 2.408 m and y D D 3.796 m 2 2 c 2008 Pearson Education, Inc., Upper Saddle River, NJ. All rights reserved. This material is protected under all copyright laws as they  currently exist. No portion of this material may be reproduced, in any form or by any means, without permission in writing from the publisher. Problem 4.171 Consider the force F acting at the origin O and the couple M given in Example 4.15. If you represent this system by a wrench, where does the line of action of the force intersect the plane y D 3 m? Solution: From Example 4.15 (see also Problem 4.170) the force is F D 3i C 6j C 2k, and the normal component of the moment is MN D 7.592i  4.816j C 3.061k. The moment produced by the force must be equal to the normal component:  i  MN D r ð F D  x 3  j k  3 z  D 6  6zi  2x  3zj C 6x  9k 6 2 D 7.592i  4.816j C 3.061k, from which xD Problem 4.172 A wrench consists of a force of magnitude 100 N acting at the origin O and a couple of magnitude 60 N-m. The force and couple point in the direction from O to the point (1, 1, 2) m. If you represent the wrench by a force F acting at point (5, 3, 1) m and a couple M, what are F and M? 9 C 3.061 6  7.592 D 2.01 m and z D D 0.2653 m 6 6 Solution: The vector parallel to the force is rF D i C j C 2k, from which the unit vector parallel to the force is eF D 0.4082i C 0.4082j C 0.8165k. The force and moment at the origin are F D jFjeOF D 40.82i C 40.82j C 81.65k (N), and M D 24.492i C 24.492j C 48.99k (N-m). The force and moment are parallel. At the point (5, 3, 1) m the equivalent force is equal to the force at the origin, given above. The moment of this force about the origin is   i  MF D r ð F D  5  40.82 j 3 40.82  k  1  81.65  D 204.13i  367.43j C 81.64k. For the moments to be equal in the two systems, the added equivalent couple must be MC D M  MF D 176.94i C 391.92j  32.65k (N-m) Problem 4.173 System 1 consists of two forces and a couple. Suppose that you want to represent it by a wrench (system 2). Determine the force F, the couple Mp , and the coordinates x and z where the line of action of F intersects the xz plane. Solution: The sum of the forces in System 1 is F D 300j C 600k (N). The equivalent force in System 2 must have this value. The unit vector parallel to the force is eF D 0.4472j C 0.8944k. The sum of the moments in System 1 is M D 6003i C 3004k C 1000i C 600j System 1 y System 2 D 2800i C 600j C 1200k (kN m). y 1000i + 600j (kN-m) The component parallel to the force is MP D 599.963j C 1199.93k (kN-m) D 600j C 1200k (kN-m). 600k (kN) 3m 300j (kN) Mp x x 4m z F z (x, 0, z) The normal component is MN D M  MP D 2800i. The moment of the force  i  MN D  x 0 j 0 300  k  z  D 300zi  600xj C 300xk D 2800i, 600  from which x D 0, z D 2800 D 9.333 m 300 c 2008 Pearson Education, Inc., Upper Saddle River, NJ. All rights reserved. This material is protected under all copyright laws as they  currently exist. No portion of this material may be reproduced, in any form or by any means, without permission in writing from the publisher. 253 y Problem 4.174 A plumber exerts the two forces shown to loosen a pipe. What total moment does he exert about the axis of the pipe? (b) If you represent the two forces by a force F acting at O and a couple M, what are F and M? (c) If you represent the two forces by a wrench consisting of the force F and a parallel couple Mp , what is Mp , and where does the line of action of F intersect the xy plane? 12 in 6 in O (a) z x 16 in 16 in 50 k (lb) –70 k (lb) Solution: The sum of the forces is  (a) F D 50k  70k D 20k (lb). The total moment exerted on the pipe is M D 1620i D 320i (ft lb). (b) The equivalent force at O is F D 20k. The sum of the moments about O is  MO D r1 ð F1  C r2 ð F2    i  D  12  0 j 16 0   k   i 0  C  18 50   0  j k  16 0  0 70  D 320i C 660j. (c) The unit vector parallel to the force is eF D k, hence the moment parallel to the force is MP D eF Ð MeF D 0, and the moment normal to the force is MN D M  MP D 320i C 660j. The force at the location of the wrench must produce this moment for the wrench to be equivalent.  i j  MN D  x y 0 0  k  0  D 20yi C 20xj D 320i C 660j, 20  from which x D 254 660 320 D 33 in, y D D 16 in 20 20 c 2008 Pearson Education, Inc., Upper Saddle River, NJ. All rights reserved. This material is protected under all copyright laws as they  currently exist. No portion of this material may be reproduced, in any form or by any means, without permission in writing from the publisher. Problem 4.175 The Leaning Tower of Pisa is approximately 55 m tall and 7 m in diamteter. The horizontal displacement of the top of the tower from the vertical is approximately 5 m. Its mass is approximately 3.2 ð 106 kg. If you model the tower as a cylinder and assume that its weight acts at the center, what is the magnitude of the moment exerted by the weight about the point at the center of the tower’s base? 5m Solution: The unstretched length of spring is 1 m and the spring N . Assume that the bar is a quarter circle, with m a radius of 4 m. The stretched length of the spring is found from the Pythagorean Theorem: The height of the attachment point is h D 4 sin ˛ m, and the distance from the center is 4 cos ˛. The stretched length of the spring is constant is k D 20 LD 4m B k 3m α  3  h2 C 4 cos ˛2 m. A The spring force is F D 20L  1 N. The angle that the spring makes with a vertical line parallel to A is  ˇ D tan1 3h 4 cos ˛  Moment at B as function of alpha 120 . The horizontal component of the spring force is FX D F cos ˇ N. The vertical component of the force is FY D F sin ˇ N. The displacement of the attachment point to the left of point A is d D 41  cos ˛ m, hence the action of the vertical component is negative, and the action of the horizontal component is positive. The moment about A is  MA D dFY C hFX . M o m e n t , 100 80 60 40 Collecting terms and equations, N m 20 h D 4 sin ˛ m, 0 FY D F sin ˇ N, 0 10 20 30 40 50 60 70 80 90 Alpha, deg FX D F cos ˇ N, F D 20L  1 N, LD  3  h2 C 4 cos ˛2 m,  ˇ D tan1 3h 4 cos ˛  . A programmable calculator or a commercial package such as TK Solver or Mathcad is almost essential to the solution of this and the following problems. The commercial package TK Solver PLUS was used here to plot the graph of M against ˛. Using the graph as a guide, the following tabular values were taken about the maximum: ˛, deg Moment, N-m 41.5 42.0 42.5 101.463 101.483 101.472 The maximum value of the moment is estimated at MB D 101.49 N-m, which occurs at approximately ˛ D 42.2° c 2008 Pearson Education, Inc., Upper Saddle River, NJ. All rights reserved. This material is protected under all copyright laws as they  currently exist. No portion of this material may be reproduced, in any form or by any means, without permission in writing from the publisher. 255 y Problem 4.176 The cable AB exerts a 300-N force on the support A that points from A toward B. Determine the magnitude of the moment the force exerts about point P. B (0.3, 0.6) m A (⫺0.4, 0.3) m Solution: F D 300 N x P (0.5, ⫺0.2) m 0.7i C 0.3j p , 0.58 rPA D 0.9i C 0.5j m MP D rPA ð F D 244 Nmk ) MP D 244 Nm Problem 4.177 Three forces act on the structure. The sum of the moments due to the forces about A is zero. Determine the magnitude of the force F. 30⬚ 45⬚ 2 kN 4 kN Solution:  b p MA D 4 kN 2b  2 kN cos 30° 3b A F C 2 kN sin 30° b C F4b D 0 2b b b Solving we find F D 2.463 kN Problem 4.178 Determine the moment of the 400-N force (a) about A, (b) about B. 30° 400 N 220 mm A Solution: Use the two dimensional description of the moment. The vertical and horizontal components of the 200 N force are 260 mm FY D 400 sin 30° D 200 N, FX D C400 cos 30° D 346.41 N. (a) B 500 mm The moment arm from A to the line of action of the horizontal component is 0.22 m. The moment arm from A to the vertical component is zero. The moment about A is negative, MA D 0.22346.41 D 76.21 N-m (b) 256 The perpendicular distances to the lines of action of the vertical and horizontal components of the force from B are d1 D 0.5 m, and d2 D 0.48 m. The action of the vertical component is positive, and the action of the horizontal component is negative. The sum of the moments: MB D C0.5200  0.48346.41 D 66.28 N-m c 2008 Pearson Education, Inc., Upper Saddle River, NJ. All rights reserved. This material is protected under all copyright laws as they  currently exist. No portion of this material may be reproduced, in any form or by any means, without permission in writing from the publisher. Problem 4.179 Determine the sum of the moments exerted about A by the three forces and the couple. A 5 ft 300 lb 800 ft-lb Solution: Establish coordinates with origin at A, x horizontal, and y vertical with respect to the page. The moment exerted by the couple is the same about any point. The moment of the 300 lb force about A is M300 D 6i  5j ð 300j D 1800k ft-lb. 200 lb 200 lb 6 ft The moment of the downward 200 lb force about A is zero since the line of action of the force passes through A. The moment of the 200 lb force which pulls to the right is 3 ft M200 D 3i  5j ð 200i D 1000k (ft-lb). The moment of the couple is MC D 800k (ft-lb). Summing the four moments, we get MA D 1800 C 0 C 1000  800k D 1600k (ft-lb) Problem 4.180 In Problem 4.179, if you represent the three forces and the couple by an equivalent system consisting of a force F acting at A and a couple M, what are the magnitudes of F and M? Solution: The equivalent force will be equal to the sum of the forces and the equivalent couple will be equal to the sum of the moments about A. From the solution to Problem 4.189, the equivalent couple will be C D MA D 1600k (ft-lb). The equivalent force will be FEQUIV. D 200i  200j C 300j D 200i C 100j (lb) Problem 4.181 The vector sum of the forces acting on the beam is zero, and the sum of the moments about A is zero. (a) (b) 30° 220 mm What are the forces Ax , Ay , and B? What is the sum of the moments about B? 400 N Ay Ax 260 mm Solution: The vertical and horizontal components of the 400 N force are: FX D 400 cos 30° D 346.41 N, 500 mm B FY D 400 sin 30° D 200 N. The sum of the forces is  FX D AX C 346.41 D 0, from which AX D 346.41 N  FY D AY C B  200 D 0. The sum of the moments about A is  MA D 0.5B  0.22346.41 D 0, from which B D 152.42 N. Substitute into the force equation to get AY D 200  B D 47.58 N (b) The moments about B are MB D 0.5AY  0.48346.41  0.26AX C 0.5200 D 0 c 2008 Pearson Education, Inc., Upper Saddle River, NJ. All rights reserved. This material is protected under all copyright laws as they  currently exist. No portion of this material may be reproduced, in any form or by any means, without permission in writing from the publisher. 257 Problem 4.182 The hydraulic piston BC exerts a 970lb force on the boom at C in the direction parallel to the piston. The angle ˛ D 40° . The sum of the moment about A due to the force exerted on the boom by the piston and the weight of the suspended load is zero. What is the weight of the suspended load? t 6f t 9f Solution: The horizontal (x) and vertical (y) coordinates of point C relative to point B are C a A x D 9 ft cos ˛  6 ft D 0.894 ft B 6 ft y D 9 ft sin ˛ D 5.79 ft The angle between the piston BC and the horizontal is ˇ D tan1 y/x D 81.2° The horizontal and vertical components of the force exerted by the piston at C are Cx D 970 lb cos ˇ D 148 lb Cy D 970 lb sin ˇ D 959 lb The sum of the moments about A due to the pistion force and the suspended weight W is MA D W15 ft cos ˛ C Cy 9 ft cos a  Cx 9 ft sin ˛ D 0 Solving, yields W D 501 lb y Problem 4.183 The force F D 60i C 60j (lb). F (a) Determine the moment of F about point A. (b) What is the perpendicular distance from point A to the line of action of F? (4, – 4, 2) ft x A (8, 2, 12) ft z Solution: The position vector of A and the point of action are (b) The magnitude of the moment is p 6002 C 6002 C 6002 D 1039.3 ft lb. rA D 8i C 2j C 12k (ft), and rF D 4i  4j C 2k. jMA j D The vector from A to F is p The magnitude of the force is jFj D 602 C 602 D 84.8528 lb. The perpendicular distance from A to the line of action is rAF D rF  rA D 4  8i C 4  2j C 2  12k DD D 4i  6j  10k. (a) 1039.3 D 12.25 ft 84.8528 The moment about A is   i  MA D rAF ð F D  4  60  j k  6 10  60 0  D 600i C 600j  600k (ft lb) 258 c 2008 Pearson Education, Inc., Upper Saddle River, NJ. All rights reserved. This material is protected under all copyright laws as they  currently exist. No portion of this material may be reproduced, in any form or by any means, without permission in writing from the publisher. y Problem 4.184 The 20-kg mass is suspended by cables attached to three vertical 2-m posts. Point A is at (0, 1.2, 0) m. Determine the moment about the base E due to the force exerted on the post BE by the cable AB. C B D A 1m 1m E 2m 0.3 m x z Solution: The strategy is to develop the simultaneous equations in the unknown tensions in the cables, and use the tension in AB to find the moment about E. This strategy requires the unit vectors parallel to the cables. The position vectors of the points are: The equilibrium conditions are TAB C TAC C TAD D W. Collect like terms in i, j, k: rOA D 1.2j,   FX D 0.2281TAB C 0TAC C 0.9284TAD i D 0 FY D C0.6082 Ð TAB C 0.6247 Ð TAC rOB D 0.3i C 2j C 1k, rOC D 2j  1k, rOD D 2i C 2j, rOE D 0.3i C 1k. C 0.3714 Ð TAD  196.2j D 0  FZ D C0.7603 Ð TAB  0.7809 Ð TAC C 0 Ð TAD k D 0 Solve: TAB D 150.04 N, The vectors parallel to the cables are: TAC D 146.08 N, rAB D rOB  rOA D 0.3i C 0.8j C 1k, TAD D 36.86 N. rAC D rOC  rOA D C0.8j  1k, rAD D rOD  rOA D C2i C 0.8j. The unit vectors parallel to the cables are: eAB D rAB D 0.2281i C 0.6082j C 0.7603k : jrAB j eAC D 0i C 0.6247j  0.7809k, The moment about E is ME D rEB ð TAB eAB  D TAB rEB ð eAB    i  D 150  0  0.2281 j 2 C0.6082   k   0  C0.7603  D 228i  68.43k (N-m) eAD D C0.9284i C 0.3714j C 0k. The tensions in the cables are TAB D TAB eAB , TAC D TAC eAC , and TAD D TAD eAD . c 2008 Pearson Education, Inc., Upper Saddle River, NJ. All rights reserved. This material is protected under all copyright laws as they  currently exist. No portion of this material may be reproduced, in any form or by any means, without permission in writing from the publisher. 259 Problem 4.185 What is the total moment due to the two couples? (a) Express the answer by giving the magnitude and stating whether the moment is clockwise or counterclockwise. (b) Express the answer as a vector. y 100 N 4m 100 N 2m x 2m 100 N 100 N Solution: (a) (b) 4m The couple in which the forces are 4 m apart exerts a counterclockwise moment of magnitude 100 N4 m D 400 N-m. The couple in which the forces are 8 m apart exerts a clockwise moment of magnitude 100 N8 m D 800 N-m. The sum of their moments is a clockwise moment of 400 N-m. The vector representation of the clockwise moment of 400 N-m magnitude is 400k (N-m). This expression can also be obtained by calculating the sum of the moments of the four forces about any point. The sum of the moments about the origin is M D 2 mi ð 100 Nj C 2 mi ð 100 Nj C 4 mj ð 100 Ni C 4 mj ð 100 Ni D 400 N-mk (a) 400 N-m clockwise (b)  400k N-m Problem 4.186 The bar AB supporting the lid of the grand piano exerts a force F D 6i C 35j  12k (lb) at B. The coordinates of B are (3, 4, 3) ft. What is the moment of the force about the hinge line of the lid (the x axis)? y Solution: The position vector of point B is rOB D 3i C 4j C 3k. The moment about the x axis due to the force is MX D eX Ð rOB ð F D i Ð rOB ð F   1 0  MX D  3 4  6 35  0  3  D 153 ft lb 12  B x A z 260 c 2008 Pearson Education, Inc., Upper Saddle River, NJ. All rights reserved. This material is protected under all copyright laws as they  currently exist. No portion of this material may be reproduced, in any form or by any means, without permission in writing from the publisher. Problem 4.187 Determine the moment of the vertical 800-lb force about point C. y 800 lb A (4, 3, 4) ft B D (6, 0, 0) ft Solution: The force vector acting at A is F D 800j (lb) and the x position vector from C to A is rCA D xA  xC i C yA  yC j C zA  zC k z C (5, 0, 6) ft D 4  5i C 3  0j C 4  6k D 1i C 3j  2k (ft). The moment about C is   i  MC D  1  0 j 3 800  k  2  D 1600i C 0j C 800k (ft-lb) 0  Problem 4.188 In Problem 4.187, determine the moment of the vertical 800-lb force about the straight line through points C and D. Solution: In Problem 4.197, we found the moment of the 800 lb force about point C to be given by MC D 1600i C 0j C 800j (ft-lb). The vector from C to D is given by rCD D xD  xC i C yD  yC j C zD  zC k D 6  5i C 0  0j C 0  6k D 1i C 0j  6j (ft), and its magnitude is jrCD j D p p 12 C 62 D 37 (ft). The unit vector from C to D is given by 6 1 eCD D p i  p k. 37 37 The moment of the 800 lb vertical force about line CD is given by  MCD D  D  6 1 p i  p k Ð 1600i C 0j C 800j (ft-lb) 37 37 1600  4800 p 37  (ft-lb). Carrying out the calculations, we get MCD D 1052 (ft-lb) c 2008 Pearson Education, Inc., Upper Saddle River, NJ. All rights reserved. This material is protected under all copyright laws as they  currently exist. No portion of this material may be reproduced, in any form or by any means, without permission in writing from the publisher. 261 Problem 4.189 The system of cables and pulleys supports the 300-lb weight of the work platform. If you represent the upward force exerted at E by cable EF and the upward force exerted at G by cable GH by a single equivalent force F, what is F, and where does its line of action intersect the x axis? Solution: The cable-pulley combination does not produce a moment. Hence the equivalent force does not. The equivalent force is 600 j D 300j (lb). The equal to the total supported weight, or F D C 2 8 force occurs at midpoint of the platform width, x D D 4 ft 2 H F E B y A G 60° D 60° C x 8 ft Problem 4.190 Consider the system in Problem 4.189. Solution: The vertical component of the tension is each cable must equal half the weight supported. (a) What are the tensions in cables AB and CD? (b) If you represent the forces exerted by the cables at A and C by a single equivalent force F, what is F, and where does its line of action intersect the x axis? 262 TAB sin 60° D 150 lb, from which TAB D symmetry, the tension TCD D 173.2 lb. 150 D 173.2 lb. By sin 60° The single force must equal the sum of the vertical components; since there is no resultant moment produced by the cables, the force is F D 300j lb and it acts at the platform width midpoint x D 4 ft. c 2008 Pearson Education, Inc., Upper Saddle River, NJ. All rights reserved. This material is protected under all copyright laws as they  currently exist. No portion of this material may be reproduced, in any form or by any means, without permission in writing from the publisher. Problem 4.191 The two systems are equivalent. Determine the forces Ax and Ay , and the couple MA . System 1 y 20 N 400 mm Solution: The sum of the forces for System 1 is   Ax FX D AX C 20i, x 30 N Ay FY D AY C 30j. 600 mm 400 mm The sum of forces for System 2 is   System 2 y FX D 20i and 8 N-m FY D 80  10j. Equating the two systems: 400 mm AX C 20 D 20 from which AX D 40 N 20 N MA 10 N x AY C 30 D 80  10 from which AY D 40 N 80 N The sum of the moments about the left end for System 1 is  600 mm 400 mm M1 D 0.420 C 301 D 22 N-m. The sum of moments about the left end for System 2 is  M2 D MA  101  8 D MA  18. Equating the moments for the two systems: MA D 18 C 22 D 40 N-m Problem 4.192 If you represent the equivalent systems in Problem 4.191 by a force F acting at the origin and a couple M, what are F and M? Solution: Summing the forces in System 1, F D AX C 20i C AY C 30j. Substituting from the solution in Problem 4.201, F D 20i C 70j. The moment is M D 200.4k C 30k D 22k (N-m) Problem 4.193 If you represent the equivalent systems in Problem 4.191 by a force F, what is F, and where does its line of action intersect the x axis? Solution: The force is F D 20i C 70j. The moment to be represented is   i  M D r ð F D 22k D  x  20 from which x D  j k  0 0  D 70xk, 70 0  22 D 0.3143 m 70 c 2008 Pearson Education, Inc., Upper Saddle River, NJ. All rights reserved. This material is protected under all copyright laws as they  currently exist. No portion of this material may be reproduced, in any form or by any means, without permission in writing from the publisher. 263 Problem 4.194 The two systems are equivalent. If Solution: The sum of forces in the two systems must be equal, thus F0 D F D 100i C 40j C 30k (lb). F D 100i C 40j C 30k (lb), The moment for the unprimed system is MT D r ð F C M. 0 M D 80i C 120j C 40k (in-lb), The moment for the primed system is M0T D r0 ð F C M0 . determine F0 and M. The position vectors are r D 0i C 6j C 6k, and r0 D 4i C 6j C 6k. Equating the moments and solving for the unknown moment System 1 y System 2 y M MD F 4 in M0 C r0   i   r ð F D 80i C 120j C 40k C  4  100 j 0 40  k  0  30  4 in F' D 80i C 200k (in-lb) M' 6 in D 80i C 120j C 40k  120j C 160k 6 in x x 6 in 6 in z z Problem 4.195 The tugboats A and B exert forces FA D 1 kN and FB D 1.2 kN on the ship. The angle  D 30° . If you represent the two forces by a force F acting at the origin O and a couple M, what are F and M? y A FA Solution: The sums of the forces are:   FX D 1 C 1.2 cos 30° i D 2.0392i (kN) 60 m O x FY D 1.2 sin 30° j D 0.6j (kN). 60 m FB The equivalent force at the origin is FEQ D 2.04i C 0.6j θ B The moment about O is MO D rA ð FA C rB ð FB . The vector positions are 25 m rA D 25i C 60j (m), and rB D 25i  60j (m). The moment:   i  MO D  25  1   j k   i 60 0  C  25 0 0   1.0392  j k  60 0  0.6 0  D 12.648k D 12.6k (kN-m) Check: Use a two dimensional description: The moment is MO D 25FB sin 30° C 60FB cos 30°   60FA  D 39.46FB  60FA D 12.6 kN-m 264 c 2008 Pearson Education, Inc., Upper Saddle River, NJ. All rights reserved. This material is protected under all copyright laws as they  currently exist. No portion of this material may be reproduced, in any form or by any means, without permission in writing from the publisher. Problem 4.196 The tugboats A and B in Problem 4.195 exert forces FA D 600 N and FB D 800 N on the ship. The angle  D 45° . If you represent the two forces by a force F, what is F, and where does its line of action intersect the y axis? Solution: The equivalent force is Check: Use a two dimensional description: F D 0.6 C 0.8 cos 45° i C 0.8 sin 45° j D 1.1656i C 0.5656j (kN). MO D 25FB sin 45° C 60FB cos 45°  60FA The moment produced by the two forces is D 24.75FB  60FA D 16.20 kN-m. The single force must produce this moment. MO D rA ð FA C rB ð FB . rA D 25i C 60j (m), and rB D 25i  60j (m).   i  MO D  0  1.1656 The moment: from which The vector positions are   j k   i 60 0  C  25 0 0   0.5656   i  MO D  25  0.6  j k  60 0  D 16.20k (kN-m) 0.5656 0  yD  j k  y 0  D 1.1656yk D 16.20k, 0.5656 0  16.20 D 13.90 m 1.1656 Problem 4.197 The tugboats A and B in Problem 4.195 want to exert two forces on the ship that are equivalent to a force F acting at the origin O of 2-kN magnitude. If FA D 800 N, determine the necessary values of FB and angle . Solution: The equivalent force at the origin is FA C FB cos 2 C sin 2 D FB must be zero:  20002 . The moment about the origin due to FA and FB MO D 60FA C 60FB cos   25FB sin  D 0. These are two equations in two unknowns FB sin  and FB cos . For brevity write x D FB cos , y D FB sin , so that the two equations become x2 C 2FA x C F2A C y 2 D 20002 and 60x  25y  60FA D 0. Eliminate y by solving each equation for y2 and equating the results:  y 2 D 20002  x 2  2FA x  F2A D 60 60  FA C x 25 25 Reduce to obtain the quadratic in x:   1C 60 25 2   C 1C  x 2 C 2FA 1   60 25  60 25 Substitute FA D 800 N to obtain 6.76x2  7616x C 326400 D 0. In canonical form: x2 C 2bx C c D 0, where p b D 563.31, and c D 48284.0, with the solutions x D b š b2  c D 1082.0, D 44.62. From the second equation, y D 1812.9, D 676.81. The force FB has two solutions: Solve for FB and : (1) FB D p 44.62 C 1812.92 D 1813.4 N at the angle   D tan1 2 . FB D 1812.9 44.6  D 88.6° , and 2 p 676.82 C 1082.02 D 1276.2 N, at the angle 2  x   D tan1 676.8 1082.0  D 32.0° 2  F2A  20002 D 0. c 2008 Pearson Education, Inc., Upper Saddle River, NJ. All rights reserved. This material is protected under all copyright laws as they  currently exist. No portion of this material may be reproduced, in any form or by any means, without permission in writing from the publisher. 265 y Problem 4.198 If you represent the forces exerted by the floor on the table legs by a force F acting at the origin O and a couple M, what are F and M? 2m 1m Solution: The sum of the forces is the equivalent force at the origin. F D 50 C 48 C 50 C 42j D 190j (N). The position vectors of the legs are, numbering the legs counterclockwise from the lower left in the sketch: 50 N x r1 D C1k, z 42 N 48 N r2 D 2i C 1k, 50 N r3 D 2i, r4 D 0. The sum of the moments about the origin is  i  MO D  0 0 j 0 48   k   i 1  C  2 0 0 j 0 50   k   i 1  C  2 0 0  j k  0 0  42 0  D 98i C 184k (N-m). This is the couple that acts at the origin. Problem 4.199 If you represent the forces exerted by the floor on the table legs in Problem 4.198 by a force F, what is F, and where does its line of action intersect the xz plane? Solution: From the solution to Problem 4.198 the equivalent force is F D 190j. This force must produce the moment M D 98i C 184k obtained in Problem 4.198.  i  M D  x 0  j k  0 z  D 190zi C 190xk D 98i C 184k, 190 0  from which 266 xD 184 D 0.9684 m and 190 zD 98 D 0.5158 m 190 c 2008 Pearson Education, Inc., Upper Saddle River, NJ. All rights reserved. This material is protected under all copyright laws as they  currently exist. No portion of this material may be reproduced, in any form or by any means, without permission in writing from the publisher. Problem 4.200 Two forces are exerted on the crankshaft by the connecting rods. The direction cosines of FA are cos x D 0.182, cos y D 0.818, and cos z D 0.545, and its magnitude is 4 kN. The direction cosines of FB are cos x D 0.182, cos y D 0.818, and cos z D 0.545, and its magnitude is 2 kN. If you represent the two forces by a force F acting at the origin O and a couple M, what are F and M? Solution: The equivalent force is the sum of the forces: FA D 40.182i C 0.818j C 0.545k D 0.728i C 3.272j C 2.18k (kN) FB D 20.182i C 0.818j  0.545k D 0.364iC1.636j1.09k (kN). The sum: FA C FB D 0.364i C 4.908j C 1.09k (kN) y FB The equivalent couple is the sum of the moments. M D rA ð FA C rB ð FB . The position vectors are: FA rA D 0.16i C 0.08k, 360 mm O rB D 0.36i  0.08k. The sum of the moments: z 160 mm 80 mm 80 mm x   i  M D  0.16  0.728 j 0 3.272   k   i 0.08  C  0.36 2.180   0.364 j 0 1.636  k  0.08  1.090  M D 0.1309i  0.0438j C 1.1125k (kN-m) Problem 4.201 If you represent the two forces exerted on the crankshaft in Problem 4.200 by a wrench consisting of a force F and a parallel couple Mp , what are F and Mp , and where does the line of action of F intersect the xz plane? Solution: From the solution to Problem 4.200, F D 0.364i C 4.908j C 1.09k (kN) and M D 0.1309i  0.0438j C 1.1125k (kN-m). The unit vector parallel to F is eF D F D 0.0722i C 0.9737j C 0.2162k. jFj The moment parallel to the force is MP D eF Ð MeF . Carrying out the operations: MP D 0.2073eF D 0.01497i C 0.2019j C 0.0448k (kN-m). This is the equivalent couple parallel to F. The component of the moment perpendicular to F is MN D M  MP D 0.1159i  0.2457j C 1.0688k. The force exerts this moment about the origin.   i  MN D  x  0.364 j 0 4.908  k  z  1.09  D 4.908zi  1.09x C 0.364zj C 4.908xk D 0.1159i  0.2457j C 1.06884k. From which xD 1.0688 D 0.2178 m, 4.908 zD 0.1159 D 0.0236 m 4.908 c 2008 Pearson Education, Inc., Upper Saddle River, NJ. All rights reserved. This material is protected under all copyright laws as they  currently exist. No portion of this material may be reproduced, in any form or by any means, without permission in writing from the publisher. 267 Problem 5.1 In Active Example 5.1, suppose that the beam is subjected to a 6kN-m counterclockwise couple at the right end in addition to the 4-kN downward force. Draw a sketch of the beam showing its new loading. Draw the free-body diagram of the beam and apply the equilibrium equations to determine the reactions at A. 4 kN A 2m Solution: The equilibrium equations are Fx : Ax D 0 Fy : Ay  4 kN D 0 MA : MA  4 kN2 m C 6 kN-m D 0 Solving yields Ax D 0 Ay D 4 kN MA D 2 kN-m Problem 5.2 The beam has a fixed support at A and is loaded by two forces and a couple. Draw the free-body diagram of the beam and apply equilibrium to determine the reactions at A. 4 kN A 2 kN 6 kN-m 60⬚ 1m 1.5 m 1.5 m Solution: The free-body diagram is drawn. The equilibrium equations are Fx : Ax C 2 kN cos 60° D 0 Fy : Ay C 4 kN C 2 kN sin 60° D 0 MA : MA C 6 kN-m C 4 kN2.5 m C 2 kN sin 60° 4 m D 0 We obtain: Ax D 1 kN, Ay D 5.73 kN, MA D 22.9 kN-m 268 c 2008 Pearson Education, Inc., Upper Saddle River, NJ. All rights reserved. This material is protected under all copyright laws as they  currently exist. No portion of this material may be reproduced, in any form or by any means, without permission in writing from the publisher. Problem 5.3 The beam is subjected to a load F D 400 N and is supported by the rope and the smooth surfaces at A and B. (a) (b) Draw the free-body diagram of the beam. What are the magnitudes of the reactions at A and B? F A B 30° 45° 1.2 m y Solution:   C  FX D 0: A cos 45°  B sin 30° D 0 FY D 0: A sin 45° C B cos 30°  T  400 N D 0 MA D 0:  1.2T  2.7400 C 3.7B cos 30° D 0 1.5 m 1m A F 45° x B 1.5 m 1.2 m 1m 30° T Solving, we get A D 271 N B D 383 N T D 124 N Problem 5.4 (a) Draw the free-body diagram of the beam. (b) Determine the tension in the rope and the reactions at B. 30⬚ 30⬚ 600 lb B A 5 ft 9 ft Solution: Let T be the tension in the rope. The equilibrium equations are: Fx : T sin 30°  600 lb sin 30° C Bx D 0 Fy : T cos 30°  600 lb cos 30° C By D 0 MB : 600 lb cos 30° 9 ft  T cos 30° 14 ft D 0 Solving yields T D 368 lb, Bx D 493 lb, By D 186 lb c 2008 Pearson Education, Inc., Upper Saddle River, NJ. All rights reserved. This material is protected under all copyright laws as they  currently exist. No portion of this material may be reproduced, in any form or by any means, without permission in writing from the publisher. 269 Problem 5.5 (a) Draw the free-body diagram of the 60-lb drill press, assuming that the surfaces at A and B are smooth. (b) Determine the reactions at A and B. Solution: The system is in equilibrium. (a) (b) The free body diagram is shown. The sum of the forces:  FX D 0,  FY D FA C FB  60 D 0 The sum of the moments about point A:  MA D 1060 C 24FB  D 0, from which FB D 60 lb 600 D 25 lb 24 Substitute into the force balance equation: FA D 60  FB D 35 lb A B 10 in 14 in 60 lb A B 10 in FA 270 14 in FB c 2008 Pearson Education, Inc., Upper Saddle River, NJ. All rights reserved. This material is protected under all copyright laws as they  currently exist. No portion of this material may be reproduced, in any form or by any means, without permission in writing from the publisher. Problem 5.6 The masses of the person and the diving board are 54 kg and 36 kg, respectively. Assume that they are in equilibrium. (a) (b) Draw the free-body diagram of the diving board. Determine the reactions at the supports A and B. Solution: (a) (b)    A FX D 0: AX D 0 FY D 0: AY C BY  549.81  369.81 D 0 MA D 0: 1.2BY  2.4369.81 B  4.6549.81 D 0 WD WP AX D 0 N Solving: 1.2 m AY D 1.85 kN 2.4 m BY D 2.74 kN 4.6 m 4.6 m 2.4 m 1.2 m AX Draw the free-body diagram of the ironing board. Determine the reactions at A and B. A B x 3 lb 10 lb 12 in 10 in 20 in Solution: The system is in equilibrium. Substitute into the force balance equation: (a) (b) FA D 13  FB D C15.833 lb The free-body diagram is shown. The sums of the forces are:   FX D 0, WP y Problem 5.7 The ironing board has supports at A and B that can be modeled as roller supports. (a) (b) WD BY AY y A B x FY D FA C FB  10  3 D 0. 12 in 10 in 10 lb 20 in 3 lb FB 10 lb 3 lb The sum of the moments about A is  MA D 12FB  2210  423 D 0, from which FB D FA 346 D 28.833 in. 12 c 2008 Pearson Education, Inc., Upper Saddle River, NJ. All rights reserved. This material is protected under all copyright laws as they  currently exist. No portion of this material may be reproduced, in any form or by any means, without permission in writing from the publisher. 271 Problem 5.8 The distance x D 9 m. (a) Draw the free-body diagram of the beam. (b) Determine the reactions at the supports. 10 kN A B 6m x Solution: 10 kN (a) The FBD (b) The equilibrium equations    x= 9 m Ax Fx : Ax D 0 6m Fy : Ay C By  10 kN D 0 Ay By MA : By 6 m  10 kN9 m D 0 Solving we find Ax D 0, Ay D 5 kN, By D 15 kN Problem 5.9 In Example 5.2, suppose that the 200-lb downward force and the 300 ft-lb counterclockwise couple change places; the 200-lb downward force acts at the right end of the horizontal bar, and the 300 ft-lb counterclockwise couple acts on the horizontal bar 2 ft to the right of the support A. Draw a sketch of the object showing the new loading. Draw the free-body diagram of the object and apply the equilibrium equations to determine the reactions at A. 100 lb 30 2 ft 200 lb A 300 ft-lb 2 ft 2 ft 2 ft Solution: The sketch and free-body diagram are shown. The equilibrium equations are Fx : Ax C 100 lb cos 30° D 0 Fy : Ay C 100 lb sin 30°  200 lb D 0 MA : MA C 300 ft-lb C 100 lb sin 30° 4 ft  100 lb cos 30° 2 ft  200 lb6 ft D 0 We obtain Ax D 86.6 lb Ay D 150 lb MA D 873 ft-lb 272 c 2008 Pearson Education, Inc., Upper Saddle River, NJ. All rights reserved. This material is protected under all copyright laws as they  currently exist. No portion of this material may be reproduced, in any form or by any means, without permission in writing from the publisher. Problem 5.10 (a) Draw the free-body diagram of the beam. (b) Determine the reactions at the supports. 100 lb 400 lb 900 ft-lb A B 3 ft Solution: (a) Both supports are roller supports. The free body diagram is shown. (b) The sum of the forces:   and 4 ft 3 ft 100 lb 4 ft 400 lb 3 ft 4 ft 3 ft 4 ft FX D 0, A FY D FA C FB C 100  400 D 0. 900 ft lb 100 lb 3 ft 4 ft 3 ft B 400 lb 4 ft The sum of the moments about A is  900 ft lb MA D 3100 C 900  7400 C 11FB D 0. From which FB D FA FB 2200 D 200 lb 11 Substitute into the force balance equation to obtain FA D 300  FB D 100 lb Problem 5.11 The person exerts 20-N forces on the pliers. The free-body diagram of one part of the pliers is shown. Notice that the pin at C connecting the two parts of the pliers behaves like a pin support. Determine the reactions at C and the force B exerted on the pliers by the bolt. Solution: The equilibrium equations  MC :B25 mm  20 N cos 45° 80 mm  20 N sin 45° 50 mm D 0   25 mm B C Fx :Cx  20 N sin 45° D 0 Cx Fy :Cy  B  20 N cos 45° 80 mm D0 Solving: Cy 50 mm 45⬚ 20 N C B D 73.5 N, Cx D 14.14 N, Cy D 87.7 N 20 N 20 N c 2008 Pearson Education, Inc., Upper Saddle River, NJ. All rights reserved. This material is protected under all copyright laws as they  currently exist. No portion of this material may be reproduced, in any form or by any means, without permission in writing from the publisher. 273 Problem 5.12 (a) Draw the free-body diagram of the beam. 8 kN 8 kN 2 kN-m A B 30⬚ (b) Determine the reactions at the pin support A. 600 mm Solution: 500 mm 8 kN 600 mm 8 kN (a) The FBD (b) The equilibrium equations  MA :  8 kN0.6 m C 8 kN1.1 m  2 kNm 600 mm 30° 2 kN-m B Ax  B cos 30° 2.3 m D 0   Ay Fx :Ax  B sin 30° D 0 Fy :Ay  8 kN C 8 kN  B cos 30° D 0 Solving Ax D 0.502 kN, Ay D 0.870 kN, B D 1.004 kN Problem 5.13 (a) Draw the free-body diagram of the beam. y A (b) Determine the reactions at the supports. 6m 40 kN B x 8m 12 m Solution: A (a) (b) The FBD The equilibrium equations    MB : 40 kN4 m C A6 m D 0 40 kN Bx Fx : A C Bx D 0 Fy : 40 kN C By D 0 Solving we find By A D Bx D 26.7 kN, By D 40 kN 274 c 2008 Pearson Education, Inc., Upper Saddle River, NJ. All rights reserved. This material is protected under all copyright laws as they  currently exist. No portion of this material may be reproduced, in any form or by any means, without permission in writing from the publisher. Problem 5.14 (a) Draw the free-body diagram of the beam. A (b) If F D 4 kN, what are the reactions at A and B? 2 kN-m F 0.2 m 0.3 m 0.2 m 0.3 m 0.4 m B 2 kN-m Solution: Ax (a) (b) The free-body diagram The equilibrium equations    F = 4 kN MA : 2 kN-m  4 kN0.2 m C B1.0 m D 0 Ay Fx : Ax  4 kN D 0 Fy : Ay C B D 0 Solving: B Ax D 4 kN, Ay D 2.8 kN, B D 2.8 kN c 2008 Pearson Education, Inc., Upper Saddle River, NJ. All rights reserved. This material is protected under all copyright laws as they  currently exist. No portion of this material may be reproduced, in any form or by any means, without permission in writing from the publisher. 275 Problem 5.15 In Example 5.3, suppose that the attachment point for the suspended mass is moved toward point B such that the horizontal distance from A to the attachment point increases from 2 m to 3 m. Draw a sketch of the beam AB showing the new geometry. Draw the free-body diagram of the beam and apply the equilibrium equations to determine the reactions at A to B. 2m 2m B 3m A Solution: From Example 5.3, we know that the mass of the suspended object is 2-Mg. The sketch and free-body diagram are shown. The equilibrium equations are Fx : Ax C Bx D 0 Fy : By  2000 kg9.81 m/s2  D 0 MB : Ax 3 m C 2000 kg9.81 m/s2 1 m D 0 We obtain Ax D 6.54 kN Bx D 6.54 kN By D 19.6 kN 276 c 2008 Pearson Education, Inc., Upper Saddle River, NJ. All rights reserved. This material is protected under all copyright laws as they  currently exist. No portion of this material may be reproduced, in any form or by any means, without permission in writing from the publisher. Problem 5.16 A person doing push-ups pauses in the position shown. His 180-lb weight W acts at the point shown. The dimensions a = 15 in, b = 42 in, and c = 16 in. Determine the normal force exerted by the floor on each of his hands and on each of his feet. c W a b Solution: The free-body diagram is shown. The equilibrium equations are Fy : 2H C 2F  180 lb D 0 MH : Wa C 2Fa C b 0 We find that H D 66.3 lb, F D 23.7 lb Thus 66.3 lb on each hand 23.7 lb on each foot Problem 5.17 The hydraulic piston AB exerts a 400-lb force on the ladder at B in the direction parallel to the piston. Determine the weight of the ladder and the reactions at C. 6 ft W 3 ft A B C 6 ft 3 ft Solution: The free-body diagram of the ladder is shown. The angle between the piston AB and the horizontal is ˛ D tan1 3/6 D 26.6° The equilibrium equations are Fx : Cx C 400 lb cos ˛ D 0 Fy : Cy C 400 lb sin ˛  W D 0 MC : W6 ft  400 lb cos ˛3 ft  400 lb sin ˛3 ft D 0 Solving yields Cx D 358 lb, Cy D 89.4 lb, W D 268 lb c 2008 Pearson Education, Inc., Upper Saddle River, NJ. All rights reserved. This material is protected under all copyright laws as they  currently exist. No portion of this material may be reproduced, in any form or by any means, without permission in writing from the publisher. 277 Problem 5.18 Draw the free-body diagram of the structure by isolating it from its supports at A and E. Determine the reactions at A and E. D 400 lb 2 ft 200 ft-lb A 1 ft B C 100 lb 1 ft E 2 ft 2 ft 2 ft Solution: The free-body diagram is shown. The equilibrium equations are Fx : Ax C 100 lb D 0 Fy : Ay  400 lb C Ey D 0 MA : 100 lb1 ft  400 lb6 ft  200 ft-lb C Ey 4 ft D 0 Solving yields Ax D 100 lb Ay D 225 lb Ey D 625 lb Problem 5.19 (a) Draw the free-body diagram of the beam. (b) Determine the tension in the cable and the reactions at A. A B 30° 30 in 30 in Solution: 800 lb 30 in T (a) The FBD (b) The equilibrium equations  C T 30° Ax MA :  800 lb60 in C T30 in C T sin 30° 90 in D 0   Fx :Ax  T cos 30° D 0 Ay 800 lb Fy :Ay C T C T sin 30°  800 lb D 0 Solving: Ax D 554 lb, Ay D 160 lb, T D 640 lb 278 c 2008 Pearson Education, Inc., Upper Saddle River, NJ. All rights reserved. This material is protected under all copyright laws as they  currently exist. No portion of this material may be reproduced, in any form or by any means, without permission in writing from the publisher. Problem 5.20 The unstretched length of the spring CD is 350 mm. Suppose that you want the lever ABC to exert a 120-N normal force on the smooth surface at A. Determine the necessary value of the spring constant k and the resulting reactions at B. C k 230 mm D 450 mm 20⬚ 180 mm B A Solution: We have F D k  0.23 m2 C 0.3 m2   0.35 330 mm m 300 mm A D 120 N  30 F0.45 m C A cos 20° 0.18 m MB :  p 1429 F 23 30 C A sin 20° 0.33 m D 0   Fx :A cos 20° C Bx C p 30 1429 Fy :  A sin 20° C By  p FD0 23 1429 Bx FD0 A By Solving we find: 20° k D 3380 N/m, Bx D 188 N, By D 98.7 N c 2008 Pearson Education, Inc., Upper Saddle River, NJ. All rights reserved. This material is protected under all copyright laws as they  currently exist. No portion of this material may be reproduced, in any form or by any means, without permission in writing from the publisher. 279 Problem 5.21 The mobile is in equilibrium. The fish B weighs 27 oz. Determine the weights of the fish A, C, and D. (The weights of the crossbars are negligible.) 12 in 3 in A 6 in 2 in B 7 in 2 in C D Solution: Denote the reactions at the supports by FAB , FCD , and FBCD as shown. Start with the crossbar supporting the weights C and D. The sum of the forces is  FCD D C 7 in 2 in FY D C  D C FCD D 0, FBCD from which FCD D C C D. 6 in For the cross bar supporting the weight B, the sum of the forces is  FY D B C FBCD  FCD D 0, from which, substituting, FBCD D B C C C D. B FCD 2 in FAB FBCD A 12 in 3 in For the crossbar supporting C and D, the sum of the moments about the support is  MCD D 7D C 2C D 0, from which D D 2C . 7 For the crossbar supporting B, the sum of the moments is  MBCD D 6FCD  2B D 0, from which, substituting from above FCD D 2C 9C 2B DCCD DCC D , 6 7 7 or C D 7B/27 D 7 oz, and D D 2C/7 D 2 oz. The sum of the moments about the crossbar supporting A is  MAB D 12A  3FBCD D 0, from which, substituting from above, AD 280 27 C 7 C 2 3B C C C D D D 9 oz 12 4 c 2008 Pearson Education, Inc., Upper Saddle River, NJ. All rights reserved. This material is protected under all copyright laws as they  currently exist. No portion of this material may be reproduced, in any form or by any means, without permission in writing from the publisher. y Problem 5.22 The car’s wheelbase (the distance between the wheels) is 2.82 m. The mass of the car is 1760 kg and its weight acts at the point x D 2.00 m, y D 0.68 m. If the angle ˛ D 15° , what is the total normal force exerted on the two rear tires by the sloped ramp? x α W Solution: Split W into components: y x α W cos ˛ acts ? to the incline W sin ˛ acts parallel to the incline    R FX : f  W sin ˛ D 0 FY : NR C NF  W cos ˛ D 0 0.68 NF α = 15° W = (1760X9.81) N α m f MR : 2m m 2.82 NR 2W cos ˛ C 0.68W sin ˛ C 2.82NF D 0 Solving: NR D 5930 N, NF D 10750 N Problem 5.23 The link AB exerts a force on the bucket of the excavator at A that is parallel to the link. The weight W = 1500 lb. Draw the free-body diagram of the bucket and determine the reactions at C. (The connection at C is equivalent to a pin support of the bucket.) 14 in 16 in B A 4 in C D W 8 in 8 in Solution: The free-body diagram is shown. The angle between the link AB and the horizontal is ˛ D tan1 12/14 D 40.6° The equilibrium equations are Fx : Cx C TAB cos ˛ D 0 Fy : Cy C TAB sin ˛  1500 lb D 0 MC : 1500 lb8 in  TAB cos ˛4 in  TAB sin ˛16 in D 0 Solving yields TAB D 892 lb, Cx D 677 lb, Cy D 919 lb c 2008 Pearson Education, Inc., Upper Saddle River, NJ. All rights reserved. This material is protected under all copyright laws as they  currently exist. No portion of this material may be reproduced, in any form or by any means, without permission in writing from the publisher. 281 Problem 5.24 The 14.5-lb chain saw is subjected to the loads at A by the log it cuts. Determine the reactions R, Bx , and By that must be applied by the person using the saw to hold it in equilibrium. Solution: The sum of the forces are   y FX D 5 C BX  R cos 60° D 0. FY D 10  14.5 C BY  R sin 60° D 0. R The sum of the moments about the origin is 60°  By 1.5 in 7 in x A Bx 5 lb MO D 7R cos 60° C 8BY  214.5  1310  51.5 D 0. From which 7R cos 60° C 8BY  166.5 D 0. Collecting equations and reducing to 3 equations in 3 unknowns: 14.5 lb 10 lb BX C 0BY  0.5R D 5 13 in 6 in 2 in 0BX C BY  0.866R D 4.5 0BX C 8BY C 3.5R D 166.5. Solving: BX D 11.257 lb, BY D 15.337 lb, and R D 12.514 lb Problem 5.25 The mass of the trailer is 2.2 Mg (megagrams). The distances a D 2.5 m and b D 5.5 m. The truck is stationary, and the wheels of the trailer can turn freely, which means that the road exerts no horizontal force on them. The hitch at B can be modeled as a pin support. (a) Draw the free-body diagram of the trailer. (b) Determine the total normal force exerted on the rear tires at A and the reactions exerted on the trailer at the pin support B. B Solution: (a) (b) The free body diagram is shown. The sum of forces:   FX D BX D 0. FY D FA  W C FB D 0. The sum of the moments about A:  MA D aW C a C bFB D 0, from which FB D W A 2.52.2 ð 103 9.81 aW D D 6.744 kN aCb 2.5 C 5.5 Substitute into the force equation: a b FA D W  FB D 14.838 kN B BX FB W A FA 282 a b c 2008 Pearson Education, Inc., Upper Saddle River, NJ. All rights reserved. This material is protected under all copyright laws as they  currently exist. No portion of this material may be reproduced, in any form or by any means, without permission in writing from the publisher. Problem 5.26 The total weight of the wheelbarrow and its load is W = 100 lb. (a) What is the magnitude of the upward force F necessary to lift the support at A off the ground? (b) What is the magnitude of the downward force necessary to raise the wheel off the ground? F W B A 40 in 12 in 14 in Solution: The free-body diagram is shown. The equilibrium equations are Fy : A C B C F  W D 0 MA : B26 in  W12 in  F40 in D 0 (a) Set A D 0 and solve. We find that F D 21.2 lb (b) Set B D 0 and solve. We find that F D 30 lb So we have a 21.2 lb, b 30 lb. c 2008 Pearson Education, Inc., Upper Saddle River, NJ. All rights reserved. This material is protected under all copyright laws as they  currently exist. No portion of this material may be reproduced, in any form or by any means, without permission in writing from the publisher. 283 Problem 5.27 The airplane’s weight is W D 2400 lb. Its brakes keep the rear wheels locked. The front (nose) wheel can turn freely, and so the ground exerts no horizontal force on it. The force T exerted by the airplane’s propeller is horizontal. (a) Draw the free-body diagram of the airplane. Determine the reaction exerted on the nose wheel and the total normal reaction on the rear wheels (b) when T D 0, (c) when T D 250 lb. T 4 ft W A 5 ft B 2 ft Solution: (a) The free body diagram is shown. (b) The sum of the forces:   FX D BX D 0 FY D AY  W C BY D 0. The sum of the moments about A is  MA D 5W C 7BY D 0, from which BY D 5W D 1714.3 lb 7 Substitute from the force balance equation: AY D W  BY D 685.7 lb (c) The sum of the forces:  FX D 250 C BX D 0, from which BX D 250 lb  FY D AY  W C BY D 0. The sum of the moments about A:  MA D 2504  5W C 7BY D 0, from which BY D 1571.4 lb. Substitute into the force balance equation to obtain: AY D 828.6 lb 4 ft AY 284 W A 5 ft B 2 ft BX BY c 2008 Pearson Education, Inc., Upper Saddle River, NJ. All rights reserved. This material is protected under all copyright laws as they  currently exist. No portion of this material may be reproduced, in any form or by any means, without permission in writing from the publisher. Problem 5.28 A safety engineer establishing limits on the load that can be carried by a forklift analyzes the situation shown. The dimensions are a = 32 in, b = 30 in, and c = 26 in. The combined weight of the forklift and operator is WF = 1200 lb. As the weight WL supported by the forklift increases, the normal force exerted on the floor by the rear wheels at B decreases. The forklift is on the verge of tipping forward when the normal force at B is zero. Determine the value of WL that will cause this condition. WL WF A a B b c Solution: The equilibrium equations and the special condition for this problem are Fy : A C B  WL  1200 lb D 0 MA : WL a  WF b C Bc D 0 BD0 We obtain WL D 1125 lb c 2008 Pearson Education, Inc., Upper Saddle River, NJ. All rights reserved. This material is protected under all copyright laws as they  currently exist. No portion of this material may be reproduced, in any form or by any means, without permission in writing from the publisher. 285 Problem 5.29 Paleontologists speculate that the stegosaur could stand on its hind limbs for short periods to feed. Based on the free-body diagram shown and assuming that m D 2000 kg, determine the magnitudes of the forces B and C exerted by the ligament–muscle brace and vertebral column, and determine the angle ˛. 580 mm 160 mm mg C B 22° Solution: Take the origin to be at the point of application of the force C. The position vectors of the points of application of the forces B and W are: α 415 mm 790 mm rB D 415i C 160j (mm), rW D 790i C 580j (mm). The forces are C D Ci cos90°  ˛ C j sin90°  ˛ D Ci sin ˛ C j cos ˛. B D Bi cos270°  22°  C j sin270°  22°  D B0.3746i  0.9272j. W D 29.81j D 19.62j (kN). The moments about C,   i   MC D  415   0.3746B   i   C  790   0  k   160 0  0.9272B 0  j  k   580 0 D 0  19.62 0  j D 444.72B  15499.8 D 0, from which BD 15499.8 D 34.85 kN. 444.72 The sums of the forces:  FX D C sin ˛  0.3746Bi D 0, from which C sin ˛ D 13.06 kN.  FY D C cos ˛  0.9272B  19.62j D 0, from which C cos ˛ D 51.93 kN. The angle ˛ is  ˛ D tan1 13.06 51.93  D 14.1° . The magnitude of C, CD 286 p 13.062 C 51.932 D 53.55 kN c 2008 Pearson Education, Inc., Upper Saddle River, NJ. All rights reserved. This material is protected under all copyright laws as they  currently exist. No portion of this material may be reproduced, in any form or by any means, without permission in writing from the publisher. Problem 5.30 The weight of the fan is W D 20 lb. Its base has four equally spaced legs of length b D 12 in. Each leg has a pad near the end that contacts the floor and supports the fan. The height h D 32 in. If the fan’s blade exerts a thrust T D 2 lb, what total normal force is exerted on the two legs at A? T b W h T A B Side View Top View Solution: The free-body diagram is shown. The equilibrium equations are Fy : A C B  W D 0 b 2b MB : W p  A p  Th D 0 2 2 We obtain A D 6.23 lb Problem 5.31 The weight of the fan is W D 20 lb. Its base has four equally spaced legs of length b D 12 in. Each leg has a pad near the end that contacts the floor and supports the fan. The height h D 32 in. As the thrust T of the fan increases, the normal force supported by the two legs at A decreases. When the normal force at A is zero, the fan is on the verge of tipping over. Determine the value of T that will cause this condition. T b W h T A B Side View Top View Solution: The free-body diagram is shown. The equilibrium equations are Fy : A C B  W D 0 2b b MB : W p  A p  Th D 0 2 2 We set A D 0 and solve to obtain T D 5.30 lb c 2008 Pearson Education, Inc., Upper Saddle River, NJ. All rights reserved. This material is protected under all copyright laws as they  currently exist. No portion of this material may be reproduced, in any form or by any means, without permission in writing from the publisher. 287 Problem 5.32 In a measure to decrease costs, the manufacturer of the fan described in Problem 5.31 proposes to support the fan with three equally spaced legs instead of four. An engineer is assigned to analyze the safety implications of the change. The weight of the fan decreases to W D 19.6 lb. The dimensions b and h are unchanged. What thrust T will cause the fan to be on the verge of tipping over in this case? Compare your answer to the answer to Problem 5.31. b T Solution: The free-body diagram is shown. The equilibrium equations are Fy : A C B  W D 0 MB : Wb cos 60°  Ab C b cos 60°   Th D 0 We set A D 0 and solve to obtain T D 3.68 lb This configuration is less stable than the one in Problem 5.31 using four legs. Problem 5.33 A force F D 400 N acts on the bracket. What are the reactions at A and B? F A 80 mm B 320 mm Solution: The joint A is a pinned joint; B is a roller joint. The pinned joint has two reaction forces AX , AY . The roller joint has one reaction force BX . The sum of the forces is   FX D AX C BX D 0, F AY AX 80 mm BX 320 mm FY D AY  F D 0, from which AY D F D 400 N. The sum of the moments about A is  MA D 0.08BX  0.320F D 0, from which BX D 0.320400 D 1600 N. 0.08 Substitute into the sum of forces equation to obtain: AX D BX D 1600 N 288 c 2008 Pearson Education, Inc., Upper Saddle River, NJ. All rights reserved. This material is protected under all copyright laws as they  currently exist. No portion of this material may be reproduced, in any form or by any means, without permission in writing from the publisher. Problem 5.34 The sign’s weight WS D 32 lb acts at the point shown. The 10-lb weight of bar AD acts at the midpoint of the bar. Determine the tension in the cable AE and the reactions at D. 11 in 30 in 11 in E 30⬚ 15⬚ A B C D Ws 33 in Solution: Treat the bar AD and sign as one single object. Let TAE be the tension in the cable. The equilibium equations are Fx : TAE cos 15° C Dx D 0 Fy : TAE sin 15° C Dy  Ws  10 lb D 0 MD : TAE cos 15° 52 in tan 30°  TAE sin 15° 52 in C 32 lb33 in C 10 lb26 in D 0 Solving yields TAE D 31.0 lb Dx D 29.9 lb Dy D 34.0 lb c 2008 Pearson Education, Inc., Upper Saddle River, NJ. All rights reserved. This material is protected under all copyright laws as they  currently exist. No portion of this material may be reproduced, in any form or by any means, without permission in writing from the publisher. 289 Problem 5.35 The device shown, called a swape or shadoof, helps a person lift a heavy load. (Devices of this kind were used in Egypt at least as early as 1550 B.C. and are still in use in various parts of the world.) The dimensions a D 3.6 m and b D 1.2 m. The mass of the bar and counterweight is 90 kg, and their weight W acts at the point shown. The mass of the load being lifted is 45 kg. Determine the vertical force the person must exert to support the stationary load (a) when the load is just above the ground (the position shown); (b) when the load is 1 m above the ground. Assume that the rope remains vertical. Solution:  MO : 441 N  F3.6 m cos   883 N1.2 m cos  D 0 Solving we find F D 147.2 N Notice that the angle  is not a part of this answer therefore (a) F D 147.2 N (b) F D 147.2 N a Oy b θ 25° Ox F W 883 N 441 N Problem 5.36 This structure, called a truss, has a pin support at A and a roller support at B and is loaded by two forces. Determine the reactions at the supports. Strategy: Draw a free-body diagram, treating the entire truss as a single object. 30° 2 kN 45° 4 kN Solution:  p MA : 4 kN 2b  2 kN cos 30° 3 b C 2 kN sin 30° b C B4 b D 0   Fx : Ax C 4 kN sin 45°  2 kN sin 30° D 0 Fy : Ay  4 kN cos 45°  2 kN cos 30° C B D 0 Solving: b Ax D 1.828 kN, Ay D 2.10 kN, B D 2.46 kN B A b b b b 45° 4 kN 30° 2 kN Ax Ay 290 B c 2008 Pearson Education, Inc., Upper Saddle River, NJ. All rights reserved. This material is protected under all copyright laws as they  currently exist. No portion of this material may be reproduced, in any form or by any means, without permission in writing from the publisher. Problem 5.37 An Olympic gymnast is stationary in the “iron cross” position. The weight of his left arm and the weight of his body not including his arms are shown. The distances are a D b D 9 in and c D 13 in. Treat his shoulder S as a fixed support, and determine the magnitudes of the reactions at his shoulder. That is, determine the force and couple his shoulder must support. S 8 lb 144 lb a b c Solution: The shoulder as a built-in joint has two-force and couple reactions. The left hand must support the weight of the left arm and half the weight of the body: FH D  FH 8 lb 144 C 8 D 80 lb. 2 The sum of the forces on the left arm is the weight of his left arm and the vertical reaction at the shoulder and hand:  FH 8 lb 144 lb FH SX M FX D SX D 0. 8 lb SY b FY D FH  SY  8 D 0, c from which SY D FH  8 D 72 lb. The sum of the moments about the shoulder is  MS D M C b C cFH  b8 D 0, where M is the couple reaction at the shoulder. Thus  M D b8  b C cFH D 1688 in lb D 1688 (in lb) 1 ft 12 in  D 140.67 ft lb c 2008 Pearson Education, Inc., Upper Saddle River, NJ. All rights reserved. This material is protected under all copyright laws as they  currently exist. No portion of this material may be reproduced, in any form or by any means, without permission in writing from the publisher. 291 Problem 5.38 Determine the reactions at A. A 5 ft 300 lb 800 ft-lb 200 lb 200 lb 6 ft 3 ft Solution: The built-in support at A is a two-force and couple reaction support. The sum of the forces for the system is  FX D AX C 200 D 0, from which AX D 200 lb  FY D AY C 300  200 D 0, from which AY D 100 lb The sum of the moments about A:  M D 6300 C 5200  800 C MA D 0, from which MA D 1600 ft lb which is the couple at A. AY MA 300 lb AX 5 ft 800 ft-lb 200 lb 200 lb 6 ft 292 3 ft c 2008 Pearson Education, Inc., Upper Saddle River, NJ. All rights reserved. This material is protected under all copyright laws as they  currently exist. No portion of this material may be reproduced, in any form or by any means, without permission in writing from the publisher. Problem 5.39 The car’s brakes keep the rear wheels locked, and the front wheels are free to turn. Determine the forces exerted on the front and rear wheels by the road when the car is parked (a) on an up slope with ˛ D 15° ; (b) on a down slope with ˛ D 15° . n 70 i n 36 i n 20 i y α 3300 lb Solution: The rear wheels are two force reaction support, and the front wheels are a one force reaction support. Denote the rear wheels by A and the front wheels by B, and define the reactions as being parallel to and normal to the road. The sum of forces:  AX D 854.1 lb.  20 in. α FX D AX  3300 sin 15° D 0, from which x BY 3300 lb AX AY 36 in. 70 in. FY D AY  3300 cos 15° C BY D 0. Since the mass center of the vehicle is displaced above the point A, a component of the weight (20W sin ˛) produces a positive moment about A, whereas the other component (36W cos ˛) produces a negative moment about A. The sum of the moments about A:  MA D 363300 cos 15°  C 203300 sin 15°  C BY 106 D 0, from which BY D C97669 D 921.4 lb. 106 Substitute into the sum of forces equation to obtain AY D 2266.1 lb (b) For the car parked down-slope the sum of the forces is  FX D AX C 3300 sin 15° D 0, from which AX D 854 lb  FY D AY  3300 cos 15° C BY D 0. The component (20W sin ˛) now produces a negative moment about A. The sum of the moments about A is  MA D 330036 cos 15°  330020 sin 15° C 106BY D 0, from which BY D 131834 D 1243.7 lb. 106 Substitute into the sum of forces equation to obtain AY D 1943.8 lb c 2008 Pearson Education, Inc., Upper Saddle River, NJ. All rights reserved. This material is protected under all copyright laws as they  currently exist. No portion of this material may be reproduced, in any form or by any means, without permission in writing from the publisher. 293 Problem 5.40 The length of the bar is L D 4 ft. Its weight W D 6 lb acts at the midpoint of the bar. The floor and wall are smooth. The spring is unstretched when the angle ˛ = 0. If the bar is in equilibrium when ˛ D 40° , what is the spring constant k? k α L Solution: The free-body diagram is shown. The stretch in the spring is L  L cos ˛, so the upward force exerted on the bar by the spring is F D kL 1  cos ˛. Let N and R be the normal forces exerted by the floor and the wall, respectively. The equilibrium equations for the bar are Fx : R D 0 Fy : F C N  W D 0 Mbottom : W L sin ˛  RL cos ˛ 2  FL sin ˛ D 0 Because R D 0, the moment equation can be solved for the force exerted by the spring. F D 0.5W D 3 lb D kL 1  cos ˛ Solving yields k D 3.21 lb/ft 294 c 2008 Pearson Education, Inc., Upper Saddle River, NJ. All rights reserved. This material is protected under all copyright laws as they  currently exist. No portion of this material may be reproduced, in any form or by any means, without permission in writing from the publisher. Problem 5.41 The weight W of the bar acts at its midpoint. The floor and wall are smooth. The spring is unstretched when the angle ˛ D 0. Determine the angle ˛ at which the bar is in equilibrium in terms of W, k, and L. k α L Solution: The free-body diagram is shown. The stretch in the spring is L  L cos ˛, so the upward force exerted on the bar by the spring is F D kL1  cos ˛. Let N and R be the normal forces exerted by the floor and the wall, respectively. The equilibrium equations for the bar are Fx : R D 0 Fy : F C N  W D 0 Mbottom : W L sin ˛  RL cos ˛ 2  FL sin ˛ D 0 Because R D 0, the moment equation can be solved for the force exerted by the spring. FD W D kL1  cos ˛ 2   W Solving yields ˛ D cos1 1  2L c 2008 Pearson Education, Inc., Upper Saddle River, NJ. All rights reserved. This material is protected under all copyright laws as they  currently exist. No portion of this material may be reproduced, in any form or by any means, without permission in writing from the publisher. 295 Problem 5.42 The plate is supported by a pin in a smooth slot at B. What are the reactions at the supports? 2 kN-m 6 kN-m A B 60° 2m Solution: The pinned support is a two force reaction support. The smooth pin is a roller support, with a one force reaction. The reaction at B forms an angle of 90° C 60° D 150° with the positive x axis. The sum of the forces:   6 kN-m 2 kN-m FX D AX C B cos 150° D 0 FY D AY C B sin 150° D 0 A 60° B The sum of the moments about B is  2m MB D 2AY C 2  6 D 0, from which AY D  4 D 2 kN. 2 2 kN-m 6 kN-m Substitute into the force equations to obtain BD AY D 4 kN, sin 150° and AX D B cos 150° D 3.464 kN. AX 150° AY B 2m The horizontal and vertical reactions at B are BX D 4 cos 150° D 3.464 kN, and BY D 4 sin 150° D 2 kN. 296 c 2008 Pearson Education, Inc., Upper Saddle River, NJ. All rights reserved. This material is protected under all copyright laws as they  currently exist. No portion of this material may be reproduced, in any form or by any means, without permission in writing from the publisher. Problem 5.43 Determine the reactions at the fixed support A. Solution: The free-body diagram is shown. The equilibrium equations are y 30 lb 40 lb 150 ft-lb 45⬚ A x Fx : Ax C 40 lb cos 45° D 0 3 ft 3 ft 6 ft Fy : Ay C 30 lb C 40 lb sin 45° D 0 MA : MA C 30 lb3 ft C 40 lb sin 45° 6 ft C 150 ft-lb D 0 Solving yields Ax D 28.3 lb, Ay D 58.3 lb, MA D 410 ft-lb. Problem 5.44 Suppose the you want to represent the two forces and couple acting on the beam in Problem 5.43 by an equivalent force F as shown. (a) Determine F and the distance D at which its line of action crosses the x axis. (b) Assume that F is the only load acting on the beam and determine the reactions at the fixed support A. Compare your answers to answers to Problem 5.43. y F A x D Solution: The free-body diagram is shown. (a) To be equivalnet, F must equal the sum of the two forces: F D 30 lbj C 40 lbcos 45° i C sin 45° j F D 28.3i C 58.3j lb The force F must be placed so that the moment about a point due to F is equal to the moment about the same point due to the two forces and couple. Evaluating the moments about the origin, 58.3 lbD D 30 lb3 ft C 40 lb sin 45° 6 ft C 150 ft-lb The distance D D 7.03 ft (b) The equilibrium equations are Fx : Ax C 28.3 lb D 0 Fy : Ay C 58.3 lb D 0 MA : MA C 58.3 lb7.03 ft D 0 Solving yields Ax D 28.3 lb, Ay D 58.3 lb, MA D 410 ft-lb. c 2008 Pearson Education, Inc., Upper Saddle River, NJ. All rights reserved. This material is protected under all copyright laws as they  currently exist. No portion of this material may be reproduced, in any form or by any means, without permission in writing from the publisher. 297 Problem 5.45 The bicycle brake on the right is pinned to the bicycle’s frame at A. Determine the force exerted by the brake pad on the wheel rim at B in terms of the cable tension T. T 35° 40 mm B Brake pad Wheel rim 45 mm A 40 mm Solution: From the force balance equation for the cables: the force TB on the brake mechanism TB in terms of the cable tension T is 35° T  2TB sin 35° D 0, from which TB D T D 0.8717T. 2 sin 35° 40 mm B Take the origin of the system to be at A. The position vector of the point of attachment of B is rB D 45j (mm). The position vector of the point of attachment of the cable is rC D 40i C 85j (mm). The force exerted by the brake pad is B D Bi. The force vector due the cable tension is AY 45 mm AX 40 mm TB D TB i cos 145° C j sin 145°  D TB 0.8192i C 0.5736j. The moment about A is MA D rB ð B C rC ð TB D 0   i   MA D  0   B j 45 0   i k     45  C  40   0   0.8192  k   85 85  TB D 0  0.5736 0  j MA D 45B C 92.576TB k D 0, from which B D 92.576TB D 2.057TB . 45 Substitute the expression for the cable tension: B D 2.0570.8717T D 1.793T 298 c 2008 Pearson Education, Inc., Upper Saddle River, NJ. All rights reserved. This material is protected under all copyright laws as they  currently exist. No portion of this material may be reproduced, in any form or by any means, without permission in writing from the publisher. Problem 5.46 The mass of each of the suspended weights is 80 kg. Determine the reactions at the supports at A and E. A B C 300 mm D E 200 mm Solution: From the free body diagram, the equations of equilibrium for the rigid body are   and  Fx D AX C EX D 0, 200 mm y AY 0.2 m 0.2 m AX A x mg 0.3 m Fy D AY  2809.81 D 0, mg E EX MA D 0.3EX  0.2809.81  0.4809.81 D 0. We have three equations in the three components of the support reactions. Solving for the unknowns, we get the values AX D 1570 N, AY D 1570 N, and EX D 1570 N. Problem 5.47 The suspended weights in Problem 5.46 are each of mass m. The supports at A and E will each safely support a force of 6 kN magnitude. Based on this criterion, what is the largest safe value of m? Solution: Written with the mass value of 80 kg replaced by the symbol m, the equations of equilibrium from Problem 5.46 are   and  Fx D AX C EX D 0, Fy D AY  2 m9.81 D 0, MA D 0.3EX  0.2 m9.81  0.4 m9.81 D 0. We also need the relation  jAj D A2X C A2Y D 6000 N. We have four equations in the three components of the support reactions plus the magnitude of A. This is four equations in four unknowns. Solving for the unknowns, we get the values AX D 4243 N, AY D 4243 N, EX D 4243 N, and m D 216.5 kg. Note: We could have gotten this result by a linear scaling of all of the numbers in Problem 5.46. c 2008 Pearson Education, Inc., Upper Saddle River, NJ. All rights reserved. This material is protected under all copyright laws as they  currently exist. No portion of this material may be reproduced, in any form or by any means, without permission in writing from the publisher. 299 Problem 5.48 The tension in cable BC is 100 lb. Determine the reactions at the built-in support. C 6 ft A B 300 ft-lb 200 lb 3 ft 3 ft 6 ft Solution: The cable does not exert an external force on the system, and can be ignored in determining reactions. The built-in support is a two-force and couple reaction support. The sum of forces:   FX D AX D 0. MA AY 300 ft-lb AX 200 lb 3 ft FY D AY  200 D 0, from which AY D 200 lb. The sum of the moments about A is  M D MA  3200  300 D 0, from which MA D 900 ft lb Problem 5.49 The tension in cable AB is 2 kN. What are the reactions at C in the two cases? 60° A B 2m C A 1m 2m (a) Solution: First Case: The sum of the forces:  FX D CX  T cos 60° D 0, from which CX D 20.5 D 1 kN  T T C 1m (b) CY 2m 1m Case (a) MC CX CY from which CY D 1.8662 D 3.732 kN.  B 60° FY D CY C T sin 60° C T D 0, The sum of the moments about C is 60° Case (b) MC CX M D MC  T sin 60°  3T D 0, from which MC D 3.8662 D 7.732 kN Second Case: The weight of the beam is ignored, hence there are no external forces on the beam, and the reactions at C are zero. 300 c 2008 Pearson Education, Inc., Upper Saddle River, NJ. All rights reserved. This material is protected under all copyright laws as they  currently exist. No portion of this material may be reproduced, in any form or by any means, without permission in writing from the publisher. Problem 5.50 Determine the reactions at the supports. 6 in 5 in 50 lb A 3 in 100 in-lb Solution: The reaction at A is a two-force reaction. The reaction 3 in at B is one-force, normal to the surface. B The sum of the forces:   30° FX D AX  B cos 60°  50 D 0. AX FY D AY C B sin 60° D 0. 50 lb AY The sum of the moments about A is  6 in. 100 MA D 100 C 11B sin 60°  6B cos 60° D 0, B 11 in. from which 60° 100 D 15.3 lb. BD 11 sin 60°  6 cos 60°  Substitute into the force equations to obtain AY D B sin 60° D 13.3 lb and AX D B cos 60° C 50 D 57.7 lb Problem 5.51 The weight W D 2 kN. Determine the tension in the cable and the reactions at A. 30° A W 0.6 m AY Solution: Equilibrium Eqns:   C  FX D 0: 0.6 m T AX C T cos 30° D 0 AX FY D 0: AY C T C T sin 30° MA D 0: 0, 6W C 0.6T sin 30°  WD0 T 30° 0.6 m 0.6 m W = 2 kN = 2000 N C 1, 2T D 0 Solving, we get AX D 693 N, AY D 800 N, T D 800 N c 2008 Pearson Education, Inc., Upper Saddle River, NJ. All rights reserved. This material is protected under all copyright laws as they  currently exist. No portion of this material may be reproduced, in any form or by any means, without permission in writing from the publisher. 301 Problem 5.52 The cable shown in Problem 5.51 will safely support a tension of 6 kN. Based on this criterion, what is the largest safe value of the weight W? Solution: The equilibrium equations in the solution of problem are   C  FX D 0: AX C T cos 30° D 0 FY D 0: AY C T C T sin 30°  W D 0 MA D 0: 0, 6W C 0, 6T sin 30°  C 1, 2T D 0 We previously had 3 equations in the 3 unknowns AX , AY and T (we knew W). In the current problem, we know T but don’t know W. We again have three equations in three unknowns (AX , AY , and W). Setting T D 6 kN, we solve to get AX D 5.2 kN AY D 6.0 kN W D 15.0 kN Problem 5.53 The blocks being compressed by the clamp exert a 200-N force on the pin at D that points from A toward D. The threaded shaft BE exerts a force on the pin at E that points from B toward E. 50 mm (a) 50 mm Draw a free-body diagram of the arm DCE of the clamp, assuming that the pin at C behaves like a pin support. (b) Determine the reactions at C.  E A C 50 mm D FBE (a) The free-body diagram (b) The equilibrium equations  125 mm B Solution:  125 mm 125 mm Cy MC : 200 N0.25 m  FBE 0.1 m D 0 200 N Fx : Cx C FBE D 0 Cx Fy : Cy  200 N D 0 Solving Cx D 500 N, Cy D 200 N 302 c 2008 Pearson Education, Inc., Upper Saddle River, NJ. All rights reserved. This material is protected under all copyright laws as they  currently exist. No portion of this material may be reproduced, in any form or by any means, without permission in writing from the publisher. Problem 5.54 Consider the clamp in Problem 5.53. The blocks being compressed by the clamp exert a 200N force on the pin at A that points from D toward A. The threaded shaft BE exerts a force on the pin at B that points from E toward B. (a) Draw a free-body diagram of the arm ABC of the clamp, assuming that the pin at C behaves like a pin support. Determine the reactions at C. (b) FBE Solution: (a) (b) The free-body diagram The equilibrium equations    Cx MC : 200 N0.25 m C FBE 0.1 m D 0 Fx : FBE C Cx D 0 Cy 200 N Fy : 200 N C Cy D 0 Solving we find Cx D 500 N, Cy D 200 N Problem 5.55 Suppose that you want to design the safety valve to open when the difference between the pressure p in the circular pipe diameter D 150 mm and the atmospheric pressure is 10 MPa (megapascals; a pascal is 1 N/m2 ). The spring is compressed 20 mm when the valve is closed. What should the value of the spring constant be? 250 mm 150 mm k A p 150 mm Solution: The area of the valve is  aD 0.15 2 150 mm 2 250 mm D 17.671 ð 103 m2 . k The force at opening is A F D 10a ð 106 D 1.7671 ð 105 N. 150 mm The force on the spring is found from the sum of the moments about A,  MA D 0.15F  0.4kL D 0. k∆L A Solving, F 0.15F 0.151.7671 ð 105  kD D 0.4L 0.40.02 D 3.313 ð 106 0.15 m 0.25 m N m c 2008 Pearson Education, Inc., Upper Saddle River, NJ. All rights reserved. This material is protected under all copyright laws as they  currently exist. No portion of this material may be reproduced, in any form or by any means, without permission in writing from the publisher. 303 Problem 5.56 The 10-lb weight of the bar AB acts at the midpoint of the bar. The length of the bar is 3 ft. Determine the tension in the string BC and the reactions at A. C B 3 ft A 30⬚ 1 ft Solution: Geometry: tan  D 3 ft  3 ft sin 30° D 0.4169 )  D 22.63° 1 ft C 3 ft cos 30° The equilibrium equations  MA : TBC cos 3 ft sin 30°  C TBC sin 3 ft cos 30°   10 lb1.5 ft cos 30°  D 0   Fx : TBC cos  C Ax D 0 Fy : TBC sin   10 lb C Ay D 0 Solving: Ax D 5.03 lb, Ay D 7.90 lb, T D 5.45 lb TBC θ 10 lb Ax Ay 304 c 2008 Pearson Education, Inc., Upper Saddle River, NJ. All rights reserved. This material is protected under all copyright laws as they  currently exist. No portion of this material may be reproduced, in any form or by any means, without permission in writing from the publisher. Problem 5.57 The crane’s arm has a pin support at A. The hydraulic cylinder BC exerts a force on the arm at C in the direction parallel to BC. The crane’s arm has a mass of 200 kg, and its weight can be assumed to act at a point 2 m to the right of A. If the mass of the suspended box is 800 kg and the system is in equilibrium, what is the magnitude of the force exerted by the hydraulic cylinder? C A 2.4 m 1m B 1.8 m 1.2 m 7m Solution: The free-body  diagram of the arm is shown.  2.4 D 63.4° 1.2 The equilibrium equations are The angle  D tan1 Fx : Ax C FH cos  D 0 Fy : Ay C FH sin   200 kg9.81 m/s2   800 kg9.81 m/s2  D 0 MA : FH sin 3 m  FH cos 1.4 m  200 kg9.81 m/s2 2 m  800 kg9.81 m/s2 7 m D 0 We obtain Ax D 12.8 kN, Ay D 15.8 kN, FH D 28.6 kN Thus FH D 28.6 kN Problem 5.58 In Problem 5.57, what is the magnitude of the force exerted on the crane’s arm by the pin support at A? Solution: See the solution to Problem 5.57. Ax D 12.8 kN, Ay D 15.8 kN, FH D 28.6 kN jAj D  12.8 kN2 C 15.8 kN2 D 20.3 kN C A 2.4 m 1m B Thus jAj D 20.3 kN 1.8 m 1.2 m 7m c 2008 Pearson Education, Inc., Upper Saddle River, NJ. All rights reserved. This material is protected under all copyright laws as they  currently exist. No portion of this material may be reproduced, in any form or by any means, without permission in writing from the publisher. 305 Problem 5.59 A speaker system is suspended by the cables attached at D and E. The mass of the speaker system is 130 kg, and its weight acts at G. Determine the tensions in the cables and the reactions at A and C. 0.5 m 0.5 m 0.5 m 0.5 m 1m E C A 1m B D G Solution: The weight of the speaker is W D mg D 1275 N. The 1m AY equations of equilibrium for the entire assembly are   1.5 m CY E A Fx D CX D 0, Fy D AY C CY  mg D 0 C CX B D (where the mass m D 130 kg), and  mg MC D 1AY  1.5mg D 0. Solving these equations, we get 1.5 m CX D 0, 1m T2 CY D 3188 N, T1 and AY D 1913 N. From the free body diagram of the speaker alone, we get  and  Mleft mg Fy D T1 C T2  mg D 0, support D 1mg C 1.5T2 D 0. Solving these equations, we get T1 D 425. N and T2 D 850 N 306 c 2008 Pearson Education, Inc., Upper Saddle River, NJ. All rights reserved. This material is protected under all copyright laws as they  currently exist. No portion of this material may be reproduced, in any form or by any means, without permission in writing from the publisher. Problem 5.60 The weight W1 D 1000 lb. Neglect the weight of the bar AB. The cable goes over a pulley at C. Determine the weight W2 and the reactions at the pin support A. B 50° 35° W1 A C W2 Solution: The strategy is to resolve the tensions at the end of bar AB into x- and y-components, and then set the moment about A to zero. The angle between the cable and the positive x axis is 35° . The tension vector in the cable is T2 D W2 i cos35°  C j sin35° . 35° T2 rB T1 50° AY D W2 0.8192i  0.5736jlb. AX Assume a unit length for the bar. The angle between the bar and the positive x axis is 180°  50° D 130° . The position vector of the tip of the bar relative to A is rB D i cos130°  C j sin130° , D 0.6428i C 0.7660j. The tension exerted by W1 is T1 D 1000j. The sum of the moments about A is:  MA D rB ð T1  C rB ð T2  D rB ð T1 C T2    i  D L  0.6428  0.8191W2    j   0.7660  0.5736W2  1000  MA D 0.2587W2 C 642.8k D 0, from which W2 D 2483.5 lb The sum of the forces:  FX D AX C W2 0.8192i D 0, from which AX D 2034.4 lb  FY D AY  W2 0.5736  1000j D 0, from which AY D 2424.5 lb c 2008 Pearson Education, Inc., Upper Saddle River, NJ. All rights reserved. This material is protected under all copyright laws as they  currently exist. No portion of this material may be reproduced, in any form or by any means, without permission in writing from the publisher. 307 Problem 5.61 The dimensions a D 2 m and b D 1 m. The couple M D 2400 N-m. The spring constant is k D 6000 N/m, and the spring would be unstretched if h D 0. The system is in equilibrium when h D 2 m and the beam is horizontal. Determine the force F and the reactions at A. k h A M F a Solution: We need to know the unstretched length of the spring, l0 b Unstretched l0 D a C b D 3 m (a + b) We also need the stretched length l2 D h2 C a C b2 AY θ M l D 3.61 m AX a FS D kl  l0  tan  D F b h a C b  D 33.69° Equilibrium eqns:   C  FX : AX  FS cos  D 0 FY : AY C FS sin   F D 0 MA :  M  aF C a C bFS sin  D 0 a D 2 m, b D 1 m, M D 2400 N-m, h D 2 m, k D 6000 N/m. Substituting in and solving, we get FS D 6000l  l0  D 3633 N and the equilibrium equations yield AX D 3023 N AY D 192 N F D 1823 N 308 c 2008 Pearson Education, Inc., Upper Saddle River, NJ. All rights reserved. This material is protected under all copyright laws as they  currently exist. No portion of this material may be reproduced, in any form or by any means, without permission in writing from the publisher. Problem 5.62 The bar is 1 m long, and its weight W acts at its midpoint. The distance b D 0.75 m, and the angle ˛ D 30° . The spring constant is k D 100 N/m, and the spring is unstretched when the bar is vertical. Determine W and the reactions at A. k α W A b p Solution: The unstretched length of the spring is L D b2 C 12 D 1.25 m. The obtuse angle is 90 C ˛, so the stretched length can be determined from the cosine law: L22 D 12 C 0.752  20.75 cos90 C ˛ D 2.3125 β m2 α W from which L2 D 1.5207 m. The force exerted by the spring is A T D kL D 1001.5207  1.25 D 27.1 N. b The angle between the spring and the bar can be determined from the sine law: 1.5207 b D , sin ˇ sin90 C ˛ T β from which sin ˇ D 0.4271, α ˇ D 25.28° . W The angle the spring makes with the horizontal is 180  25.28  90  ˛ D 34.72° . The sum of the forces:  AX FX D AX  T cos 34.72° D 0, AY from which AX D 22.25 N.  FY D AY  W  T sin 34.72° D 0. The sum of the moments about A is   MA D T sin 25.28°  W 2  sin ˛ D 0, from which WD 2T sin 25.28° D 46.25 N. sin ˛ Substitute into the force equation to obtain: AY D W C T sin 34.72° D 61.66 N c 2008 Pearson Education, Inc., Upper Saddle River, NJ. All rights reserved. This material is protected under all copyright laws as they  currently exist. No portion of this material may be reproduced, in any form or by any means, without permission in writing from the publisher. 309 B Problem 5.63 The boom derrick supports a suspended 15-kip load. The booms BC and DE are each 20 ft long. The distances are a D 15 ft and b D 2 ft, and the angle  D 30° . Determine the tension in cable AB and the reactions at the pin supports C and D. E θ Solution: Choose a coordinate system with origin at point C, with the y axis parallel to CB. The position vectors of the labeled points are: C A D rD D 2i a b rE D rD C 20i sin 30° C j cos 30°  D 12i C 17.3j, The components: rB D 20j, Dx D 0.6jDj D 7.67 kip, rA D 15i. The unit vectors are: eDE D rE  r D D 0.5i C 0.866j, jrE  rD j eEB D rB  r E D 0.976i C 0.2179j. jrB  rE j eCB D rB  r C D 1j, jrB  rC j eAB D rA  r B D 0.6i  0.8j. jrA  rB j Dy D 0.866jDj D 13.287 kip, and Cy D 1jCj D 11.94 kip B E θ A C  Fx D 0.5jDj  0.976jTEB j D 0, TAB Fy D 0.866jDj C 0.2179jTEB j  15 D 0, from which b a Isolate the juncture at E: The equilibrium conditions are  D TEB TEB 15 kip C D Juncture B Juncture E jDj D 15.34 kip and jTEB j D 7.86 kip. Isolate the juncture at B: The equilibrium conditions are:  and  Fx D 0jCj  0.6jTAB j C 0.976jTEB j, Fy D 1jCj  0.6jTAB j  0.2179jTEB j D 0, from which jTAB j D 12.79 kip, and jCj D 11.94 kip. 310 c 2008 Pearson Education, Inc., Upper Saddle River, NJ. All rights reserved. This material is protected under all copyright laws as they  currently exist. No portion of this material may be reproduced, in any form or by any means, without permission in writing from the publisher. Problem 5.64 The arrangement shown controls the elevators of an airplane. (The elevators are the horizontal control surfaces in the airplane’s tail.) The elevators are attached to member EDG. Aerodynamic pressures on the elevators exert a clockwise couple of 120 in-lb. Cable BG is slack, and its tension can be neglected. Determine the force F and the reactions at pin support A. A D 2.5 in C F 3.5 in Elevator E B 6 in 120 in-lb G 2 in 2.5 in 2.5 in 1.5 in 120 in (Not to scale) Solution: Begin at the elevator. The moment arms at E and G are 6 in. The angle of the cable EC with the horizontal is ˛ D tan1 12 D 5.734° . 119.5 Denote the horizontal and vertical components of the force on point E by FX and FY . The sum of the moments about the pinned support on the member EG is  2 in FX α C A TEC 2.5 in F 3.5 in E FY D TEC α C 6 in 120 in-lb 2.5 in MEG D 2.5FY C 6FX  120 D 0. This is the tension in the cable EC. Noting that FX D TEC cos ˛, and FY D TEC sin ˛, then TEC D 120 . 2.5 sin ˛ C 6 cos ˛ The sum of the moments about the pinned support BC is  MBC D 2TEC sin ˛ C 6TEC cos ˛  2.5F D 0. Substituting:  FD 120 2.5  6 cos ˛  2 sin ˛ 6 cos ˛ C 2.5 sin ˛  D 480.9277 D 44.53 lb. The sum of the forces about the pinned joint A:  Fx D Ax  F C TEC cos ˛ D 0 from which Ax D 25.33 lb,  Fy D Ay C TEC sin ˛ D 0 from which Ay D 1.93 lb c 2008 Pearson Education, Inc., Upper Saddle River, NJ. All rights reserved. This material is protected under all copyright laws as they  currently exist. No portion of this material may be reproduced, in any form or by any means, without permission in writing from the publisher. 311 Problem 5.65 In Example 5.4 suppose that ˛ D 40° , d D 1 m, a D 200 mm, b D 500 mm, R D 75 mm, and the mass of the luggage is 40 kg. Determine F and N. Solution: (See Example 5.4.) The sum of the moments about the center of the wheel:  MC D dF cos ˛ C aW sin ˛  bW cos ˛ D 0, from which F D b  a tan ˛W D 130.35 N. d The sum of the forces:  FY D N  W C F D 0, F from which N D 262.1 N d A d b h a a W α R C W b F R h a C N Problem 5.66 In Example 5.4 suppose that ˛ D 35° , d D 46 in, a D 10 in, b D 14 in, R D 3 in, and you don’t want the user to have to exert a force F larger than 20 lb. What is the largest luggage weight that can be placed on the carrier? N Solution: (See Example 5.4.) From the solution to Problem 5.65, the force is FD b  a tan ˛W . d Solve for W: WD Fd . b  a tan ˛ For F D 20 lb, W D 131.47 D 131.5 lb Problem 5.67 One of the difficulties in making design decisions is that you don’t know how the user will place the luggage on the carrier in Example 5.4. Suppose you assume that the point where the weight acts may be anywhere within the “envelope” R  a  0.75c and 0  b  0.75d. If ˛ D 30° , c D 14 in, d D 48 in, R D 3 in, and W D 80 lb, what is the largest force F the user will have to exert for any luggage placement? Solution: (See Example 5.4.) From the solution to Problem 5.65, the force is FD b  a tan ˛W . d The force is maximized as b ! 0.75d, and a ! R. Thus FMAX D 312 0.75d  R tan ˛W D 57.11 lb d c 2008 Pearson Education, Inc., Upper Saddle River, NJ. All rights reserved. This material is protected under all copyright laws as they  currently exist. No portion of this material may be reproduced, in any form or by any means, without permission in writing from the publisher. Problem 5.68 In our design of the luggage carrier in Example 5.4, we assumed a user that would hold the carrier’s handle at h D 36 in above the floor. We assumed that R D 3 in, a D 6 in, and b D 12 in, and we chose the dimension d D 4 ft. The resulting ratio of the force the user must exert to the weight of the luggage is F/W D 0.132. Suppose that people with a range of heights use this carrier. Obtain a graph of F/W as a function of h for 24  h  36 in. Solution: (See Example 5.4.) From the solution to Problem 5.67, the force that must be exerted is FD b  a tan ˛W , d b  a tan ˛ F D . W d from which The angle a is given by  ˛ D sin1 hR d  . F / W , d i m e n s i o n l e x e F/W versus height .2 .19 .18 .17 .16 .15 .14 .13 24 26 28 30 32 height h, in 34 36 The commercial package TK Solver Plus was used to plot a graph of F as a function of h. W Problem 5.69 (a) Draw the free-body diagram of the beam and show that it is statically indeterminate. (See Active Example 5.5.) (b) Determine as many of the reactions as possible. 20 N-m A 800 mm Solution: (a) The free body diagram shows that there are four unknowns, whereas only three equilibrium equations can be written. (b) The sum of moments about A is  MA D M C 1.1BY D 0, from which BY D  20 D 18.18 N. 1.1 The sum of forces in the vertical direction is  300 mm 20 N-m A 800 mm B B 300 mm BX AX AY 800 mm 300 mm BY FY D AY C BY D 0, from which AY D BY D 18.18 N. The sum of forces in the horizontal direction is  FX D AX C BX D 0, from which the values of AX and BX are indeterminate. c 2008 Pearson Education, Inc., Upper Saddle River, NJ. All rights reserved. This material is protected under all copyright laws as they  currently exist. No portion of this material may be reproduced, in any form or by any means, without permission in writing from the publisher. 313 Problem 5.70 Consider the beam in Problem 5.69. Choose supports at A and B so that it is not statically indeterminate. Determine the reactions at the supports. Solution: One possibility is shown: the pinned support at B is replaced by a roller support. The equilibrium conditions are:  20 N-m A B FX D AX D 0. 800 mm The sum of moments about A is  AX MA D M C 1.1BY D 0, from which BY D  300 mm 20 N-m AY 800 mm 300 mm BY 20 D 18.18 N. 1.1 The sum of forces in the vertical direction is  FY D AY C BY D 0, from which AY D BY D 18.18 N. Problem 5.71 (a) Draw the free-body diagram of the beam and show that it is statically indeterminate. (The external couple M0 is known.) M0 A B (b) By an analysis of the beam’s deflection, it is determined that the vertical reaction B exerted by the roller support is related to the couple M0 by B D 2M0 /L. What are the reactions at A? Solution:  (a)  C  L Eqn (3) and Eqn (4) yield FX : AX D 0 (1) MA D MO  2MO FY : AY C B D 0 (2) MA D MO MA : MA  MO C BL D 0 (3) MA was assumed counterclockwise Unknowns: MA , AX , AY , B. 3 Eqns in 4 unknowns MA D jMO j clockwise AX D 0 AY D 2MO /L ∴ Statistically indeterminate (b) Given B D 2MO /L (4) We now have 4 eqns in 4 unknowns and can solve. Eqn (1) yields AX D 0 AY MO MA AX L B Eqn (2) and Eqn (4) yield AY D 2MO /L 314 c 2008 Pearson Education, Inc., Upper Saddle River, NJ. All rights reserved. This material is protected under all copyright laws as they  currently exist. No portion of this material may be reproduced, in any form or by any means, without permission in writing from the publisher. Problem 5.72 Consider the beam in Problem 5.71. Choose supports at A and B so that it is not statically indeterminate. Determine the reactions at the supports. Solution: This result is not unique. There are several possible answers  FX :   A MO L AX D 0 FY : AY C BY D 0 MA :  Mo C BL D 0 O MO AX AX D 0 L B AY B D MO /L AY D MO /L Problem 5.73 Draw the free-body diagram of the L-shaped pipe assembly and show that it is statically indeterminate. Determine as many of the reactions as possible. B 80 N A Strategy: Place the coordinate system so that the x axis passes through points A and B. 300 mm Solution: The free body diagram shows that there are four reactions, hence the system is statically indeterminate. The sum of the forces:  and  FX D AX C BX  D 0, FY D AY C BY C F D 0. 300 mm 100 N-m 700 mm The moment about the point A is MA D LBN  0.3F C M D 0, from which BN D 76 M C 0.3F D D 99.79 N, L 0.76157 from which A strategy for solving some statically indeterminate problems is to select a coordinate system such that the indeterminate reactions vanish from the sum of the moment equations. The choice here is to locate the x axis on a line passing through both A and B, with the origin at A. Denote the reactions at A and B by AN , AP , BN , and BP , where the subscripts indicate the reactions are normal to and parallel to the new x axis. Denote The sum of the forces normal to the new axis is F D 80 N, The reactions parallel to the new axis are indeterminate.  FN D AN C BN C F cos  D 0, from which AN D BN  F cos  D 26.26 lb M D 100 N-m. The length from A to B is LD p 0.32 C 0.72 D 0.76157 m. The angle between the new axis and the horizontal is   D tan1 0.3 0.7 BN 80 N AN  D 23.2° . 300 mm BP 300 mm AP 100 N-m 700 mm c 2008 Pearson Education, Inc., Upper Saddle River, NJ. All rights reserved. This material is protected under all copyright laws as they  currently exist. No portion of this material may be reproduced, in any form or by any means, without permission in writing from the publisher. 315 Problem 5.74 Consider the pipe assembly in Problem 5.73. Choose supports at A and B so that it is not statically indeterminate. Determine the reactions at the supports. Solution: This problem has no unique solution. Problem 5.75 State whether each of the L-shaped bars shown is properly or improperly supported. If a bar is properly supported, determine the reactions at its supports. (See Active Example 5.6.) F C –12 L F L –1 L 2 Solution: (1)  is properly constrained. The sum of the forces FX D F C BX D 0, A B L (1) (2) FY D BY C Ay D 0, –12 L F –12 L A MB D LAY C LF D 0, B 45° from which AY D F, and By D F (2) is improperly constrained. The reactions intersect at B, while the force produces a moment about B. (3) is properly constrained. The forces are neither concurrent nor parallel. The sum of the forces:   45° C 45° from which By D Ay . The sum of the moments about B:  B L from which BX D F.  A L (3) FX D C cos 45°  B  A cos 45° C F D 0. FY D C sin 45°  A sin 45° D 0 from which A D C. The sum of the moments about A:  MA D  21 LF C LC cos 45° C LC sin 45° D 0, F F from which C D p . Substituting and combining: A D p , 2 2 2 2 F BD 2 316 c 2008 Pearson Education, Inc., Upper Saddle River, NJ. All rights reserved. This material is protected under all copyright laws as they  currently exist. No portion of this material may be reproduced, in any form or by any means, without permission in writing from the publisher. Problem 5.76 State whether each of the L-shaped bars shown is properly or improperly supported. If a bar is properly supported, determine the reactions at its supports. (See Active Example 5.6.) C C –12 L F F –12 L –12 L Solution: (1) (2) (3) is improperly constrained. The reactions intersect at a point P, and the force exerts a moment about that point. is improperly constrained. The reactions intersect at a point P and the force exerts a moment about that point. is properly constrained. The sum of the forces:  A L 45° (2) (1) C FX D C  F D 0, –12 L F –12 L from which C D F.  A B FY D A C B D 0, from which A D B. The sum of the moments about B: LA C L 1 1 F  LC D 0, from which A D F, and B D F 2 2 2 B A B L –12 L L (3) c 2008 Pearson Education, Inc., Upper Saddle River, NJ. All rights reserved. This material is protected under all copyright laws as they  currently exist. No portion of this material may be reproduced, in any form or by any means, without permission in writing from the publisher. 317 Problem 5.77 The bar AB has a built-in support at A and is loaded by the forces y A FB D 2i C 6j C 3k (kN), FB z FC D i  2j C 2k (kN). B 1m (a) Draw the free-body diagram of the bar. (b) Determine the reactions at A. C 1m Strategy: (a) Draw a diagram of the bar isolated from its supports. Complete the free-body diagram of the bar by adding the two external forces and the reactions due to the built-in support (see Table 5.2). (b) Use the scalar equilibrium equations (5.16)–(5.21) to determine the reactions. FC Solution: x AY MA = MAX i + MAY j + MAZ K AX MA D MAX i C MAY j C MAZ k FB (b) Equilibrium Eqns (Forces)    1m FX : AX C FBX C FCX D 0 FY : AY C FBY C FCY D 0 FZ : AZ B C 1m x FC AZ C FBZ C FCZ D 0 Equilibrium Equations (Moments) Sum moments about A rAB ð FB D 1i ð 2i C 6j C 3k kN-m rAB ð FB D 3j C 6k (kN-m) rAC ð FC D 2i ð 1i  2j C 2k kN-m rAC ð FC D 4j  4k (kN-m) x: y: z:    MA : M AX D 0 MA : M AY  3  4 D 0 MA : M AZ C 6  4 D 0 Solving, we get AX D 3 kN, AY D 4 kN, AZ D 5 kN MAx D 0, MAy D 7 kN-m, MAz D 2 kN-m 318 c 2008 Pearson Education, Inc., Upper Saddle River, NJ. All rights reserved. This material is protected under all copyright laws as they  currently exist. No portion of this material may be reproduced, in any form or by any means, without permission in writing from the publisher. y Problem 5.78 The bar AB has a built-in support at A. The tension in cable BC is 8 kN. Determine the reactions at A. A z C (3,0.5,–0.5)m 2m B x Solution: AY MA = MAX i + MAY j + MAZ K AX MA D MAx i C MAy j C MAz k AZ We need the unit vector eBC eBC 2m xC  xB i C yC  yB j C zC  zB k D  xC  xB 2 C yC  yB 2 C zC  zB 2 TBC C (3, 0.5, −0.5) B (2, 0, 0) x eBC D 0.816i C 0.408j  0.408k TBC D 8 kNeBC TBC D 6.53i C 3.27j  3.27k (kN) The moment of TBC about A is MBC D rAB ð TBC   i  D  2  6.53 j 0 3.27  k  0  3.27  MBC D rAB ð TBC D 0i C 6.53j C 6.53k (kN-m) Equilibrium Eqns.  FX : AX C TBCX D 0 FY : AY C TBCY D 0 FZ : AZ C TBCZ D 0 MX : MAX C MBCX D 0 MY : MAY C MBCY D 0 MZ : MAZ C MBCZ D 0      Solving, we get AX D 6.53 (kN), AY D 3.27 (kN), AZ D 3.27 (kN) MAx D 0, MAy D 6.53 (kN-m), MAz D 6.53 (kN-m) c 2008 Pearson Education, Inc., Upper Saddle River, NJ. All rights reserved. This material is protected under all copyright laws as they  currently exist. No portion of this material may be reproduced, in any form or by any means, without permission in writing from the publisher. 319 y Problem 5.79 The bar AB has a fixed support at A. The collar at B is fixed to the bar. The tension in the rope BC is 300 lb. (a) Draw the free-body diagram of the bar. (b) Determine the reactions at A. B (6, 6, 2) ft A x C (8, 0, 3) ft z Solution: (a) The free-body diagram is shown. (b) We need to express the force exerted by the rope in terms of its components. The vector from B to C is rBC D [8  6i C 0  6j C 3  2k] ft D 2i  6j C k ft The force in the rope can now be written T D TBC rBC D TBC 0.312i  0.937j C 0.156k jrBC j The equilibrium equations for the bar are Fx : Ax C 0.312TBC D 0 Fy : Ay  0.937TBC D 0 Fz : Az C 0.156TBC D 0   i  MA : MAx i C MAy j C MAz k C  6 ft  0.312TBC j 6 ft 0.937TBC   k  2 ft  D 0 0.156TBC  These last equations can be written as MAx D 0.218 ftTBC , MAy D 0.312 ftTBC , MAz D 7.50 ftTBC Setting TBC D 300 lb, expanding and solving we have Ax D 93.7 lb, Ay D 281 lb, Az D 46.9 lb MAx D 843 ft-lb, MAy D 93.7 ft-lb, MAz D 2250 ft-lb 320 c 2008 Pearson Education, Inc., Upper Saddle River, NJ. All rights reserved. This material is protected under all copyright laws as they  currently exist. No portion of this material may be reproduced, in any form or by any means, without permission in writing from the publisher. y Problem 5.80 The bar AB has a fixed support at A. The collar at B is fixed to the bar. Suppose that you don’t want the support at A to be subjected to a couple of magnitude greater than 3000 ft-lb. What is the largest allowable tension in the rope BC? B (6, 6, 2) ft Solution: See the solution to Problem 5.79. The magnitude of the couple at A can be expressed in terms of the tension in the rope as A  2 MAx C MAy C MAz jMA j D D 2  2 x 2.81 ft2 C 0.312 ft2 C 7.50 ft2 TBC Setting jMA j D 3000 ft-lb and solving for TBC yields TBC D 374 lb C (8, 0, 3) ft z Problem 5.81 The total force exerted on the highway sign by its weight and the most severe anticipated winds is F D 2.8i  1.8j (kN). Determine the reactions at the fixed support. y F 8m Solution: The applied load is F D 2.8i  1.8j kN applied at r D 8j C 8k m The force reaction at the base is 8m O x R D Ox i C Oy j C Oz k The moment reaction at the base is MO D MOx i C MOy j C MOz k z For equilibrium we need   Fx : 2.8 kN C Ox D 0     FDFCRD0) Fy : 1.8 kN C Oy D 0     Fz : 0 C Oz D 0 Ox D 2.8 kN ) Oy D 1.8 kN Oz D 0   Mx : 14.4 kN-m C MOx D 0     M D r ð F C MO D 0 ) My : 22.4 kN-m C MOy D 0     Mz : 22.4 kN-m C MOz D 0 MOx D 14.4 kN-m ) MOy D 22.4 kN-m MOz D 22.4 kN-m c 2008 Pearson Education, Inc., Upper Saddle River, NJ. All rights reserved. This material is protected under all copyright laws as they  currently exist. No portion of this material may be reproduced, in any form or by any means, without permission in writing from the publisher. 321 Problem 5.82 The tension in cable AB is 800 lb. Determine the reactions at the fixed support C. y 4 ft Solution: The force in the cable is  F D 800 lb 2i  4j  k p 21 C  5 ft We also have the position vector 4 ft rCA D 4i C 5k ft A x The force reaction at the base is R D Cx i C Cy j C Cz k B The moment reaction at the base is z (6, 0, 4) ft MC D MCx i C MCy j C MCz k For equilibrium we need   Fx : Cx C 349 lb D 0     FDFCRD0) Fy : Cy  698 lb D 0      Fz : Cz  175 lb D 0 Cx D 349 lb ) Cy D 698 lb Cz D 175 lb   Mx : MCx C 3490 ft-lb D 0     MDrCFCRD0) My : MCy C 2440 ft-lb D 0      Mz : MCz  2790 ft-lb D 0 MCx D 3490 ft-lb ) MCy D 2440 ft-lb MCz D 2790 ft-lb 322 c 2008 Pearson Education, Inc., Upper Saddle River, NJ. All rights reserved. This material is protected under all copyright laws as they  currently exist. No portion of this material may be reproduced, in any form or by any means, without permission in writing from the publisher. Problem 5.83 The tension in cable AB is 24 kN. Determine the reactions in the built-in support D. 2m C A 2m Solution: The force acting on the device is D B F D FX i C FY j C FZ k D 24 kNeAB , 3m and the unit vector from A toward B is given by eAB D 1m 1i  2j C 1k p . 6 The force, then, is given by F D 9.80i  19.60j C 9.80k kN. The position from D to A is r D 2i C 2j C 0k m. The force equations of equilibrium are DX C FX D 0, DY C FY D 0, and DZ C FZ D 0. The moment equation, in vector form, is  M D MD C r ð F. Expanded, we get    i  M D MDX i C MDY j C MDZ k C  2  9.80 j 2 19.60  k  0  D 0. 9.80  The corresponding scalar equations are MDX C 29.80 D 0, MDY  29.80 D 0, and MDZ C 219.60  29.80 D 0. Solving for the support reactions, we get DX D 9.80 kN, OY D 19.60 kN, OZ D 9.80 kN. MDX D 19.6 kN-m, MDY D 19.6 kN-m, and MDZ D 58.8 kN-m. c 2008 Pearson Education, Inc., Upper Saddle River, NJ. All rights reserved. This material is protected under all copyright laws as they  currently exist. No portion of this material may be reproduced, in any form or by any means, without permission in writing from the publisher. 323 Problem 5.84 The robotic manipulator is stationary and the y axis is vertical. The weights of the arms AB and BC act at their midpoints. The direction cosines of the centerline of arm AB are cos x D 0.174, cos y D 0.985, cos z D 0, and the direction cosines of the centerline of arm BC are cos x D 0.743, cos y D 0.557, cos z D 0.371. The support at A behaves like a built-in support. y 600 mm C 160 N B (a) What is the sum of the moments about A due to the weights of the two arms? (b) What are the reactions at A? 600 mm 200 N A z x Solution: Denote the center of mass of arm AB as D1 and that of BC as D2 . We need rAD , (a) rAB , and rBD2 . We now have the geometry determined and are ready to determine the moments of the weights about A.  MW D rAD1 ð W1 C rAD2 ð W2 where   i  rAD1 ð W1 D  0.0522  0 To get these, use the direction cosines to get the unit vectors eAB and eBC . Use the relation e D cos X i C cos Y j C cos Z k  j k  0.2955 0  200 0  rAD1 ð W1 D 10.44k N-m eAB D 0.174i C 0.985j C 0k and   i  rAD2 ð W2 D  0.3273  0 eBC D 0.743i C 0.557j  0.371k rAD1 D 0.3eAB m j 0.7581 160   k  0.1113   0 rAD2 ð W2 D 17.81i  52.37k rAB D 0.6eAB m Thus,  MW D 17.81i  62.81k (N-m) rBC D 0.6eBC m rBD2 D 0.3eBC m (b) WAB D 200j N Equilibrium Eqns  FX : AX D 0 WBC D 160j N  Thus rAD1 D 0.0522i C 0.2955j m rAB D 0.1044i C 0.5910j m rBD2 D 0.2229i C 0.1671j  0.1113k m rBC D 0.4458i C 0.3342j  0.2226k m and rAD2 D rAB C rBD2 rAD2 D 0.3273i C 0.7581j  0.1113k m 324  FY : AY  W1  W2 D 0 FZ : AZ D 0 Sum Moments about  A : MA C MW D 0    MX : MAx  17.81 D 0 (N-m) MY : MAy C 0 D 0 MZ : MZ  62.81 D 0 (N-m) c 2008 Pearson Education, Inc., Upper Saddle River, NJ. All rights reserved. This material is protected under all copyright laws as they  currently exist. No portion of this material may be reproduced, in any form or by any means, without permission in writing from the publisher. 5.84 (Continued ) Thus: AX D 0, AY D 360 (N), AZ D 0, MAx D 17.81 (N-m), MAy D 0, MAz D 62.81 (N-m) C W2 W1 D2 B D1 MA MA = MAXi + MAYj + MAZk W1 = 200 N W2 = 160 N AX AZ AY c 2008 Pearson Education, Inc., Upper Saddle River, NJ. All rights reserved. This material is protected under all copyright laws as they  currently exist. No portion of this material may be reproduced, in any form or by any means, without permission in writing from the publisher. 325 Problem 5.85 The force exerted on the grip of the exercise machine is F D 260i  130j (N). What are the reactions at the built-in support at O? 150 mm y F O 200 mm z 250 mm Solution: MO D MOx i C MOy j C MOz k OX rOP D 0.25i C 0.2j  0.15k z Equilibrium (Forces)    0.15 P m y MO OZ x OY 0.2 F = 260 i – 130 j (N) 5m 0.2 m x FX : OX C FX D OX C 260 D 0 (N) FY : OY C FY D OY  130 D 0 (N) FZ : OZ C FZ D OZ D 0 (N) Thus, OX D 260 N, OY D 130 N, OZ D 0 Summing Moments about O    MX : MOX C MFX D 0 MY : MOY C MFY D 0 MZ : MOZ C MFZ D 0 where   i  MF D rOP ð F D  0.25  260 j 0.2 130  k  0.15  0  MF D 19.5i  39j  84.5k (N-m) and from the moment equilibrium eqns, MOX D 19.5 (N-m) MOY D 39.0 (N-m) MOZ D 84.5 (N-m) 326 c 2008 Pearson Education, Inc., Upper Saddle River, NJ. All rights reserved. This material is protected under all copyright laws as they  currently exist. No portion of this material may be reproduced, in any form or by any means, without permission in writing from the publisher. y Problem 5.86 In Active Example 5.7, suppose that cable BD is lengthened and the attachment point D moved form (0, 600, 400) mm to (0, 600, 600) mm. (The end B of bar AB remains where it is.) Draw a sketch of the bar and its supports showing cable BD in its new position. Draw the free-body diagram of the bar and apply equilibrium to determine the tensions in the cables and the reactions at A. 400 mm 1000 mm C B D 600 mm 600 mm A x ⫺200j (N) z Solution: The sketch and free-body diagram are shown. We must express the force exerted on the bar by cable BD in terms of its components. The vector from B to D is rBD D [0  1000i C 600  600j C 600  400k] mm D 1000i C 200j mm The force exerted by cable BD can be expressed as TBD rBD D TBD 0.981i C 0.196k jrBD j The equilibrium equations are Fx : Ax  0.981TBD D 0 Fy : Ay  200 N D 0 Fz : Az C 0.196TBD  TBC D 0   i  1 MA :   0.981TBD     i j k    C  0.5 0.6 0.4   0 0.196TBD  TBC   0 j 0.3 200  k  0.2  D 0 0  Expanding and solving these equations, we find Ax D 166.7 N, Ay D 200 N, Az D 66.7 N, TBC D 100 N, TBD D 170 N c 2008 Pearson Education, Inc., Upper Saddle River, NJ. All rights reserved. This material is protected under all copyright laws as they  currently exist. No portion of this material may be reproduced, in any form or by any means, without permission in writing from the publisher. 327 Problem 5.87 The force F acting on the boom ABC at C points in the direction of the unit vector 0.512i  0.384j C 0.768k and its magnitude is 8 kN. The boom is supported by a ball and socket at A and the cables BD and BE. The collar at B is fixed to the boom. y 1.5 m 2m D E (a) Draw the free-body diagram of the boom. (b) Determine the tensions in the cables and the reactions at A. 1m 2m A B z 2m C 2m x Solution: F (a) The free-body diagram (b) We identify the following forces, position vectors, and reactions rAC D 4 mi, F D 8 kN0.512i  0.384j C 0.768k    2i C 2j C 1.5k   p T D T  BD BD  10.25 D 2 mi,    2i C j  2k    TBE D TBE 3 rAB Az Ax TBE B Ay TBD C R D Ax i C Ay j C Az k Force equilibrium requires:  F D R C TBD C TBE C F D 0. F In component form we have   2 2 TBD  TBE D 0 Fx : Ax C 8 kN0.512  p 3 10.25 2 1 TBD C TBE D 0 Fy : Ay  8 kN0.384 C p 3 10.25  1.5 2 TBD  TBE D 0 Fz : Az C 8 kN0.768 C p 3 10.25 Moment equilibrium requires:  MA D rAB ð TBD C TBE  C rAC ð F D 0. In components:   Mx : 0 D 0 My : 8 kN0.7684 m  p C  TBD 2 m 2 TBE 2 m D 0 3 Mz : 8 kN0.3844 m C p C 1.5 10.25 2 10.25 TBD 2 m 1 TBE 2 m D 0 3 Solving five equations for the five unknowns we find Ax D 8.19 kN, Ay D 3.07 kN, Az D 6.14 kN, TBD D 0, TBE D 18.43 kN 328 c 2008 Pearson Education, Inc., Upper Saddle River, NJ. All rights reserved. This material is protected under all copyright laws as they  currently exist. No portion of this material may be reproduced, in any form or by any means, without permission in writing from the publisher. Problem 5.88 The cables BD and BE in Problem 5.87 will each safely support a tension of 25 kN. Based on this criterion, what is the largest acceptable magnitude of the force F? Solution: We have the force and distances: rAC D 4 mi, F D F0.512i  0.384j C 0.768k rAB    2i C 2j C 1.5k   p   TBD D TBD 10.25 D 2 mi,    2i C j  2k    TBE D TBE 3 The moment equations are   1.5 2 TBD 2 m C TBE 2 m D 0 My : F0.7684 m  p 3 10.25 2 1 TBD 2 m C TBE 2 m D 0 Mz : F0.3844 m C p 3 10.25 Solving we find TBE D 2.304F, TBD D 0 Thus: 25 kN D 2.304F ) F D 10.85 kN c 2008 Pearson Education, Inc., Upper Saddle River, NJ. All rights reserved. This material is protected under all copyright laws as they  currently exist. No portion of this material may be reproduced, in any form or by any means, without permission in writing from the publisher. 329 Problem 5.89 The suspended load exerts a force F D 600 lb at A, and the weight of the bar OA is negligible. Determine the tensions in the cables and the reactions at the ball and socket support O. y C (0, 6, –10) ft A (8, 6, 0) ft B (0, 10, 4) ft –Fj x O Solution: From the diagram, the important points in this problem z are A (8, 6, 0), B (0, 10, 4), C (0, 6, 10), and the origin O (0, 0, 0) with all dimensions in ft. We need unit vectors in the directions A to B and A to C. Both vectors are of the form eAP D xP  xA i C yP  yA j C zP  zA k, If we carry through these operations in the sequence described, we get the following vectors: where P can be either A or B. The forces in cables AB and AC are TAB D TAB eAB D TABX i C TABY j C TABZ k, and TAC D TAC eAB D TACX i C TACY j C TACZ k. eAB D 0.816i C 0.408j C 0.408k, eAC D 0.625i C 0j  0.781k, TAB D 387.1i C 193.5j C 193.5k lb, The weight force is jTAB j D 474.1 lb, F D 0i  600j C 0k, and the support force at the ball joint is TAC D 154.8i C 0j  193.5k lb, jTAC j D 247.9 lb, S D SX i C SY j C SZ k. The vector form of the force equilibrium equation (which gives three scalar equations) for the bar is MAB D rOA ð TAB D 1161i  1548j C 3871k ft-lb, MAC D rOA ð TAC D 1161i C 1548j C 929k ft-lb, TAB C TAC C F C S D 0. Let us take moments about the origin. The moment equation, in vector form, is given by  and S D 541.9i C 406.5j C 0k lb MO D rOA ð TAB C rOA ð TAC C rOA ð F D 0, where rOA D 8i C 6j C 0k. The cross products are evaluated using the form   i  M D r ð H D  8  HX j 6 HY  k  0  , HZ  where H can be any of the three forces acting at point A. The vector moment equation provides another three equations of equilibrium. Once we have evaluated and applied the unit vectors, we have six vector equations of equilibrium in the five unknowns TAB , TAC , SX , SY , and SZ (there is one redundant equation since all forces pass through the line OA). Solving these equations yields the required values for the support reactions at the origin. 330 c 2008 Pearson Education, Inc., Upper Saddle River, NJ. All rights reserved. This material is protected under all copyright laws as they  currently exist. No portion of this material may be reproduced, in any form or by any means, without permission in writing from the publisher. Problem 5.90 In Problem 5.89, suppose that the suspended load exerts a force F D 600 lb at A and bar OA weighs 200 lb. Assume that the bar’s weight acts at its midpoint. Determine the tensions in the cables and the reactions at the ball and socket support O. Solution: Point G is located at (4, 3, 0) and the position vector of G with respect to the origin is rOG D 4i C 3j C 0k ft. The weight of the bar is WB D 0i  200j C 0k lb, and its moment around the origin is MWB D 0i C 0j  800k ft-lb. The mathematical representation for all other forces and moments from Problem 5.89 remain the same (the numbers change!). Each equation of equilibrium has a new term reflecting the addition of the weight of the bar. The new force equilibrium equation is TAB C TAC C F C S C WB D 0. The new moment equilibrium equation is  MO D rOA ð TAB C rOA ð TAC C rOA ð F C rOG ð WB D 0. As in Problem 5.89, the vector equilibrium conditions can be reduced to six scalar equations of equilibrium. Once we have evaluated and applied the unit vectors, we have six vector equations of equilibrium in the five unknowns TAB , TAC , SX , SY , and SZ (As before, there is one redundant equation since all forces pass through the line OA). Solving these equations yields the required values for the support reactions at the origin. If we carry through these operations in the sequence described, we get the following vectors: eAB D 0.816i C 0.408j C 0.408k, eAC D 0.625i C 0j  0.781k, TAB D 451.6i C 225.8j C 225.8k lb, jTAB j D 553.1 lb, TAC D 180.6i C 0j  225.8k lb, jTAC j D 289.2 lb, MAB D rOA ð TAB D 1355i  1806j C 4516k ft-lb, MAC D rOA ð TAC D 1354i C 1806j C 1084k ft-lb, and S D 632.3i C 574.2j C 0k lb c 2008 Pearson Education, Inc., Upper Saddle River, NJ. All rights reserved. This material is protected under all copyright laws as they  currently exist. No portion of this material may be reproduced, in any form or by any means, without permission in writing from the publisher. 331 Problem 5.91 The 158,000-kg airplane is at rest on the ground (z D 0 is ground level). The landing gear carriages are at A, B, and C. The coordinates of the point G at which the weight of the plane acts are (3, 0.5, 5) m. What are the magnitudes of the normal reactions exerted on the landing gear by the ground? 21 m 6m B G A x C 6m y Solution:   mg 3m FY D NL C NR  C NF  W D 0 MR D 3 mg C 21NF D 0 21 m Side View x R Solving, NF D 221.4 kN F (NL + NR) (1) NF Z NL C NR  D 1328.6 kN  (2) 0.5 m W FY D NR C NL C NF  W D 0 (same equation as before) C  MO D 0.5 W  6NR  C 6NL  D 0 (3) Front View y Solving (1), (2), and (3), we get 6 6 NF D 221.4 kN NR D 728.9 kN NF NR NL z NL D 599.7 kN 332 c 2008 Pearson Education, Inc., Upper Saddle River, NJ. All rights reserved. This material is protected under all copyright laws as they  currently exist. No portion of this material may be reproduced, in any form or by any means, without permission in writing from the publisher. Problem 5.92 The horizontal triangular plate is suspended by the three vertical cables A, B, and C. The tension in each cable is 80 N. Determine the x and z coordinates of the point where the plate’s weight effectively acts. y A B C 0.3 m 0.4 m (x, 0, z) x z Solution:   80 N 80 N Mx : 240 Nz  80 N0.4 m D 0 Mz : 80 N0.3 m  240 Nx D 0 z 80 N x X Solving x D 0.1 m, z D 0.1333 m 240 N Z Problem 5.93 The 800-kg horizontal wall section is supported by the three vertical cables, A, B, and C. What are the tensions in the cables? B 7m Solution: All dimensions are in m and all forces are in N. Forces A, B, C, and W act on the wall at (0, 0, 0), (5, 14, 0), (12, 7, 0), and (4, 6, 0), respectively. All forces are in the z direction. The force equilibrium equation in the z direction is A C B C C  W D 0. The moments are calculated from MB D rOB ð Bk, C A 7m 7m 6m 4m 8m mg MC D rOC ð Ck, and MG D rOG ð Wk. The moment equilibrium equation is  MO D MB C MC C MG D 0. Carrying out these operations, we get A D 3717 N, B D 2596 N, C D 1534 N, and W D 7848 N. c 2008 Pearson Education, Inc., Upper Saddle River, NJ. All rights reserved. This material is protected under all copyright laws as they  currently exist. No portion of this material may be reproduced, in any form or by any means, without permission in writing from the publisher. 333 Problem 5.94 The bar AC is supported by the cable BD and a bearing at A that can rotate about the z axis. The person exerts a force F D 10j (lb) at C. Determine the tension in the cable and the reactions at A. y A x B C 8 in 14 in z (18, ⫺8, 7) in D Solution: The force in the cable is  TBD D TBD 10i  8j C 7k p 213  Ax, MAx Az F = 10 lbj We have the following six equilibrium equations       10 TBD D 0 Fx : Ax C p 213 8 TBD C 10 lb D 0 Fy : Ay  p 213 Ay MAy TBD 7 TBD D 0 Fz : Az C p 213 Mx : MAx D 0 My : MAy  p 7 213 TBD 8 in D 0 8 TBD 8 in C 10 lb22 in D 0 Mz : p 213 Solving we find Ax D 34.4 lb, Ay D 17.5 lb, Az D 24.1 lb MAx D 0, MAy D 192.5 lb in, TBD D 50.2 lb 334 c 2008 Pearson Education, Inc., Upper Saddle River, NJ. All rights reserved. This material is protected under all copyright laws as they  currently exist. No portion of this material may be reproduced, in any form or by any means, without permission in writing from the publisher. y Problem 5.95 The L-shaped bar is supported by a bearing at A and rests on a smooth horizontal surface at B. The vertical force F D 4 kN and the distance b D 0.15 m. Determine the reactions at A and B. F b A x Solution: Equilibrium Eqns:    B FX : 0.2 m OD0 0.3 m z FY : AY C B  F D 0 FZ : AZ D 0 y Sum moments around A F b x: Fb  0.3B D 40.15  0.3B D 0 y: M AY D 0 z: MAZ C 0.2F  0.2B D 0 MZ 0.2 m A B 3 0. m x AY b = 0.15 m F = 4 kN AZ B z (MAX ≡ 0) AX ≡ 0 MY Solving, AX D 0, AY D 2 (kN), AZ D 0 MAX D 0, MAY D 0, MAZ D 0.4 (kN-m) Problem 5.96 In Problem 5.95, the vertical force F D 4 kN and the distance b D 0.15 m. If you represent the reactions at A and B by an equivalent system consisting of a single force, what is the force and where does its line of action intersect the xz plane? Solution: We want to represent the forces at A & B by a single zR 4 D C0.32 force. From Prob. 5.95 zR D C0.15 m A D C2j (kN), xR 4 D 0.22 B D C2j (kN) xR D 0.1 m MA D 0.4k (kN-m) y We want a single equivalent force, R that has the same resultant force and moment about A as does the set A, B, and MA . R F b A R D A C B D 4j (kN) x Let R pierce the xz plane at xR , zR    B MX :  zR R D 0.3B z MZ : 0.2 m 0.3 m  xR R D 0.2AY c 2008 Pearson Education, Inc., Upper Saddle River, NJ. All rights reserved. This material is protected under all copyright laws as they  currently exist. No portion of this material may be reproduced, in any form or by any means, without permission in writing from the publisher. 335 Problem 5.97 In Problem 5.95, the vertical force F D 4 kN. The bearing at A will safely support a force of 2.5-kN magnitude and a couple of 0.5 kN-m magnitude. Based on these criteria, what is the allowable range of the distance b? Solution: The solution to Prob. 5.95 produced the relations AY C B  F D 0 F D 4 kN Fb  0.3B D 0 MAZ C 0.2F  0.2B D 0 AX D AZ D MAX D MAY D 0 Set the force at A to its limit of 2.5 kN and solve for b. In this case, MAZ D 0.5 (kN-m) which is at the moment limit. The value for b is b D 0.1125 m We make AY unknown, b unknown, and B unknown F D 4 kN, MAY D C0.5 (kN-m), and solve we get AY D 2.5 at b D 0.4875 m However, 0.3 is the physical limit of the device. Thus, 0.1125 m  b  0.3 m y F b A x B z 336 0.2 m 0.3 m c 2008 Pearson Education, Inc., Upper Saddle River, NJ. All rights reserved. This material is protected under all copyright laws as they  currently exist. No portion of this material may be reproduced, in any form or by any means, without permission in writing from the publisher. Problem 5.98 The 1.1-m bar is supported by